Download as docx, pdf, or txt
Download as docx, pdf, or txt
You are on page 1of 595

Prayer Before the Exam

O Lord, before You lie my strength and my


University of San Agustin weakness; Preserve the one, heal the other. Before
You lie my knowledge and my ignorance. Come to
COLLEGE OF COMMERCE my assistance as I prepare for this test. Help me to
remember all that is important. Let me remember
Department of Accountancy You, let me understand You, let me love You.
AE 19 Increase these things in me until You refashion me
to Your will. Amen.
MANAGEMENT ADVISORY SERVICES (Inspired by: St. Augustine, The Trinity XV, 51)
CAPITAL BUDGETING
THEORIES
1st Semester, 2020-2021

BASIC CONCEPTS
Capital Budgeting Defined
1
. The capital budget is a(n)
a. Plan to insure that there are sufficient funds available for the operating needs of the company.
b. Exercise that sets the long-range goals of the company including the consideration of external influences.
c. Plan that coordinates and communicates a company’s plan for the coming year to all departments and divisions.
d. Plan that assesses the long-term needs of the company for plant and equipment purchases. CMA 0695 3-17
2
. In planning and controlling capital expenditures, the most logical sequence is to begin with
a. Analyzing capital addition proposals.
b. Making capital and expenditure decisions.
c. Analyzing and evaluating all promising alternatives.
d. Identifying capital addition projects and other capital needs. CMA 0696 3-11

21. Which of the following involves significant financial investments in projects to develop new products, expand production capacity,
or remodel current production facilities? (E)
a. Capital budgeting c. Master budgeting
b. Working capital d. Project-cost budgeting Horngren

*. The detailed plan for the acquisition and replacement of major portions of property, plant, and equipment is known as the
a. capital budget. c. commitments budget.
b. purchases budget. d. treasury budget. Barfield

2. _____________ decisions are concerned with the process of planning, setting goals and priorities, arranging financing, and
identifying criteria for making long-term investments.
a. Limited resources c. Capital investment
b. Sell now or process further d. Make or buy H&M

Capital Budgeting vs. Strategic Planning


3. Which of the following statements regarding capital budgeting and strategic planning is true?
A. Capital budgeting and strategic planning are bottom-up processes
B. Capital budgeting and strategic planning are top down processes B&M
C. Capital budgeting is a top-down process, while strategic planning is a bottom-up process
D. Capital budgeting is a bottom-up process, while strategic planning is a top-down process

Agency Problem in Capital Budgeting


15. The following are agency problems in capital budgeting except:
A. Reduced effort C. Empire building
B. Need for good information D. Perks B&M

16. The following are agency problems in capital budgeting except:


A. Empire building C. Avoiding risks
B. Entrenching investment D. Reducing forecast bias B&M

17. The following are agency problems in capital budgeting except:


A. Avoiding risks
B. The free-rider problem
C. Compensation
D. All of the above are agency problems B&M

18. The following are agency problems in capital budgeting except:


A. Monitoring C. Avoiding risks
B. Compensation D. Economic value added B&M

Uses
107.A capital budget is used by management to determine
Barfield a. b. c. d.
In what to invest No No Yes Yes
How much to invest No Yes No Yes

3
. Capital budgeting is concerned with
A. Decisions affecting only capital intensive industries.
B. Analysis of short-range decisions.
C. Analysis of long-range decisions.
D. Scheduling office personnel in office buildings. Gleim
4
. Capital budgeting is used for the decision analysis of
A. Adding product lines or facilities. C. Lease-or-buy decisions.
B. Multiple profitable alternatives. D. All of the answers are correct. Gleim
5
. Capital budgeting techniques are least likely to be used in evaluating the (E)
a. Acquisition of new aircraft by a cargo company.
b. Design and implementation of a major advertising program.
c. Trade for a star quarterback by a football team. CMA 0693 4-19
d. Adoption of a new method of allocating non-traceable costs to product lines.

Application
2. A company can replace the machinery currently used to manufacture its product with more efficient machinery. The new
machinery will reduce labor and also will reduce the percentage of spoiled units. It is expected to have a useful life of 5 years.
The most appropriate technique for determining whether or not the company should replace its machinery with the new, more
efficient machinery is:
A. cost-volume-profit analysis C. regression analysis
B. capital-budgeting analysis D. linear programming CIA adapted

Not-for-Profit Entities
10. Not-for-profit entities
a. Cannot use capital budgeting techniques because profitability is irrelevant to them.
b. Cannot use discounted cash flow techniques because the time value of money is irrelevant to them.
c. Might have serious problems in quantifying the benefits expected from an investment.
d. Should use the IRR method to make investment decisions. L&H

Life Cycle Costing


22. The accounting system that corresponds to the project dimension in capital budgeting is the (E)
a. net present value method. c. accrual accounting rate of return.
b. internal rate of return. d. life-cycle costing. Horngren

Stages
Identification Stage
23. The stage of the capital budgeting process which distinguishes which types of capital expenditure projects are necessary to
accomplish organization objectives is the (E)
a. identification stage. c. information-acquisition stage.
b. search stage. d. selection stage. Horngren

Information-Acquisition Stage
25. The stage of the capital budgeting process which considers the expected costs and the expected benefits of alternative capital
investments is the (E)
a. identification stage. c. information-acquisition stage.
b. search stage. d. selection stage. Horngren

Search Stage
24. The stage of the capital budgeting process which explores alternative capital investments that will achieve organization objectives
is the (E)
a. identification stage. c. information-acquisition stage.
b. search stage. d. selection stage. Horngren

Selection Stage
26. The stage of the capital budgeting process which chooses projects for implementation is the (E)
a. selection stage. c. identification stage.
b. search stage. d. management-control stage. Horngren

Appropriation Requests
8. Which of the following statements regarding appropriation requests is true?
A. Usually submitted by head office staff
B. Represents the first step in the capital budgeting process
C. Authorization tends to be reserved for senior management
D. None of the above B&M

9. Which of the following statements regarding appropriation requests is true?


A. Usually submitted by head office staff
B. Usually requires a detailed analysis using more than one investment criterion
C. Usually submitted to a single review at the head office
D. None of the above B&M

Implementation & Control Stage


27. The stage of the capital-budgeting process in which projects get underway and performance is monitored is the (E)
a. implementation and control stage. c. identification stage.
b. search stage. d. management-control stage. Horngren

Acquisition Considerations
6. Effective planning and control is important for the effective administration of a capital expenditure program because:
A. the long-term commitment increases financial risk
B. the magnitude of expenditures is substantial and the economic penalties for unwise decisions are usually severe
C. decisions made in this area provide the structure for operation of the firm
D. all of the above Carter & Usry

7. A company manual used for detailing policies and procedures required for administering the capital expenditure program should:
A. encourage people to work on and submit new ideas
B. focus attention on useful analytical tasks
C. facilitate rapid project development and expeditious review
D. all of the above Carter & Usry

14. The normal methods of analyzing investments


a. Cannot be used by not-for-profit entities.
b. Do not apply if the project will not produce revenues.
c. Cannot be used if the company plans to finance the project with funds already available internally.
d. Require forecasts of cash flows expected from the project. L&H
6
. High-Tech Industries is considering the acquisition of a new state-of-the-art manufacturing machine to replace a less efficient
machine. Hi-Tech has completed a net present value analysis and found it to be favorable. Which one of the following factors
should not be of concern to Hi-Tech in its acquisition considerations?
A. The availability of any necessary financing.
B. The probability of near-term technological changes to the manufacturing process.
C. The investment tax credit. CMA 1290 4-18
D. Maintenance requirements, warranties, and availability of service arrangements.

8. A number of evaluations of a single capital expenditure proposal may be necessary because of:
A. circumstances that change during the time span from the origin of the project idea to its completion
B. alternative solutions of the problem for which the project is designed
C. assumptions that vary as to the amount and timing of cash flows
D. all of the above Carter & Usry

23. Which of the following events is most likely to increase the number of investments that meet a company’s acceptance criteria? (M)
a. Top management raises the target rate of return.
b. The interest rate on long-term debt rises.
c. The income tax rate rises.
d. The IRS allows companies to expense purchases of fixed assets, instead of depreciating them over their lives. L&H

Qualitative Factors
37. Qualitative issues could increase the acceptability of a project under which of the following conditions?
a. The IRR is less than the company’s cutoff rate.
b. The project has a negative NPV.
c. The payback period is longer than the company’s cutoff period.
d. All of the above. L&H

40. Qualitative factors can influence managers to


a. Accept an investment project having a negative NPV.
b. Reject an investment project having an IRR greater than the company’s cutoff rate.
c. Raise the “ranking” of an investment project.
d. Take any of the above courses of action. L&H

76. Which of the following statements is (are) true about automation?


a. Automation is inexpensive.
b. Automation should be adopted as soon as new technology is available.
c. Automation should be adopted after a company makes the most efficient use of existing technology.
d. All of the above are true H&M

Ethical Consideration
4. Common problems related to ethical considerations in the capital budgeting include all of the following, except:
A. superiors and associates sometimes apply pressure to circumvent the approval process
B. pressure may exist to write-off or devalue assets below their true value to justify replacement
C. the economic benefit of capital projects may be exaggerated to increase the likelihood of approval
D. the accountant may mistakenly go to the individuals involved in the ethical conflict first, rather than first discussing it with the
accounting supervisor
E. all of the above are ethical problems related to capital budgeting AICPA adapted

Post- Investment Audit


69. Comparison of the actual results for a project to the costs and benefits expected at the time the project was selected is referred to
as (E)
a. the audit trail. c. a postinvestment audit.
b. management control. d. a cost-benefit analysis. Horngren

10. Post audit is conducted:


A. Before starting a project
B. Before authorizing a project
C. Shortly after a project has started operating B&M
D. Long after the project has been completed and the salvage has been realized

77. A post audit compares


a. estimated benefits and costs with budgeted benefits and cost
b. estimated benefits with estimated costs
c. actual benefits with actual costs
d. actual benefits and costs with estimated benefits and costs H&M

27 The post-audit is used to(E)


a. Improve cash flow forecasts.
b. Stimulate management to improve operations and bring results into line with forecasts.
c. Eliminate potentially profitable but risky projects.
d. Statements a and b are correct.
e. All of the statements above are correct. Brigham

11. A post audit will:


A. Identify the problem that needs to be fixed
B. Check the accuracy of the cash flow forecasts
C. Suggest questions that should have been asked before
D. All of the above B&M

Type of Capital Expenditure


Replacement Expenditures
9. The following capital expenditures that compare the future costs of the old assets with the future costs of the new assets as a basis
for making a decision are:
A. replacement expenditures C. improvement expenditures
B. expansion expenditures D. allowance expenditures Carter & Usry

Expansion Expenditures
10. In which of the following types of capital expenditure decisions does the basis for a decision most markedly shift from cost savings
to increased profits and cash flow?
A. replacement expenditures C. improvement expenditures
B. expansion expenditures D. allowance expenditures Carter & Usry

Improvement Expenditures
11. The capital expenditures in which the benefits are most difficult to quantify are:
A. replacement expenditures C. improvement expenditures
B. expansion expenditures D. allowance expenditures Carter & Usry

Not a Capital Expenditure


6. The following capital expenditures may not appear in the capital budget except:
A. Investment in information technology
B. Investment in research and development
C. Investment in training and personal development
D. Investment in a new office building B&M
7. The following capital expenditures may not appear in the capital budget except:
A. Marketing
B. Training and personnel development
C. Investment in a new machine
D. Investment in research and development B&M

4. Which of the following capital expenditure may not appear in capital budget?
A. Investment in a new plant
B. Investment in a new machine
C. Investment in information technology
D. All of the above are included in capital budget B&M

5. Which of the following capital expenditures may not appear in capital budget?
A. Investment in a new building
B. Investment in a new machine
C. Investment in research and development
D. All of the above are included in capital budget B&M

Types of Projects
Independent Projects
1. ______________ are projects that when accepted or rejected will NOT affect the cash flows of another project.
a. Independent projects c. Dependent projects
b. Mutually exclusive projects d. Both b and c H&M

Mutually Exclusive Projects


4. ______________are projects that when accepted preclude the acceptance of competing projects.
a. Independent projects c. Dependent projects
b. Mutually exclusive projects d. Both b and c H&M

Inflation Element
*. The “inflation element” refers to the
a. Impact that future price increases will have on the original cost of a capital expenditure.
b. Fact that the real purchasing power of a monetary unit usually increases over time.
c. Future deterioration of the general purchasing power of the monetary unit.
d. Future increases in the general purchasing power of the monetary unit. RPCPA 0597

Relevant and Irrelevant Costs


28. Capital budgeting emphasizes two factors (E)
a. qualitative and nonfinancial. c. quantitative and financial
b. quantitative and nonfinancial. d. qualitative and financial. Horngren

29. Which of the following are NOT included in the formal financial analysis of a capital budgeting program? (E)
a. Quality of the output c. Cash flow
b. Safety of employees d. Neither (a) nor (b) are included Horngren

56. The focus in capital budgeting should be on (E)


a. the tax consequences of different investment strategies.
b. the internal rate of return of different strategies.
c. expected future cash flows that differ between alternatives.
d. none of the above. Horngren

32. The only future costs that are relevant to deciding whether to accept an investment are those that will
a. Be different if the project is accepted rather than rejected.
b. Be saved if the project is accepted rather than rejected.
c. Be deductible for tax purposes.
d. Affect net income in the period that they are incurred. L&H
7
. Which of the following rules are essential to successful cash flow estimates, and ultimately, to successful capital budgeting? (M)
a. The return on invested capital is the only relevant cash flow.
b. Only incremental cash flows are relevant to the accept/reject decision.
c. Total cash flows are relevant to capital budgeting analysis and the accept/reject decision.
d. Statements a and b are correct.
e. All of the statements above are correct. Brigham

Salvage Value
60. The relevant terminal disposal price of a machine equals (M)
a. the difference between the salvage value of the old machine and the ultimate salvage value of the new machine.
b. the total of the salvage values of the old machine and the new machine.
c. the salvage value of the old machine.
d. the salvage value of the new machine. Horngren

*. Karen Company is considering replacing an old machine with a new machine. Which of the following items is economically
relevant to Karen’s decisions? (M)
RPCPA 0598 a. b. c. d.
Carrying amount of old machine Yes Yes No No
Disposal value of new machine Yes No Yes No

*. You are the treasurer of the Hibang Corp. The company is considering a proposed project which has an expected economic life of
seven years. Net present value is the capital budgeting technique the president wants you to use. Salvage value of the project
would be (M)
a. Treated as cash inflow at estimated salvage value.
b. Treated as cash flow at its present value.
c. Irrelevant cash flow item.
d. Treated as cash inflow at the future value. RPCPA 1096

Working Capital
66. In the analysis of a capital budgeting proposal, for which of the following items are there no after-tax consequences? (E)
a. Cash flow from operations
b. Gain or loss on the disposal of the asset
c. Reduction of working capital balances at the end of the useful life of the capital asset
d. There are no after-tax consequences of any of the above. Horngren

11. A major difference between an investment in working capital and one in depreciable assets is that (M)
a. An investment in working capital is never returned, while most depreciable assets have some residual value.
b. An investment in working capital is returned in full at the end of the project’s life, while an investment in depreciable assets has
no residual value.
c. An investment in working capital is not tax-deductible when made, not taxable when returned, while an investment in
depreciable assets does allow tax deductions.
d. Because an investment in working capital is usually returned in full at the end of the project’s life, it is ignored in computing the
amount of the investment required for the project. L&H

*. Mahlin Movers, Inc. is planning to purchase equipment to make its operations more efficient. This equipment has an estimated
useful life of six years. As part of this acquisition, a P150,000 investment in working capital is required. In a discounted cash flow
analysis, this investment in working capital should be (E)
a. Amortized over the useful life of the equipment.
b. Disregarded because no cash is involved. RPCPA 1095
c. Treated as a recurring annual cash flow that is recovered at the end of six years.
d. Treated as an immediate cash outflow that is recovered at the end of six years.
8
. Fast Freight, Inc. is planning to purchase equipment to make its operations more efficient. This equipment has an estimated life of
6 years. As part of this acquisition, a $75,000 investment in working capital is anticipated. In a discounted cash flow analysis, the
investment in working capital (E)
a. Should be amortized over the useful life of the equipment.
b. Should be treated as a recurring cash outflow over the life of the equipment.
c. Should be treated as an immediate cash outflow. CMA 0691 4-20
d. Should be treated as an immediate cash outflow recovered at the end of 6 years.

32. In connection with a capital budgeting project, an investment in working capital is normally recovered
a. At the end of the project’s life. c. Evenly through the project’s life. L&H
b. In the first year of the project’s life. d. When the company goes out of business.

12. The proper treatment of an investment in receivables and inventory is to


a. Ignore it.
b. Add it to the required investment in fixed assets.
c. Add it to the required investment in fixed assets and subtract it from the annual cash flows.
d. Add it to the investment in fixed assets and add the present value of the recovery to the present value of the annual cash
flows. L&H

34. The cash inflow from the return on an investment in working capital is
a. Adjusted for taxes due.
b. Discounted to present value.
c. Ignored if any depreciable assets also involved in the project have no expected residual value.
d. Not real. L & H, RPCPA 1001
29. XYZ Co. is adopting just-in-time principles. When evaluating an investment project that would reduce inventory, how should XYZ
treat the reduction?
a. Ignore it.
b. Decrease the cost of the investment and decrease cash flows at the end of the project’s life.
c. Decrease the cost of the investment.
d. Decrease the cost of the investment and increase the cash flow at the end of the project’s life. L&H

4. Net Working Capital should be considered in project cash flows because:


A. They are sunk costs
B. Firms must invest cash in short-term assets to produce finished goods
C. Firms need positive NPV projects for investment
D. None of the above B&M

Opportunity Costs
10. The value of a previously purchased machine expected to be used by a proposed project is an example of:
A. Sunk costs C. Fixed costs
B. Opportunity costs D. None of the above B&M

Cash Outflows
14. All of the following are common cash outflows from capital expenditure programs, except:
A. equipment installation D. increased working capital requirements
B. employee training E. salvage value at the end of the project
C. computer programming and fine tuning Carter & Usry

33. For investments that have only costs (no revenues or cost savings), an appropriate decision rule is to accept the project that has
the
a. Longest payback period.
b. Lower present value of cash outflows.
c. Higher present value of future cash outflows.
d. Lowest internal rate of return. L&H

Cash Inflows
15. As to a capital investment, net cash inflow is equal to the
a. cost savings resulting from the investment.
b. sum of all future revenues from the investment.
c. net increase in cash receipts over cash payments.
d. net increase in cash payments over cash receipts. Barfield

8. Which of the following describes the annual returns that are discounted in determining the NPV of an investment?
a. Net incomes expected to be earned by the project.
b. Pre-tax cash flows expected from the project.
c. After-tax cash flows expected from the project.
d. After-tax cash flows adjusted for the time value of money. L&H

57. Annual after-tax corporate net income can be converted to annual after-tax cash flow by (E)
a. adding back the depreciation amount.
b. deducting the depreciation amount. Barfield
c. adding back the quantity (t x depreciation deduction), where t is the corporate tax rate.
d. deducting the quantity [(1- t) x depreciation deduction], where t is the corporate tax rate.

. To approximate annual cash inflow, depreciation is


a. Added back to net income because it is an inflow of cash.
b. Subtracted from net income because it is an outflow of cash.
c. Subtracted from net income because it is an expense.
d. Added back to net income because it is not an outflow of cash. RPCPA 1001

59. Depreciation is usually not considered an operating cash flow in capital budgeting because (E)
a. depreciation is usually a constant amount each year over the life of the capital investment.
b. deducting depreciation from operating cash flows would be counting the lump-sum amount twice.
c. depreciation usually does not result in an increase in working capital.
d. depreciation usually has no effect on the disposal price of the machine. Horngren

25. Which of these could occur in practice where the capital expenditure relates to the production of an established product or service,
the demand for which is expected to vary in response to temporary changes in consumer taste?
A. perfectly correlated cash flows C. independent cash flows
B. negative cash flows D. mixed cash flows Carter & Usry

3. Which of the following is NOT relevant in calculating annual net cash flows for an investment?
a. Interest payments on funds borrowed to finance the project.
b. Depreciation on fixed assets purchased for the project.
c. The income tax rate.
d. Lost contribution margin if sales of the product invested in will reduce sales of other products. L&H

39. Which of the following is NOT relevant in calculating net cash flows for Project N?
a. Interest payments on funds that would be borrowed to finance Project N.
b. Depreciation on assets purchased for Project N.
c. The contribution margin the company would lose if sales of the product introduced by Project N will reduce sales of other
products.
d. The income tax rate applicable to the entity. L&H

13. All of the following are common cash inflows related to capital expenditure proposals, except:
A. additional revenues from increased sales
B. increased working capital requirements
C. reduction in inventory carrying costs
D. salvage value at the end of the project Carter & Usry

Tax Shield on Depreciation


8. Which of the following cash flows should be treated as incremental flows when deciding whether to go ahead with an electric car?
A. The cost of research and development undertaken for developing the electric car in the past three years
B. The annual depreciation charge
C. The reduction in taxes resulting from the depreciation charges
D. Dividend payments B&M

Irrelevant Costs
9. Money that a firm has already spent or committed to spend regardless of whether a project is taken is called:
A. Sunk costs C. Fixed costs
B. Opportunity costs D. None of the above B&M

*. In capital expenditures decisions, the following are relevant in estimating operating costs except (E)
a. Future costs. c. Differential costs.
b. Cash costs. d. Historical costs. RPCPA 1094

58. An example of a sunk cost in a capital budgeting decision for new equipment is (E)
a. increase in working capital required by a particular investment choice.
b. the book value of the old equipment.
c. the necessary transportation costs on the new equipment.
d. all of the above are examples of sunk costs. Horngren

7. The following cash flows should be treated as incremental flows when deciding whether to go ahead with an electric car except:
(M)
A. The consequent deduction in sales of the company's existing gasoline models
B. The expenditure on new plants and equipment
C. The value of tools that can be transferred from the company's existing plants
D. Interest payment on debt B&M

Comprehensive
9
. Which of the following statements is most correct? (E)
a. When evaluating corporate projects it is important to include all sunk costs in the estimated cash flows.
b. When evaluating corporate projects it is important to include all relevant externalities in the estimated cash flows.
c. Interest expenses should be included in project cash flows.
d. Statements a and b are correct. Brigham
10
. When evaluating potential projects, which of the following factors should be incorporated as part of a project’s estimated cash
flows? (E)
a. Any sunk costs that were incurred in the past prior to considering the proposed project.
b. Any opportunity costs that are incurred if the project is undertaken.
c. Any externalities (both positive and negative) that are incurred if the project is undertaken.
d. Statements b and c are correct. Brigham
11
. Which one of the following statements concerning cash flow determination for capital budgeting purposes is not correct?
a. Tax depreciation must be considered since it affects cash payments for taxes.
b. Book depreciation is relevant since it affects net income.
d. Net working capital changes should be included in cash flow forecasts.
c. Sunk costs are not incremental flows and should not be included. CMA 1295 4-11
12
. A company is considering a new project. The company’s CFO plans to calculate the project’s NPV by discounting the relevant
cash flows (which include the initial up-front costs, the operating cash flows, and the terminal cash flows) at the company’s cost of
capital (WACC). Which of the following factors should the CFO include when estimating the relevant cash flows? (E)
a. Any sunk costs associated with the project.
b. Any interest expenses associated with the project.
c. Any opportunity costs associated with the project.
d. Statements b and c are correct. Brigham
13
. Which of the following statements is correct? (M)
a. An asset that is sold for less than book value at the end of a project’s life will generate a loss for the firm and will cause an
actual cash outflow attributable to the project.
b. Only incremental cash flows are relevant in project analysis and the proper incremental cash flows are the reported
accounting profits because they form the true basis for investor and managerial decisions.
c. It is unrealistic to expect that increases in net operating working capital that are required at the start of an expansion project
are simply recovered at the project’s completion. Thus, these cash flows are included only at the start of a project.
d. Equipment sold for more than its book value at the end of a project’s life will increase income and, despite increasing taxes,
will generate a greater cash flow than if the same asset is sold at book value. Brigham
14
. Which of the following statements is most correct? (E)
a. The rate of depreciation will often affect operating cash flows, even though depreciation is not a cash expense.
b. Corporations should fully account for sunk costs when making investment decisions.
c. Corporations should fully account for opportunity costs when making investment decisions.
d. Statements a and c are correct. Brigham

. Which of the following is not a cash flow that results from the decision to accept a project? (E)
15

a. Changes in net operating working capital. d. Opportunity costs.


b. Shipping and installation costs. e. Externalities.
c. Sunk costs. Brigham
16
. Adams Audio is considering whether to make an investment in a new type of technology. Which of the following factors should the
company consider when it decides whether to undertake the investment? (M)
a. The company has already spent $3 million researching the technology.
b. The new technology will affect the cash flows produced by its other operations.
c. If the investment is not made, then the company will be able to sell one of its laboratories for $2 million.
d. Statements b and c should be considered. Brigham
17
. Laurier Inc. is a household products firm that is considering developing a new detergent. In evaluating whether to go ahead with
the new detergent project, which of the following items should Laurier explicitly include in its cash flow analysis? (M)
a. The company will produce the detergent in a vacant facility that they renovated five years ago at a cost of $700,000.
b. The company will need to use some equipment that it could have leased to another company. This equipment lease could
have generated $200,000 per year in after-tax income.
c. The new detergent is likely to significantly reduce the sales of the other detergent products the company currently sells.
d. Statements b and c are correct. Brigham
18
. Sanford & Son Inc. is thinking about expanding their business by opening another shop on property they purchased 10 years ago.
Which of the following items should be included in the analysis of this endeavor? (M)
a. The property was cleared of trees and brush 5 years ago at a cost of $5,000.
b. The new shop is expected to affect the profitability of the existing shop since some current customers will transfer their
business to the new shop. Sanford and Son estimate that profits at the existing shop will decrease by 10 percent.
c. Sanford & Son can lease the entire property to another company (that wants to grow flowers on the lot) for $5,000 per year.
d. Both statements b and c should be included in the analysis. Brigham
19
. Pickles Corp. is a company that sells bottled iced tea. The company is thinking about expanding its operations into the bottled
lemonade business. Which of the following factors should the company incorporate into its capital budgeting decision as it decides
whether or not to enter the lemonade business? (M)
a. If the company enters the lemonade business, its iced tea sales are expected to fall 5 percent as some consumers switch from
iced tea to lemonade.
b. Two years ago the company spent $3 million to renovate a building for a proposed project that was never undertaken. If the
project is adopted, the plan is to have the lemonade produced in this building.
c. If the company doesn’t produce lemonade, it can lease the building to another company and receive after-tax cash flows of
$500,000 a year.
d. Statements a and c are correct. Brigham
20
. Which of the following statements is correct? (M)
a. In a capital budgeting analysis where part of the funds used to finance the project are raised as debt, failure to include interest
expense as a cost in the cash flow statement when determining the project’s cash flows will lead to an upward bias in the
NPV.
b. The preceding statement would be true if “upward” were replaced with “downward.”
c. The existence of “externalities” reduces the NPV to a level below the value that would exist in the absence of externalities.
d. If one of the assets that would be used by a potential project is already owned by the firm, and if that asset could be leased to
another firm if the project is not undertaken, then the net rent that could be obtained should be charged as a cost to the project
under consideration.
e. The rent referred to in statement d is a sunk cost, and as such it should be ignored. Brigham

. Which of the following constitutes an example of a cost that is not incremental, and therefore, not relevant in an accept/reject
21

decision? (M)
a. A firm has a parcel of land that can be used for a new plant site, or alternatively, can be used to grow watermelons.
b. A firm can produce a new cleaning product that will generate new sales, but some of the new sales will be from customers
who switch from another product the company currently produces.
c. A firm orders and receives a piece of new equipment that is shipped across the country and requires $25,000 in installation
and set-up costs.
d. Statements a, b, and c are examples of incremental cash flows, and therefore, relevant cash flows. Brigham

. Which of the following is not considered a relevant concern in deter- mining incremental cash flows for a new product? (M)
22

a. The use of factory floor space that is currently unused but available for production of any product.
b. Revenues from the existing product that would be lost as a result of some customers switching to the new product.
c. Shipping and installation costs associated with preparing the machine to be used to produce the new product.
d. The cost of a product analysis completed in the previous tax year and specific to the new product. Brigham

Uncertainty
1. Which of the following best identifies the reason for using probabilities in capital budgeting decisions?
A. uncertainty C. time value of money
B. cost of capital D. projects with unequal lives AICPA adapted

*. Which of the following best identifies the reason for using probabilities in capital budgeting is (E)
a. Different life of projects. c. Uncertainty. RPCPA 0577, 0588, 1093
b. Cost of capital. d. Time value of money.

26. In capital expenditure analysis, which of the following can be constructed to evaluate alternative levels of investment?
A. normal distribution D. pie chart
B. bar graph E. payoff table
C. nonnormal distribution Carter & Usry

28. The standard deviation of the expected net present value is determined by summing the discounted standard deviations for each
period over the life of the project when the cash flows in each of the periods are:
A. independent D. negative
B. positive E. perfectly correlated
C. mixed Carter & Usry

12. Cinzano Inc. wants to use discounted cash flow techniques when analyzing its capital investment projects. The company is aware
of the uncertainty involved in estimating future cash flows. A simple method some companies employ to adjust for the uncertainty
inherent in their estimates is to:
A. ignore salvage values
B. average the expectations of several different managers
C. use accelerated depreciation
D. adjust the minimum desired rate of return
E. increase the estimates of the cash flows CMA adapted

Tax Factor
In general
58. Income taxes are levied on
a. net cash flow.
b. income as measured by accounting rules.
c. net cash flow plus depreciation.
d. income as measured by tax rules. Barfield

18. Which of the following statements is true?


a. All revenue is taxed.
b. All expenses are tax-deductible.
c. Some revenues and expenses have no tax effects.
d. Income taxes are based solely on revenues and expenses. L&H

*. In capital budgeting decisions, the following items are considered among others: (M)
1. Cash outflow for the investment.
2. Increase in working capital requirements.
3. Profit on sale of old asset
4. Loss on write-off of old asset.
For which of the above items would taxes be relevant? (D)
a. Items 1 and 3 only. c. All items.
b. Items 3 and 4 only. d. Items 1, 3 and 4 only. RPCPA 0594

6. The government could encourage increases in investment by


a. Increasing tax rates.
b. Lengthening the MACRS period.
c. Letting a company expense fixed assets in the year acquired instead of through annual depreciation charges.
d. Taking actions that would increase interest rates. L&H

61. Which of the following are tax deductible under U.S. tax law? (M)
a. interest payments to bondholders c. common stock dividends
b. preferred stock dividends d. all of the above Barfield

No Income Tax Situation


21. If there were no income taxes,
a. Depreciation would be ignored in capital budgeting.
b. The NPV method would not work.
c. Income would be discounted instead of cash flow.
d. All potential investments would be desirable. L&H

35. If a company is NOT subject to income tax, which of the following is true of a proposed investment?
a. The project’s IRR equals the entity’s cost of capital.
b. The project’s NPV is zero.
c. Depreciation on assets required for the project is irrelevant to the evaluation.
d. The expected annual increase in future cash flows equals the investment required to undertake the project. L & H

53. The pre-tax and after-tax cash flows would be the same for all of the following items except (D)
a. the liquidation of working capital at the end of a project's life.
b. the initial (outlay) cost of an investment.
c. the sale of an asset at its book value.
d. a cash payment for salaries and wages. Barfield

Depreciation Tax Shield


*. The accounting area in which the only objective of depreciation accounting relates to the effect of depreciation charges upon tax
payments is (E)
a. Income determination. c. Cost/volume/profit analysis.
b. Financial reporting. d. Capital budgeting. RPCPA 0587

29. Which statement describes the relevance of depreciation in calculating cash flows?
a. Depreciation is relevant only when income taxes exist.
b. Depreciation is always relevant.
c. Depreciation is never relevant.
d. Depreciation is relevant only with discounted cash flow methods. L&H

56. Multiplying the depreciation deduction by the tax rate yields a measure of the depreciation tax (D)
a. shield. c. payable.
b. benefit. d. loss. Barfield

3. Depreciation is incorporated explicitly in the cash flow analysis of an investment proposal because it:
A. is a cost of operations that cannot be avoided
B. results in an annual cash outflow
C. is a cash inflow
D. reduces the cash outlay for income taxes
E. represents the initial cash outflow spread over the life of the investment CMA adapted
23
. A depreciation tax shield is
a. An after-tax cash outflow.
b. A reduction in income taxes.
c. The cash provided by recording depreciation.
d. The expense caused by depreciation. CMA 1293 4-14, RPCPA 0596
24
. The annual tax depreciation expense on an asset reduces income taxes by an amount equal to
a. The firm’s average tax rate times the depreciation amount.
b. One minus the firm’s average tax rate times the depreciation amount.
c. The firm’s marginal tax rate times the depreciation amount.
d. One minus the firm’s marginal tax rate times the depreciation amount. CMA 1293 4-20
Accelerated Method vs. Straight-line Method
25
. The use of an accelerated method instead of the straight-line method of depreciation in computing the net present value of a
project has the effect of
a. Raising the hurdle rate necessary to justify the project.
b. Lowering the net present value of the project.
c. Increasing the present value of the depreciation tax shield.
d. Increasing the cash outflows at the initial point of the project. CMA 0695 4-3

Changes in Tax Rates


*. Your company is purchasing a transport equipment as part of its territorial expansion strategy. The technical services department
indicated that this equipment needs overhauling in year 4 or year 5 of its useful life. The overhauling cost will be expected during
the year the overhauling is done. The finance officer insists that the overhauling be done in year 4, not in year 5. The most likely
reason is (M) RPCPA 0594
a. There is lower tax rate in year 5. c. The time value of money is considered.
b. There is higher tax rate in year 5 d. Due statements A and C above.

Optimal Capital Budget


26
. An optimal capital budget is determined by the point where the marginal cost of capital is (D)
A. Minimized.
B. Equal to the average cost of capital.
C. Equal to the rate of return on total assets.
D. Equal to the marginal rate of return on investment. CIA 1187 IV-43
27
. A firm seeking to optimize its capital budget has calculated its marginal cost of capital and projected rates of return on several
potential projects. The optimal capital budget is determined by
A. Calculating the point at which marginal cost of capital meets the projected rate of return, assuming that the most profitable
projects are accepted first.
B. Calculating the point at which average marginal cost meets average projected rate of return, assuming the largest projects are
accepted first.
C. Accepting all potential projects with projected rates of return exceeding the lowest marginal cost of capital.
D. Accepting all potential projects with projected rates of return lower than the highest marginal cost of capital. CIA 1191 IV-57
28
. Shanahan Inc. has two divisions: Division A makes up 50 percent of the company, while Division B makes up the other 50 percent.
Shanahan’s beta is 1.2. Looking at stand-alone competitors, Shanahan’s CFO estimates that Division A’s beta is 1.5, while
Division B’s beta is 0.9. The risk-free rate is 5 percent and the market risk premium is 5 percent. The company is 100 percent
equity-financed. (WACC = ks, the cost of equity).
Division B is considering the following projects given below. Each of the projects has the same risk and all have the same risk as a
“typical” Division B project.
Project Capital required IRR
1 $400 million 14.0%
2 300 million 10.7
3 250 million 10.5
4 320 million 10.0
5 230 million 9.0
The company is debating which cost of capital they should use to evaluate Division B’s projects. John Green argues that
Shanahan should use the same cost of capital for each of its divisions, and believes it should base the cost of equity on
Shanahan’s overall beta. Becky White argues that the cost of capital should vary for each division, and that Division B’s beta
should be used to estimate the cost of equity for Division B’s projects.
If the company uses White’s approach, how much larger will the capital budget be than if it uses Green’s approach? (E)
a. Capital budget is $320 million larger using White’s approach.
b. Capital budget is $550 million larger using White’s approach.
c. Capital budget is $870 million larger using White’s approach.
d. Capital budget is $1,200 million larger using White’s approach.
e. The capital budget is the same using the two approaches. Brigham

INVESTMENT OPTIONS
Abandonment option
29
. Which of the following statements best describes the likely impact that an abandonment option will have on a project’s expected
cash flow and risk? (E)
a. No impact on expected cash flow, but risk will increase.
b. Expected cash flow increases and risk decreases.
c. Expected cash flow increases and risk increases.
d. Expected cash flow decreases and risk decreases.
e. Expected cash flow decreases and risk increases. Brigham

Investment timing option


30
. Commodore Corporation is deciding whether it makes sense to invest in a project today, or to postpone this decision for one year.
Which of the following statements best describes the issues that Commodore faces when considering this investment timing
option? (E)
a. The investment timing option does not affect the expected cash flows and should therefore have no impact on the project’s
risk.
b. The more uncertainty about the project’s future cash flows the more likely it is that Commodore will go ahead with the project
today.
c. If the project has a positive expected NPV today, this means that its expected NPV will be even higher if it chooses to wait a
year.
d. All of the above statements are correct.
e. None of the above statements is correct. Brigham

Real options
31
. Which of the following is an example of a flexibility option? (E)
a. A company has the option to invest in a project today or to wait a year.
b. A company has the option to back out of a project that turns out to be unproductive.
c. A company pays a higher cost today in order to be able to reconfigure the project’s input or outputs at a later date.
d. A company invests in a project today that may lead to enhanced technological improvements that allow it to expand
into different markets at a later date.
e. All of the statements above are correct. Brigham

4. Which of the following are not real options? (M)


a. The option to expand production if the product is successful.
b. The option to buy additional shares of stock if the stock price goes up.
c. The option to expand into a new geographic region.
d. The option to abandon a project.
e. The option to switch sources of fuel used in an industrial furnace. Brigham

5. Which of the following will not increase the value of a real option? (M)
a. An increase in the time remaining until the real option must be exercised.
b. An increase in the volatility of the underlying source of risk.
c. An increase in the risk-free rate.
d. An increase in the cost of exercising the real option.
e. Statements b and d. Brigham

Abandonment and growth options


32
. Clueless Corporation never considers abandonment options or growth options when estimating its optimal capital budget. What
impact does this policy have on the company’s optimal capital budget? (M)
a. Its estimated capital budget is too small because it fails to consider abandonment and growth options.
b. Its estimated capital budget is too large because it fails to consider abandonment and growth options.
c. Failing to consider abandonment options makes the optimal capital budget too large, but failing to consider growth options
makes the optimal capital budget too small, so it is unclear what impact this policy has on the overall capital budget.
d. Failing to consider abandonment options makes the optimal capital budget too small, but failing to consider growth options
makes the optimal capital budget too large, so it is unclear what impact this policy has on the overall capital budget.
e. Neither abandonment nor growth options should have an effect on the company’s optimal capital budget. Brigham

CAPITAL BUDGET EVALUATION METHODS


In general
*. “Net present value” is an example of which concept? (E)
a. Capital budgeting. c. Managerial control.
b. Project feasibility. d. Management by exception. RPCPA 0580

*. “Net present value” is an example of which concept? (E)


a. Capital budgeting c. Management control
b. Project feasibility d. Management by objectives RPCPA 0577

*. All of the following are methods that aid management in analyzing the expected results of capital budgeting decisions, except (E)
Horngren, RPCPA 1095, 1096
a. Accrual accounting rate of return. c. Future value cash flow.
b. Payback period. d. Discounted cash flow rate of return.

1. The following measures are used by firms when making capital budgeting decisions except:
A. Payback period C. Net present value
B. Internal rate of return D. P/E ratio B&M

Use of Net Income


*. A number of techniques are commonly used in the analysis of capital budgeting decisions. Each method involves the
measurement of cash flows, except the (E)
a. Internal rate of return. c. Average rate of return method.
b. Payback period method. d. Net present value. RPCPA 1097

20. The technique that does NOT use cash flows is


a. Payback. c. IRR.
b. NPV. d. Book rate of return. L&H

9. Which of the following capital budgeting methods does NOT consider the time value of money?
a. IRR. c. Time-adjusted rate of return.
b. Book rate of return. d. NPV. L&H
33
. Which one of the following capital investment evaluation methods does not take the time value of money into consideration?
a. Net present value. c. Internal rate of return.
b. Discounted payback. d. Accounting rate of return. CMA 0696 4-26

Use of Cash Flows


In general
*. In capital budgeting, these techniques are applied: payback (PB) method, net present value (NPV) method and time-adjusted rate
of return (TARR) method. PB method has this in common with NPV and TARR methods. (M)
a. Use of cash flows.
b. Consideration of the time value of money.
c. Use of discounting.
d. Use of accrual method of accounting. RPCPA 1094

57. All of the following are major categories of cash flows in capital investment decisions EXCEPT (E)
a. the initial investment in machines and working capital.
b. recurring operating cash flows.
c. the initial working capital investment
d. depreciation expense reported on the income statement. Horngren

Without Time Value of Money


1. Which of the following capital budgeting techniques ignores the time value of money?
a. payback period c. internal rate of return
b. net present value d. profitability index Barfield

With Time Value of Method


28. Which of the following is a discounted cash flow method?
a. NPV. c. Book rate of return.
b. Payback. d. All of the above. L&H

8. The method of project selection that considers the time value of money in a capital budgeting decision computes the:
A. accounting rate of return on average investment
B. internal rate of return
C. payback period
D. return on investment
E. accounting rate of return on initial investment AICPA adapted

52. Which of the following methods consider the time value of money?
a. payback and accounting rate of return
b. payback and internal rate of return
c. internal rate of return and accounting rate of return
d. internal rate of return and net present value H&M

*. Which of the following capital investment rating procedures recognize(s) the time value of money? (E)
RPCPA 0590 a. b. c. d.
Profitability index Yes No Yes No
Discounted rate of return Yes Yes No No

1. Which of the following groups of capital budgeting techniques uses the time value of money? (E)
a. Book rate of return, payback, and profitability index.
b. IRR, payback, and NPV.
c. IRR, NPV, and profitability index.
d. IRR, book rate of return, and profitability index. L&H

30. Which of the following combinations of capital budgeting techniques includes only discounted cash flow techniques?
a. Book rate of return, payback, and profitability index.
b. NPV, IRR and profitability index.
c. IRR, payback, and NPV.
d. Profitability index, NPV, and payback. L&H

Use of Time Value of Money vs. No Time Value of Money


27. In contrast to the payback period and book rate of return methods, the NPV and IRR methods
a. Consider the time value of money. c. Use after-tax cash flows.
b. Ignore depreciation. d. All of the above. L&H

18. The net present value and internal rate of return methods of capital budgeting are superior to the payback method in that they: (M)
a. are easier to implement.
b. consider the time value of money.
c. require less input.
d. reflect the effects of depreciation and income taxes. AICPA adapted

ACCOUNTING RATE OF RETURN


Definition
*. A capital budgeting method that provides a rough approximation of an investment’s profitability as measured with net income from
the income statement is known as: (E)
a. Average rate of return method. c. Payback period. RPCPA 1097
b. Net present value method. d. Internal rate of return method.
34
. The technique that measures the estimated performance of a capital investment by dividing the project's annual after-tax net
income by the average investment cost is called the (E)
A. Bail-out payback method. C. Profitability index method. CMA 0692 4-21
B. Internal rate of return method. D. Accounting rate of return method.
35
. The technique that measures the estimated performance of a capital investment by dividing the project's annual after-tax net
income by the average investment cost is called the
A. Average rate of return method. C. Capital asset pricing model.CMA 1290 4-15
B. Internal rate of return method. D. Accounting rate of return method.

31. The capital budgeting method that divides a project's annual incremental net income by the initial investment is the: (M)
a. internal rate of return method.
b. the simple ( or accounting) rate of return method.
c. the payback method.
d. the net present value method. CMA adapted

Characteristics
36
. The accounting rate of return
A. Is synonymous with the internal rate of return.
B. Focuses on income as opposed to cash flows.
C. Is inconsistent with the divisional performance measure known as return on investment.
D. Recognizes the time value of money. CMA 0691 4-18

11. Which of the following methods uses income instead of cash flows?
a. payback c. internal rate of return
b. accounting rate of return d. net present value H&M
37
. The capital budgeting technique known as accounting rate of return uses
a. b. c. d.
Revenue over life of project No No Yes Yes
Depreciation expense Yes No No Yes
99. The capital budgeting technique known as accounting rate of return uses (D)
Barfield a. b. c. d.
Salvage value No No Yes Yes
Time value of money No Yes Yes No

Formula
38
. The method that divides a project’s annual after-tax net income by the average investment cost to measure the estimated
performance of a capital investment is the
a. Internal rate of return method. c. Payback method. CMA 1294 4-24
b. Accounting rate of return method. d. Net present value (NPV) method.

100.In computing the accounting rate of return, the ______ level of investment should be used as the denominator.
a. Average c. Residual
b. Initial d. Cumulative Barfield

16. The accounting rate of return on original investment is calculated as


a. original investment/net income c. net income/original investment
b. net income/debt d. assets/debt H&M

64. The approach to capital budgeting which divides an accounting measure of income by an accounting measure of investment is (E)
a. net present value. c. payback method. Horngren
b. internal rate of return. d. accrual accounting rate of return.

18. The book rate of return on a project is calculated as:


A. Book Cash Flow/book assets C. Book assets/book income
B. Book income/book assets D. None of the above B&M

Limitation
*. The following statements refer to the accounting rate of return (ARR)
1. The ARR is based on the accrual basis, not cash basis.
2. The ARR does not consider the time value of money.
3. The profitability of the project is considered.
From the above statements, which are considered limitations of the ARR concept? (M)
a. Statements 2 and 3 only. c. All the 3 statements.
b. Statements 3 and 1 only. d. Statements 1 and 2 only. RPCPA 1094

65. For capital budgeting decisions, the use of the accrual accounting rate of return for evaluating performance is often a stumbling
block to the implementation of the (E)
a. net cash flow.
b. most effective goal-congruence choice.
c. discounted cash flow method for capital budgeting.
d. most effective tax strategy. Horngren

19. The disadvantages of the book rate of return method is/are:


A. It uses net income instead of cash flows
B. The pattern of income has no impact on the book rate of return
C. There is no clear cut decision rule
D. All of the above B&M

PAYBACK PERIOD
Definition
10. The payback period is the
a. length of time over which the investment will provide cash inflows.
b. length of time over which the initial investment is recovered.
c. shortest length of time over which an investment may be depreciated.
d. shortest length of time over which the net present value will be positive. Barfield
39
. The length of time required to recover the initial cash outlay of a capital project is determined by using the CMA 1294 4-20
a. Discounted cash flow method. c. Weighted net present value method.
b. Payback method. d. Net present value method.

*. An investment rating approach which measures the length of time required to recover the initial outlay for a particular investment
proposal is the (E)
a. Accounting rate of return c. Net present value
b. Payback period d. Present value index RPCPA 0590
40
. The technique that measures the number of years required for the after-tax cash flows to recover the initial investment in a project
is called the
A. Net present value method. C. Profitability index method. CMA 1290 4-17
B. Payback method. D. Accounting rate of return method.
*. The method that measures how quickly investment pesos may be recovered is the (E)
a. Payback method c. Simple rate of return method
b. Time adjusted rate of return d. Least squares methodRPCPA 1074, 10/77

*. The payback method measures (E)


a. Profitability of an investment c. Time to recover investment RPCPA 1093
b. Economic life of the investment d. Cash flow from an investment

27. The payback method measures: (E)


a. how quickly investment dollars may be recovered.
b. the cash flow from an investment.
c. the economic life of an investment.
d. the profitability of an investment. CMA adapted

61. The method that measures the time it will take to recoup, in the form of future cash inflows, the total dollars invested in a project is
called (E)
a. the accrued accounting rate-of-return method.
b. payback method.
c. internal rate-of-return method.
d. the book-value method. Horngren

63. The payback method of capital budgeting approach to the investment decision highlights (E)
a. cash flow over the life of the investment.
b. the liquidity of the investment.
c. the tax savings of the depreciation amounts.
d. having as lengthy payback time as possible. Horngren

Characteristics
19. The technique most concerned with liquidity is (M)
a. Payback. c. IRR.
b. NPV. d. Book rate of return. L&H

2. The payback criterion for capital investment decisions


a. Is conceptually superior to the IRR criterion.
b. Takes into consideration the time value of money.
c. Gives priority to rapid recovery of cash.
d. Emphasizes the most profitable projects. L&H

8. The payback period rule:


A. Varies the cut-off point with the interest rate
B. Determines a cut-off point so that all projects accepted by the NPV rule will be accepted by the payback period rule.
C. Requires an arbitrary choice of a cut-off point
D. Both A and C B&M

2. Which of the following capital budgeting techniques may potentially ignore part of a project's relevant cash flows? (M)
a. net present value c. payback period
b. internal rate of return d. profitability index Barfield
41
. A characteristic of the payback method (before taxes) is that it (E)
a. Incorporates the time value of money.
b. Neglects total project profitability.
c. Uses accrual accounting inflows in the numerator of the calculation. CMA 0694 4-17
d. Uses the estimated expected life of the asset in the denominator of the calculation.

4. Which of the following capital budgeting techniques does not routinely rely on the assumption that all cash flows occur at the end
of the period?
a. internal rate of return c. profitability index
b. net present value d. payback period Barfield

7. The payback method assumes that all cash inflows are reinvested to yield a return equal to
a. the discount rate. c. the internal rate of return.
b. the hurdle rate. d. zero. Barfield

39. Assuming that a project has already been evaluated using the following techniques, the evaluation under which technique is least
likely to be affected by an increase in the estimated residual value of the project?
a. Payback period. c. NPV.
b. IRR. d. PI. L&H
30. The evaluation of an investment having uneven cash flows using the payback method: (M)
a. cannot be done.
b. can be done only by matching cash inflows and investment outflows on a year-by-year basis.
c. will product essentially the same results as those obtained through the use of discounted cash flow techniques.
d. requires the use of a sophisticated calculator or computer software. G & N 9e

10. Which of the following investment rules may not use all possible cash flows in its calculations?
A. Payback period. C. IRR
B. NPV D. All of the above B&M

Advantage
*. An advantage of the payback method is (E)
a. Not based on cash flow data. c. Precise in estimate of profits.
b. Insensitive of the life of the project. d. Easy to apply. RPCPA 1093

*. An advantage of using the payback method of evaluating capital budgeting alternatives is that payback is
a. Insensitive to the life of the project considered.
b. Precise estimate of profitability.
c. Based on cash flow data.
d. Easy to apply. RPCPA 0577, 5/80, 5/98

12. The advantage of the payback rule is:


A. Adjustment for uncertainty of early cash flows
B. It is simple to use
C. Does not discount cash flows
D. Both A and C B&M

Disadvantage
*. This technique is criticised because it fails to consider investment profitability (E)
a. Time adjusted ROI c. Average return on investment
b. Payback method d. Present value method RPCPA 1093

11. Which of the following capital budgeting techniques has been criticized because it fails to consider investment profitability? (E)
a. payback method c. net present value method
b. accounting rate of return d. internal rate of return Barfield
42
. Which one of the following statements about the payback method of investment analysis is correct? The payback method
a. Does not consider the time value of money.
b. Consider cash flows after the payback has been reached.
c. Uses discounted cash flow techniques. CMA 1295 4-1
d. Generally leads to the same decision as other methods for long-term projects.

34. Which of the following methods FAILS to distinguish between return of investment and return on investment.
a. NPV. c. Payback.
b. IRR. d. Book rate of return. L&H

15. Which of the following is NOT a defect of the payback method?


a. It ignores cash flow because it uses net income.
b. It ignores profitability.
c. It ignores the present values of cash flows.
d. It ignores the pattern of cash flows beyond the payback period. L&H

14. Deficiencies associated with using the payback method to evaluate investment alternatives include all of the following, except that:
A. the present value of cash inflows is ignored
B. inflows of different time periods are treated equally
C. it may be used to select those investments yielding a quick return of cash
D. cash flows after the payback period are ignored CIA adapted

1. A major disadvantage of the payback period method is that it (E)


a. Is useless as a risk indicator.
b. Ignores cash flows beyond the payback period.
c. Does not directly account for the time value of money.
d. Statements b and c are correct. Brigham

15. The following are disadvantages of using the payback rule except:
A. The payback rule ignores all cash flow after the cutoff date
B. The payback rule does not use the time value of money
C. The payback period is easy to calculate and use
D. The payback rule does not have the value additive property B&M

Formula
. The cash payback formula is:
a. Cost of Capital Investment / Net Income.
b. Average Investment / Net Annual Cash Inflow.
c. Cost of Capital Investment / Net Annual Cash Inflow.
d. Average Investment / Net Income. RPCPA 1001

5. Assume that a project consists of an initial cash outlay of $100,000 followed by equal annual cash inflows of $40,000 for 4 years.
In the formula X = $100,000/$40,000, X represents the (E)
a. payback period for the project. c. internal rate of return for the project.
b. profitability index of the project. d. project's discount rate. Barfield

Decision Criteria
9. The payback period rule accepts all projects for which the payback period is:
A. Greater than the cut-off value C. Is positive
B. Less than the cut-off value D. An integer B&M

Payback Reciprocal
43
. The payback reciprocal can be used to approximate a project’s
a. Profitability index
b. Net present value.
c. Accounting rate of return if the cash flow pattern is relatively stable.
d. Internal rate of return if the cash flow pattern is relatively stable. CMA 0693 4-27

Relevant Items
1. In order to calculate the payback period for a project, it is necessary to know the:
A. salvage value D. net present value
B. useful life E. annual cash flow
C. minimum desired rate of return Carter & Usry

1. Calculating the payback period for a capital project requires knowing which of the following?
a. Useful life of the project.
b. The company’s minimum required rate of return.
c. The project’s NPV.
d. The project’s annual cash flow. L&H

Irrelevant Items
*. As a capital budgeting technique, the payback period considers depreciation expenses (DE) and time value of money (TVM) as
follows: (M)
RPCPA 1095 a. b. c. d.
DE relevant irrelevant Irrelevant relevant
TVM relevant irrelevant Relevant irrelevant

44
. The capital budgeting technique known as payback period uses
a. b. c. d.
Depreciation expense Yes Yes No No
Time value of money Yes No No Yes

BAILOUT PAYBACK
Definition
*. The bailout payback period is (E)
a. The payback period used by firms with government insured loans.
b. The length of time for payback using cash flows plus the salvage value to recover the original investment
c. (a) and (b)
d. None of the above. RPCPA 1090
45
. The bailout payback method (E)
A. Is used by firms with federally insured loans.
B. Calculates the payback period using the sum of the net cash flows and the salvage value.
C. Calculates the payback period using the difference between net cash inflow and the salvage value.
D. Estimates short-term profitability. Gleim

Use
46
. The bailout payback method
a. Incorporates the time value of money.
b. Equals the recovery period from normal operations.
c. Eliminates the disposal value from the payback calculation.
d. Measures the risk if a project is terminated. CMA 1292 4-11
DISCOUNTED CASH FLOW ANALYSIS
37. The consumption opportunity increases when an investment with positive NPV is available because:
A. The investment is better than what is available in the market
B. The project rate of return is less than market rate
C. The market is irrelevant to the investment criteria
D. All of the above are reasons for increase in consumption B&M

Factors
14. When using one of the discounted cash flow methods to evaluate the desirability of a capital budgeting project, which of the
following factors is generally not important? (E)
a. method of financing the project under consideration
b. timing of cash flows relating to the project
c. impact of the project on income taxes to be paid
d. amounts of cash flows relating to the project Barfield

Assumptions
52. In a typical (conservative assumptions) after-tax discounted cash flow analysis, depreciation expense is assumed to accrue at
a. the beginning of the period.
b. the middle of the period.
c. the end of the period.
d. irregular intervals over the life of the investment. Barfield

Time Value Of Money


53. There are two reasons for discounting future cash flow. They are:
A. A dollar today is worth more than a dollar tomorrow
B. A safe dollar is worth more than a risky one
C. The value of a dollar is changing all the time
D. A and B above B&M

29. The line that connects the maximum that one can consume this year (now) and the maximum one can consume next year:
A. Has a slope of (1+r) C. Has a slope of r
B. Has a slope of -(1+r) D. Has a slope of 1/r B&M

*. The method of project selection which considers the time value of money in a capital budgeting decision is accomplished by
computing the (E)
a. Accounting rate of return on initial investment
b. Payback period.
c. Accounting rate of return on average investment.
d. Discounted cash flow. RPCPA 0598

2. The component of the capital investment decision that would most likely concern an accountant is the:
A. social responsibility factors D. imponderables
B. competition E. legal restrictions
C. time value of money Carter & Usry

*. The fact that an amount of money that is to be received in the future is not equivalent to the same amount of money to be received
now is referred to as: (E)
a. Present value of money. c. Future value of money.
b. Time value of money. d. Discounted value of money. RPCPA 0587
47
. What is the time value of money?
A. Interest. C. Future value.
B. Present value. D. Annuity. Gleim

12. The time value of money is explicitly recognized through the process of
a. interpolating. c. annuitizing.
b. discounting. d. budgeting. Barfield

13. The time value of money is considered in long-range investment decisions by (M)
a. assuming equal annual cash flow patterns.
b. investing only in short-term projects.
c. assigning greater value to more immediate cash flows.
d. ignoring depreciation and tax implications of the investment. Barfield
48
. Basic time value of money concepts concern
Gleim A. B. C. D.
Interest Factors Yes Yes No No
Risk Yes No Yes No
Cost of Capital No Yes No Yes

49
. The present value may be calculated for discounted cash
Gleim A. B. C. D.
Inflows Yes Yes No No
Outflows Yes No Yes No
Annuities Yes Yes No Yes

9. The net present value and the internal rate of return methods of decision making in capital budgeting are superior to the payback
method in that they:
A. consider the time value of money
B. are easier to implement
C. consider accrual-based accounting income
D. require less input
E. reflect the effects of depreciation and income taxes AICPA adapted

Application
11. A company is considering the purchase of a new conveyor belt system for carrying parts and subassemblies from building to
building within its plant complex. It is expected that the system will have a useful life of at least ten years and that it will
substantially reduce labor and waiting-time costs. If the company's average cost of capital is about 15% and if some evaluation
must be made of cost/benefit relationships (including the effects of interest) to determine the desirability of the purchase, the most
relevant quantitative technique for evaluating the investment is:
A. present value (or internal rate of return) analysis
B. Program Evaluation and Review Technique (PERT)
C. accounting rate of return analysis
D. cost-volume-profit analysis
E. payback analysis AICPA adapted

2. Discounted cash flow techniques for analyzing capital budgeting decisions are NOT normally applied to projects
a. Requiring no investment after the first year of life.
b. Having useful lives shorter than one year.
c. That are essential to the business.
d. Involving replacement of existing assets. L&H

Definition of Discounted Cash Flow


*. Which of the following methods measures the cash flows and outflows of a project as if they occurred at a single point in time? (M)
a. Cash flow based payback period. c. Payback method.
b. Capital budgeting. d. Discounted cash flow. RPCPA 0595

*. The method of project selection which considers the value of money in a capital budgeting decision is accomplished by computing
the
a. Payback period.
b. Accounting rate of return on initial investment.
c. Accounting rate of return on average investment.
d. Discounted cash flow. RPCPA 0577, 10/79, 10/93

*. There are several evaluation techniques for determining the acceptability of an investment. The method that considers the time
value of money in an investment budgeting decision is accomplished by determining the (E)
a. Cash-flow payback method. c. Average rate of return.
b. Accounting rate of return. d. Discounted cash flow. RPCPA 1081

*. The method of project selection which considers the time value of money in a capital budgeting decision is accomplished by
computing the (E)
a. Accounting rate of return on initial investment.
b. Accounting rate of return on average investment.
c. Discounted cash flow.
d. Payback period. RPCPA 0580

*. The modern approach used by decision-makers in evaluating investment opportunities by comparing the original investment with
the cash generation for the entire life of the project to come up with the rate of return on investment is (E)
a. Discounted cash flow method d. Profitability index
b. Accounting rate of return approach e. Cash payoff method
c. Cash flow projections RPCPA 1077
50
. All of the following items are included in discounted cash flow analysis except
a. Future operating cash savings.
b. The disposal prices of the current assets
c. The future asset depreciation expense.
d. The tax effects of future asset depreciation. CMA 0694 4-18
51
. The capital budgeting model that is ordinarily considered the best model for long-range decision making is the CMA 1294 4-25
a. Payback model. c. Unadjusted rate of return model.
b. Accounting rate of return model. d. Discounted cash flow model.

*. When using one of the discounted-cash-flow methods to evaluate the feasibility of a capital budgeting project, which of the
following factors generally is not important? (M)
a. The method of financing the project under consideration.
b. The impact of the project on income taxes to be paid.
c. The timing of cash flows relating to the project.
d. The amount of cash flows relating to the project. RPCPA 0598
52
. Discounted cash flow concepts concern
Gleim a. b. c. d.
Interest Factors Yes Yes No No
Risk Yes No Yes No

53
. Depreciation is incorporated explicitly in the discounted cash flow analysis of an investment proposal because it
A. Is a cost of operations that cannot be avoided.
B. Is a cash inflow.
C. Reduces the cash outlay for income taxes.
D. Represents the initial cash outflow spread over the life of the investment. CMA 1277 5-14

31. Discounted cash flow methods for capital budgeting focus on (E)
a. cash inflows. c. cash outflows.
b. operating income. d. both (a) and (c). Horngren

Rate of Return
Discount Rate
18. The discount rate for safe projects is the:
A. Market rate of return C. Market risk premium
B. Risk-free rate D. None of the above B&M

19. The discount rate for a project with a risk the same as the market risk is the:
A. Market rate of return C. Market risk premium
B. Risk-free rate D. None of the above B&M

Cost of Capital
27. Hurdle rate for capital budgeting decisions is:
A. The cost of capital C. The cost of equity
B. The cost of debt D. All of the above B&M

*. In an investment in plant the return that should keep the market price of the firm stock unchanged is (M)RPCPA 0577, 1077, 0588,
1093
a. Payback c. Net present value
b. Discounted rate of return d. Cost of capital

25. For a project such as plant investment, the return that should leave the market price of the firm's stock unchanged is known as the
a. cost of capital. c. payback rate.
b. net present value. d. internal rate of return. Barfield

16. A company with cost of capital of 15% plans to finance an investment with debt that bears 10% interest. The rate it should use to
discount the cash flows is
a. 10%. c. 25%.
b. 15%. d. Some other rate. L&H

1. The term cost of capital for a project depends on:


A. The use to which the capital is put, i.e. the project
B. The company's cost of capital
C. The industry cost of capital
D. All of the above B&M

Weighted-Average Cost of Capital


54
. A company had made the decision to finance next year’s capital projects through debt rather than additional equity. The
benchmark cost of capital for these projects should be (M)
a. The before-tax cost of new-debt financing.
b. The after-tax cost of new-debt financing.
c. The cost of equity financing.
d. The weighted-average cost of capital. CIA 0597 IV-42

Opportunity Cost of Capital


32. The ___________________ is the highest rate of return that can be earned from the most attractive, alternative capital project
available to the firm. (D)
a. accounting rate of return c. hurdle rate
b. internal rate of return d. opportunity cost of capital Barfield

21. The opportunity cost of capital for a risky project is


A. The expected rate of return on a government security having the same maturity as the project
B. The expected rate of return on a well diversified portfolio of common stocks
C. The expected rate of return on a portfolio of securities of similar risks as the project
D. None of the above B&M

Comprehensive
. All of the following refer to the discount rate used by a firm in capital budgeting except (M)
a. Hurdle rate. c. Opportunity cost.
b. Required rate of return. d. Opportunity cost of capital. RPCPA 1096

Present Value Formula


94. If r is the discount rate, the formula [1/(1 + r)] refers to the
a. future value interest factor associated with r for one period.
b. present value of some future cash flow.
c. present value interest factor associated with r for one period.
d. future value interest factor for an annuity with a duration of r periods. Barfield

97. Which of the following indicates that the first cash flow is at the end of a period? (M)
Barfield a. b. c. d.
Ordinary annuity Yes Yes No No
Annuity due No Yes Yes No

Future Value Formula


95. Future value is the
a. sum of dollars-in discounted to time zero.
b. sum of dollars-out discounted to time zero.
c. difference of dollars-in and dollars-out.
d. value of dollars-in minus dollars-out for future periods adjusted for any interest-compounding factor. Barfield

98. Assume that X represents a sum of money that Bill has available to invest in a project that will yield a return of r. In the formula Y =
X(1 + r), Y represents the
a. future value of X in one period.
b. future value interest factor associated with r.
c. present value of X.
d. present value interest factor associated with r. Barfield

Discounted Cash Flow Methods


*. A mechanized system of handling parts from one assembly line to another is being contemplated by the Moonbeam Co. The
technical evaluation indicated that the system will reduce labor and waiting time costs substantially. An assessment has to be
made of cost/benefit relationship including the effects of interest. The most relevant quantitative technique to evaluate the project
is (M)
a. Regression analysis. c. Time adjusted rate of return analysis.
b. PERT-CPM. d. Payback period analysis. RPCPA 0594
55
. The discount rate (hurdle rate of return) must be determined in advance for the (E)
a. Payback period method. c. Net present value method.
b. Time adjusted rate of return method. d. Internal rate of return method.
56
. Amster Corporation has not yet decided on its hurdle rate for use in the evaluation of capital budgeting projects. This lack of
information will prohibit Amster from calculating a project’s (M)
CMA 0693 4-20 a. b. c. d.
Accounting Rate of Return No Yes No No
Net Present Value No Yes Yes Yes
Internal Rate of Return No Yes Yes No
30. Which capital budgeting technique(s) measure all expected future cash inflows and outflows as if they occurred at a single point in
time? (E)
a. Net present value c. Payback
b. Internal rate of return d. Both (a) and (b). Horngren

2. Which of the following investment rules does not use the time value of the money concept?
A. The payback period
B. Internal rate of return
C. Net present value
D. All of the above use the time value concept B&M

BREAKEVEN TIME
14. Which of the following statements regarding the discounted payback period rule is true?
A. The discounted payback rule uses the time value of money concept.
B. The discounted payback rule is better than the NPV rule
C. The discounted payback rule considers all cash flows
D. The discounted payback rule exhibits the value additive property B&M
57
. When evaluating projects, breakeven time is best described as (M)
a. Annual fixed costs  monthly contribution margin.
b. Project investment  annual net cash inflows.
c. The point at which cumulative cash inflows on a project equal total cash outflows.
d. The point at which discounted cumulative cash inflows on a project equal discounted total cash outflows. CMA 0693 4-28

PROFITABILITY INDEX
Definition
3. The profitability index
a. Does not use present values of cash flows.
b. Is generally preferable to any other approach for evaluating mutually exclusive investment alternatives.
c. Produces the same ranking of investment alternatives as does the IRR criterion.
d. Is a discounted cash flow method. L&H

*. Which of the following capital budgeting techniques is computed by dividing present value of future inflows by the initial
investment? (M)
a. Accounting rate of return c. Payback reciprocal
b. Time-adjusted rate of return d. Profitability index RPCPA 0589

41. The profitability index is (E)


a. the ratio of net cash flows to the original investment.
b. the ratio of the present value of cash flows to the original investment.
c. a capital budgeting evaluation technique that doesn't use discounted values.
d. a mandatory technique when capital rationing is used. Barfield
58
. The technique that reflects the time value of money and is calculated by dividing the present value of the future net after-tax cash
inflows that have been discounted at the desired cost of capital by the initial cash outlay for the investment is the
a. Capital rationing method. c. Profitability index method.
b. Average rate of return method. d. Accounting rate of return. CMA 1290 4-14
59
. The profitability index (present value index)
a. Represents the ratio of the discounted net cash outflows to cash inflows.
b. Is the relationship between the net discounted cash inflows less the discounted cash outflows.
c. Is calculated by dividing the discounted profits by the cash outflows.
d. Is the ratio of the discounted net cash inflows to discounted cash outflows. CMA 0695 4-4

19. The profitability index is the ratio of


a. Total cash inflows to the cost of the investment.
b. The present value of cash inflows to the cost of the investment.
c. The NPV of the investment to the cost of the investment.
d. The IRR to the company’s cost of capital. L&H

Assumption
42. Which method of evaluating capital projects assumes that cash inflows can be reinvested at the discount rate?
a. internal rate of return c. profitability index
b. payback period d. accounting rate of return Barfield

Formula
33. Profitability index is the ratio of:
A. Present value of cash flow to initial investment
B. Net present value cash flow to initial investment
C. Net present value of cash flow to IRR
D. Present value of cash flow to IRR B&M

Benefit Cost Ratio


34. Benefit cost ratio is defined as:
A. Present value of cash flow to initial investment
B. Net present value cash flow to initial investment
C. Net present value of cash flow to IRR
D. Present value of cash flow to IRR B&M

NET PRESENT VALUE METHOD


Definition
60
. The method that recognizes the time value of money by discounting the after-tax cash flows over the life of a project, using the
company's minimum desired rate of return is the
A. Accounting rate of return method. C. Internal rate of return method.
B. Net present value method. D. Payback method. CMA 1294 4-23
61
. The technique that recognizes the time value of money by discounting the after-tax cash flows for a project over its life to time
period zero using the company’s minimum desired rate of return is the CMA 1290 4-13
a. Net present value method. c. Average rate of return method.
b. Payback method. d. Accounting rate of return method.

22. Probabilistic estimates are most frequently used with which of the following methods of capital expenditure evaluation?
A. payback C. internal rate of return
B. present value D. accounting rate of return Carter & Usry

3. The net present value of a proposed project represents the:


A. cash flows less the original investment
B. present value of the cash flows plus the present value of the original investment less the original investment
C. present value of the cash flows less the original investment
D. present value of the cash flows less the cost of the old machine being replaced
E. cash flows less the present value of the cash flows Carter & Usry

34. The capital budgeting method which calculates the expected monetary gain or loss from a project by discounting all expected
future cash inflows and outflows to the present point in time using the required rate of return is the (E)
a. payback method.
b. accrual accounting rate-of-return method.
c. sensitivity method.
d. net present value method. Horngren

70. A capital budgeting tool management can use to summarize the difference in the future net cash inflows from an intangible asset at
two different points in time is referred to as (E)
a. the accrual accounting rate-of-return method.
b. the net present value method.
c. sensitivity analysis
d. the payback method. Horngren

Underlying Theory
*. The excess present value method is anchored on the theory that the future returns, expressed in terms of present value, must at
least be (E)
a. Equal to the amount of investment c. More than the amount of investment
b. Less than the amount of investment RPCPA 1074
62
. If a firm identifies (or creates) an investment opportunity with a present value <List A> its cost, the value of the firm and the price of
its common stock will <List B> (M)
CIA 1195 IV-44 a. b. c. d.
List A Greater than Greater than Equal to Equal to
List B Increase Decrease Increase Decrease

Characteristics
35. The net present value method of evaluating proposed investments
a. measures a project's internal rate of return.
b. ignores cash flows beyond the payback period.
c. applies only to mutually exclusive investment proposals.
d. discounts cash flows at a minimum desired rate of return. Barfield
63
. In evaluating a capital budget project, the use of the net present value (NPV) model is ordinarily not affected by the (E)
a. Method of funding the project.
b. Initial cost of the project.
c. Amount of added working capital needed for operations during the term of the project.
d. Project’s salvage value. CMA 1294 4-21
e. Amount of the project’s associated depreciation tax allowance.
64
. The capital budgeting technique known as net present value uses
AICPA 1180 T-48 a. b. c. d.
Cash flow over life of project No No Yes Yes
Time value of money Yes No No Yes

42. The net present value method focuses on (E)


a. cash inflows. c. cash outflows.
b. accrual-accounting net income. d. both (a) and (c). Horngren

41. The net present value rule is valid for:


A. Two period certain cash flows
B. Two period uncertain cash flows
C. Uncertain cash flows that extend far into the future
D. All of the above B&M

16. Which of the following capital budgeting methods has the value additive property?
A. NPV C. Payback period
B. IRR D. Discounted payback period B&M

Assumption
33. If an analyst desires a conservative net present value estimate, he/she will assume that all cash inflows occur at (M)
a. mid year. c. year end.
b. the beginning of the year. d. irregular intervals. Barfield

*. The common assumption in capital budgeting analysis is that cash inflows occur in lump sums at the end of individual years during
the life of an investment project when in fact they flow more or less continuously during those years (M)
a. Results in understated estimates of NPV.
b. Is done because present value tables for continuous flows cannot be constructed.
c. Will result in inconsistent errors being made on estimating NPVs such that project cannot be evaluated reliably.
d. Results in higher estimate for the IRR on the investment. RPCPA 1095
65
. The accountant of Ronier, Inc. has prepared an analysis of a proposed capital project using discounted cash flow techniques. One
manager has questioned the accuracy of the results because the discount factors employed in the analysis have assumed the
cash flows occurred at the end of the year when the cash flows actually occurred uniformly throughout each year. The net present
value calculated by the accountant will
A. Not be in error.
B. Be slightly overstated.
C. Be unusable for actual decision making.
D. Be slightly understated but usable. CMA 1278 5-10

21. The net present value method assumes that all cash inflows can be immediately reinvested at the
a. cost of capital. c. internal rate of return. Barfield
b. discount rate. d. rate on the corporation's short-term debt.

21. The net present value method of capital budgeting assumes that cash flows are reinvested at: (E)
a. the internal rate of return on the project.
b. the rate of return on the company's debt.
c. the discount rate used in the analysis.
d. a zero rate of return. CMA adapted
66
. The net present value (NPV) method of investment project analysis assumes that the project’s cash flows are reinvested at the
CMA 0692 4-16, RPCPA 0596
a. Computed internal rate of return. c. Discount rate used in the NPV calculation.
b. Risk-free interest rate. d. Firm’s accounting rate of return.
67
. The net present value method of capital budgeting assumes that cash flows are reinvested at
a. The risk-free rate. c. The internal rate of return of the project.
b. The cost of debt. d. The discount rate used in the analysis.
CMA 1295 4-9

36. Which of the following statements is true regarding capital budgeting methods? (D)
a. The Fisher rate can never exceed a company's cost of capital.
b. The internal rate of return measure used for capital project evaluation has more conservative assumptions than the net
present value method, especially for projects that generate a positive net present value.
c. The net present value method of project evaluation will always provide the same ranking of projects as the profitability index
method.
d. The net present value method assumes that all cash inflows can be reinvested at the project's cost of capital. Barfield

10. The advantage of the Net Present Value method over the Internal Rate of Return method for screening investment projects is that
it:
a. does not consider the time value of money
b. implicitly assumes that the company is able to reinvest cash flows from the project at the company’s discount rate
c. implicitly assumes that the company is able to reinvest cash flows from the project at the internal rate of return
d. fails to consider the timing of cash flows Pol Bobadilla

16. The capital budgeting method that assumes that funds are reinvested at the company's cost of capital is:
A. accounting rate of return D. return on investment
B. net present value E. payback
C. internal rate of return AICPA adapted

36. The net present value rule assumes that:


A. Borrowing and lending rate are equal C. Both A and B are true
B. Financial markets are well functioning D. None of the above are true B&M

Advantage
68
. An advantage of the net present value method over the internal rate of return model in discounted cash flow analysis is that the net
present value method
a. Computes a desired rate of return for capital projects. CMA 0695 4-1
b. Can be used when there is no constant rate of return required for each year of the project.
c. Uses a discount rate that equates the discounted cash inflows with the outflows.
d. Uses discounted cash flows whereas the internal rate of return model does not.

28. The net present value capital budgeting technique can be used when cash flows from period to period are:
AICPA adapted A. B. C. D.
Uniform No No Yes Yes
Uneven Yes No No Yes

53. In situations where the required rate of return is not constant for each year of the project, it is advantageous to use (E)
a. the adjusted rate-of-return method. c. the net present value method.
b. the internal rate-of-return method. d. sensitivity analysis. Horngren

Disadvantage
69
. A disadvantage of the net present value method of capital expenditure evaluations is that it (M)
a. Is calculated using sensitivity analysis.
b. Computes the true interest rate.
c. Does not provide the true rate of return on investment.
d. Is difficult to apply because it uses a trial-and-error approach.CMA 1295 4-16 RPCPA 0597

Application
35. NPV is appropriate to use to analyze which decision relating to a joint-products company?
a. Whether or not to sell facilities now used for additional processing of one of the joint products.
b. Whether or not to acquire facilities needed for additional processing of one of the joint products.
c. Whether or not to sell facilities now used to operate the joint process.
d. All of the above. L&H

Variables
Required Rate of Return
32. Net present value is calculated using (E)
a. the internal rate of return.
b. the required rate of return.
c. the rate of return required by the investment bankers.
d. none of the above. Horngren

13. The rate of return is also called:


A. Discount rate C. Opportunity cost of capital
B. Hurdle rate D. All of the above. B&M

39. Which of the following is NOT an appropriate term for the required rate of return? (E)
a. Discount rate c. Cost of capital Horngren
b. Hurdle rate d. All of the above are appropriate terms
18. The interest rate used to find the present value of a future cash flow is the
a. prime rate. c. cutoff rate.
b. discount rate. d. internal rate of return. Barfield

54. By using the required rate of return of an equivalent security traded in the financial markets as a discount rate in the NPV
calculations, we are:
A. Discounting for time C. A and B above
B. Discounting for risk D. None of the above B&M

52. The discount rate is used for calculating the NPV is:
A. Determined by the financial market
B. Found by the government
C. Found by the CEO
D. None of the above B&M
70
. The discount rate ordinarily used in present value calculation is the
a. Federal Reserve rate.
b. Treasury bill rate.
c. Minimum desired rate of return set by the firm.
d. Prime rate. Gleim

19. A firm's discount rate is typically based on (M)


a. the interest rates related to the firm's bonds.
b. a project's internal rate of return.
c. its cost of capital.
d. the corporate Aa bond yield. Barfield

20. In capital budgeting, a firm's cost of capital is frequently used as the (M)
a. internal rate of return. c. discount rate.
b. accounting rate of return. d. profitability index. Barfield
71
. When using the net present value method for capital budgeting analysis, the required rate of return is called all of the following
except the
A. Risk-free rate. C. Discount rate.
B. Cost of capital. D. Cutoff rate. CMA 1292 4-16
72
. All of the following are the rates used in net present value analysis except for the
a. Cost of capital. c. Discount rate.
b. Hurdle rate. d. Accounting rate of return. CMA 0694 4-15

Net Investment
73
. A project’s net present value, ignoring income tax considerations, is normally affected by the
a. Proceeds from the sale of the asset to be replaced.
b. Carrying amount of the asset to be replaced by the project.
c. Amount of annual depreciation on the asset to be replaced. AICPA 0593 T-47
d. Amount of annual depreciation on fixed assets used directly on the project.

Working Capital
22. Some investment projects require that a company expand its working capital to service the greater volume of business that will be
generated. Under the net present value method, the investment of working capital should be treated as: (M)
a. an initial cash outflow for which no discounting is necessary.
b. a future cash inflow for which discounting is necessary.
c. both an initial cash outflow for which no discounting is necessary and a future cash inflow for which discounting is necessary.
d. irrelevant to the net present value analysis. G & N 9e

Salvage Value
31. A proposed project has an expected economic life of eight years. In the calculation of the net present value (NPV) of the project,
salvage value would be:
A. excluded from the calculation of the NPV
B. included as a cash inflow at the estimated salvage value AICPA adapted
C. included as a cash inflow at the future amount of the estimated salvage value
D. included as a cash inflow at the present value of the estimated salvage value

Comprehensive
19. How are the following used in the calculation of the net present value of a proposed project? Ignore income tax considerations. (M)
AICPA adapted A. B. C. D.
Depreciation expense Include Include Exclude Exclude
Salvage value Include Exclude Include Exclude

20. The net present value method takes into account: (M)
AICPA adapted A. B. C. D.
Cash Flow Over Life of Project No No Yes Yes
Time Value of Money Yes No No Yes

Formula
50. The present value formula for one period cash flow is:
A. PV = C1(1 + r) C. PV = C1/(1 + r)
B. PV = C1/r D. None of the above B&M

51. The net present value formula for one period is:
A. NPV = PV cash flows - initial investment C. NPV = C0/[C1(1 + r)]
B. NPV = C0/C1 D. None of the above B&M

Decision Criteria
43. The managers of a firm can maximize stockholder wealth by:
A. Taking all projects with positive NPVs
B. Taking all projects with NPVs greater than the cost of investment
C. Taking all projects with NPVs greater than present value of cash flow
D. All of the above B&M

43. If the net present value for a project is zero or positive, this means (E)
a. the project should be accepted.
b. the project should not be accepted.
c. the expected rate of return is below the required rate of return.
d. both (a) and (c). Horngren

32. According to the net present value rule, an investment in a project should be made if the:
A. Net present value is greater than the cost of investment
B. Net present value is greater than the present value of cash flows
C. Net present value is positive
D. Net present value is negative B&M

35. Which of the following statements regarding the net present value rule and the rate of return rule is true?
A. Accept a project if the rate of return is positive
B. Accept a project the rate of return on a risky project exceeds the risk-free rate
C. Accept a project if the net present value is positive
D. None of the above statements are true B&M

38. In using the net present value method, only projects with a zero or positive net present value are acceptable because (E)
a. the return from these projects equals or exceeds the cost of capital.
b. a positive net present value on a particular project guarantees company profitability.
c. the company will be able to pay the necessary payments on any loans secured to finance the project.
d. of both (a) and (b). Horngren

33. Which of the following statements regarding the net present value rule and the rate of return rule is not true?
A. Accept a project if NPV > cost of investment
B. Accept a project if NPV is positive
C. Accept a project if return on investment exceeds the rate of return on an equivalent investment in the financial market
D. All of the above statements are true B&M

17. The following statements regarding the NPV rule and the rate of return rule are true except:
A. Accept a project if its NPV > 0
B. Reject a project if its NPV < 0
C. Accept a project if its rate of return > 0
D. Accept a project if its rate of return > opportunity cost of capital B&M

34. According to the rate of return rule an investment in a risky project should be made if:
A. The return on investment exceeds the risk-free rate
B. The return on investment is positive
C. The return on investments exceeds the rate of return on an equivalent investment in the financial market
D. None of the above statements are true B&M

NPV in an Inflationary Environment


13. Which of the following statements is true?
A. Nominal cash flows are discounted using nominal discount rate
B. Nominal cash flows are discounted using the real discount rate
C. Real cash flows are discounted using the nominal discount rate
D. None of the above statements are true B&M
74
. When determining net present value in an inflationary environment, adjustments should be made to
a. Increase the discount rate, only.
b. Increase the estimated cash inflows and increase the discount rate.
c. Increase the estimated cash inflows but not the discount rate.
d. Decrease the estimated cash inflows and increase the discount rate. CMA 1293 4-21

18. Proper treatment of inflation in the NPV calculation involves:


A. Discounting nominal cash flows using the nominal discount rate
B. Discounting real cash flows using the real discount rate
C. Discounting nominal cash flows using the real discount rates
D. A and B B&M

20. The NPV value obtained by discounting nominal cash flows using the nominal discount rate is:
A. The same as the NPV value obtained by discounting real cash flows using the real discount rate
B. The same as the NPV value obtained by discounting real cash flows using the nominal discount rate
C. The same as the NPV value obtained by discounting nominal cash flows using the real discount rate
D. None of the above B&M

INTERNAL RATE OF RETURN


Definition
*. The discount rate that equates the present value of the expected cash flows with the cost of the investment is the (E)
a. Net present value c. Accounting rate of return
b. Internal rate of return d. Payback period RPCPA 0593
75
. The technique that incorporates the time value of money by determining the compound interest rate of an investment such that the
present value of the after-tax cash inflows over the life of the investment is equal to the initial investment is called the
A. Internal rate of return method. C. Profitability index method. CMA 1290 4-16
B. Capital asset pricing model. D. Accounting rate of return method.
76
. The internal rate of return (IRR) is the
a. Hurdle rate.
b. Rate of interest for which the net present value is greater than 1.0.
c. Rate of interest for which the net present value is equal to zero.
d. Rate of return generated from the operational cash flows. CMA 0694 4-16, RPCPA 1096

24. The IRR is defined as: (E)


A. The discount rate that makes the NPV equal to zero
B. The difference between the cost of capital and the present value of the cash flows
C. The discount rate used in the NPV method
D. The discount rate used in the discounted payback period method B&M

46. The rate of interest that produces a zero net present value when a project's discounted cash operating advantage is netted against
its discounted net investment is the
a. cost of capital. c. cutoff rate.
b. discount rate. d. internal rate of return. Barfield
77
. The internal rate of return for a project can be determined
A. If the internal rate of return is greater than the firm's cost of capital.
B. Only if the project cash flows are constant. CMA 1293 4-12
C. By finding the discount rate that yields a net present value of zero for the project.
D. By subtracting the firm's cost of capital from the project's profitability index.
78
. The internal rate of return is
a. The discount rate at which the NPV of the cash flows is zero.
b. The breakeven borrowing rate for the project in question.
c. The yield rate/effective rate of interest quoted on long-term debt and other instruments.
d. All of the answers are correct. AICPA 1181 I-39
79
. The internal rate of return is
A. The breakeven borrowing rate for the project in question.
B. The yield rate/effective rate of interest quoted on long-term debt and other instruments.
C. Favorable when it exceeds the hurdle rate.
D. All of the answers are correct. Gleim
17. When a project has uneven projected cash inflows over its life, an analyst may be forced to use _______________ to find the
project's internal rate of return.
a. a screening decision c. a post investment audit
b. a trial-and-error approach d. a time line Barfield

47. The capital budgeting method that calculates the discount rate at which the present value of expected cash inflows from a project
equals the present value of expected cash outflows is the (E)
a. net present value method.
b. accrual accounting rate-of-return method.
c. payback method.
d. internal rate of return. Horngren

21 The internal rate of return of a capital investment(M)


a. Changes when the cost of capital changes.
b. Is equal to the annual net cash flows divided by one half of the project’s cost when the cash flows are an annuity.
c. Must exceed the cost of capital in order for the firm to accept the investment.
d. Is similar to the yield to maturity on a bond.
e. Statements c and d are correct. Brigham

Variables
*. The capital budgeting technique known as internal rate of return uses (E)
RPCPA 0598 a. b. c. d.
Cash flow over entire life of project No Yes Yes No
Time value of money Yes Yes No No

80
. How are the following used in the calculation of the internal rate of return of a proposed project? Ignore income tax considerations.
a. b. c. d.
Residual sales value of project Exclude Include Exclude Include
Depreciation expense Include Include Exclude Exclude

38. If Co. X wants to use IRR to evaluate long-term decisions and to establish a cutoff rate of return, X must be sure the cutoff rate is
(E)
a. At least equal to its cost of capital.
b. At least equal to the rate used by similar companies.
c. Greater than the IRR on projects accepted in the past.
d. Greater than the current book rate of return. L&H

Assumption/Disadvantage
48. Which of the following capital expenditure planning and control techniques has been criticized because it might mistakenly imply
that earnings are reinvested at the rate of return earned by the investment? (M)
a. payback method c. net present value method
b. accounting rate of return d. internal rate of return Barfield
81
. The net present value (NPV) method and the internal rate of return (IRR) method are used to analyze capital expenditures. The
IRR method, as contrasted with the NPV method,
A. Is considered inferior because it fails to calculate compounded interest rates.
B. Incorporates the time value of money whereas the NPV method does not.
C. Assumes that the rate of return on the reinvestment of the cash proceeds is at the indicated rate of return of the project
analyzed rather than at the discount rate used.
D. Is preferred in practice because it is able to handle multiple desired hurdle rates, which is impossible with the NPV method.
CMA 1291 4-7
82
. A weakness of the internal rate of return (IRR) approach for determining the acceptability of investments is that it (E)
a. Does not consider the time value of money.
b. Is not a straightforward decision criterion.
c. Implicitly assumes that the firm is able to reinvest project cash flows at the firm’s cost of capital.
d. Implicitly assumes that the firm is able to reinvest project cash flows at the project’s internal rate of return. CMA 1292 4-13

25. A weakness of the internal rate of return method for screening investment projects is that it: (M)
a. does not consider the time value of money.
b. implicitly assumes that the company is able to reinvest cash flows from the project at the company's discount rate.
c. implicitly assumes that the company is able to reinvest cash flows from the project at the internal rate of return.
d. does not take into account all of the cash flows from a project. CMA adapted

10. Which of the following capital expenditure planning and control techniques has been criticized because it might mistakenly imply
that earnings are reinvested at the rate of return earned by the investment?
A. internal rate of return method
B. accounting rate of return on initial investment method
C. payback method
D. average return on investment method
E. present value method AICPA adapted

25. The following are some of the shortcomings of the IRR method except: (E)
A. IRR is conceptually easy to communicate
B. Projects can have multiple IRRs B&M
C. IRR method cannot distinguish between a borrowing project and a lending project
D. It is very cumbersome to evaluate mutually exclusive projects using the IRR method

Advantage
83
. Which of the following characteristics represent an advantage of the internal rate of return techniques over the accounting rate of
return technique in evaluating a project? (M)
I Recognition of the project’s salvage value.
II Emphasis on cash flows.
III Recognition of the time value of money.
a. I only. c. II and III.
b. I and II. d. I, II, and III. AICPA 1192 T-49

Decision Criteria
48. In capital budgeting, a project is accepted only if the internal rate of return (E)
a. equals or exceeds the required rate of return.
b. equals or is less than the required rate of return.
c. equals or exceeds the net present value.
d. equals or exceeds the accrual accounting rate of return. Horngren

Comprehensive
23. Your company is comparing internal rate of return to net present value computations as alternative criteria for evaluating potential
capital investments. Which of the following best describes these computations?
A. The internal rate of return method ignores the initial cost of the investment in its computations.
B. The net present value method ignores the company's cost of capital.
C. The net present value method is more appropriate to use during periods of inflation.
D. The two methods will give the same rankings because they both consider the time value of money. CIA adapted
E. The internal rate of return method assumes that the positive cash flows generated each year are reinvested at the computed
rate of return for the investment being evaluated.

11. A company is considering the purchase of a new conveyor belt system for carrying parts and subassemblies from building to
building within its plant complex. It is expected that the system will have a useful life of at least ten years and that it will
substantially reduce labor and waiting-time costs. If the company's average cost of capital is about 15% and if some evaluation
must be made of cost/benefit relationships (including the effects of interest) to determine the desirability of the purchase, the most
relevant quantitative technique for evaluating the investment is:
A. present value (or internal rate of return) analysis
B. Program Evaluation and Review Technique (PERT)
C. accounting rate of return analysis
D. cost-volume-profit analysis
E. payback analysis AICPA adapted

*. Statement 1 The internal rate of return is the discount rate that equals the amount invested at a given date with the present value
of the expected cash inflows from the investment.
Statement 2 If the minimum desired rate of return exceeds the internal rate of return expected from a project, the project should
be rejected.
Statement 3 The internal rate of return can be more easily applied to situations with uneven periodic cash flows than can the net
present value. (M)
RPCPA 0592 a. b. c. d.
Statement 1 True False True False
Statement 2 True False True False
Statement 3 True False False True

MODIFIED INTERNAL RATE OF RETURN


22 Which of the following statements is most correct? The modified IRR (MIRR) method: (M)
a. Always leads to the same ranking decision as NPV for independent projects.
b. Overcomes the problem of multiple rates of return.
c. Compounds cash flows at the cost of capital.
d. Overcomes the problems of cash flow timing and project size that lead to criticism of the regular IRR method.
e. Statements b and c are correct. Brigham
84
. Which of the following statements is most correct? (M)
a. The MIRR method will always arrive at the same conclusion as the NPV method.
b. The MIRR method can overcome the multiple IRR problem, while the NPV method cannot.
c. The MIRR method uses a more reasonable assumption about reinvestment rates than the IRR method.
d. Statements a and c are correct.
e. All of the statements above are correct. Brigham

RELATIONSHIP among Payback, ARR, PI, NPV & IRR


ARR vs. IRR
11. Which of the following is a basic difference between the IRR and the book rate of return (BRR) criteria for evaluating investments?
a. IRR emphasizes expenses and BRR emphasizes expenditures.
b. IRR emphasizes revenues and BRR emphasizes receipts.
c. IRR is used for internal investments and BRR is used for external investments.
d. IRR concentrates on receipts and expenditures and BRR concentrates on revenues and expenses. L & H

Payback & NPV


12. If a project has a payback period shorter than its life,
a. Its NPV may be negative.
b. Its IRR is greater than cost of capital.
c. It will have a positive NPV.
d. Its incremental cash flows may not cover its cost. L&H

Payback & IRR


5. The relationship between payback period and IRR is that
a. A payback period of less than one-half the life of a project will yield an IRR lower than the target rate.
b. The payback period is the present value factor for the IRR.
c. A project whose payback period does not meet the company’s cutoff rate for payback will not meet the company’s criterion for
IRR.
d. None of the above. L&H

PI and NPV
31. An investment whose profitability index is 1.00
a. Has an IRR equal tot eh prevailing interest rate.
b. Returns to the company only the cash outlay for the investment.
c. Has a payback period equal to its useful life.
d. Has an NPV of zero. L&H

38. If a project generates a net present value of zero, the profitability index for the project will
a. equal zero. c. equal -1.
b. equal 1. d. be undefined. Barfield

40. If a project's profitability index is less than 1, the project's (E)


a. discount rate is above its cost of capital. c. payback period is infinite.
b. internal rate of return is less than zero. d. net present value is negative. Barfield

39. If the profitability index for a project exceeds 1, then the project's
a. net present value is positive.
b. internal rate of return is less than the project's discount rate.
c. payback period is less than 5 years.
d. accounting rate of return is greater than the project's internal rate of return. Barfield

15. An investment with a positive NPV also has


a. A positive profitability index.
b. A profitability index of one.
c. A profitability index less than one.
d. A profitability index greater than one. L&H
85
. If an investment project has a profitability index of 1.15, the
a. Project’s internal rate of return is 15%.
b. Project’s cost of capital is greater than its internal rate of return.
c. Project’s internal rate of return exceeds its net present value.
d. Net present value of the project is positive. CMA 1293 4-11

PI & IRR
21. If the profitability index is less than one,
a. The IRR is less than cost of capital. c. The IRR is greater than cost of capital.
b. The IRR is the same as cost of capital. d. None of the above is true. L&H
48. A project has an IRR less than the cost of capital. The profitability index for this project would be
a. Less than zero.
b. Between zero and one.
c. Greater than one.
d. Cannot be determined without more information. L&H

47. A project has an IRR in excess of the cost of capital. The profitability index for this project would be (M)
a. Less than zero.
b. Between zero and one.
c. Greater than one.
d. Cannot be determined without more information. L&H

NPV and IRR


86
. Polo Co. requires higher rates of return for projects with a life span greater than 5 years. Projects extending beyond 5 years must
earn a higher specified rate of return. Which of the following capital budgeting techniques can readily accommodate this
requirement? (M)
AICPA 0590 T-48 a. b. c. d.
Internal Rate of Return Yes No No Yes
Net Present Value No Yes No Yes

49. If the discount rate that is used to evaluate a project is equal to the project's internal rate of return, the project's _________ is zero.
a. profitability index c. present value of the investment
b. internal rate of return d. net present value Barfield

22. If an investment project (normal project) has an IRR equal to the cost of capital, the NPV for that project is: (E)
A. Positive C. Zero
B. Negative D. Unable to be determined B&M

26. If the net present value of a project is zero based on a discount rate of sixteen percent, then the time-adjusted rate of return: (M)
a. is equal to sixteen percent.
b. is less than sixteen percent.
c. is greater than sixteen percent.
d. cannot be determined from the information given. G & N 9e

4. If the present value of the future cash flows for an investment equals the required investment, the IRR is (D)
a. Equal to the cutoff rate.
b. Equal to the cost of borrowed capital.
c. Equal to zero.
d. Lower than the company’s cutoff rate of return. L&H
87
. If a prospective investment has a positive net present value at a company's cost of capital of 15%, it can be concluded that
A. The accounting rate of return of the project is greater than 15%.
B. The internal rate of return of the project is equal to the accounting rate of return.
C. The payback period of the associated asset is shorter than its life.
D. The internal rate of return of the project is greater than 15%. CIA 0R98 IV-37

45. If an investment has a positive net present value, the


a. internal rate of return is higher than the discount rate.
b. discount rate is higher than the hurdle rate of return.
c. internal rate of return is lower than the discount rate of return.
d. hurdle rate of return is higher than the discount rate. Barfield

7. If an investment has a positive NPV


a. Its IRR is greater than the company’s cost of capital.
b. Cost of capital exceeds the cutoff rate of return.
c. Its IRR is less than the company’s cutoff rate of return.
d. The cutoff rate of return exceeds cost of capital. L&H

*. Lenders Inc. is considering an investment that has a positive net present value based on its 16% hurdle rate. The internal rate of
return would be (M)
a. More than 16%. c. 16%.
b. Less than 16%. d. Zero. RPCPA 1095
88
. Neu Co. is considering the purchase of an investment that has a positive net present value based on Neu’s 12% hurdle rate. The
internal rate of return would be
a. 0%. c. >12%
b. 12%. d. < 12%
18. An investment has a positive NPV discounting the cash flows at a 14% cost of capital. Which statement is true?
a. The IRR is lower than 14%. c. The payback period is less than 14 years.
b. The IRR is higher than 14%. d. The book rate of return is 14%. L&H
89
. The net present value of a proposed investment is negative; therefore, the discount rate used must be
A. Greater than the project's internal rate of return.
B. Less than the project's internal rate of return.
C. Greater than the firm's cost of equity.
D. Less than the risk-free rate. CMA 1295 4-14, RPCPA 0596

25. At a company's cost of capital of 15%, a prospective investment has a negative net present value. Based on this information, it can
be concluded that:
A. the internal rate of return is greater than 15%
B. the payback period is shorter than the life of the asset
C. the accounting rate of return is less than 15%
D. the accounting rate of return is greater than 15%
E. the internal rate of return is less than 15% CIA adapted
90
. Which of the following statements is most correct? (E)
a. If a project’s internal rate of return (IRR) exceeds the cost of capital, then the project’s net present value (NPV) must be
positive.
b. If Project A has a higher IRR than Project B, then Project A must also have a higher NPV.
c. The IRR calculation implicitly assumes that all cash flows are reinvested at a rate of return equal to the cost of capital.
d. Statements a and c are correct. Brigham
91
. Which of the following statements is most correct? (M)
a. If a project with normal cash flows has an IRR which exceeds the cost of capital, then the project must have a positive NPV.
b. If the IRR of Project A exceeds the IRR of Project B, then Project A must also have a higher NPV.
c. The modified internal rate of return (MIRR) can never exceed the IRR.
d. Statements a and c are correct. Brigham
92
. Project A has an internal rate of return (IRR) of 15 percent. Project B has an IRR of 14 percent. Both projects have a cost of
capital of 12 percent. Which of the following statements is most correct? (E)
a. Both projects have a positive net present value (NPV).
b. Project A must have a higher NPV than Project B.
c. If the cost of capital were less than 12 percent, Project B would have a higher IRR than Project A.
d. Statements a and c are correct.
e. All of the statements above are correct. Brigham
93
. Project A has an IRR of 15 percent. Project B has an IRR of 18 percent. Both projects have the same risk. Which of the following
statements is most correct? (E)
a. If the WACC is 10 percent, both projects will have a positive NPV, and the NPV of Project B will exceed the NPV of Project A.
b. If the WACC is 15 percent, the NPV of Project B will exceed the NPV of Project A.
c. If the WACC is less than 18 percent, Project B will always have a shorter payback than Project A.
d. If the WACC is greater than 18 percent, Project B will always have a shorter payback than Project A.
e. If the WACC increases, the IRR of both projects will decline. Brigham
94
. Project J has the same internal rate of return as Project K. Which of the following statements is most correct? (M)
a. If the projects have the same size (scale) they will have the same NPV, even if the two projects have different levels of risk.
b. If the two projects have the same risk they will have the same NPV, even if the two projects are of different size.
c. If the two projects have the same size (scale) they will have the same discounted payback, even if the two projects have
different levels of risk.
d. All of the statements above are correct.
e. None of the statements above is correct. Brigham

NPV, IRR & WACC


95
. Assume a project has normal cash flows (that is, the initial cash flow is negative, and all other cash flows are positive). Which of
the following statements is most correct? (E)
a. All else equal, a project’s IRR increases as the cost of capital declines.
b. All else equal, a project’s NPV increases as the cost of capital declines.
c. All else equal, a project’s MIRR is unaffected by changes in the cost of capital.
d. Statements a and b are correct.
e. Statements b and c are correct. Brigham

. Which of the following statements is incorrect? (M)


96

a. Assuming a project has normal cash flows, the NPV will be positive if the IRR is less than the cost of capital.
b. If the multiple IRR problem does not exist, any independent project acceptable by the NPV method will also be acceptable by
the IRR method.
c. If IRR = k (the cost of capital), then NPV = 0.
d. NPV can be negative if the IRR is positive.
e. The NPV method is not affected by the multiple IRR problem. Brigham
97
. A project has an up-front cost of $100,000. The project’s WACC is 12 percent and its net present value is $10,000. Which of the
following statements is most correct? (E)
a. The project should be rejected since its return is less than the WACC.
b. The project’s internal rate of return is greater than 12 percent.
c. The project’s modified internal rate of return is less than 12 percent.
d. All of the statements above are correct. Brigham

NPV, IRR & Payback


37. A project that has a negative NPV.
a. Has a payback period longer than its life.
b. Has a negative profitability index.
c. Must be rejected.
d. Doesn’t necessarily fit any of the above descriptions. L&H
98
. Project X has an internal rate of return of 20 percent. Project Y has an internal rate of return of 15 percent. Both projects have a
positive net present value. Which of the following statements is most correct? (M)
a. Project X must have a higher net present value than Project Y.
b. If the two projects have the same WACC, Project X must have a higher net present value.
c. Project X must have a shorter payback than Project Y.
d. Statements b and c are correct.
e. None of the statements above is correct. Brigham

PI, NPV & IRR


22. Which of the following combinations is possible? L&H
Profitability Index NPV IRR
a. Greater than 1 Positive Equal cost of capital
b. Greater than 1 Negative Less that cost of capital
c. Less than 1 Negative Less than cost of capital
d. Less than 1 Positive Less than cost of capital

23. Which of the following combinations is NOT possible? (E) L&H


Profitability Index NPV IRR
a. Greater than 1 Positive More than cost of capital
b. Equals 1 Zero Equals cost of capital
c. Less than 1 Negative Less than cost of capital
d. Less than 1 Positive Less than cost of capital

NPV, IRR, MIRR, and Payback


99
. A proposed project has normal cash flows. In other words, there is an up-front cost followed over time by a series of positive cash
flows. The project’s internal rate of return is 12 percent and its WACC is 10 percent. Which of the following statements is most
correct? (E)
a. The project’s NPV is positive.
b. The project’s MIRR is greater than 10 percent but less than 12 percent.
c. The project’s payback period is greater than its discounted payback period.
d. Statements a and b are correct. Brigham

COMPARISION among Payback, ARR, PI, NPV & IRR


NPV vs.IRR
52. An important advantage of the net present value method of capital budgeting over the internal rate-of-return method is (E)
a. the net present value method is expressed as a percentage.
b. the net present values of individual projects can be added to determine the effects of accepting a combination of projects.
c. no advantage.
d. both (a) and (b). Horngren

Depreciation
23. (Ignore income taxes in this problem.) How is depreciation handled by the following capital budgeting techniques? (M)
CMA adapted A. B. C. D.
Internal Rate of Return Excluded Included Excluded Included
Simple Rate of Return Included Excluded Excluded Included
Payback Excluded Included Excluded

19. If income tax considerations are ignored, how is depreciation expense used in the following capital budgeting techniques?
AICPA adapted A. B. C. D.
Internal Rate of Return Excluded Included Included Excluded
Payback Included Excluded Included Excluded

*. If income tax considerations are ignored, how is depreciation used in the following capital budgeting techniques? (E)
RPCPA 0595 a. b. c. d.
Internal rate of return Included Excluded Excluded Included
Accounting rate of return Excluded Included Excluded Included

30. If income tax considerations are ignored, how is depreciation expense used in the following capital budgeting techniques?
AICPA adapted A. B. C. D.
Internal Rate of Return Excluded Excluded Included Included
Net Present Value Excluded Included Excluded Included

100
. If income tax considerations are ignored, how is depreciation handled by the following budgeting technique?
CMA 1293 4-17 a. b. c. d.
Internal Rate of Return Excluded Included Excluded Included
Accounting Rate of Return Included Excluded Excluded Included
Payback Excluded Included Included Included

Cash Flow
24. Which of the following capital budgeting techniques consider(s) cash flow over the entire life of the project? (E)
AICPA adapted A. B. C. D.
Internal Rate of Return Yes Yes No No
Payback Yes No Yes No

PROJECT SCREENING (Accept/Reject Decision for Independent Project)


59. Which of the following best represents a screening decision?
a. determining which project has the highest net present value
b. determining if a project's internal rate of return exceeds the firm's cost of capital
c. determining which projects are mutually exclusive
d. determining which are the best projects Barfield

Security Market Line (SML) Concept


. Using the Security Market Line concept in capital budgeting, which of the following is correct? (M)
a. If the expected rate of return on a given capital project lies above the SML, the project should be accepted even if its beta is
above the beta of the firm’s average project.
b. If a project’s return lies below the SML, it should be rejected if it has a beta greater than the firm’s existing beta but accepted if
its beta is below the firm’s beta.
c. If two mutually exclusive projects’ expected returns are both above the SML, the project with the lower risk should be
accepted.
d. If a project’s expected rate of return is greater than the expected rate of return on an average project, it should be accepted.
Brigham

23. On a graph with common stock returns on the Y axis and market returns on the X-axis, the slope of the regression line represents
the:
A. Alpha C. R-squared
B. Beta D. Adjusted beta B&M

15. The historical returns data for the past three years for Company A's stock is -6.0%, 15%, 15% and that of the market portfolio is
10%, 10% and 16%. According to the SML, the Stock A is:
A. over priced C. Correctly priced
B. Under priced D. Need more information B&M

22. The historical returns data for the past three years for Stock B and the stock market portfolio are: Stock B:- 24%, 0%, 24%, Market
Portfolios:- 10%, 12%, 20%. According to the SML the stock B is:
A. Overpriced C. Correctly priced
B. Underpriced B&M

Single Project
101
. The profitability index approach to investment analysis (M)
A. Fails to consider the timing of project cash flows.
B. Considers only the project's contribution to net income and does not consider cash flow effects.
C. Always yields the same accept/reject decisions for independent projects as the net present value method.
D. Always yields the same accept/reject decisions for mutually exclusive projects as the net present value method. CMA
1292 4-14, RPCPA 0596
14. The NPV and IRR methods give
a. The same decision (accept or reject) for any single investment.
b. The same choice from among mutually exclusive investments.
c. Different rankings of projects with unequal lives.
d. The same rankings of projects with different required investments. L&H

29. Which of the following is always true of the net present value (NPV) approach?
A. If a project is found to be acceptable under the NPV approach, it would also be acceptable under the internal rate of return
(IRR) approach.
B. The NPV and the IRR approaches will always rank projects in the same order.
C. If a project is found to be acceptable under the NPV approach, it would also be acceptable under the payback approach.
CIA adapted
D. The NPV and the payback approaches will always rank projects in the same order.

Questions 140 and 141 are based on the following information. CIA 0594 IV-45 & 46
The financial management team of a company is assessing an investment proposal involving a $100,000 outlay today. Manager one
expects the project to provide cash inflows of $20,000 at the end of each year for 6 years. She considers the project to be of low risk,
requiring only a 10% rate of return. Manager two expects the project to provide cash inflows of $5,000 at the end of the first year,
followed by $23,000 at the end of each year in years two through six. He considers the project to be of medium risk, requiring a 14%
rate of return. Manager three expects the project to be of high risk, providing one large cash inflow of $135,000 at the end of the sixth
year. She proposes a 15% rate of return for the project.
Additional Information:
Number Discount Present Value of $1 Due at Annuity of $1 per Period
of Years Rate (%) the End of n Periods (PVIF) for n Periods (PVIFA)
1 10 .9091 .9091
1 14 .8772 .8772
1 15 .8696 .8696
5 10 .6209 3.7908
5 14 .5194 3.4331
5 15 .4972 3.3522
6 10 .5645 4.3553
6 14 .4556 3.8887
6 15 .4323 3.7845
102
. According to the net present value criterion, which of the following is true?
A. Manager one will recommend that the project be accepted.
B. Manager two will recommend that the project be accepted.
C. All three managers will recommend acceptance of the project.
D. All three managers will recommend rejection of the project.
103
. Which manager will assess the project as having the shortest payback period?
A. Manager one.
B. Manager two.
C. Manager three.
D. All three managers will agree on the payback period.

Group Project
6. You are given a job to make a decision on project X, which is composed of three projects A, B, and C which have NPVs of +$50, -
$20 and +$100, respectively. How would you go about making the decision about whether to accept or reject the project? (M)
A. Accept the firm's joint project as it has a positive NPV
B. Reject the joint project
C. Break up the project into its components: accept A and C and reject B
D. None of the above B&M

Unlimited Capital
104
. A company has unlimited capital funds to invest. The decision rule for the company to follow in order to maximize shareholders'
wealth is to invest in all projects having a(n)
A. Present value greater than zero.
B. Net present value greater than zero.
C. Internal rate of return greater than zero. CMA 1293 4-15
D. Accounting rate of return greater than the hurdle rate used in capital budgeting analyses.
105
. Future, Inc. is in the enviable situation of having unlimited capital funds. The best decision rule, in an economic sense, for it to
follow would be to invest in all projects in which the
A. Accounting rate of return is greater than the earnings as a percent of sales.
B. Payback reciprocal is greater than the internal rate of return.
C. Internal rate of return is greater than zero.
D. Net present value is greater than zero. CMA 1278 5-12
106
. Barker Inc. has no capital rationing constraint and is analyzing many independent investment alternatives. Barker should accept
all investment proposals
a. If debt financing is available for them.
b. That have positive cash flows.
c. That provide returns greater than the after-tax cost of debt.
d. That have a positive net present value. CMA 1295 4-2

Independent Projects
107
. An organization is using capital budgeting techniques to compare two independent projects. It could accept one, both, or neither
of the projects. Which of the following statements is true about the use of net-present-value (NPV) and internal-rate-of-return (IRR)
methods for evaluating these two projects?
a. NPV and IRR criteria will always lead to the same accept or reject decision for two independent projects.
b. If the first project’s IRR is higher than the organization’s cost of capital, the first project will be accepted but the second project
will not.
c. If the NPV criterion leads to accepting or rejecting the first project, one cannot predict whether the IRR criterion will lead to
accepting or rejecting the first project.
d. If the NPV criterion leads to accepting the first project, the IRR criterion will never lead to accepting the first project. CIA
0597 IV-43
108
. Which of the following is always true with regard to the net present value (NPV) approach?
A. If a project is found to be acceptable under the NPV approach, it would also be acceptable under the internal rate of return
(IRR) approach.
B. The NPV and the IRR approaches will always rank projects in the same order.
C. If a project is found to be acceptable under the NPV approach, it would also be acceptable under the payback approach.
CIA 0586 IV-29
D. The NPV and payback approaches will always rank projects in the same order.

PROJECT RANKING METHOD


Capital Rationing
38. Soft rationing is imposed in order to: (E)
A. Control managers' actions C. Cope with market imperfections
B. Limit runaway growth D. A and B B&M

39. Hard rationing is imposed by the: (E)


A. Market C. Management
B. Majority stockholder D. Both B and C B&M

Mutually Inclusive Projects


63. If management judges one project in a mutually inclusive set to be acceptable for investment,
a. all the other projects in the set are rejected.
b. only one other project in the set can be accepted.
c. all other projects in the set are also accepted.
d. only one project in the set will be rejected. Barfield

Mutually Exclusive Projects


Net Present Value (Preferred Method)
7. In choosing from among mutually exclusive investments the manager should normally select the one with the highest (M)
a. NPV. c. Profitability index.
b. IRR. d. Book rate of return. L&H

57. Which of the following capital investment models would be preferred when choosing among mutually exclusive alternatives? (M)
a. payback period c. IRR
b. accounting rate of return d. NPV H&M

Profitability Index
109
. When ranking two mutually exclusive investments with different initial amounts, management should give first priority to the project
A. That generates cash flows for the longer period of time.
B. Whose net after-tax flows equal the initial investment.
C. That has the greater accounting rate of return.
D. That has the greater profitability index. CMA 1291 4-6

NPV & IRR


110
. The internal rate of return on an investment
A. Usually coincides with the company's hurdle rate.
B. Disregards discounted cash flows.
C. May produce different rankings from the net present value method on mutually exclusive projects.
D. Would tend to be reduced if a company used an accelerated method of depreciation for tax purposes rather than the straight-
line method. CMA 0691 4-19
111
. The rankings of mutually exclusive investments determined using the internal rate of return method (IRR) and the net present
value method (NPV) may be different when
a. The lives of the multiple projects are equal and the size of the required investments are equal.
b. The required rate of return equals that IRR of each project.
c. The required rate of return is higher than the IRR of each project.
d. Multiple projects have unequal lives and the size of the investment for each project is different. CMA 1292 4-15, RPCPA
1096

50. Why do the NPV method and the IRR method sometimes produce different rankings of mutually exclusive investment projects?
A. The NPV method does not assume reinvestment of cash flows while the IRR method assumes the cash flows will be
reinvested at the internal rate of return.
B. The NPV method assumes a reinvestment rate equal to the discount rate while the IRR method assumes a reinvestment rate
equal to the internal rate of return.
C. The IRR method does not assume reinvestment of the cash flows while the NPV assumes the reinvestment rate is equal to
the discount rate.
D. The NPV method assumes a reinvestment rate equal to the bank loan interest rate while the IRR rate method assumes a
reinvestment rate equal to the discount rate. Pol Bobadilla

Ranking Decision
112
. Which mutually exclusive project would you select, if both are priced at $1,000 and your discount rate is 15%; Project A with three
annual cash flows of $1,000, or Project B, with 3 years of zero cash flow followed by 3 years of $1,500 annually?
A. Project A.
B. Project B.
C. The IRRs are equal, hence you are indifferent.
D. The NPVs are equal, hence you are indifferent. Gleim

Independent Projects
Definition
*. The kind of investment project which has no direct relationship with other projects and can therefore be implemented or rejected
independently of others (E)
a. Independent investment project
b. Complimentary investment project
c. None of these RPCPA 0588

Examples
60. Below are pairs of projects. Which pair best represents independent projects?
a. buy computer; buy software package
b. buy computer #1; buy computer #2
c. buy computer; buy computer security system
d. buy computer; repave parking lot Barfield

Profitability Index
35. Profitability index is useful under: (E)
A. Capital rationing C. Non-normal projects
B. Mutually exclusive projects D. None of the above B&M

40. The profitability index can be used for ranking projects under: (E)
A. Soft capital rationing C. Capital rationing at t = 0
B. Hard capital rationing D. Both A and B B&M
113
. The recommended technique for evaluating projects when capital is rationed and there are no mutually exclusive projects from
which to choose is to rank the projects by
A. Accounting rate of return. C. Internal rate of return.
B. Payback. D. Profitability index. CMA 0692 4-15
114
. The technique used to evaluate all possible capital projects of different dollar amounts and then rank them according to their
desirability is the (M)
a. Profitability index method. c. Payback method. CMA 1294 4-26
b. Net present value method. d. Discounted cash flow method.

Ranking Decision
37. A company is evaluating three possible investments. Information relating to the company and the investments follow:
Fisher rate for the three projects 7%
Cost of capital 8%
Based on this information, we know that (D)
a. all three projects are acceptable.
b. none of the projects are acceptable.
c. the capital budgeting evaluation techniques profitability index, net present value, and internal rate of return will provide a
consistent ranking of the projects.
d. the net present value method will provide a ranking of the projects that is superior to the ranking obtained using the internal
rate of return method. Barfield

*. Several proposed capital projects which are economically acceptable may have to be ranked due to constraints in financial
resources. In ranking these projects, the least pertinent is this statement. (M)
a. If the internal rate of return method is used in the capital rationing problem, the higher the rate, the better the project.
b. In selecting the required rate of return, one may either calculate the organization’s cost of capital or use a rate generally
acceptable in the industry.
c. A ranking procedure on the basis of quantitative criteria may be established by specifying a minimum desired rate of return,
which rate is used in calculating the net present value of each project.
d. If the net present value method is used, the profitability index is calculated to rank the projects. The lower the index, the better
the project. RPCPA 1094

Profitability Index
115
. Capital budgeting methods are often divided into two classifications: project screening and project ranking. Which one of the
following is considered a ranking method rather than a screening method?
A. Net present value. C. Profitability index.
B. Time-adjusted rate of return. D. Accounting rate of return. CMA 0691 4-17

IRR
116
. A company has analyzed seven new projects, each of which has its own internal rate of return. It should consider each project
whose internal rate of return is _____ its marginal cost of capital and accept those projects in _____ order of their internal rate of
return.
A. Below; decreasing. C. Above; increasing.
B. Above; decreasing. D. Below; increasing. CIA 0593 IV-55

Internal Rate of Return & Net Present Value


22. The three frequently used methods for ranking investment proposals are payback, net present value, and internal rate of return.
One of the three is defined as the interest rate that equates the present value of expected cash flows and the cost of the project. A
second method finds the present value of expected cash flows and subtracts the initial cost of the project. The following terms that
match these respective definitions are:
A. internal rate of return and net present value
B. internal rate of return and payback
C. net present value and internal rate of return
D. net present value and payback CIA adapted

RISK ANALYSIS
Risk
Risk-free Rate
117
. The proper discount rate to use in calculating certainty equivalent net present value is the (E)
a. Risk-adjusted discount rate. d. Cost of equity capital.
b. Cost of capital. c. Risk-free rate. CMA 1292 4-19
118
. The following data are related to the cash flows of a risky capital-budgeting alternative:
Col. 1 Col. 2 Col. 3
Period Expected Cash Flows Certainty Equivalent Factors
1 1,000 .85
2 1,000 .75
3 1,000 .70
The discount rates available for this analysis are: risk-free rate = 5%, cost of capital = 10%, and risk-adjusted discount rate = 15%.
How should these cash flows be discounted using the certainty-equivalent method (CE) and the risk-adjusted discount rate method
(RADR)?
CE RADR
A. (Col.2xCol.3) at 10% (Col.2xCol.3) at 5%
B. (Col.2xCol.3) at 5% (Col.2xCol.3) at 10%
C. (Col.2xCol.3) at 10% Col.2 at 15%
D. (Col.2xCol.3) at 5% Col.2 at 15%
CIA 0586 IV-32
119
. A firm has negotiated a contract with the government and has locked in the payment it will receive in each of the future years from
this project. However, the firm's costs for this project are uncertain. How should the certainty-equivalent (CE) approach be applied
in this situation?
A. Discount cash inflows using cost of capital and CE values of cost using cost of capital.
B. Discount cash inflows using cost of capital and CE values of cost using risk-free rate.
C. Determine net cash inflows using CE values of cost and discount using cost of capital.
D. Determine net cash inflows using CE values of cost and discount using risk-free rate.
CIA 0586 IV-34

Uncertainties
120
. Carco, Inc. wants to use discounted cash flow techniques when analyzing its capital investment projects. The company is aware of
the uncertainty involved in estimating future cash flows. A simple method some companies employ to adjust for the uncertainty
inherent in their estimates is to
A. Prepare a direct analysis of the probability of outcomes.
B. Use accelerated depreciation.
C. Adjust the minimum desired rate of return.
D. Increase the estimates of the cash flows. CMA 1278 5-8

23. To reflect greater uncertainty (greater risk) about a future cash inflow, an analyst could
a. increase the discount rate for the cash flow.
b. decrease the discounting period for the cash flow.
c. increase the expected value of the future cash flow before it is discounted.
d. extend the acceptable length for the payback period. Barfield

Risk Factor
121
. For capital budgeting purposes, management would select a high hurdle rate of return for certain projects because management
a. Wants to use equity funding exclusively.
b. Believes too many proposals are being rejected.
c. Believes bank loans are riskier than capital investments.
d Wants to factor risk into its consideration of projects. CMA 1294 4-22

16. In a discounted cash flow analysis, which of the following would not be consistent with adjusting a project's cash flows to account
for higher-than-normal risk?
a. increasing the expected amount for cash outflows
b. increasing the discounting period for expected cash inflows
c. increasing the discount rate for cash outflows
d. decreasing the amount for expected cash inflows Barfield

Different Risk Levels


122
. When the risks of the individual components of a project’s cash flows are different, an acceptable procedure to evaluate these
cash flows is to
a. Divide each cash flow by the payback period.
b. Compute the net present value of each cash flow using the firm’s cost of capital.
c. Compare the internal rate of return from each cash flow to its risk. CMA 1295 4-6
d. Discount each cash flow using a discount rate that reflects the degree of risk.

Risk-Adjusted Discount Rates


123
. Risk in a revenue-producing project can best be adjusted for by(E)
a. Ignoring it.
b. Adjusting the discount rate upward for increasing risk.
c. Adjusting the discount rate downward for increasing risk.
d. Picking a risk factor equal to the average discount rate.
e. Reducing the NPV by 10 percent for risky projects. Brigham

Risk and Project Selection


. If a company uses the same discount rate for evaluating all projects, which of the following results is likely? (M)
a. Accepting poor, high-risk projects. d. Accepting no projects.
b. Rejecting good, low-risk projects. e. Statements a and b are correct.
c. Accepting only good, low risk projects. Brigham

. If a typical U. S. company uses the same discount rate to evaluate all projects, the firm will most likely become (M)
a. Riskier over time, and its value will decline.
b. Riskier over time, and its value will rise.
c. Less risky over time, and its value will rise.
d. Less risky over time, and its value will decline.
e. There is no reason to expect its risk position or value to change over time as a result of its use of a single discount rate.
Brigham
124
. A company estimates that an average-risk project has a WACC of 10 percent, a below-average risk project has a WACC of 8
percent, and an above-average risk project has a WACC of 12 percent. Which of the following independent projects should the
company accept? (E)
a. Project A has average risk and an IRR = 9 percent.
b. Project B has below-average risk and an IRR = 8.5 percent.
c. Project C has above-average risk and an IRR = 11 percent.
d. All of the projects above should be accepted.
e. None of the projects above should be accepted. Brigham
125
. A firm is considering the purchase of an asset whose risk is greater than the current risk of the firm, based on any method for
assessing risk. In evaluating this asset, the decision maker should(E)
a. Increase the IRR of the asset to reflect the greater risk.
b. Increase the NPV of the asset to reflect the greater risk.
c. Reject the asset, since its acceptance would increase the risk of the firm.
d. Ignore the risk differential if the asset to be accepted would comprise only a small fraction of the total assets of the firm.
e. Increase the cost of capital used to evaluate the project to reflect the higher risk of the project. Brigham
126
. Downingtown Industries has an overall (composite) WACC of 10 percent. This cost of capital reflects the cost of capital for a
Downingtown project with average risk; however, there are large differences among the projects. The company estimates that low-
risk projects have a cost of capital of 8 percent and high-risk projects have a cost of capital of 12 percent. The company is
considering the following projects:
Project Expected Return Risk
A 15% High
B 12 Average
C 11 High
D 9 Low
E 6 Low
Which of the projects will the company select? (E)
a. A and B. d. A, B, C, and D.
b. A, B, and C. e. A, B, C, D, and E.
c. A, B, and D. Brigham
127
. Mega Inc., a large conglomerate with operating divisions in many industries, uses risk-adjusted discount rates in evaluating capital
investment decisions. Consider the following statements concerning Mega's use of risk-adjusted discount rates.
I. Mega may accept some investments with internal rates of return less than Mega's overall average cost of capital.
II. Discount rates vary depending on the type of investment.
III. Mega may reject some investments with internal rates of return greater than the cost of capital.
IV. Discount rates may vary depending on the division.
Which of the above statements are correct?
A. I and III only. C. II, III, and IV only.
B. II and IV only. D. I, II, III, and IV. CMA Samp Q4-5
128
. Kemp Consolidated has two divisions of equal size: a computer division and a restaurant division. Stand-alone restaurant
companies typically have a cost of capital of 8 percent, while stand-alone computer companies typically have a 12 percent cost of
capital. Kemp’s restaurant division has the same risk as a typical restaurant company, and its computer division has the same risk
as a typical computer company. Consequently, Kemp estimates that its composite corporate cost of capital is 10 percent. The
company’s consultant has suggested that they use an 8 percent hurdle rate for the restaurant division and a 12 percent hurdle rate
for the computer division. However, Kemp has chosen to ignore its consultant, and instead, chooses to assign a 10 percent cost of
capital to all projects in both divisions. Which of the following statements is most correct? (M)
a. While Kemp’s decision to not risk adjust its cost of capital will lead it to accept more projects in its computer division and fewer
projects in its restaurant division, this should not affect the overall value of the company.
b. Kemp’s decision to not risk adjust means that it is effectively subsidizing its restaurant division, which means that its restaurant
division is likely to become a larger part of the overall company over time.
c. Kemp’s decision to not risk adjust means that the company will accept too many projects in the computer business and too
few projects in the restaurant business. This will lead to a reduction in the overall value of the company.
d. Statements a and b are correct. Brigham
129
. The Barabas Company has an equal amount of low-risk projects, average-risk projects, and high-risk projects. Barabas estimates
that the overall company’s WACC is 12 percent. This is also the correct cost of capital for the company’s average-risk projects.
The company’s CFO argues that, even though the company’s projects have different risks, the cost of capital for each project
should be the same because the company obtains its capital from the same sources. If the company follows the CFO’s advice,
what is likely to happen over time? (M)
a. The company will take on too many low-risk projects and reject too many high-risk projects.
b. The company will take on too many high-risk projects and reject too many low-risk projects.
c. Things will generally even out over time, and therefore, the risk of the firm should remain constant over time.
d. Statements a and c are correct. Brigham
130
. The Oneonta Chemical Company is evaluating two mutually exclusive pollution control systems. Since the company’s revenue
stream will not be affected by the choice of control systems, the projects are being evaluated by finding the PV of each set of
costs. The firm’s required rate of return is 13 percent, and it adds or subtracts 3 percentage points to adjust for project risk
differences. System A is judged to be a high-risk project (it might end up costing much more to operate than is expected). System
A’s risk-adjusted cost of capital is(M)
a. 10 percent; this might seem illogical at first, but it correctly adjusts for risk where outflows, rather than inflows, are being
discounted.
b. 13 percent; the firm’s cost of capital should not be adjusted when evaluating outflow only projects.
c. 16 percent; since A is more risky, its cash flows should be discounted at a higher rate, because this correctly penalizes the
project for its high risk.
d. Somewhere between 10 percent and 16 percent, with the answer depending on the riskiness of the relevant inflows.
e. Indeterminate, or, more accurately, irrelevant, because for such projects we would simply select the process that meets the
requirements with the lowest required investment. Brigham

Methods of Analyzing Risk


131
. Which of the following is not a method for analyzing risk in capital budgeting? (E)
a. Sensitivity analysis.
b. Beta, or CAPM, analysis.
c. Monte Carlo simulation.
d. Scenario analysis.
e. All of the statements above are methods of analyzing risk in capital budgeting. Brigham
132
. Which of the following statements is correct? (M)
a. Sensitivity analysis is incomplete because it fails to consider the range of likely values of key variables as reflected in their
probability distributions.
b. In comparing two projects using sensitivity analysis, the one with the steeper lines would be considered less risky, because a
small error in estimating a variable, such as unit sales, would produce only a small error in the project’s NPV.
c. The primary advantage of simulation analysis over scenario analysis is that scenario analysis requires a relatively powerful
computer, coupled with an efficient financial planning software package, whereas simulation analysis can be done using a PC
with a spreadsheet program or even a calculator.
d. Sensitivity analysis is a risk analysis technique that considers both the sensitivity of NPV to changes in key variables and the
likely range of variable values. Brigham

Simulation and Sensitivity Analysis


133
. A company is deciding whether to purchase an automated machine to manufacture one of its products. Expected net cash flows
from this decision depend on several factors, interactions among those factors and the probabilities associated with different levels
of those factors. The method that the company should use to evaluate the distribution of net cash flows from this decision and
changes in net cash flows resulting from changes in levels of various factors is
a. Simulation and sensitivity analysis. c. Correlation analysis.
b. Linear programming. d. Differential analysis. CIA 1194 III-61

Simulation Analysis
134
. A firm is evaluating a large project. It desires to develop not only the best guess of the outcome of the project, but also a list (or
distribution) of outcomes that might occur. This firm would best achieve its objective by using
A. The net-present-value (NPV) approach for capital budgeting.
B. The profitability-index approach for capital budgeting.
C. Simulation as applied to capital budgeting.
D. The internal-rate-of-return (IRR) approach for capital budgeting. CIA 0589 IV-51
135
. A statistical technique used to evaluate possible rates of return for a capital budgeting project is
A. Regression analysis. C. Markov chain analysis.
B. Simulation analysis. D. Gantt charting. CMA 0689 5-15
136
. Which of the following statements is most correct? (E)
a. Sensitivity analysis is a good way to measure market risk because it explicitly takes into account the effects of diversification.
b. One advantage of sensitivity analysis relative to scenario analysis is it explicitly takes into account the probability of certain
effects occurring, whereas scenario analysis does not take into account probabilities.
c. Simulation analysis is a computerized version of scenario analysis that uses continuous probability distributions of the input
variables.
d. Statements a and b are correct.
e. All of the statements above are correct. Brigham

Sensitivity Analysis
137
. Sensitivity analysis, if used with capital projects (M)
a. Is used extensively when cash flows are known with certainty.
b. Measures the change in the discounted cash flows when using the discounted payback method rather than the net present
value method.
c. Is a “what-if” technique that asks how a given outcome will change if the original estimates of the capital budgeting model are
changed.
d. Is a technique used to rank capital expenditure requests. CMA 0695 4-2, RPCPA 0596
138
. A manager wants to know the effect of a possible change in cash flows on the net present value of a project. The technique used
for this purpose is
A. Sensitivity analysis. C. Cost behavior analysis. CMA 1286 5-4
B. Risk analysis. D. Return on investment analysis.
62. Sensitivity analysis is
a. an appropriate response to uncertainty in cash flow projections.
b. useful in measuring the variance of the Fisher rate.
c. typically conducted in the post investment audit.
d. useful to compare projects requiring vastly different levels of initial investment. Barfield
139
. Sensitivity analysis is used in capital budgeting to
A. Estimate a project's internal rate of return.
B. Determine the amount that a variable can change without generating unacceptable results.
C. Simulate probabilistic customer reactions to a new product.
D. Identify the required market share to make a new product viable and produce acceptable results. CMA 1293 4-16

24. In capital budgeting, sensitivity analysis is used


a. To determine whether an investment is profitable.
b. To see how a decision would be affected by changes in variables.
c. To test the relationship of the IRR and NPV.
d. To evaluate mutually exclusive investments. L&H

17. Which of the following makes investments more desirable than they had been?
a. An increase in income tax rate.
b. An increase in interest rates.
c. An increase in the number of years over which assets must be depreciated.
d. None of the above. L&H

28. Which statement could express the results of a sensitivity analysis of an investment decision?
a. The NPV of the project is $50,000.
b. A 5% decline in volume will make the project unprofitable.
c. This project ranks third out of the five available.
d. This project does not meet the cutoff rate of return. L&H

36. If X Co. expects to get a one-year bank loan to help cover the initial financing of capital project Q, the analysis of Q should (D)
a. Offset the loan against any investment in inventory or receivables required by the project.
b. Show the loan as an increase in the investment.
c. Show the loan as a cash outflow in the second year of the project’s life.
d. Ignore the loan. L&H

54. A "what-if" technique that examines how a result will change if the original predicted data are not achieved or if an underlying
assumption changes is called (E)
a. sensitivity analysis. c. internal rate-of-return analysis. Horngren
b. net present value analysis. d. adjusted rate-of-return analysis.

Monte Carlo simulation


140
. Monte Carlo simulation(M)
a. Can be useful for estimating a project’s stand-alone risk.
b. Is capable of using probability distributions for variables as input data instead of a single numerical estimate for each variable.
c. Produces both an expected NPV (or IRR) and a measure of the riskiness of the NPV or IRR.
d. Statements a and b are correct.
e. All of the statements above are correct. Brigham

Net Investment
64. All other factors equal, which of the following would affect a project's internal rate of return, net present value, and payback period?
(M)
a. an increase in the discount rate c. an increase in the initial cost of the project
b. a decrease in the life of the project d. all of the above Barfield

Tax Effect on Transactions


Change in Depreciation Rate
27. If the tax law were changed so that owners of apartment buildings had to depreciate them over 50 years instead of the current 31.5
years,
a. Rents would rise.
b. Rents would fall because annual depreciation charges would fall.
c. Rents would stay about the same.
d. More people would invest in apartment buildings. L&H

47. For a profitable company, an increase in the rate of depreciation on a specific project could
a. increase the project's profitability index.
b. increase the project's payback period.
c. decrease the project's net present value.
d. increase the project's internal rate of return. Barfield

Depreciation and Savings on Cash Operating Costs


41. If depreciation of a new asset exceeds its savings in cash operating costs, which of the following is true?
a. The project is usually unacceptable.
b. The annual after-tax cash flow on the new asset will be greater than the savings in cash operating costs.
c. The project has a negative NPV.
d. All of the above. L&H

Sale of Old Plant Assets


26. Because of idle capacity, a company is considering two assets for sale. They are identical in all respects except that asset A has a
higher tax basis than asset B. Only one need be sold now and the market price is the same for both assets. Which of the
following is true?
a. The cash flow is greater from selling asset A.
b. The cash flow is greater from selling asset B.
c. The cash flow is the same no matter which one is sold.
d. It is not possible to determine how the cash flows from sale of the assets will differ. L & H

Sale of Old Plant Asset at a Loss


51. When a profitable corporation sells an asset at a loss, the after-tax cash flow on the sale will (D)
a. exceed the pre-tax cash flow on the sale.
b. be less than the pre-tax cash flow on the sale.
c. be the same as the pre-tax cash flow on the sale.
d. increase the corporation's overall tax liability. Barfield

50. As the marginal tax rate goes up, the benefit from the depreciation tax shield
a. decreases.
b. increases.
c. stays the same. Barfield
d. can move up or down depending on whether the firm's cost of capital is high or low.

Payback Period
In general
6. All other factors equal, a large number is preferred to a smaller number for all capital project evaluation measures except (E)
a. net present value. c. internal rate of return.
b. payback period. d. profitability index. Barfield

26. Risk can be controlled in capital budgeting situations by assuming a:


A. high accounting rate of return C. high net income
B. large net present value D. short payback period CIA adapted

3. In comparing two projects, the _______ is often used to evaluate the relative riskiness of the projects. (D)
a. payback period c. internal rate of return
b. net present value d. discount rate Barfield

Effect of Cash Flow


28. An investment project that requires a present investment of $210,000 will have cash inflows of "R" dollars each year for the next
five years. The project will terminate in five years. Consider the following statements (ignore income tax considerations):
I. If "R" is less than $42,000, the payback period exceeds the life of the project.
II. If "R" is greater than $42,000, the payback period exceeds the life of the project.
III. If "R" equals $42,000, the payback period equals the life of the project. (M)
Which statement(s) is (are) true?
a. Only I and II. c. Only II and III.
b. Only I and III. d. I, II, and III. G & N 9e

Relative Profitability
24. Investment A has a payback period of 5.4 years, investment B one of 6.7 years. From this information we can conclude
a. That investment A has a higher NPV than B.
b. That investment A has a higher IRR than B.
c. That investment A’s book rate of return is higher than B’s.
d. None of the above. L&H

9. If investment A has a payback period of three years and investment B has a payback period of four years, then
a. A is more profitable than B.
b. A is less profitable than B.
c. A and B are equally profitable. Barfield
d. the relative profitability of A and B cannot be determined from the information given.
Accounting Rate of Return
25. Investment A has a book rate of return of 26%, investment B one of 18%. From this information we can conclude
a. That investment A has a higher NPV than B.
b. That investment A has a higher IRR than B.
c. That investment A has a shorter payback period than B.
d. None of the above. L&H

Discount Rate
Effect of Change in Acceptability of Projects
10. All other things being equal, as cost of capital increases
a. More capital projects will probably be acceptable.
b. Fewer capital projects will probably be acceptable.
c. The number of capital projects that are acceptable will change, but the direction of the change is not determinable just by
knowing the direction of the change in cost of capital.
d. The company will probably want to borrow money rather than issue stock. L&H

Effect on Payback Period, Profitability Index & Accounting Rate of Return


*. Payback period (PP), profitability index (PI), and simple accounting rate of return (SARR) are some of the capital budgeting
techniques. What is the effect of an increase in the cost of capital on these techniques? (M)
RPCPA 0594 a. b. c. d.
PP Increase No change No change Decrease
PI Decrease Decrease Increase No change
SARR Increase No change Decrease No change

Present Value Factors


Effect of Discount Rate
30. As the discount rate increases
a. Present value factors increase.
b. Present value factors decrease.
c. Present value factors remain constant.
d. It is impossible to tell what happens to present value factors. L&H

Effect of Length of an Annuity


31. As the length of an annuity increases
a. Present value factors increase.
b. Present value factors decrease.
c. Present value factors remain constant.
d. It is impossible to tell what happen to present value factors. L&H

96. All other things being equal, as the time period for receiving an annuity lengthens,
a. the related present value factors increase.
b. the related present value factors decrease.
c. the related present value factors remain constant. Barfield
d. it is impossible to tell what happens to present value factors from the information given.

Net Present Value


Factors Affecting Net Present Value
54. The after-tax net present value of a project is affected by
a. tax-deductible cash flows. c. accounting accruals.
b. non-tax-deductible cash flows. d. all of the above. Barfield

17. Which of the following events will increase the NPV of an investment involving a new product?
a. An increase in the income tax rate.
b. An increase in the expected per-unit variable cost of the product.
c. An increase in the expected annual unit volume of the product.
d. A decrease in the expected salvage value of equipment. L&H

55. A project's after-tax net present value is increased by all of the following except
a. revenue accruals. c. depreciation deductions.
b. cash inflows. d. expense accruals. Barfield

Change in Discount Rate


24. A change in the discount rate used to evaluate a specific project will affect the project's
a. life. c. net present value.
b. payback period. d. total cash flows. Barfield
141
. The net present value (NPV) of a project has been calculated to be $215,000. Which one of the following changes in assumptions
would decrease the NPV? (M)
a. Decrease the estimated effective income tax rate.
b. Decrease the initial investment amount.
c. Extend the project life and associated cash inflows.
d. Increase the discount rate. CMA 1295 4-7

16. An increase in the discount rate: (M)


a. will increase the present value of future cash flows.
b. will have no effect on net present value.
c. will reduce the present value of future cash flows.
d. is one method of compensating for reduced risk. G & N 9e

22. Which of the following changes would not decrease the present value of the future depreciation deductions on a specific
depreciable asset? (D)
a. a decrease in the marginal tax rate
b. a decrease in the discount rate
c. a decrease in the rate of depreciation
d. an increase in the life expectancy of the depreciable asset Barfield

17. Suppose an investment has cash inflows of R dollars at the end of each year for two years. The present value of these cash
inflows using a 12% discount rate will be: (M)
a. greater than under a 10% discount rate.
b. less than under a 10% discount rate.
c. equal to that under a 10% discount rate. G & N 9e
d. sometimes greater than under a 10% discount rate and sometimes less; it depends on R.

24. A firm is evaluating a project that has a net present value of $0 when a discount rate of 8% is used. A discount rate of 10% will
result in
a. a negative net present value
b. a positive net present value
c. a net present value of $0
d. The question cannot be answered based upon the information provided. H&M

27. A firm is evaluating a project that has a net present value of $0 when a discount rate of 8% is used. A discount rate of 6% will
result in
a. a negative net present value
b. a positive net present value
c. a net present value of $0
d. The question cannot be answered based upon the information provided. H&M

12. Which of the following would decrease the net present value of a project?
A. A decrease in the income tax rate.
B. A decrease in the initial investment.
C. An increase in the useful life of the project.
D. An increase in the discount rate. Pol Bobadilla
142
. Other things held constant, which of the following would increase the NPV of a project being considered? (E)
a. A shift from MACRS to straight-line depreciation.
b. Making the initial investment in the first year rather than spreading it over the first three years.
c. A decrease in the discount rate associated with the project.
d. An increase in required net operating working capital.
e. All of the statements above will increase the project’s NPV. Brigham

*. You have determined the profitability of a planned project by finding the present value of all the cash flows from that project.
Which of the following would cause the project to look less appealing, that is, have a lower present value? (M)
a. The discount rate increases.
b. The cash flows are extended over a longer period of time.
c. The investment cost decreases without affecting the expected income and life of the project. RPCPA 0595
d. The cash flows are accelerated and the project life is correspondingly shortened.

*. Velasquez & Co. is considering an investment proposal for P10 million yielding a net present value of P450,000. The project has a
life of 7 years with salvage value of P200,000. The company uses a discount rate of 12%. Which of the following would decrease
the net present value? (M)
a. Extend the project life and associated cash inflows.
b. Increase discount rate to 15%.
c. Decrease the initial investment amount to P9.0 million.
d. Increase the salvage value. RPCPA 0597
Effect of Salvage Value
34. The salvage value of an old lathe is zero. If instead, the salvage value of the old lathe was $20,000, what would be the impact on
the net present value of the proposal to purchase a new lathe? (M)
a. It would increase the net present value of the proposal.
b. It would decrease the net present value of the proposal.
c. It would not affect the net present value of the proposal.
d. Potentially it could increase or decrease the net present value of the new lathe. Barfield

Timing of Cash Flow Stream


22. Two new products, X and Y, are alike in every way except that the sales of X will start low and rise throughout its life, while those
of Y will be the same each year. Total volumes over their five-year lives will be the same, as will selling prices, unit variable costs,
cash fixed costs, and investment. The NPV of product X
a. Will be less than that of product Y. c. Will be greater than that of product Y.
b. Will be the same as that of product Y. d. None of the above. L&H

Income Tax Rate


6. Which of the following events is most likely to reduce the expected NPV of an investment?
a. The major competitor for the product to be manufactured with the machinery being considered for purchase has been rated
“unsatisfactory” by a consumer group.
b. The interest rate on long-term debt declines.
c. The income tax rate is raised by the Congress.
d. Congress approves the use of faster depreciation than was previously available. L&H

Expected Returns
143
. Stock C has a beta of 1.2, while Stock D has a beta of 1.6. Assume that the stock market is efficient. Which of the following
statements is most correct? (E)
a. The required rates of return of the two stocks should be the same.
b. The expected rates of return of the two stocks should be the same.
c. Each stock should have a required rate of return equal to zero.
d. The NPV of each stock should equal its expected return.
e. The NPV of each stock should equal zero. Brigham

NPV profiles
144
. If the net present value profiles for two mutually exclusive capital projects are shaped as in the graph below, which of the following
statements is true?
$

NPV Profile
for Project 2 Cost of
Capital %

NPV Profile for Project 1


a. Project 2 has a higher internal rate of return that Project 1.
b. Project 1 has a higher internal rate of return than Project 2.
c. Project 1 has a higher net present value than Project 2.
d. Project 2 has a higher net present value than Project 1.

43 Projects A and B have the same expected lives and initial cash outflows. However, one project’s cash flows are larger in the early
years, while the other project has larger cash flows in the later years. The two NPV profiles are given below:

Which of the following statements is most correct? (E)


a. Project A has the smaller cash flows in the later years.
b. Project A has the larger cash flows in the later years.
c. We require information on the cost of capital in order to determine which project has larger early cash flows.
d. The NPV profile graph is inconsistent with the statement made in the problem.
e. None of the statements above is correct. Brigham
145
. Projects A and B both have normal cash flows. In other words, there is an up-front cost followed over time by a series of positive
cash flows. Both projects have the same risk and a WACC equal to 10 percent. However, Project A has a higher internal rate of
return than Project B. Assume that changes in the WACC have no effect on the projects’ cash flow levels. Which of the following
statements is most correct? (E)
a. Project NPV
A must have a higher net present value than Project B.
b. If Project A has a positive NPV, Project B must also have a positive NPV.
c. If Project A’s WACC falls, its internal rate of return will increase.
d. If Projects A andA B have the same NPV at the current WACC, Project B would have a higher NPV if the WACC of both
projects was lower. Brigham
146
. Cherry Books isBconsidering two mutually exclusive projects. Project A has an internal rate of return of 18 percent, while Project B
has an internal rate of return of 30 percent. The two projects have the same risk, the same cost of capital, and the timing of the
cash flows is similar. Each has an up-front cost k followed by a series of positive cash flows. One of the projects, however, is much
larger than the other. If the cost of capital is 16 percent, the two projects have the same net present value (NPV); otherwise, their
NPVs are different. Which of the following statements is most correct? (E)
a. If the cost of capital is 12 percent, Project B will have a higher NPV.
b. If the cost of capital is 17 percent, Project B will have a higher NPV.
c. Project B is larger than Project A.
d. Statements a and c are correct. Brigham

5. Projects L and S each have an initial cost of $10,000, followed by a series of positive cash inflows. Project L has total,
undiscounted cash inflows of $16,000, while S has total undiscounted inflows of $15,000. Further, at a discount rate of 10 percent,
the two projects have identical NPVs. Which project’s NPV will be more sensitive to changes in the discount rate? (Hint: Projects
with steeper NPV profiles are more sensitive to discount rate changes.) (M)
a. Project S.
b. Project L.
c. Both projects are equally sensitive to changes in the discount rate since their NPVs are equal at all costs of capital.
d. Neither project is sensitive to changes in the discount rate, since both have NPV profiles which are horizontal.
e. The solution cannot be determined unless the timing of the cash flows is known. Brigham

6. Two mutually exclusive projects each have a cost of $10,000. The total, undiscounted cash flows from Project L are $15,000, while
the undiscounted cash flows from Project S total $13,000. Their NPV profiles cross at a discount rate of 10 percent. Which of the
following statements best describes this situation? (M)
a. The NPV and IRR methods will select the same project if the cost of capital is greater than 10 percent; for example, 18
percent.
b. The NPV and IRR methods will select the same project if the cost of capital is less than 10 percent; for example, 8 percent.
c. To determine if a ranking conflict will occur between the two projects the cost of capital is needed as well as an additional
piece of information.
d. Project L should be selected at any cost of capital, because it has a higher IRR.
e. Project S should be selected at any cost of capital, because it has a higher IRR. Brigham
147
. A company is comparing two mutually exclusive projects with normal cash flows. Project P has an IRR of 15 percent, while Project
Q has an IRR of 20 percent. If the WACC is 10 percent, the two projects have the same NPV. Which of the following statements
is most correct? (M)
a. If the WACC is 12 percent, both projects would have a positive NPV.
b. If the WACC is 12 percent, Project Q would have a higher NPV than Project P.
c. If the WACC is 8 percent, Project Q would have a lower NPV than Project P.
d. All of the statements above are correct. Brigham
148
. Project C and Project D are two mutually exclusive projects with normal cash flows and the same risk. If the WACC were equal to
10 percent, the two projects would have the same positive NPV. However, if the WACC < 10%, Project C has a higher NPV,
whereas if the WACC > 10%, Project D has a higher NPV. On the basis of this information, which of the following statements is
most correct? (M)
a. Project D has a higher IRR, regardless of the cost of capital.
b. If the WACC < 10%, Project C has a higher IRR.
c. If the WACC < 10%, Project D’s MIRR is less than its IRR.
d. Statements a and c are correct. Brigham

7. Your assistant has just completed an analysis of two mutually exclusive projects. You must now take her report to a board of
directors meeting and present the alternatives for the board’s consideration. To help you with your presentation, your assistant also
constructed a graph with NPV profiles for the two projects. However, she forgot to label the profiles, so you do not know which line
applies to which project. Of the following statements regarding the profiles, which one is most reasonable? (D)
a. If the two projects have the same investment cost, and if their NPV profiles cross once in the upper right quadrant, at a
discount rate of 40 percent, this suggests that a NPV versus IRR conflict is not likely to exist.
b. If the two projects’ NPV profiles cross once, in the upper left quadrant, at a discount rate of minus 10 percent, then there will
probably not be a NPV versus IRR conflict, irrespective of the relative sizes of the two projects, in any meaningful, practical
sense (that is, a conflict which will affect the actual investment decision).
c. If one of the projects has a NPV profile which crosses the X-axis twice, hence the project appears to have two IRRs, your
assistant must have made a mistake.
d. Whenever a conflict between NPV and IRR exist, then, if the two projects have the same initial cost, the one with the steeper
NPV profile probably has less rapid cash flows. However, if they have identical cash flow patterns, then the one with the
steeper profile probably has the lower initial cost. Brigham
e. If the two projects both have a single outlay at t = 0, followed by a series of positive cash inflows, and if their NPV profiles
cross in the lower left quadrant, then one of the projects should be accepted, and both would be accepted if they were not
mutually exclusive.

Cross-Over Rate or Fisher Rate


149
. Project A and Project B are mutually exclusive projects with equal risk. Project A has an internal rate of return of 12 percent, while
Project B has an internal rate of return of 15 percent. The two projects have the same net present value when the cost of capital is
7 percent. (In other words, the “crossover rate” is 7 percent.) Which of the following statements is most correct? (E)
a. If the cost of capital is 10 percent, each project will have a positive net present value.
b. If the cost of capital is 6 percent, Project B has a higher net present value than Project A.
c. If the cost of capital is 13 percent, Project B has a higher net present value than Project A.
d. Statements a and b are correct.
e. Statements a and c are correct. Brigham
150
. Sacramento Paper is considering two mutually exclusive projects. Project A has an internal rate of return (IRR) of 12 percent, while
Project B has an IRR of 14 percent. The two projects have the same risk, and when the cost of capital is 7 percent the projects
have the same net present value (NPV). Assume each project has an initial cash outflow followed by a series of inflows. Given this
information, which of the following statements is most correct? (E)
a. If the cost of capital is 13 percent, Project B’s NPV will be higher than Project A’s NPV.
b. If the cost of capital is 9 percent, Project B’s NPV will be higher than Project A’s NPV.
c. If the cost of capital is 9 percent, Project B’s modified internal rate of return (MIRR) will be less than its IRR.
d. Statements a and c are correct.
e. All of the statements above are correct. Brigham

Profitability Index
*. What is the effect of changes in cash inflows, investment cost and cash outflows on profitability (present value) index (PI) (M)
a. PI will increase with an increase in cash inflows, a decrease in investment cost, or a decrease in cash outflows.
b. PI will increase with an increase in cash inflows, an increase in investment cost, or an increase in cash outflows.
c. PI will decrease with an increase in cash inflows, a decrease in investment cost, or a decrease in cash outflows.
d. PI will decrease with an increase in cash outflows, an increase in investment cost, or an increase in cash inflows. RPCPA
0594

Internal Rate of Return


IRR of Zero
40. If the IRR on an investment is zero,
a. Its NPV is positive.
b. Its annual cash flows equal its required investment.
c. It is generally a wise investment.
d. Its cash flows decrease over its life. L&H

Timing of Cash Flow Stream


151
. Everything else being equal, the internal rate of return (IRR) of an investment project will be lower if (M)
a. The investment cost is lower.
b. Cash inflows are received later in the life of the project.
c. Cash inflows are larger.
d. The project has a shorter payback period. CIA 0595 IV-37

Cash Outflow and Cash Inflow


43. If the total cash inflows associated with a project exceed the total cash outflows associated with the project, the project's (D)
a. net present value is greater than zero.
b. internal rate of return is greater than zero.
c. profitability index is greater than 1.
d. payback period is acceptable. Barfield

NPV and IRR


Change in Sales and Cost of Capital
*. You are engaged by the Baquis Co. to evaluate the introduction of a new product line with an innovative packaging. You
computed the net present value (NPV) and internal rate of return (IRR). If your client would reduce the estimate for its sales of the
new product and increase the projected cost of capital, what would be the impact of these revisions on NPV and IRR? (M)RPCPA
1094
a. NPV will increase, IRR will increase. c. NPV will increase, IRR will decrease.
b. NPV will decrease, IRR will increase. d. NPV will decrease, IRR will decrease.

36. Which of the following factors increase NPV and IRR?


a. An upward revision in expected annual cash flows.
b. An upward revision of expected life.
c. An upward revision of the residual value of the long-lived assets being acquired for the project.
d. All of the above. L&H

NPV, IRR, and MIRR


17 Assume a project has normal cash flows (that is, the initial cash flow is negative, and all other cash flows are positive). Which of
the following statements is most correct? (M)
a. All else equal, a project’s IRR increases as the cost of capital declines.
b. All else equal, a project’s NPV increases as the cost of capital declines.
c. All else equal, a project’s MIRR is unaffected by changes in the cost of capital.
d. Statements a and b are correct. Brigham

NPV & Market Value of Stocks


152
. If a firm identifies (or creates) an investment opportunity with a present value <List A> its cost, the value of the firm and the price of
its common shares will <List B>.
CIA 1195 IV-44 A. B. C. D.
List A Greater than Greater than Equal to Equal to
List B Increase Decrease Increase Decrease

153
. The economic value of the firm will rise following an increase in
A. Net cash flow. C. Unsystematic risk.
B. Systematic risk. D. The discount rate. CIA 0591 IV-47

Comprehensive
33. Which of the following is true of an investment?
a. The lower the cost of capital, the higher the NPV.
b. The lower the cost of capital, the higher the IRR.
c. The longer the project’s life, the shorted its payback period.
d. The higher the project’s NPV, the shorter its life. L&H
154
. Lieber Technologies is considering two potential projects, Project X and Project Y. In assessing the projects’ risk, the company has
estimated the beta of each project and has also conducted a simulation analysis. Their efforts have produced the following
numbers:
Project X Project Y
Expected NPV $350,000 $350,000
Standard deviation (NPV) $100,000 $150,000
Estimated project beta 1.4 0.8
Estimated correlation of Cash flows are not highly Cash flows are highly
project’s cash flows with the correlated with the cash correlated with the cash
cash flows of the Company’s flows of the existing flows of the existing
existing projects. projects. projects.

Which of the following statements is most correct? (E)


a. Project X has a higher level of stand-alone risk relative to Project Y.
b. Project X has a higher level of corporate risk relative to Project Y.
c. Project X has a higher level of market risk relative to Project Y.
d. Statements b and c are correct. Brigham

PROJECT EVALUATION & SELECTION


Categories of Projects
5. A firm might categorize its projects into:
A. Cost improvement projects D. Speculative ventures
B. Expansion projects (existing business) E. All of the above
C. New products projects B&M

6. Which of the following types of projects have the highest risk?


A. Speculation ventures C. Expansion of existing business B&M
B. New products D. Cost improvement, (known technology)

7. Which of the following type of projects has the lowest risk?


A. Speculation ventures C. Expansion of existing business
B. New products D. Cost improvement B&M
8. Which of the following type of projects has average risk?
A. Speculation ventures C. Expansion of existing business
B. New products D. Cost improvement B&M

Cost of Capital Consideration


2. If a firm uses the same company cost of capital for evaluating all projects, which of the following is likely?
A. Rejecting good low risk projects C. Both A and B
B. Accepting poor high risk projects D. Neither A nor B B&M

3. If a firm uses the same company cost of capital for evaluating all projects, which of the following is likely?
A. Accepting poor low risk projects. C. Both A and B
B. Rejecting good high risk projects D. Neither A nor B B&M

4. Using the company cost of capital to evaluate a project is:


A. Always correct
B. Always incorrect
C. Correct for projects that are about as risky as the average of the firm's other assets
D. None of the above B&M

Project Analysis
1. Project analysis includes the following procedures:
A. Sensitivity analysis C. Monte Carlo simulation
B. Break-even analysis D. All of the above B&M

Break-even Analysis
16. Firms often calculate a project's break-even sales using book earnings. Generally, break-even sales based on NPV is:
A. Higher than the one calculated using book earnings
B. Lower than the one calculated using book earnings
C. Equal to the one calculated using book earnings
D. None of the above B&M

Decision Tree
32. Which of the following statements applied to decision trees?
A. They are simple to construct and analyze
B. They should include all possible future events and decisions
C. They help the financial manager to assess the value of options to abandon or expand the project
D. All of the above B&M

Simulation Models
2. Simulation models are useful:
A. To understand the project better C. To assess the project risk
B. To forecast expected cash flows D. All of the above B&M

23. Generally, the simulation models for projects are developed using a:
A. Computer C. Pair of dice
B. Roulette wheel D. Pack of cards B&M

28. The hardest and most important part of a simulation is:


A. Simulating the cash flows
B. Specifying the inter-dependencies
C. Specifying probabilities
D. Specifying the numbers on the roulette wheel B&M

20. Which of the following simulation outputs is likely to be most useful and easy to interpret? The output shows the distribution(s) of
the project:
A. Earnings C. Cash flows
B. Internal rate of return D. Profits B&M

21. The following statements about simulation models are true except:
A. Simulation models enable the financial manager to analyze risky projects without estimating the approximate cost of capital
B. Simulation models are complex and expensive to develop
C. Simulation models are specific to the project and every project requires a new simulation model
D. Simulation models usually ignore opportunities to expand or abandon the project B & M

22. The following statements about simulation models are true except:
A. Simulation models enable the financial manager to analyze what would happen if the uncertainty about any of the variables
were reduced
B. Simulation models take into account the interdependencies between different time periods
C. Simulation models are easy to understand and communicate
D. Simulation models enable the financial manager to visualize how outcomes may be affected if the project is modified B & M

Monte Carlo Simulation


19. Monte Carlo simulation is a tool for considering the:
A. Effect of changing one variable on the NPV of the project
B. Effect of changing a limited number of plausible combination of variables on the NPV of the project
C. Effect of changing all possible combinations of variables on the NPV of the project
D. None of the above B&M

24. Monte Carlo simulation is likely to be most useful:


A. If small amounts of funds are at stake
B. If large amounts of funds are at stake
C. If moderate amounts of funds are at stake
D. Regardless of amount at stake B&M

25. Monte Carlo simulation is likely to be most useful:


A. For simple problems
B. For problems of moderate complexity
C. For very complex problems
D. Regardless of the problem's complexity B&M

26. There are three steps involved in Monte Carlo simulations. One of the following is not one of them:
A. Modeling the project C. Modeling the strategy
B. Specifying probabilities D. Simulating the cash flows B&M

27. The following is not among the steps involved in the Monte Carlo method:
A. Modeling the project
B. Specifying the numbers on the roulette wheel
C. Specifying probabilities
D. Simulating the cash flows B&M

29. The pharmaceutical companies have used the following method to analyze investments in R&D (research and development) of
new drugs:
A. Monte Carlo Simulation C. Sensitivity analysis
B. Decision trees D. None of the above B&M

30. The pharmaceutical companies face three types of uncertainty. They are the following except:
A. Scientific and clinical C. Bureaucratic
B. Production and distribution D. Market success B&M

31. According to the simulation model used by Merck and Company, the following types of variables are used in their model except:
A. Research and development
B. Manufacturing variables
C. Marketing variables
D. FDA (Food and Drug Administration) variables B&M

Basis for Decision


3. After the completion of project analysis, the final decision on the project would be from:
A. Sensitivity analysis C. Decision trees
B. Break-even analysis D. NPV B&M
Independent Projects
155
. Moynihan Motors has a cost of capital of 10.25 percent. The firm has two normal projects of equal risk. Project A has an internal
rate of return of 14 percent, while Project B has an internal rate of return of 12.25 percent. Which of the following statements is
most correct? (E)
a. Both projects have a positive net present value.
b. If the projects are mutually exclusive, the firm should always select Project A.
c. If the crossover rate (that is, the rate at which the Project’s NPV profiles intersect) is 8 percent, Project A will have a higher net
present value than Project B.
d. Statements a and b are correct.
e. Statements a and c are correct. Brigham
156
. A company estimates that its weighted average cost of capital (WACC) is 10 percent. Which of the following independent projects
should the company accept? (M)
a. Project A requires an up-front expenditure of $1,000,000 and generates a net present value of $3,200.
b. Project B has a modified internal rate of return of 9.5 percent.
c. Project C requires an up-front expenditure of $1,000,000 and generates a positive internal rate of return of 9.7 percent.
d. Project D has an internal rate of return of 9.5 percent. Brigham
157
. Project A has an internal rate of return of 18 percent, while Project B has an internal rate of return of 16 percent. However, if the
company’s cost of capital (WACC) is 12 percent, Project B has a higher net present value. Which of the following statements is
most correct? (D)
a. The crossover rate for the two projects is less than 12 percent.
b. Assuming the timing of the two projects is the same, Project A is probably of larger scale than Project B.
c. Assuming that the two projects have the same scale, Project A probably has a faster payback than Project B.
d. Statements a and b are correct.
e. Statements b and c are correct. Brigham

Mutually Exclusive Projects


11. Assume that you are comparing two mutually exclusive projects. Which of the following statements is most correct? (M)
a. The NPV and IRR rules will always lead to the same decision unless one or both of the projects are “non-normal” in the sense
of having only one change of sign in the cash flow stream, that is, one or more initial cash outflows (the investment) followed
by a series of cash inflows.
b. If a conflict exists between the NPV and the IRR, the conflict can always be eliminated by dropping the IRR and replacing it
with the MIRR.
c. There will be a meaningful (as opposed to irrelevant) conflict only if the projects’ NPV profiles cross, and even then, only if the
cost of capital is to the left of (or lower than) the discount rate at which the crossover occurs.
d. All of the statements above are correct. Brigham
158
. Jurgensen Medical is considering two mutually exclusive projects with the following characteristics:
 The two projects have the same risk and the same cost of capital.
 Both projects have normal cash flows. Specifically, each has an up-front cost followed by a series of positive cash flows.
 If the cost of capital is 12 percent, Project X’s IRR is greater than its MIRR.
 If the cost of capital is 12 percent, Project Y’s IRR is less than its MIRR.
 If the cost of capital is 10 percent, the two Project’s have the same NPV.
Which of the following statements is most correct? (M)
a. Project X’s IRR is greater than 12 percent.
b. Project Y’s IRR is less than 12 percent.
c. If the cost of capital is 8 percent, Project X has a lower NPV than Project Y.
d. All of the statements above are correct.
e. None of the statements above is correct. Brigham

Ranking methods
159
. Which of the following statements is correct? (M)
a. Because discounted payback takes account of the cost of capital, a project’s discounted payback is normally shorter than its
regular payback.
b. The NPV and IRR methods use the same basic equation, but in the NPV method the discount rate is specified and the
equation is solved for NPV, while in the IRR method the NPV is set equal to zero and the discount rate is found.
c. If the cost of capital is less than the crossover rate for two mutually exclusive projects’ NPV profiles, a NPV/IRR conflict will not
occur.
d. If you are choosing between two projects which have the same life, and if their NPV profiles cross, then the smaller project will
probably be the one with the steeper NPV profile.
e. If the cost of capital is relatively high, this will favor larger, longer-term projects over smaller, shorter-term alternatives because
it is good to earn high rates on larger amounts over longer periods. Brigham
160
. In comparing two mutually exclusive projects of equal size and equal life, which of the following statements is most correct? (M)
a. The project with the higher NPV may not always be the project with the higher IRR.
b. The project with the higher NPV may not always be the project with the higher MIRR.
c. The project with the higher IRR may not always be the project with the higher MIRR.
d. Statements a and c are correct.
e. All of the statements above are correct. Brigham

Ranking conflicts
26 Which of the following statements is most correct? (E)
a. The NPV method assumes that cash flows will be reinvested at the cost of capital while the IRR method assumes
reinvestment at the IRR.
b. The NPV method assumes that cash flows will be reinvested at the risk-free rate while the IRR method assumes reinvestment
at the IRR.
c. The NPV method assumes that cash flows will be reinvested at the cost of capital while the IRR method assumes
reinvestment at the risk-free rate.
d. The NPV method does not consider the inflation premium.
e. The IRR method does not consider all relevant cash flows, and particularly cash flows beyond the payback period. Brigham

Comprehensive
18 Which of the following statements is most correct? (D)
a. When dealing with independent projects, discounted payback (using a payback requirement of 3 or less years), NPV, IRR,
and modified IRR always lead to the same accept/reject decisions for a given project.
b. When dealing with mutually exclusive projects, the NPV and modified IRR methods always rank projects the same, but those
rankings can conflict with rankings produced by the discounted payback and the regular IRR methods.
c. Multiple rates of return are possible with the regular IRR method but not with the modified IRR method, and this fact is one
reason given by the textbook for favoring MIRR (or modified IRR) over IRR.
d. Statements a and c are correct.
e. None of the statements above is correct. Brigham

Decision-Making
161
. Which of the following statements is correct? (D)
a. There can never be a conflict between NPV and IRR decisions if the decision is related to a normal, independent project, that
is, NPV will never indicate acceptance if IRR indicates rejection.
b. To find the MIRR, we first compound CFs at the regular IRR to find the TV, and then we discount the TV at the cost of capital
to find the PV.
c. The NPV and IRR methods both assume that cash flows are reinvested at the cost of capital. However, the MIRR method
assumes reinvestment at the MIRR itself.
d. If you are choosing between two projects which have the same cost, and if their NPV profiles cross, then the project with the
higher IRR probably has more of its cash flows coming in the later years. Brigham
e. A change in the cost of capital would normally change both a project’s NPV and its IRR.
162
. In an operational audit of the finance department, the auditor observed that the department always used proper quantitative
techniques based on sound economic assumptions to evaluate proposed alternative capital investments. However, management
did not always choose the investment option with the most favorable quantitative assessment. In fact, sometimes management
opted for what appeared to be the third or fourth most favorable investment. The chief financial officer indicated that management
ultimately makes a subjective decision as to which investment is best regardless of which investment option looks best according
to the quantitative analysis. Which of the following statements is most accurate?
A. The approach is justifiable if the economic results of capital investments are highly uncertain.
B. The approach is an irrational, intuitive decision process.
C. The approach results in the organization not maximizing its profits.
D. The approach is an example of the bounded rationality model of decision making whereby managers simplify problems.
CIA 1195 II-4

COMPREHENSIVE
*. In capital budgeting decision, the following are relevant statements except: (E)
a. Since resources are scarce, all capital expenditures must be ranked according to priority.
b. The company must be able to define what falls under this category, whether they are for expansion, for replacements, or for
improvements in operations.
c. Capital investments are short-term commitments of resources, and they are decided in the same process as operating
expenses/
d. A careful analysis of the economic and non-economic reasons or justifications for these investments must be made to arrive at
the appropriate decision. RPCPA 0593

*. Which of the following statements is correct? (E)


a. One key shortcoming of discounted cash flow method is that they ignore the recovery of original investment.
b. Although a cash outlay for noncurrent asset such as a machine would be considered in a capital budgeting analysis, a cash
outlay for working capital item such as inventory would not be considered.
c. To be acceptable, a project’s time adjusted rate of return cannot be less than the company’s cost of capital.
d. If the net present value of an investment is zero, then the project should be rejected since it is not providing any return on
investment. RPCPA 1095
*. Which of the following statements is False? (M)
a. The net present value (NPV) of a project with cash flows that come in relatively slowly is more sensitive to changes in the
discount rate than is the NPV of a project with cash flows that come in rapidly.
b. Other things held constant, a decrease in the cost of capital (discount rate) will cause an increase in a project’s internal rate of
return.
c. The IRR method can be used in place of the NPV method for all independent projects because the two methods then result in
identical decisions.
d. The NPV method is preferred over the IRR method because the NPV method’s reinvestment rate assumption is the correct
assumption. RPCPA 0595
163
. Which of the following is most correct? (M)
a. The NPV and IRR rules will always lead to the same decision in choosing between mutually exclusive projects, unless one or
both of the projects are “non-normal” in the sense of having only one change of sign in the cash flow stream.
b. The Modified Internal Rate of Return (MIRR) compounds cash outflows at the cost of capital.
c. Conflicts between NPV and IRR rules arise in choosing between two mutually exclusive projects (that each have normal cash
flows) when the cost of capital exceeds the crossover point (that is, the point at which the NPV profiles cross).
d. The discounted payback method overcomes the problems that the payback method has with cash flows occurring after the
payback period.
e. None of the statements above is correct. Brigham
164
. Which of the following statements is most correct? (M)
a. The IRR method is appealing to some managers because it produces a rate of return upon which to base decisions rather
than a dollar amount like the NPV method.
b. The discounted payback method solves all the problems associated with the payback method.
c. For independent projects, the decision to accept or reject will always be the same using either the IRR method or the NPV
method.
d. Statements a and c are correct.
e. All of the statements above are correct. Brigham
165
. Which of the following statements is most correct? (M)
a. One of the disadvantages of choosing between mutually exclusive projects on the basis of the discounted payback method is
that you might choose the project with the faster payback period but with the lower total return.
b. Multiple IRRs can occur in cases when project cash flows are normal, but they are more common in cases where project cash
flows are nonnormal.
c. When choosing between mutually exclusive projects, managers should accept all projects with IRRs greater than the weighted
average cost of capital.
d. Statements a and b are correct.
e. All of the statements above are correct. Brigham
166
. Normal projects C and D are mutually exclusive. Project C has a higher net present value if the WACC is less than 12 percent,
whereas Project D has a higher net present value if the WACC exceeds 12 percent. Which of the following statements is most
correct? (M)
a. Project D has a higher internal rate of return.
b. Project D is probably larger in scale than Project C.
c. Project C probably has a faster payback.
d. Statements a and c are correct.
e. All of the statements above are correct. Brigham

MODIFIED ACCELERATED COST RECOVERY SYSTEM (MACRS)


MACRS accelerated depreciation rates should be given for many of these problems. These rates are provided in the text in Appendix
12A.

ACRS
15. The system for recovering the cost of capital expenditures through federal income tax deductions that was required for tangible,
depreciable property placed in service after 1980 is known as:
A. MACRS C. ACRS
B. 200% declining balance D. 150% declining balance Carter & Usry

MACRS
16. Under the Tax Reform Act of 1986, the system that increased the number of property classes and lengthened the recovery periods
of most kinds of depreciable property is known as:
A. MACRS C. ACRS
B. 200% declining balance D. 150% declining balance Carter & Usry

21. When computing depreciation deductions under the MACRS system, taxpayers must: (M)
a. use the half-year convention under which taxpayers are allowed to take only a half year's depreciation in the first year of an
asset's life.
b. use the half-year convention under which taxpayers are allowed to take only a half year's depreciation in the last year of an
asset's life.
c. use the half-year convention under which taxpayers are allowed to take only a half year's depreciation in the first and last
years of an asset's life.
d. calculate depreciation for partial periods using the exact number of days if the asset is acquired at some time other than the
beginning or end of the fiscal year. G & N 9e

20. Under MACRS, the depreciation on tangible personal property is computed as if the property were placed into service at the:
A. beginning of the year C. midpoint of the year
B. end of the year D. midpoint of the month Carter & Usry

21. Under MACRS, the depreciation on real property is computed as if the property were placed into service at the:
A. beginning of the year C. midpoint of the year
B. end of the year D. midpoint of the month Carter & Usry

16. Classifying an asset in a MACRS life category is based on


a. Useful life estimated by the company.
b. Asset depreciation range (ADR) guidelines.
c. The cost of the asset.
d. Any of the above factors. L&H

17. An example of 5-year property under MACRS is:


A. most manufacturing machinery C. commercial aircraft
B. railroad cars D. light trucks Carter & Usry

18. An example of 7-year property under MACRS is:


A. automobiles C. light trucks
B. most manufacturing machinery D. small aircraft Carter & Usry

19. An example of 27.5-year property under MACRS is:


A. residential rental property C. nonresidential buildings
B. commercial aircraft D. railroad cars Carter & Usry

20. With respect to income taxes, the principal advantage of MACRS over straight-line depreciation is that
a. Total taxes will be lower under MACRS.
b. Taxes will be constant from year to year under MACRS.
c. Taxes will be lower in the earlier years under MACRS.
d. Taxes will decline in future years under MACRS. L&H

4. Companies using MACRS for tax purposes and straight-line depreciation for financial reporting purposes usually find that the
relationship between the tax basis and book value of their assets is
a. The tax basis is lower than book value.
b. The tax basis is higher than book value.
c. The tax basis is the same as book value.
d. None of the above. L&H

5. A company that wants to use MACRS for tax purposes must


a. Request permission from the IRS.
b. Acquire new assets to or near the middle of the year.
c. Ignore salvage value in calculating depreciation.
d. Do none of the above. L&H

38. A company evaluates a project using straight-line depreciation over its 10-year estimated useful life and then reevaluates it using a
7-year MACRS class life. The second analysis will show
a. A lower IRR for the project.
b. The same NPV and IRR for the project.
c. A higher NPV for the project.
d. Lower total cash flows over the 10 years. L&H

13. If a company uses a five-year MACRS period to depreciate assets instead of a 10-year life with straight-line depreciation,
a. The NPV of the investment is higher.
b. The IRR of the investment is lower.
c. There is no difference in either NPV or IRR.
d. Total cash flows over the useful life would be lower. L&H
167
. Flex Corporation is studying a capital acquisition proposal in which newly acquired assets will be depreciated using the straight-line
method. Which one of the following statements about the proposal would be incorrect if a switch is made to the Modified
Accelerated Cost Recovery System (MACRS)? CMA 0693 4-29
A. The net present value will increase. C. The payback period will be shortened.
B. The internal rate of return will increase. D. The profitability index will decrease.

. Which of the following statement completions is incorrect? For a profitable firm, when MACRS accelerated depreciation is
168

compared to straight-line depreciation, MACRS accelerated allowances produce (M)


a. Higher depreciation charges in the early years of an asset’s life.
b. Larger cash flows in the earlier years of an asset’s life.
c. Larger total undiscounted profits from the project over the project’s life.
d. Smaller accounting profits in the early years, assuming the company uses the same depreciation method for tax and book
purposes.
e. None of the statements above. (All of the statements above are correct.) Brigham

Questions 74 and 75 are based on the following information. CMA 0694 4-13 & 14
The tax impact of equipment depreciation affects capital budgeting decisions. Currently, the Modified Accelerated Cost Recovery
System (MACRS) is used as the depreciation method for most assets for tax purposes.
169
. The MACRS method of depreciation for assets with 3, 5, 7, and 10-year recovery periods is most similar to which one of the
following depreciation methods used for financial reporting purposes?
A. Straight-line. C. Sum-of-the-years'-digits.
B. Units-of-production. D. 200% declining-balance.
170
. When employing the MACRS method of depreciation in a capital budgeting decision, the use of MACRS as compared with the
straight-line method of depreciation will result in
A. Equal total depreciation for both methods.
B. MACRS producing less total depreciation than straight line.
C. Equal total tax payments, after discounting for the time value of money.
D. MACRS producing more total depreciation than straight line.

MACRS vs. Optional Straight-line Method


18. In a capital budgeting decision, the use of MACRS tables as compared to the optional straight-line method will result in: (M)
a. equal total depreciation for both methods.
b. more total depreciation for the MACRS tables method.
c. more total depreciation for the optional straight-line method. CMA adapted
d. less depreciation for the MACRS tables method in the early years of asset life.

19. The use of the MACRS tables instead of the optional straight-line method of depreciation has the effect of: (M)
a. raising the hurdle rate necessary to justify the project.
b. decreasing the net present value of the project.
c. increasing the present value of the depreciation tax shield.
d. increasing the cash outflows at the beginning of the project. CMA adapted

20. Which of the following is correct? (M)


a. Use of the MACRS tables requires that salvage value be deducted in computing depreciation deductions.
b. Use of the optional straight-line method requires that salvage value not be considered in computing depreciation deductions.
c. The use of both MACRS tables and the optional straight-line method requires that salvage value be deducted in computing
depreciation deductions.
d. None of the above are true. G & N 9e

Taxes on gain on sale


171
. St. John’s Paper is considering purchasing equipment today that has a depreciable cost of $1 million. The equipment will be
depreciated on a MACRS 5-year basis, which implies the following depreciation schedule:
Year MACRS Depreciation Rate
1 0.20
2 0.32
3 0.19
4 0.12
5 0.11
6 0.06
Assume that the company sells the equipment after three years for $400,000 and the company’s tax rate is 40 percent. What
would be the tax consequences resulting from the sale of the equipment? (E)
a. There are no tax consequences.
b. The company would have to pay $44,000 in taxes.
c. The company would have to pay $160,000 in taxes.
d. The company would receive a tax credit of $124,000.
e. The company would receive a tax credit of $48,000. Brigham
Risk-adjusted discount rate
172
. Dick Boe Enterprises, an all-equity firm, has a corporate beta coefficient of 1.5. The financial manager is evaluating a project with
an IRR of 21 percent, before any risk adjustment. The risk-free rate is 10 percent, and the required rate of return on the market is
16 percent. The project being evaluated is riskier than Boe’s average project, in terms of both beta risk and total risk. Which of the
following statements is most correct? (E)
a. The project should be accepted since its IRR (before risk adjustment) is greater than its required return.
b. The project should be rejected since its IRR (before risk adjustment) is less than its required return.
c. The accept/reject decision depends on the risk-adjustment policy of the firm. If the firm’s policy were to reduce a riskier-than-
average project’s IRR by 1 percentage point, then the project should be accepted.
d. Riskier-than-average projects should have their IRRs increased to reflect their added riskiness. Clearly, this would make the
project acceptable regardless of the amount of the adjustment.
e. Projects should be evaluated on the basis of their total risk alone. Thus, there is insufficient information in the problem to make
an accept/reject decision. Brigham

Risk-adjusted discount rate


173
. The Unlimited, a national retailing chain, is considering an investment in one of two mutually exclusive projects. The discount rate
used for Project A is 12 percent. Further, Project A costs $15,000, and it would be depreciated using MACRS. It is expected to
have an after-tax salvage value of $5,000 at the end of 6 years and to produce after-tax cash flows (including depreciation) of
$4,000 for each of the 6 years. Project B costs $14,815 and would also be depreciated using MACRS. B is expected to have a
zero salvage value at the end of its 6-year life and to produce after-tax cash flows (including depreciation) of $5,100 each year for
6 years. The Unlimited’s marginal tax rate is 40 percent. What risk-adjusted discount rate will equate the NPV of Project B to that
of Project A? (M)
a. 15% d. 20%
b. 16% e. 12%
c. 18% Brigham
174
. California Mining is evaluating the introduction of a new ore production process. Two alternatives are available. Production
Process A has an initial cost of $25,000, a 4-year life, and a $5,000 net salvage value, and the use of Process A will increase net
cash flow by $13,000 per year for each of the 4 years that the equipment is in use. Production Process B also requires an initial
investment of $25,000, will also last 4 years, and its expected net salvage value is zero, but Process B will increase net cash flow
by $15,247 per year. Management believes that a risk-adjusted discount rate of 12 percent should be used for Process A. If
California Mining is to be indifferent between the two processes, what risk-adjusted discount rate must be used to evaluate B? (D)
a. 8% d. 14%
b. 10% e. 16%
c. 12% Brigham

New project NPV


175
. Given the following information, calculate the NPV of a proposed project: Cost = $4,000; estimated life = 3 years; initial decrease in
accounts receivable = $1,000, which must be restored at the end of the project’s life; estimated salvage value = $1,000; earnings
before taxes and depreciation = $2,000 per year; method of depreciation = MACRS; tax rate = 40 percent; and cost of capital = 18
percent. (MACRS table required) (M)
a. $1,137 d. $ 804
b. -$ 151 e. $ 544
c. $ 137 Brigham
176
. Mars Inc. is considering the purchase of a new machine that will reduce manufacturing costs by $5,000 annually. Mars will use the
MACRS accelerated method to depreciate the machine, and it expects to sell the machine at the end of its 5-year operating life for
$10,000. The firm expects to be able to reduce net operating working capital by $15,000 when the machine is installed, but
required net operating working capital will return to the original level when the machine is sold after 5 years. Mars’ marginal tax
rate is 40 percent, and it uses a 12 percent cost of capital to evaluate projects of this nature. If the machine costs $60,000, what is
the project’s NPV? (MACRS table required) (M)
a. -$15,394 d. -$21,493
b. -$14,093 e. -$46,901
c. -$58,512 Brigham
177
. Stanton Inc. is considering the purchase of a new machine that will reduce manufacturing costs by $5,000 annually and increase
earnings before depreciation and taxes by $6,000 annually. Stanton will use the MACRS method to depreciate the machine, and it
expects to sell the machine at the end of its 5-year operating life for $10,000 before taxes. Stanton’s marginal tax rate is 40
percent, and it uses a 9 percent cost of capital to evaluate projects of this type. If the machine’s cost is $40,000, what is the
project’s NPV? (MACRS table required) (M)
a. $1,014 d. $ 817
b. $2,292 e. $5,040
c. $7,550 Brigham
178
. Maple Media is considering a proposal to enter a new line of business. In reviewing the proposal, the company’s CFO is
considering the following facts:
 The new business will require the company to purchase additional fixed assets that will cost $600,000 at t = 0. For tax
and accounting purposes, these costs will be depreciated on a straight-line basis over three years. (Annual depreciation will
be $200,000 per year at
t = 1, 2, and 3.)
 At the end of three years, the company will get out of the business and will sell the fixed assets at a salvage value of
$100,000.
 The project will require a $50,000 increase in net operating working capital at t = 0, which will be recovered at t = 3.
 The company’s marginal tax rate is 35 percent.
 The new business is expected to generate $2 million in sales each year (at t = 1, 2, and 3). The operating costs excluding
deprecia-tion are expected to be $1.4 million per year.
 The project’s cost of capital is 12 percent.
What is the project’s net present value (NPV)? (M)
a. $536,697 d. $ 56,331
b. $ 86,885 e. $561,609
c. $ 81,243 Brigham
179
. MacDonald Publishing is considering entering a new line of business. In analyzing the potential business, their financial staff has
accumulated the following information:
 The new business will require a capital expenditure of $5 million at t = 0. This expenditure will be used to purchase new
equipment.
 This equipment will be depreciated according to the following depreciation schedule:
Year MACRS Depreciation Rate
1 0.33
2 0.45
3 0.15
4 0.07
 The equipment will have no salvage value after four years.
 If MacDonald goes ahead with the new business inventories will rise by $500,000 at t = 0, and its accounts payable will rise by
$200,000 at t = 0. This increase in net operating working capital will be recovered at t = 4.
 The new business is expected to have an economic life of four years. The business is expected to generate sales of $3 million
at t = 1, $4 million at t = 2, $5 million at t = 3, and $2 million at t = 4. Each year, operating costs excluding depreciation are
expected to be 75 percent of sales.
 The company’s tax rate is 40 percent.
 The company’s weighted average cost of capital is 10 percent.
 The company is very profitable, so any accounting losses on this project can be used to reduce the company’s overall tax
burden.
What is the expected net present value (NPV) of the new business? (M)
a. $ 740,298 d. -$1,961,833
b. -$1,756,929 e. –$5,919,974
c. -$1,833,724 Brigham
180
. Rio Grande Bookstores is considering a major expansion of its business. The details of the proposed expansion project are
summarized below:
 The company will have to purchase $500,000 in equipment at t = 0. This is the depreciable cost.
 The project has an economic life of four years.
 The cost can be depreciated on a MACRS 3-year basis, which implies the following depreciation schedule:
Year MACRS Depreciation Rate
1 0.33
2 0.45
3 0.15
4 0.07
 At t = 0, the project requires that inventories increase by $50,000 and accounts payable increase by $10,000. The change in
net operating working capital is expected to be fully recovered at t = 4.
 The project’s salvage value at the end of four years is expected to be $0.
 The company forecasts that the project will generate $800,000 in sales the first two years (t = 1 and 2) and $500,000 in sales
during the last two years (t = 3 and 4).
 Each year the project’s operating costs excluding depreciation are expected to be 60 percent of sales revenue.
 The company’s tax rate is 40 percent.
 The project’s cost of capital is 10 percent.
What is the net present value (NPV) of the proposed project? (M)
a. $159,145 d. $150,776
b. $134,288 e. -$257,060
c. $162,817 Brigham
181
. Foxglove Corp. is faced with an investment project. The following information is associated with this project:
Year Net Income* MACRS Depreciation Rate
1 $50,000 0.33
2 60,000 0.45
3 70,000 0.15
4 60,000 0.07
*Assume no interest expenses and a zero tax rate.
The project involves an initial investment of $100,000 in equipment that falls in the 3-year MACRS class and has an estimated
salvage value of $15,000. In addition, the company expects an initial increase in net operating working capital of $5,000 that will be
recovered in Year 4. The cost of capital for the project is 12 percent. What is the project’s net present value? (Round your final
answer to the nearest whole dollar.) (D)
a. $153,840 d. $168,604
b. $159,071 e. $182,344
c. $162,409 Brigham
182
. Pierce Products is deciding whether it makes sense to purchase a new piece of equipment. The equipment costs $100,000
(payable at t = 0). The equipment will provide before-tax cash inflows of $45,000 a year at the end of each of the next four years (t
= 1, 2, 3, and 4). The equipment can be depreciated according to the following schedule:
Year MACRS Depreciation Rate
1 0.33
2 0.45
3 0.15
4 0.07
At the end of four years the company expects to be able to sell the equipment for an after-tax salvage value of $10,000. The
company is in the 40 percent tax bracket. The company has an after-tax cost of capital of 11 percent. Because there is more
uncertainty about the salvage value, the company has chosen to discount the salvage value at 12 percent. What is the net present
value (NPV) of purchasing the equipment? (D)
a. $ 9,140.78 d. $22,853.90
b. $16,498.72 e. $28.982.64
c. $20,564.23 Brigham
183
. Lugar Industries is considering an investment in a proposed project that requires an initial expenditure of $100,000 at t = 0. This
expenditure can be depreciated at the following annual rates:
Year MACRS Depreciation Rate
1 0.20
2 0.32
3 0.19
4 0.12
5 0.11
6 0.06
The project has an economic life of six years. The project’s revenues are forecasted to be $90,000 a year. The project’s operating
costs (not including depreciation) are forecasted to be $50,000 a year. After six years, the project’s estimated salvage value is
$10,000. The company’s WACC is 10 percent, and its corporate tax rate is 40 percent. What is the project’s net present value
(NPV)? (D)
a. $31,684 d. $38,840
b. $33,843 e. $45,453
c. $34,667 Brigham
184
. Mills Mining is considering an expansion project. The proposed project has the following features:
 The project has an initial cost of $500,000. This is also the amount that can be depreciated using the following
depreciation schedule:
Year MACRS Depreciation Rate
1 0.33
2 0.45
3 0.15
4 0.07
 If the project is undertaken, at t = 0 the company will need to increase its inventories by $50,000, and its accounts
payable will rise by $10,000. This net operating working capital will be recovered at the end of the project’s life (t = 4).
 If the project is undertaken, the company will realize an additional $600,000 in sales over each of the next four years (t =
1, 2, 3, and 4). The company’s operating cost (not including depreciation) will equal $400,000 a year.
 The company’s tax rate is 40 percent.
 At t = 4, the project’s economic life is complete, but it will have a salvage value of $50,000.
 The project’s WACC = 10 percent.
What is the project’s net present value (NPV)? (D)
a. $11,122.87 d. $68,336.86
b. $50,330.14 e. $80,035.52
c. $54,676.59 Brigham

Risk-adjusted NPV
185
. Virus Stopper Inc., a supplier of computer safeguard systems, uses a cost of capital of 12 percent to evaluate average-risk
projects, and it adds or subtracts 2 percentage points to evaluate projects of more or less risk. Currently, two mutually exclusive
projects are under consideration. Both have a cost of $200,000 and will last 4 years. Project A, a riskier-than-average project, will
produce annual end-of-year cash flows of $71,104. Project B, a less-than-average-risk project, will produce cash flows of $146,411
at the end of Years 3 and 4 only. Virus Stopper should accept(M)
a. B with a NPV of $10,001.
b. Both A and B because both have NPVs greater than zero.
c. B with a NPV of $8,042.
d. A with a NPV of $7,177.
e. A with a NPV of $15,968. Brigham
186
. An all-equity firm is analyzing a potential project that will require an initial, after-tax cash outlay of $50,000 and after-tax cash
inflows of $6,000 per year for 10 years. In addition, this project will have an after-tax salvage value of $10,000 at the end of Year
10. If the risk-free rate is 6 percent, the return on an average stock is 10 percent, and the beta of this project is 1.50, what is the
project’s NPV? (M)
a. $13,210 d. -$ 6,158
b. $ 4,905 e. -$12,879
c. $ 7,121 Brigham
187
. Real Time Systems Inc. is considering the development of one of two mutually exclusive new computer models. Each will require a
net investment of $5,000. The cash flows for each project are shown below:
Year Project A Project B
1 $2,000 $3,000
2 2,500 2,600
3 2,250 2,900
Model B, which will use a new type of laser disk drive, is considered a high-risk project, while Model A is an average-risk project.
Real Time adds 2 percentage points to arrive at a risk-adjusted cost of capital when evaluating high-risk projects. The cost of
capital used for average-risk projects is 12 percent. Which of the following statements regarding the NPVs for Models A and B is
most correct? (M)
a. NPVA = $380; NPVB = $1,815 c. NPVA = $380; NPVB = $1,590
b. NPVA = $197; NPVB = $1,590 d. NPVA = $5,380; NPVB = $6,590 Brigham

NPV and risk-adjusted discount rate


188
. Garcia Paper is deciding whether to build a new plant. The proposed project would have an up-front cost (at t = 0) of $30 million.
The project’s cost can be depreciated on a straight-line basis over three years. Consequently, the depreciation expense will be $10
million in each of the first three years, t = 1, 2, and 3. Even though the project is depreciated over three years, the project has an
economic life of five years.
The project is expected to increase the company’s sales by $20 million. Sales will remain at this higher level for each year of the
project (t = 1, 2, 3, 4, and 5). The operating costs, not including depreciation, equal 60 percent of the increase in annual sales. The
project’s interest expense is $5 million per year and the company’s tax rate is 40 percent. The company is very profitable, so any
accounting losses on this project can be used to reduce the company’s overall tax burden. The project does not require any
additions to net operating working capital. The company estimates that the project’s after-tax salvage value at t = 5 will be $1.2
million. The project is of average risk, and, therefore, the CFO has decided to discount the operating cash flows at the company’s
overall WACC of 10 percent. However, the salvage value is more uncertain, so the CFO has decided to discount it at 12 percent.
What is the net present value (NPV) of the proposed project? (D)
a. $11.86 million d. -$12.55 million
b. $14.39 million e. -$ 1.18 million
c. -$26.04 million Brigham

Discounting risky outflows


189
. Alabama Pulp Company (APC) can control its environmental pollution using either “Project Old Tech” or “Project New Tech.” Both
will do the job, but the actual costs involved with Project New Tech, which uses unproved, new state-of-the-art technology, could
be much higher than the expected cost levels. The cash outflows associated with Project Old Tech, which uses standard proven
technology, are less risky. (They are about as uncertain as the cash flows associated with an average project.) APC’s cost of
capital for average-risk projects is normally set at 12 percent, and the company adds 3 percent for high-risk projects but subtracts
3 percent for low-risk projects. The two projects in question meet the criteria for high and average risk, but the financial manager is
concerned about applying the normal rule to such cost-only projects. You must decide which project to recommend, and you
should recommend the one with the lower PV of costs. What is the PV of costs of the better project? (M)
Cash Outflows
Years: 0 1 2 3 4
Project New Tech 1,500 315 315 315 315
Project Old Tech 600 600 600 600 600
a. 2,521 d. 2,543
b. 2,399 e. 2,422
c. 2,457 Brigham

Risky projects
190
. Cochran Corporation has a weighted average cost of capital of 11 percent for projects of average risk. Projects of below-average
risk have a cost of capital of 9 percent, while projects of above-average risk have a cost of capital equal to 13 percent. Projects A
and B are mutually exclusive, whereas all other projects are independent. None of the projects will be repeated. The following table
summarizes the cash flows, internal rate of return (IRR), and risk of each of the projects.
Year Project A Project B Project C Project D Project E
0 -$200,000 -$100,000 -$100,000 -$100,000 -$100,000
1 66,000 30,000 30,000 30,000 40,000
2 66,000 30,000 30,000 30,000 25,000
3 66,000 40,000 30,000 40,000 30,000
4 66,000 40,000 40,000 50,000 35,000

IRR 12.110% 14.038% 10.848% 16.636% 11.630%


Project Below Below Average Above Above
Risk Average Average Average Average
Which projects will the firm select for investment? (M)
a. Projects: A, B, C, D, E d. Projects: A, D
b. Projects: B, C, D, E e. Projects: B, C, D
c. Projects: B, D Brigham

Scenario analysis
191
. Klott Company encounters significant uncertainty with its sales volume and price in its primary product. The firm uses scenario
analysis in order to determine an expected NPV, which it then uses in its budget. The base-case, best-case, and worst-case
scenarios and probabilities are provided in the table below. What is Klott’s expected NPV, standard deviation of NPV, and
coefficient of variation of NPV? (M)
Probability of Unit Sales Sales Price NPV
Outcome Volume (In Thousands)
Worst case 0.30 6,000 $3,600 -$6,000
Base case 0.50 10,000 4,200 +13,000
Best case 0.20 13,000 4,400 +28,000
a. Expected NPV = $35,000; σNPV = 17,500; CVNPV = 2.00
b. Expected NPV = $35,000; σNPV = 11,667; CVNPV = 0.33
c. Expected NPV = $10,300; σNPV = 12,083; CVNPV = 1.17
d. Expected NPV = $13,900; σNPV = 8,476; CVNPV = 0.61
e. Expected NPV = $10,300; σNPV = 13,900; CVNPV = 1.35 Brigham

Questions 50 thru 53 are based on the following information Brigham


The president of Real Time Inc. has asked you to evaluate the proposed acquisition of a new computer. The computer’s price is
$40,000, and it falls into the MACRS 3-year class. Purchase of the computer would require an increase in net operating working capital
of $2,000. The computer would increase the firm’s before-tax revenues by $20,000 per year but would also increase operating costs by
$5,000 per year. The computer is expected to be used for three years and then be sold for $25,000. The firm’s marginal tax rate is 40
percent, and the project’s cost of capital is 14 percent. (MACRS table required)
192
. What is the net investment required at t = 0? (E)
a. -$42,000 d. -$37,600
b. -$40,000 e. -$36,600
c. -$38,600
193
. What is the operating cash flow in Year 2? (M)
a. $ 9,000 d. $13,453
b. $10,240 e. $16,200
c. $11,687
194
. What is the total value of the terminal year non-operating cash flows at the end of Year 3? (M)
a. $18,120 d. $25,000
b. $19,000 e. $27,000
c. $21,000
195
. What is the project’s NPV? (M)
a. $2,622 d. $5,712
b. $2,803 e. $6,438
c. $2,917

Questions 54 thru 57 are based on the following information. Brigham


You have been asked by the president of your company to evaluate the proposed acquisition of a new special-purpose truck. The
truck’s basic price is $50,000, and it will cost another $10,000 to modify it for special use by your firm. The truck falls into the MACRS
3-year class, and it will be sold after three years for $20,000. Use of the truck will require an increase in net operating working capital
(spare parts inventory) of $2,000. The truck will have no effect on revenues, but it is expected to save the firm $20,000 per year in
before-tax operating costs, mainly labor. The firm’s marginal tax rate is 40 percent. (MACRS table required)
196
. What is the net investment in the truck? (That is, what is the Year 0 net cash flow?) (E)
a. -$50,000 d. -$62,000
b. -$52,600 e. -$65,000
c. -$55,800
197
. What is the operating cash flow in Year 1? (M)
a. $17,820 d. $20,121
b. $18,254 e. $21,737
c. $19,920
198
. What is the total value of the terminal year non-operating cash flows at the end of Year 3? (M)
a. $10,000 d. $16,000
b. $12,000 e. $18,000
c. $15,680
199
. The truck’s cost of capital is 10 percent. What is its NPV? (M)
a. -$1,547 d. $ 562
b. -$ 562 e. $1,034
c. $ 0

ANSWER EXPLANATIONS
1
.Answer (D) is correct. Capital budgeting is the process of planning expenditures for long-lived assets. It involves
choosing among investment proposals using a ranking procedure. Evaluations are based on various measures involving
rate of return on investment.
Answer (A) is incorrect because capital budgeting involves long-term investment needs, not immediate operating needs.
Answer (B) is incorrect because strategic planning establishes long-term goals in the context of relevant factors in the
firm's environment. Answer (C) is incorrect because cash budgeting determines operating cash flows. Capital budgeting
evaluates the rate of return on specific investment alternatives.

NPV
$

B
A

Expected increase in annual net income k


ARR = 0 10%
Initial (or average) investment IRRB IRRA

NPV
($)

Discount rate (%)


0 16% 17% 18% 30%

NPV
$
NPV ($)
P
C

k 10% k
10% 15% 20% %
NPV NPV
$

A A

B
B n
Sigma CF t
k k t=1
0 7% 12% 15% 0 7% 12% 14%
(1 + K)t
NPV
$

X
Crossover

k
10% IRRY 12% IRRX

0 k = 12% 1 2 3 4 5 6

CFsA -15,000 4,000 4,000 4,000 4,000 4,000 4,000


Project B:
0 k = ? 1 2 6 Years
  
CFsB -14,815 5,100 5,100 5,100
Time lines:
Project A:
0 1 2 3 4 Years
k = 12%

CFsA -25,000 13,000 13,000 13,000 13,000


NPVA = ? = 17,663 Terminal value = 5,000
CF 4 = 18,000
Project B:
0 1 2 3 4 Years
k = ?

CFsB -25,000 15,247 15,247 15,247 15,247


NPVA = NPVB = 17,663 Terminal value = 0
CF 4 = 15,247
Time line:
0 k = 18% 1 2 3 Years

-3,000 1,728 1,920 1,152


Time line:
0 1 2 3 4 5 Years
k = 12%

-45,000 7,800 10,680 7,560 5,880 -1,920


Time line:
0 1 2 3 4 5 Years
k = 9%

-40,000 9,800 11,720 9,640 8,520 15,320


Time lines:
Project A:
0 k = 14% 1 2 3 4 Years

CFsA -200,000 71,104 71,104 71,104 71,104


Project B:
0 k = 10% 1 2 3 4 Years

CFsB -200,000 0 0 146,411 146,411


Time line:
0 k = 12% 1 2 10 Years
  
-50,000 6,000 6,000 6,000
Time lines:
Project A:
0 k = 12% 1 2 3 Periods

CFsA -5,000 2,000 2,500 2,250


Project B:
0 k = 14% 1 2 3 Periods

CFsB -5,000 3,000 2,600 2,900


0 1 2 3 4
| | | | |
CFsNew Tech -1,500 -315 -315 -315 -315 x x x
Time line:
0 1 2 3 Years
k = 14%

-42,000 14,280 16,200 11,400


TV = 18,120
29,520
Time line:
0 1 2 3 Years
k = 10%

-62,000 19,920 22,800 15,600


TV = 15,680
31,280
2
.Answer (D) is correct. Capital budgeting is a long-term planning process for investments. This process begins with the
identification of capital needs, that is, of projects required to achieve organizational goals. The next step is to search for
specific investments. The third step is to acquire and analyze information about the potential choices. The fourth step is
to select specific investments after considering both qualitative and quantitative factors. The fifth step is to finance the
undertakings. The final step is implementation and monitoring.
Answer (A) is incorrect because analyzing capital addition proposals is a step that is subsequent to identifying capital
addition projects and other capital needs. Answer (B) is incorrect because making expenditure decisions is a step
subsequent to identifying capital addition projects and other capital needs. Answer (C) is incorrect because analyzing
and evaluating all promising alternatives is a step that is subsequent to identifying capital addition projects and other
capital needs.
NPV
$

B
A

Expected increase in annual net income k


ARR = 0 10%
Initial (or average) investment IRRB IRRA

NPV
($)

Discount rate (%)


0 16% 17% 18% 30%

NPV
$
NPV ($)
P
C

k 10% k
10% 15% 20% %

NPV NPV
$

A A

B
B n
Sigma CF t
k k t=1
0 7% 12% 15% 0 7% 12% 14%
(1 + K)t
NPV
$

X
Crossover

k
10% IRRY 12% IRRX

0 k = 12% 1 2 3 4 5 6

CFsA -15,000 4,000 4,000 4,000 4,000 4,000 4,000


Project B:
0 k = ? 1 2 6 Years
  
CFsB -14,815 5,100 5,100 5,100
Time lines:
Project A:
0 1 2 3 4 Years
k = 12%

CFsA -25,000 13,000 13,000 13,000 13,000


NPVA = ? = 17,663 Terminal value = 5,000
CF 4 = 18,000
Project B:
0 1 2 3 4 Years
k = ?

CFsB -25,000 15,247 15,247 15,247 15,247


NPVA = NPVB = 17,663 Terminal value = 0
CF 4 = 15,247
Time line:
0 1 2 3 Years
k = 18%

-3,000 1,728 1,920 1,152


Time line:
0 1 2 3 4 5 Years
k = 12%

-45,000 7,800 10,680 7,560 5,880 -1,920


Time line:
0 k = 9% 1 2 3 4 5 Years

-40,000 9,800 11,720 9,640 8,520 15,320


Time lines:
Project A:
0 k = 14% 1 2 3 4 Years

CFsA -200,000 71,104 71,104 71,104 71,104


Project B:
0 k = 10% 1 2 3 4 Years

CFsB -200,000 0 0 146,411 146,411


Time line:
0 k = 12% 1 2 10 Years
  
-50,000 6,000 6,000 6,000
Time lines:
Project A:
0 k = 12% 1 2 3 Periods

CFsA -5,000 2,000 2,500 2,250


Project B:
0 k = 14% 1 2 3 Periods

CFsB -5,000 3,000 2,600 2,900


0 1 2 3 4
| | | | |
CFsNew Tech -1,500 -315 -315 -315 -315 x x x
Time line:
0 1 2 3 Years
k = 14%

-42,000 14,280 16,200 11,400


TV = 18,120
29,520
Time line:
0 1 2 3 Years
k = 10%

-62,000 19,920 22,800 15,600


TV = 15,680
31,280
3
.Answer (C) is correct. Capital budgeting is concerned with long-range decisions, such as whether to add a product line,
to build new facilities, or to lease or buy equipment. Any decision regarding cash inflows and outflows over a period of
more than 1 year probably needs capital budgeting analysis.
Answer (A) is incorrect because capital budgeting is useful for all long-range decision making. Answer (B) is incorrect
because capital budgeting is not useful for short-range decisions. Answer (D) is incorrect because it is a nonsense
answer.

NPV
$

B
A

Expected increase in annual net income k


ARR = 0 10%
Initial (or average) investment IRRB IRRA

NPV
($)

Discount rate (%)


0 16% 17% 18% 30%
NPV
$
NPV ($)
P
C

k 10% k
10% 15% 20% %

NPV NPV
$

A A

B
B n
Sigma CF t
k k t=1
0 7% 12% 15% 0 7% 12% 14%
(1 + K)t
NPV
$

X
Crossover

k
10% IRRY 12% IRRX

0 k = 12% 1 2 3 4 5 6

CFsA -15,000 4,000 4,000 4,000 4,000 4,000 4,000


Project B:
0 k = ? 1 2 6 Years
  
CFsB -14,815 5,100 5,100 5,100
Time lines:
Project A:
0 1 2 3 4 Years
k = 12%

CFsA -25,000 13,000 13,000 13,000 13,000


NPVA = ? = 17,663 Terminal value = 5,000
CF 4 = 18,000
Project B:
0 1 2 3 4 Years
k = ?

CFsB -25,000 15,247 15,247 15,247 15,247


NPVA = NPVB = 17,663 Terminal value = 0
CF 4 = 15,247
Time line:
0 k = 18% 1 2 3 Years

-3,000 1,728 1,920 1,152


Time line:
0 1 2 3 4 5 Years
k = 12%

-45,000 7,800 10,680 7,560 5,880 -1,920


Time line:
0 1 2 3 4 5 Years
k = 9%

-40,000 9,800 11,720 9,640 8,520 15,320


Time lines:
Project A:
0 k = 14% 1 2 3 4 Years

CFsA -200,000 71,104 71,104 71,104 71,104


Project B:
0 k = 10% 1 2 3 4 Years

CFsB -200,000 0 0 146,411 146,411


Time line:
0 k = 12% 1 2 10 Years
  
-50,000 6,000 6,000 6,000
Time lines:
Project A:
0 k = 12% 1 2 3 Periods

CFsA -5,000 2,000 2,500 2,250


Project B:
0 k = 14% 1 2 3 Periods

CFsB -5,000 3,000 2,600 2,900


0 1 2 3 4
| | | | |
CFsNew Tech -1,500 -315 -315 -315 -315 x x x
Time line:
0 1 2 3 Years
k = 14%

-42,000 14,280 16,200 11,400


TV = 18,120
29,520
Time line:
0 1 2 3 Years
k = 10%

-62,000 19,920 22,800 15,600


TV = 15,680
31,280
4
.Answer (D) is correct. The capital budgeting process is a method of planning the efficient expenditure of the firm's
resources on capital projects. Such planning is essential in view of the rising costs of scarce resources.
Answer (A) is incorrect because capital budgeting may also be used for analysis of multiple profitable alternatives and of
lease-or-buy decisions. Answer (B) is incorrect because capital budgeting permits analysis of adding or discontinuing
product lines or facilities and of lease-or-buy decisions. Answer (C) is incorrect because the lease-or-buy decision is just
one specific example of an appropriate use of capital budgeting techniques.

NPV
$

B
A

Expected increase in annual net income k


ARR = 0 10%
Initial (or average) investment IRRB IRRA

NPV
($)

Discount rate (%)


0 16% 17% 18% 30%

NPV
$
NPV ($)
P
C

k 10% k
10% 15% 20% %

NPV NPV
$

A A

B
B n
Sigma CF t
k k t=1
0 7% 12% 15% 0 7% 12% 14%
(1 + K)t
NPV
$

X
Crossover

k
10% IRRY 12% IRRX

0 k = 12% 1 2 3 4 5 6

CFsA -15,000 4,000 4,000 4,000 4,000 4,000 4,000


Project B:
0 k = ? 1 2 6 Years
  
CFsB -14,815 5,100 5,100 5,100
Time lines:
Project A:
0 1 2 3 4 Years
k = 12%

CFsA -25,000 13,000 13,000 13,000 13,000


NPVA = ? = 17,663 Terminal value = 5,000
CF 4 = 18,000
Project B:
0 1 2 3 4 Years
k = ?

CFsB -25,000 15,247 15,247 15,247 15,247


NPVA = NPVB = 17,663 Terminal value = 0
CF 4 = 15,247
Time line:
0 1 2 3 Years
k = 18%

-3,000 1,728 1,920 1,152


Time line:
0 1 2 3 4 5 Years
k = 12%

-45,000 7,800 10,680 7,560 5,880 -1,920


Time line:
0 k = 9% 1 2 3 4 5 Years

-40,000 9,800 11,720 9,640 8,520 15,320


Time lines:
Project A:
0 k = 14% 1 2 3 4 Years

CFsA -200,000 71,104 71,104 71,104 71,104


Project B:
0 k = 10% 1 2 3 4 Years

CFsB -200,000 0 0 146,411 146,411


Time line:
0 k = 12% 1 2 10 Years
  
-50,000 6,000 6,000 6,000
Time lines:
Project A:
0 k = 12% 1 2 3 Periods

CFsA -5,000 2,000 2,500 2,250


Project B:
0 k = 14% 1 2 3 Periods

CFsB -5,000 3,000 2,600 2,900


0 1 2 3 4
| | | | |
CFsNew Tech -1,500 -315 -315 -315 -315 x x x
Time line:
0 1 2 3 Years
k = 14%

-42,000 14,280 16,200 11,400


TV = 18,120
29,520
Time line:
0 1 2 3 Years
k = 10%

-62,000 19,920 22,800 15,600


TV = 15,680
31,280
5
.Answer (D) is correct. Capital budgeting is the process of planning expenditures for investments on which the returns
are expected to occur over a period of more than 1 year. Thus, capital budgeting concerns the acquisition or disposal of
long-term assets and the financing ramifications of such decisions. The adoption of a new method of allocating
nontraceable costs to product lines has no effect on a company's cash flows, does not concern the acquisition of long-
term assets, and is not concerned with financing. Hence, capital budgeting is irrelevant to such a decision.
Answer (A) is incorrect because a new aircraft represents a long-term investment in a capital good. Answer (B) is
incorrect because a major advertising program is a high cost investment with long-term effects. Answer (C) is incorrect
because a star quarterback is a costly asset who is expected to have a substantial effect on the team's long-term
profitability.
NPV
$

B
A

Expected increase in annual net income k


ARR = 0 10%
Initial (or average) investment IRRB IRRA

NPV
($)

Discount rate (%)


0 16% 17% 18% 30%

NPV
$
NPV ($)
P
C

k 10% k
10% 15% 20% %

NPV NPV
$

A A

B
B n
Sigma CF t
k k t=1
0 7% 12% 15% 0 7% 12% 14%
(1 + K)t
NPV
$

X
Crossover

k
10% IRRY 12% IRRX

0 k = 12% 1 2 3 4 5 6

CFsA -15,000 4,000 4,000 4,000 4,000 4,000 4,000


Project B:
0 k = ? 1 2 6 Years
  
CFsB -14,815 5,100 5,100 5,100
Time lines:
Project A:
0 1 2 3 4 Years
k = 12%

CFsA -25,000 13,000 13,000 13,000 13,000


NPVA = ? = 17,663 Terminal value = 5,000
CF 4 = 18,000
Project B:
0 1 2 3 4 Years
k = ?

CFsB -25,000 15,247 15,247 15,247 15,247


NPVA = NPVB = 17,663 Terminal value = 0
CF 4 = 15,247
Time line:
0 1 2 3 Years
k = 18%

-3,000 1,728 1,920 1,152


Time line:
0 1 2 3 4 5 Years
k = 12%

-45,000 7,800 10,680 7,560 5,880 -1,920


Time line:
0 k = 9% 1 2 3 4 5 Years

-40,000 9,800 11,720 9,640 8,520 15,320


Time lines:
Project A:
0 k = 14% 1 2 3 4 Years

CFsA -200,000 71,104 71,104 71,104 71,104


Project B:
0 k = 10% 1 2 3 4 Years

CFsB -200,000 0 0 146,411 146,411


Time line:
0 k = 12% 1 2 10 Years
  
-50,000 6,000 6,000 6,000
Time lines:
Project A:
0 k = 12% 1 2 3 Periods

CFsA -5,000 2,000 2,500 2,250


Project B:
0 k = 14% 1 2 3 Periods

CFsB -5,000 3,000 2,600 2,900


0 1 2 3 4
| | | | |
CFsNew Tech -1,500 -315 -315 -315 -315 x x x
Time line:
0 1 2 3 Years
k = 14%

-42,000 14,280 16,200 11,400


TV = 18,120
29,520
Time line:
0 1 2 3 Years
k = 10%

-62,000 19,920 22,800 15,600


TV = 15,680
31,280
6
.Answer (C) is correct. The investment tax credit is of no concern because it no longer exists. The 1986 Tax Reform Act
eliminated the investment tax credit.
Answer (A) is incorrect because the availability of any necessary financing should be considered even though the net
present value method indicates that the project is acceptable. Answer (B) is incorrect because the probability of near-
term technological changes to the manufacturing process should be considered even though the net present value
method indicates that the project is acceptable. Answer (D) is incorrect because maintenance requirements, warranties,
and availability of service arrangements should be considered even though the net present value method indicates that
the project is acceptable.
NPV
$

B
A

Expected increase in annual net income k


ARR = 0 10%
Initial (or average) investment IRRB IRRA

NPV
($)

Discount rate (%)


0 16% 17% 18% 30%

NPV
$
NPV ($)
P
C

k 10% k
10% 15% 20% %

NPV NPV
$

A A

B
B n
Sigma CF t
k k t=1
0 7% 12% 15% 0 7% 12% 14%
(1 + K)t
NPV
$

X
Crossover

k
10% IRRY 12% IRRX

0 k = 12% 1 2 3 4 5 6

CFsA -15,000 4,000 4,000 4,000 4,000 4,000 4,000


Project B:
0 k = ? 1 2 6 Years
  
CFsB -14,815 5,100 5,100 5,100
Time lines:
Project A:
0 1 2 3 4 Years
k = 12%

CFsA -25,000 13,000 13,000 13,000 13,000


NPVA = ? = 17,663 Terminal value = 5,000
CF 4 = 18,000
Project B:
0 1 2 3 4 Years
k = ?

CFsB -25,000 15,247 15,247 15,247 15,247


NPVA = NPVB = 17,663 Terminal value = 0
CF 4 = 15,247
Time line:
0 1 2 3 Years
k = 18%

-3,000 1,728 1,920 1,152


Time line:
0 1 2 3 4 5 Years
k = 12%

-45,000 7,800 10,680 7,560 5,880 -1,920


Time line:
0 k = 9% 1 2 3 4 5 Years

-40,000 9,800 11,720 9,640 8,520 15,320


Time lines:
Project A:
0 k = 14% 1 2 3 4 Years

CFsA -200,000 71,104 71,104 71,104 71,104


Project B:
0 k = 10% 1 2 3 4 Years

CFsB -200,000 0 0 146,411 146,411


Time line:
0 k = 12% 1 2 10 Years
  
-50,000 6,000 6,000 6,000
Time lines:
Project A:
0 k = 12% 1 2 3 Periods

CFsA -5,000 2,000 2,500 2,250


Project B:
0 k = 14% 1 2 3 Periods

CFsB -5,000 3,000 2,600 2,900


0 1 2 3 4
| | | | |
CFsNew Tech -1,500 -315 -315 -315 -315 x x x
Time line:
0 1 2 3 Years
k = 14%

-42,000 14,280 16,200 11,400


TV = 18,120
29,520
Time line:
0 1 2 3 Years
k = 10%

-62,000 19,920 22,800 15,600


TV = 15,680
31,280
7
. Estimating cash flows Answer: b Diff: M
NPV
$

B
A

Expected increase in annual net income k


ARR = 0 10%
Initial (or average) investment IRRB IRRA

NPV
($)

Discount rate (%)


0 16% 17% 18% 30%
NPV
$
NPV ($)
P
C

k 10% k
10% 15% 20% %

NPV NPV
$

A A

B
B n
Sigma CF t
k k t=1
0 7% 12% 15% 0 7% 12% 14%
(1 + K)t
NPV
$

X
Crossover

k
10% IRRY 12% IRRX

0 k = 12% 1 2 3 4 5 6

CFsA -15,000 4,000 4,000 4,000 4,000 4,000 4,000


Project B:
0 k = ? 1 2 6 Years
  
CFsB -14,815 5,100 5,100 5,100
Time lines:
Project A:
0 1 2 3 4 Years
k = 12%

CFsA -25,000 13,000 13,000 13,000 13,000


NPVA = ? = 17,663 Terminal value = 5,000
CF 4 = 18,000
Project B:
0 1 2 3 4 Years
k = ?

CFsB -25,000 15,247 15,247 15,247 15,247


NPVA = NPVB = 17,663 Terminal value = 0
CF 4 = 15,247
Time line:
0 k = 18% 1 2 3 Years

-3,000 1,728 1,920 1,152


Time line:
0 1 2 3 4 5 Years
k = 12%

-45,000 7,800 10,680 7,560 5,880 -1,920


Time line:
0 1 2 3 4 5 Years
k = 9%

-40,000 9,800 11,720 9,640 8,520 15,320


Time lines:
Project A:
0 k = 14% 1 2 3 4 Years

CFsA -200,000 71,104 71,104 71,104 71,104


Project B:
0 k = 10% 1 2 3 4 Years

CFsB -200,000 0 0 146,411 146,411


Time line:
0 k = 12% 1 2 10 Years
  
-50,000 6,000 6,000 6,000
Time lines:
Project A:
0 k = 12% 1 2 3 Periods

CFsA -5,000 2,000 2,500 2,250


Project B:
0 k = 14% 1 2 3 Periods

CFsB -5,000 3,000 2,600 2,900


0 1 2 3 4
| | | | |
CFsNew Tech -1,500 -315 -315 -315 -315 x x x
Time line:
0 1 2 3 Years
k = 14%

-42,000 14,280 16,200 11,400


TV = 18,120
29,520
Time line:
0 1 2 3 Years
k = 10%

-62,000 19,920 22,800 15,600


TV = 15,680
31,280
8
.Answer (D) is correct. The investment in a new project includes more than the initial cost of new capital equipment. In
addition, funds must be provided for increases in receivables and inventories. This investment in working capital is
treated as an initial cost of the investment that will be recovered in full at the end of the project's life.
Answer (A) is incorrect because the investment in working capital will be needed throughout the life of the investment.
Answer (B) is incorrect because cash will be needed to fund the investments in receivables and inventory. Answer (C) is
incorrect because the initial investment should be treated as an initial cash outflow, but one that will be recovered at the
end of the project.

NPV
$

B
A

Expected increase in annual net income k


ARR = 0 10%
Initial (or average) investment IRRB IRRA

NPV
($)

Discount rate (%)


0 16% 17% 18% 30%

NPV
$
NPV ($)
P
C

k 10% k
10% 15% 20% %

NPV NPV
$

A A

B
B n
Sigma CF t
k k t=1
0 7% 12% 15% 0 7% 12% 14%
(1 + K)t
NPV
$

X
Crossover

k
10% IRRY 12% IRRX

0 k = 12% 1 2 3 4 5 6

CFsA -15,000 4,000 4,000 4,000 4,000 4,000 4,000


Project B:
0 k = ? 1 2 6 Years
  
CFsB -14,815 5,100 5,100 5,100
Time lines:
Project A:
0 1 2 3 4 Years
k = 12%

CFsA -25,000 13,000 13,000 13,000 13,000


NPVA = ? = 17,663 Terminal value = 5,000
CF 4 = 18,000
Project B:
0 1 2 3 4 Years
k = ?

CFsB -25,000 15,247 15,247 15,247 15,247


NPVA = NPVB = 17,663 Terminal value = 0
CF 4 = 15,247
Time line:
0 1 2 3 Years
k = 18%

-3,000 1,728 1,920 1,152


Time line:
0 1 2 3 4 5 Years
k = 12%

-45,000 7,800 10,680 7,560 5,880 -1,920


Time line:
0 k = 9% 1 2 3 4 5 Years

-40,000 9,800 11,720 9,640 8,520 15,320


Time lines:
Project A:
0 k = 14% 1 2 3 4 Years

CFsA -200,000 71,104 71,104 71,104 71,104


Project B:
0 k = 10% 1 2 3 4 Years

CFsB -200,000 0 0 146,411 146,411


Time line:
0 k = 12% 1 2 10 Years
  
-50,000 6,000 6,000 6,000
Time lines:
Project A:
0 k = 12% 1 2 3 Periods

CFsA -5,000 2,000 2,500 2,250


Project B:
0 k = 14% 1 2 3 Periods

CFsB -5,000 3,000 2,600 2,900


0 1 2 3 4
| | | | |
CFsNew Tech -1,500 -315 -315 -315 -315 x x x
Time line:
0 1 2 3 Years
k = 14%

-42,000 14,280 16,200 11,400


TV = 18,120
29,520
Time line:
0 1 2 3 Years
k = 10%

-62,000 19,920 22,800 15,600


TV = 15,680
31,280
9
. Relevant cash flows Answer: b Diff: E N

Sunk costs are never included in project cash flows, so statement a is false.
Externalities are always included, so statement b is true. Since the weighted
average cost of capital includes the cost of debt, and this is the discount
rate used to evaluate project cash flows, interest expense should not be
included in project cash flows. Therefore, statement c is false.
NPV
$

B
A

Expected increase in annual net income k


ARR = 0 10%
Initial (or average) investment IRRB IRRA

NPV
($)

Discount rate (%)


0 16% 17% 18% 30%
NPV
$
NPV ($)
P
C

k 10% k
10% 15% 20% %

NPV NPV
$

A A

B
B n
Sigma CF t
k k t=1
0 7% 12% 15% 0 7% 12% 14%
(1 + K)t
NPV
$

X
Crossover

k
10% IRRY 12% IRRX

0 k = 12% 1 2 3 4 5 6

CFsA -15,000 4,000 4,000 4,000 4,000 4,000 4,000


Project B:
0 k = ? 1 2 6 Years
  
CFsB -14,815 5,100 5,100 5,100
Time lines:
Project A:
0 1 2 3 4 Years
k = 12%

CFsA -25,000 13,000 13,000 13,000 13,000


NPVA = ? = 17,663 Terminal value = 5,000
CF 4 = 18,000
Project B:
0 1 2 3 4 Years
k = ?

CFsB -25,000 15,247 15,247 15,247 15,247


NPVA = NPVB = 17,663 Terminal value = 0
CF 4 = 15,247
Time line:
0 k = 18% 1 2 3 Years

-3,000 1,728 1,920 1,152


Time line:
0 1 2 3 4 5 Years
k = 12%

-45,000 7,800 10,680 7,560 5,880 -1,920


Time line:
0 1 2 3 4 5 Years
k = 9%

-40,000 9,800 11,720 9,640 8,520 15,320


Time lines:
Project A:
0 k = 14% 1 2 3 4 Years

CFsA -200,000 71,104 71,104 71,104 71,104


Project B:
0 k = 10% 1 2 3 4 Years

CFsB -200,000 0 0 146,411 146,411


Time line:
0 k = 12% 1 2 10 Years
  
-50,000 6,000 6,000 6,000
Time lines:
Project A:
0 k = 12% 1 2 3 Periods

CFsA -5,000 2,000 2,500 2,250


Project B:
0 k = 14% 1 2 3 Periods

CFsB -5,000 3,000 2,600 2,900


0 1 2 3 4
| | | | |
CFsNew Tech -1,500 -315 -315 -315 -315 x x x
Time line:
0 1 2 3 Years
k = 14%

-42,000 14,280 16,200 11,400


TV = 18,120
29,520
Time line:
0 1 2 3 Years
k = 10%

-62,000 19,920 22,800 15,600


TV = 15,680
31,280
10
. Relevant cash flows Answer: d Diff: E N

Sunk costs should be ignored, but externalities and opportunity costs should
be included in the project evaluation. Therefore, the correct choice is
statement d.

NPV
$

B
A

Expected increase in annual net income k


ARR = 0 10%
Initial (or average) investment IRRB IRRA

NPV
($)

Discount rate (%)


0 16% 17% 18% 30%

NPV
$
NPV ($)
P
C

k 10% k
10% 15% 20% %

NPV NPV
$

A A

B
B n
Sigma CF t
k k t=1
0 7% 12% 15% 0 7% 12% 14%
(1 + K)t
NPV
$

X
Crossover

k
10% IRRY 12% IRRX

0 k = 12% 1 2 3 4 5 6

CFsA -15,000 4,000 4,000 4,000 4,000 4,000 4,000


Project B:
0 k = ? 1 2 6 Years
  
CFsB -14,815 5,100 5,100 5,100
Time lines:
Project A:
0 1 2 3 4 Years
k = 12%

CFsA -25,000 13,000 13,000 13,000 13,000


NPVA = ? = 17,663 Terminal value = 5,000
CF 4 = 18,000
Project B:
0 1 2 3 4 Years
k = ?

CFsB -25,000 15,247 15,247 15,247 15,247


NPVA = NPVB = 17,663 Terminal value = 0
CF 4 = 15,247
Time line:
0 1 2 3 Years
k = 18%

-3,000 1,728 1,920 1,152


Time line:
0 1 2 3 4 5 Years
k = 12%

-45,000 7,800 10,680 7,560 5,880 -1,920


Time line:
0 k = 9% 1 2 3 4 5 Years

-40,000 9,800 11,720 9,640 8,520 15,320


Time lines:
Project A:
0 k = 14% 1 2 3 4 Years

CFsA -200,000 71,104 71,104 71,104 71,104


Project B:
0 k = 10% 1 2 3 4 Years

CFsB -200,000 0 0 146,411 146,411


Time line:
0 k = 12% 1 2 10 Years
  
-50,000 6,000 6,000 6,000
Time lines:
Project A:
0 k = 12% 1 2 3 Periods

CFsA -5,000 2,000 2,500 2,250


Project B:
0 k = 14% 1 2 3 Periods

CFsB -5,000 3,000 2,600 2,900


0 1 2 3 4
| | | | |
CFsNew Tech -1,500 -315 -315 -315 -315 x x x
Time line:
0 1 2 3 Years
k = 14%

-42,000 14,280 16,200 11,400


TV = 18,120
29,520
Time line:
0 1 2 3 Years
k = 10%

-62,000 19,920 22,800 15,600


TV = 15,680
31,280
11
.Answer (B) is correct. Tax depreciation is relevant to cash flow analysis because it affects the amount of income taxes
that must be paid. However, book depreciation is not relevant because it does not affect the amount of cash generated
by an investment.
Answer (A) is incorrect because it is a true statement relating to capital budgeting. Answer (C) is incorrect because it is
a true statement relating to capital budgeting. Answer (D) is incorrect because it is a true statement relating to capital
budgeting.

NPV
$

B
A

Expected increase in annual net income k


ARR = 0 10%
Initial (or average) investment IRRB IRRA

NPV
($)

Discount rate (%)


0 16% 17% 18% 30%
NPV
$
NPV ($)
P
C

k 10% k
10% 15% 20% %

NPV NPV
$

A A

B
B n
Sigma CF t
k k t=1
0 7% 12% 15% 0 7% 12% 14%
(1 + K)t
NPV
$

X
Crossover

k
10% IRRY 12% IRRX

0 k = 12% 1 2 3 4 5 6

CFsA -15,000 4,000 4,000 4,000 4,000 4,000 4,000


Project B:
0 k = ? 1 2 6 Years
  
CFsB -14,815 5,100 5,100 5,100
Time lines:
Project A:
0 1 2 3 4 Years
k = 12%

CFsA -25,000 13,000 13,000 13,000 13,000


NPVA = ? = 17,663 Terminal value = 5,000
CF 4 = 18,000
Project B:
0 1 2 3 4 Years
k = ?

CFsB -25,000 15,247 15,247 15,247 15,247


NPVA = NPVB = 17,663 Terminal value = 0
CF 4 = 15,247
Time line:
0 k = 18% 1 2 3 Years

-3,000 1,728 1,920 1,152


Time line:
0 1 2 3 4 5 Years
k = 12%

-45,000 7,800 10,680 7,560 5,880 -1,920


Time line:
0 1 2 3 4 5 Years
k = 9%

-40,000 9,800 11,720 9,640 8,520 15,320


Time lines:
Project A:
0 k = 14% 1 2 3 4 Years

CFsA -200,000 71,104 71,104 71,104 71,104


Project B:
0 k = 10% 1 2 3 4 Years

CFsB -200,000 0 0 146,411 146,411


Time line:
0 k = 12% 1 2 10 Years
  
-50,000 6,000 6,000 6,000
Time lines:
Project A:
0 k = 12% 1 2 3 Periods

CFsA -5,000 2,000 2,500 2,250


Project B:
0 k = 14% 1 2 3 Periods

CFsB -5,000 3,000 2,600 2,900


0 1 2 3 4
| | | | |
CFsNew Tech -1,500 -315 -315 -315 -315 x x x
Time line:
0 1 2 3 Years
k = 14%

-42,000 14,280 16,200 11,400


TV = 18,120
29,520
Time line:
0 1 2 3 Years
k = 10%

-62,000 19,920 22,800 15,600


TV = 15,680
31,280
12
. Relevant cash flows Answer: c Diff: E
The correct answer is c. Sunk costs should be excluded from the analysis, and
interest expense is incorporated in the WACC and not the cash flows.
NPV
$

B
A

Expected increase in annual net income k


ARR = 0 10%
Initial (or average) investment IRRB IRRA

NPV
($)

Discount rate (%)


0 16% 17% 18% 30%

NPV
$
NPV ($)
P
C

k 10% k
10% 15% 20% %

NPV NPV
$

A A

B
B n
Sigma CF t
k k t=1
0 7% 12% 15% 0 7% 12% 14%
(1 + K)t
NPV
$

X
Crossover

k
10% IRRY 12% IRRX

0 k = 12% 1 2 3 4 5 6

CFsA -15,000 4,000 4,000 4,000 4,000 4,000 4,000


Project B:
0 k = ? 1 2 6 Years
  
CFsB -14,815 5,100 5,100 5,100
Time lines:
Project A:
0 1 2 3 4 Years
k = 12%

CFsA -25,000 13,000 13,000 13,000 13,000


NPVA = ? = 17,663 Terminal value = 5,000
CF 4 = 18,000
Project B:
0 1 2 3 4 Years
k = ?

CFsB -25,000 15,247 15,247 15,247 15,247


NPVA = NPVB = 17,663 Terminal value = 0
CF 4 = 15,247
Time line:
0 1 2 3 Years
k = 18%

-3,000 1,728 1,920 1,152


Time line:
0 1 2 3 4 5 Years
k = 12%

-45,000 7,800 10,680 7,560 5,880 -1,920


Time line:
0 k = 9% 1 2 3 4 5 Years

-40,000 9,800 11,720 9,640 8,520 15,320


Time lines:
Project A:
0 k = 14% 1 2 3 4 Years

CFsA -200,000 71,104 71,104 71,104 71,104


Project B:
0 k = 10% 1 2 3 4 Years

CFsB -200,000 0 0 146,411 146,411


Time line:
0 k = 12% 1 2 10 Years
  
-50,000 6,000 6,000 6,000
Time lines:
Project A:
0 k = 12% 1 2 3 Periods

CFsA -5,000 2,000 2,500 2,250


Project B:
0 k = 14% 1 2 3 Periods

CFsB -5,000 3,000 2,600 2,900


0 1 2 3 4
| | | | |
CFsNew Tech -1,500 -315 -315 -315 -315 x x x
Time line:
0 1 2 3 Years
k = 14%

-42,000 14,280 16,200 11,400


TV = 18,120
29,520
Time line:
0 1 2 3 Years
k = 10%

-62,000 19,920 22,800 15,600


TV = 15,680
31,280
13
. Cash flows and accounting measures Answer: d Diff: M
NPV
$

B
A

Expected increase in annual net income k


ARR = 0 10%
Initial (or average) investment IRRB IRRA

NPV
($)

Discount rate (%)


0 16% 17% 18% 30%
NPV
$
NPV ($)
P
C

k 10% k
10% 15% 20% %

NPV NPV
$

A A

B
B n
Sigma CF t
k k t=1
0 7% 12% 15% 0 7% 12% 14%
(1 + K)t
NPV
$

X
Crossover

k
10% IRRY 12% IRRX

0 k = 12% 1 2 3 4 5 6

CFsA -15,000 4,000 4,000 4,000 4,000 4,000 4,000


Project B:
0 k = ? 1 2 6 Years
  
CFsB -14,815 5,100 5,100 5,100
Time lines:
Project A:
0 1 2 3 4 Years
k = 12%

CFsA -25,000 13,000 13,000 13,000 13,000


NPVA = ? = 17,663 Terminal value = 5,000
CF 4 = 18,000
Project B:
0 1 2 3 4 Years
k = ?

CFsB -25,000 15,247 15,247 15,247 15,247


NPVA = NPVB = 17,663 Terminal value = 0
CF 4 = 15,247
Time line:
0 k = 18% 1 2 3 Years

-3,000 1,728 1,920 1,152


Time line:
0 1 2 3 4 5 Years
k = 12%

-45,000 7,800 10,680 7,560 5,880 -1,920


Time line:
0 1 2 3 4 5 Years
k = 9%

-40,000 9,800 11,720 9,640 8,520 15,320


Time lines:
Project A:
0 k = 14% 1 2 3 4 Years

CFsA -200,000 71,104 71,104 71,104 71,104


Project B:
0 k = 10% 1 2 3 4 Years

CFsB -200,000 0 0 146,411 146,411


Time line:
0 k = 12% 1 2 10 Years
  
-50,000 6,000 6,000 6,000
Time lines:
Project A:
0 k = 12% 1 2 3 Periods

CFsA -5,000 2,000 2,500 2,250


Project B:
0 k = 14% 1 2 3 Periods

CFsB -5,000 3,000 2,600 2,900


0 1 2 3 4
| | | | |
CFsNew Tech -1,500 -315 -315 -315 -315 x x x
Time line:
0 1 2 3 Years
k = 14%

-42,000 14,280 16,200 11,400


TV = 18,120
29,520
Time line:
0 1 2 3 Years
k = 10%

-62,000 19,920 22,800 15,600


TV = 15,680
31,280
14. Relevant cash flows
Answer:
d Diff: E
Statements a and c are correct; therefore, statement d is the correct answer.
Net cash flow = Net income + depreciation; therefore, depreciation affects
operating cash flows. Sunk costs should be disregarded when making investment
decisions, while opportunity costs should be considered when making
investment decisions, as they represent the best alternative use of an asset.

NPV
$

B
A

Expected increase in annual net income k


ARR = 0 10%
Initial (or average) investment IRRB IRRA

NPV
($)

Discount rate (%)


0 16% 17% 18% 30%

NPV
$
NPV ($)
P
C

k 10% k
10% 15% 20% %

NPV NPV
$

A A

B
B n
Sigma CF t
k k t=1
0 7% 12% 15% 0 7% 12% 14%
(1 + K)t
NPV
$

X
Crossover

k
10% IRRY 12% IRRX

0 k = 12% 1 2 3 4 5 6

CFsA -15,000 4,000 4,000 4,000 4,000 4,000 4,000


Project B:
0 k = ? 1 2 6 Years
  
CFsB -14,815 5,100 5,100 5,100
Time lines:
Project A:
0 1 2 3 4 Years
k = 12%

CFsA -25,000 13,000 13,000 13,000 13,000


NPVA = ? = 17,663 Terminal value = 5,000
CF 4 = 18,000
Project B:
0 1 2 3 4 Years
k = ?

CFsB -25,000 15,247 15,247 15,247 15,247


NPVA = NPVB = 17,663 Terminal value = 0
CF 4 = 15,247
Time line:
0 1 2 3 Years
k = 18%

-3,000 1,728 1,920 1,152


Time line:
0 1 2 3 4 5 Years
k = 12%

-45,000 7,800 10,680 7,560 5,880 -1,920


Time line:
0 k = 9% 1 2 3 4 5 Years

-40,000 9,800 11,720 9,640 8,520 15,320


Time lines:
Project A:
0 k = 14% 1 2 3 4 Years

CFsA -200,000 71,104 71,104 71,104 71,104


Project B:
0 k = 10% 1 2 3 4 Years

CFsB -200,000 0 0 146,411 146,411


Time line:
0 k = 12% 1 2 10 Years
  
-50,000 6,000 6,000 6,000
Time lines:
Project A:
0 k = 12% 1 2 3 Periods

CFsA -5,000 2,000 2,500 2,250


Project B:
0 k = 14% 1 2 3 Periods

CFsB -5,000 3,000 2,600 2,900


0 1 2 3 4
| | | | |
CFsNew Tech -1,500 -315 -315 -315 -315 x x x
Time line:
0 1 2 3 Years
k = 14%

-42,000 14,280 16,200 11,400


TV = 18,120
29,520
Time line:
0 1 2 3 Years
k = 10%

-62,000 19,920 22,800 15,600


TV = 15,680
31,280
15

. Relevant cash flows Answer: c Diff: E


NPV
$

B
A

Expected increase in annual net income k


ARR = 0 10%
Initial (or average) investment IRRB IRRA

NPV
($)

Discount rate (%)


0 16% 17% 18% 30%
NPV
$
NPV ($)
P
C

k 10% k
10% 15% 20% %

NPV NPV
$

A A

B
B n
Sigma CF t
k k t=1
0 7% 12% 15% 0 7% 12% 14%
(1 + K)t
NPV
$

X
Crossover

k
10% IRRY 12% IRRX

0 k = 12% 1 2 3 4 5 6

CFsA -15,000 4,000 4,000 4,000 4,000 4,000 4,000


Project B:
0 k = ? 1 2 6 Years
  
CFsB -14,815 5,100 5,100 5,100
Time lines:
Project A:
0 1 2 3 4 Years
k = 12%

CFsA -25,000 13,000 13,000 13,000 13,000


NPVA = ? = 17,663 Terminal value = 5,000
CF 4 = 18,000
Project B:
0 1 2 3 4 Years
k = ?

CFsB -25,000 15,247 15,247 15,247 15,247


NPVA = NPVB = 17,663 Terminal value = 0
CF 4 = 15,247
Time line:
0 k = 18% 1 2 3 Years

-3,000 1,728 1,920 1,152


Time line:
0 1 2 3 4 5 Years
k = 12%

-45,000 7,800 10,680 7,560 5,880 -1,920


Time line:
0 1 2 3 4 5 Years
k = 9%

-40,000 9,800 11,720 9,640 8,520 15,320


Time lines:
Project A:
0 k = 14% 1 2 3 4 Years

CFsA -200,000 71,104 71,104 71,104 71,104


Project B:
0 k = 10% 1 2 3 4 Years

CFsB -200,000 0 0 146,411 146,411


Time line:
0 k = 12% 1 2 10 Years
  
-50,000 6,000 6,000 6,000
Time lines:
Project A:
0 k = 12% 1 2 3 Periods

CFsA -5,000 2,000 2,500 2,250


Project B:
0 k = 14% 1 2 3 Periods

CFsB -5,000 3,000 2,600 2,900


0 1 2 3 4
| | | | |
CFsNew Tech -1,500 -315 -315 -315 -315 x x x
Time line:
0 1 2 3 Years
k = 14%

-42,000 14,280 16,200 11,400


TV = 18,120
29,520
Time line:
0 1 2 3 Years
k = 10%

-62,000 19,920 22,800 15,600


TV = 15,680
31,280
16
. Relevant cash flows Answer: d Diff: M
Statements b and c are correct; therefore, statement d is the correct answer.
The $3 million spent on researching the technology is a sunk cost.
NPV
$

B
A

Expected increase in annual net income k


ARR = 0 10%
Initial (or average) investment IRRB IRRA

NPV
($)

Discount rate (%)


0 16% 17% 18% 30%

NPV
$
NPV ($)
P
C

k 10% k
10% 15% 20% %

NPV NPV
$

A A

B
B n
Sigma CF t
k k t=1
0 7% 12% 15% 0 7% 12% 14%
(1 + K)t
NPV
$

X
Crossover

k
10% IRRY 12% IRRX

0 k = 12% 1 2 3 4 5 6

CFsA -15,000 4,000 4,000 4,000 4,000 4,000 4,000


Project B:
0 k = ? 1 2 6 Years
  
CFsB -14,815 5,100 5,100 5,100
Time lines:
Project A:
0 1 2 3 4 Years
k = 12%

CFsA -25,000 13,000 13,000 13,000 13,000


NPVA = ? = 17,663 Terminal value = 5,000
CF 4 = 18,000
Project B:
0 1 2 3 4 Years
k = ?

CFsB -25,000 15,247 15,247 15,247 15,247


NPVA = NPVB = 17,663 Terminal value = 0
CF 4 = 15,247
Time line:
0 1 2 3 Years
k = 18%

-3,000 1,728 1,920 1,152


Time line:
0 1 2 3 4 5 Years
k = 12%

-45,000 7,800 10,680 7,560 5,880 -1,920


Time line:
0 k = 9% 1 2 3 4 5 Years

-40,000 9,800 11,720 9,640 8,520 15,320


Time lines:
Project A:
0 k = 14% 1 2 3 4 Years

CFsA -200,000 71,104 71,104 71,104 71,104


Project B:
0 k = 10% 1 2 3 4 Years

CFsB -200,000 0 0 146,411 146,411


Time line:
0 k = 12% 1 2 10 Years
  
-50,000 6,000 6,000 6,000
Time lines:
Project A:
0 k = 12% 1 2 3 Periods

CFsA -5,000 2,000 2,500 2,250


Project B:
0 k = 14% 1 2 3 Periods

CFsB -5,000 3,000 2,600 2,900


0 1 2 3 4
| | | | |
CFsNew Tech -1,500 -315 -315 -315 -315 x x x
Time line:
0 1 2 3 Years
k = 14%

-42,000 14,280 16,200 11,400


TV = 18,120
29,520
Time line:
0 1 2 3 Years
k = 10%

-62,000 19,920 22,800 15,600


TV = 15,680
31,280
17
. Relevant cash flows Answer: d Diff: M N

Statement a is a sunk cost and sunk costs are never included in the capital
budgeting analysis. Therefore, statement a is not included. Statement b is an
opportunity cost and should be included in the capital budgeting analysis.
Statement c is the cannibalization of existing products, which will cause the
company to forgo cash flows and profits in another division. Therefore, it is
included in the capital budgeting analysis. Therefore, the correct answer is
statement d.
NPV
$

B
A

Expected increase in annual net income k


ARR = 0 10%
Initial (or average) investment IRRB IRRA

NPV
($)

Discount rate (%)


0 16% 17% 18% 30%

NPV
$
NPV ($)
P
C

k 10% k
10% 15% 20% %

NPV NPV
$

A A

B
B n
Sigma CF t
k k t=1
0 7% 12% 15% 0 7% 12% 14%
(1 + K)t
NPV
$

X
Crossover

k
10% IRRY 12% IRRX

0 k = 12% 1 2 3 4 5 6

CFsA -15,000 4,000 4,000 4,000 4,000 4,000 4,000


Project B:
0 k = ? 1 2 6 Years
  
CFsB -14,815 5,100 5,100 5,100
Time lines:
Project A:
0 1 2 3 4 Years
k = 12%

CFsA -25,000 13,000 13,000 13,000 13,000


NPVA = ? = 17,663 Terminal value = 5,000
CF 4 = 18,000
Project B:
0 1 2 3 4 Years
k = ?

CFsB -25,000 15,247 15,247 15,247 15,247


NPVA = NPVB = 17,663 Terminal value = 0
CF 4 = 15,247
Time line:
0 1 2 3 Years
k = 18%

-3,000 1,728 1,920 1,152


Time line:
0 1 2 3 4 5 Years
k = 12%

-45,000 7,800 10,680 7,560 5,880 -1,920


Time line:
0 k = 9% 1 2 3 4 5 Years

-40,000 9,800 11,720 9,640 8,520 15,320


Time lines:
Project A:
0 k = 14% 1 2 3 4 Years

CFsA -200,000 71,104 71,104 71,104 71,104


Project B:
0 k = 10% 1 2 3 4 Years

CFsB -200,000 0 0 146,411 146,411


Time line:
0 k = 12% 1 2 10 Years
  
-50,000 6,000 6,000 6,000
Time lines:
Project A:
0 k = 12% 1 2 3 Periods

CFsA -5,000 2,000 2,500 2,250


Project B:
0 k = 14% 1 2 3 Periods

CFsB -5,000 3,000 2,600 2,900


0 1 2 3 4
| | | | |
CFsNew Tech -1,500 -315 -315 -315 -315 x x x
Time line:
0 1 2 3 Years
k = 14%

-42,000 14,280 16,200 11,400


TV = 18,120
29,520
Time line:
0 1 2 3 Years
k = 10%

-62,000 19,920 22,800 15,600


TV = 15,680
31,280
18
. Relevant cash flows Answer: d Diff: M
Statements b and c are correct; therefore, statement d is the correct answer.
The cost of clearing the land is a sunk cost and should not be considered in
the analysis. The expected impact of the new store on the existing store
should be considered. In addition, the opportunity to lease the land
represents an opportunity cost of opening a new store on the land and should
be considered.
NPV
$

B
A

Expected increase in annual net income k


ARR = 0 10%
Initial (or average) investment IRRB IRRA

NPV
($)

Discount rate (%)


0 16% 17% 18% 30%
NPV
$
NPV ($)
P
C

k 10% k
10% 15% 20% %

NPV NPV
$

A A

B
B n
Sigma CF t
k k t=1
0 7% 12% 15% 0 7% 12% 14%
(1 + K)t
NPV
$

X
Crossover

k
10% IRRY 12% IRRX

0 k = 12% 1 2 3 4 5 6

CFsA -15,000 4,000 4,000 4,000 4,000 4,000 4,000


Project B:
0 k = ? 1 2 6 Years
  
CFsB -14,815 5,100 5,100 5,100
Time lines:
Project A:
0 1 2 3 4 Years
k = 12%

CFsA -25,000 13,000 13,000 13,000 13,000


NPVA = ? = 17,663 Terminal value = 5,000
CF 4 = 18,000
Project B:
0 1 2 3 4 Years
k = ?

CFsB -25,000 15,247 15,247 15,247 15,247


NPVA = NPVB = 17,663 Terminal value = 0
CF 4 = 15,247
Time line:
0 k = 18% 1 2 3 Years

-3,000 1,728 1,920 1,152


Time line:
0 1 2 3 4 5 Years
k = 12%

-45,000 7,800 10,680 7,560 5,880 -1,920


Time line:
0 1 2 3 4 5 Years
k = 9%

-40,000 9,800 11,720 9,640 8,520 15,320


Time lines:
Project A:
0 k = 14% 1 2 3 4 Years

CFsA -200,000 71,104 71,104 71,104 71,104


Project B:
0 k = 10% 1 2 3 4 Years

CFsB -200,000 0 0 146,411 146,411


Time line:
0 k = 12% 1 2 10 Years
  
-50,000 6,000 6,000 6,000
Time lines:
Project A:
0 k = 12% 1 2 3 Periods

CFsA -5,000 2,000 2,500 2,250


Project B:
0 k = 14% 1 2 3 Periods

CFsB -5,000 3,000 2,600 2,900


0 1 2 3 4
| | | | |
CFsNew Tech -1,500 -315 -315 -315 -315 x x x
Time line:
0 1 2 3 Years
k = 14%

-42,000 14,280 16,200 11,400


TV = 18,120
29,520
Time line:
0 1 2 3 Years
k = 10%

-62,000 19,920 22,800 15,600


TV = 15,680
31,280
19

. Relevant cash flows Answer: d Diff: M


Statements a and c are correct; therefore, statement d is correct.
Externalities and opportunity costs should be considered, while sunk costs
should not be included in the analysis.

NPV
$

B
A

Expected increase in annual net income k


ARR = 0 10%
Initial (or average) investment IRRB IRRA

NPV
($)

Discount rate (%)


0 16% 17% 18% 30%

NPV
$
NPV ($)
P
C

k 10% k
10% 15% 20% %

NPV NPV
$

A A

B
B n
Sigma CF t
k k t=1
0 7% 12% 15% 0 7% 12% 14%
(1 + K)t
NPV
$

X
Crossover

k
10% IRRY 12% IRRX

0 k = 12% 1 2 3 4 5 6

CFsA -15,000 4,000 4,000 4,000 4,000 4,000 4,000


Project B:
0 k = ? 1 2 6 Years
  
CFsB -14,815 5,100 5,100 5,100
Time lines:
Project A:
0 1 2 3 4 Years
k = 12%

CFsA -25,000 13,000 13,000 13,000 13,000


NPVA = ? = 17,663 Terminal value = 5,000
CF 4 = 18,000
Project B:
0 1 2 3 4 Years
k = ?

CFsB -25,000 15,247 15,247 15,247 15,247


NPVA = NPVB = 17,663 Terminal value = 0
CF 4 = 15,247
Time line:
0 1 2 3 Years
k = 18%

-3,000 1,728 1,920 1,152


Time line:
0 1 2 3 4 5 Years
k = 12%

-45,000 7,800 10,680 7,560 5,880 -1,920


Time line:
0 k = 9% 1 2 3 4 5 Years

-40,000 9,800 11,720 9,640 8,520 15,320


Time lines:
Project A:
0 k = 14% 1 2 3 4 Years

CFsA -200,000 71,104 71,104 71,104 71,104


Project B:
0 k = 10% 1 2 3 4 Years

CFsB -200,000 0 0 146,411 146,411


Time line:
0 k = 12% 1 2 10 Years
  
-50,000 6,000 6,000 6,000
Time lines:
Project A:
0 k = 12% 1 2 3 Periods

CFsA -5,000 2,000 2,500 2,250


Project B:
0 k = 14% 1 2 3 Periods

CFsB -5,000 3,000 2,600 2,900


0 1 2 3 4
| | | | |
CFsNew Tech -1,500 -315 -315 -315 -315 x x x
Time line:
0 1 2 3 Years
k = 14%

-42,000 14,280 16,200 11,400


TV = 18,120
29,520
Time line:
0 1 2 3 Years
k = 10%

-62,000 19,920 22,800 15,600


TV = 15,680
31,280
20
. Cash flow estimation Answer: d Diff: M
Statement d is true--the forgone rent is an “opportunity cost” which should
be charged to the project under consideration. Note that Statements a and b
are both false—the cash flows should not take account of interest, because
financial costs are dealt with by discounting at the WACC. If interest were
deducted to find cash flows, then this cost would be “double counted,” and
the NPV would be downward biased. Ignoring interest when determining cash
flows produces no bias in the NPV whatever. Note also that externalities can
be either positive or negative—they tend to be negative if the new project is
a substitute for existing products, but positive if the new project is
complementary to the firm’s other products.
NPV
$

B
A

Expected increase in annual net income k


ARR = 0 10%
Initial (or average) investment IRRB IRRA

NPV
($)

Discount rate (%)


0 16% 17% 18% 30%

NPV
$
NPV ($)
P
C

k 10% k
10% 15% 20% %

NPV NPV
$

A A

B
B n
Sigma CF t
k k t=1
0 7% 12% 15% 0 7% 12% 14%
(1 + K)t
NPV
$

X
Crossover

k
10% IRRY 12% IRRX

0 k = 12% 1 2 3 4 5 6

CFsA -15,000 4,000 4,000 4,000 4,000 4,000 4,000


Project B:
0 k = ? 1 2 6 Years
  
CFsB -14,815 5,100 5,100 5,100
Time lines:
Project A:
0 1 2 3 4 Years
k = 12%

CFsA -25,000 13,000 13,000 13,000 13,000


NPVA = ? = 17,663 Terminal value = 5,000
CF 4 = 18,000
Project B:
0 1 2 3 4 Years
k = ?

CFsB -25,000 15,247 15,247 15,247 15,247


NPVA = NPVB = 17,663 Terminal value = 0
CF 4 = 15,247
Time line:
0 1 2 3 Years
k = 18%

-3,000 1,728 1,920 1,152


Time line:
0 1 2 3 4 5 Years
k = 12%

-45,000 7,800 10,680 7,560 5,880 -1,920


Time line:
0 k = 9% 1 2 3 4 5 Years

-40,000 9,800 11,720 9,640 8,520 15,320


Time lines:
Project A:
0 k = 14% 1 2 3 4 Years

CFsA -200,000 71,104 71,104 71,104 71,104


Project B:
0 k = 10% 1 2 3 4 Years

CFsB -200,000 0 0 146,411 146,411


Time line:
0 k = 12% 1 2 10 Years
  
-50,000 6,000 6,000 6,000
Time lines:
Project A:
0 k = 12% 1 2 3 Periods

CFsA -5,000 2,000 2,500 2,250


Project B:
0 k = 14% 1 2 3 Periods

CFsB -5,000 3,000 2,600 2,900


0 1 2 3 4
| | | | |
CFsNew Tech -1,500 -315 -315 -315 -315 x x x
Time line:
0 1 2 3 Years
k = 14%

-42,000 14,280 16,200 11,400


TV = 18,120
29,520
Time line:
0 1 2 3 Years
k = 10%

-62,000 19,920 22,800 15,600


TV = 15,680
31,280
21
. Incremental cash flows Answer: d Diff: M
NPV
$

B
A

Expected increase in annual net income k


ARR = 0 10%
Initial (or average) investment IRRB IRRA

NPV
($)

Discount rate (%)


0 16% 17% 18% 30%
NPV
$
NPV ($)
P
C

k 10% k
10% 15% 20% %

NPV NPV
$

A A

B
B n
Sigma CF t
k k t=1
0 7% 12% 15% 0 7% 12% 14%
(1 + K)t
NPV
$

X
Crossover

k
10% IRRY 12% IRRX

0 k = 12% 1 2 3 4 5 6

CFsA -15,000 4,000 4,000 4,000 4,000 4,000 4,000


Project B:
0 k = ? 1 2 6 Years
  
CFsB -14,815 5,100 5,100 5,100
Time lines:
Project A:
0 1 2 3 4 Years
k = 12%

CFsA -25,000 13,000 13,000 13,000 13,000


NPVA = ? = 17,663 Terminal value = 5,000
CF 4 = 18,000
Project B:
0 1 2 3 4 Years
k = ?

CFsB -25,000 15,247 15,247 15,247 15,247


NPVA = NPVB = 17,663 Terminal value = 0
CF 4 = 15,247
Time line:
0 k = 18% 1 2 3 Years

-3,000 1,728 1,920 1,152


Time line:
0 1 2 3 4 5 Years
k = 12%

-45,000 7,800 10,680 7,560 5,880 -1,920


Time line:
0 1 2 3 4 5 Years
k = 9%

-40,000 9,800 11,720 9,640 8,520 15,320


Time lines:
Project A:
0 k = 14% 1 2 3 4 Years

CFsA -200,000 71,104 71,104 71,104 71,104


Project B:
0 k = 10% 1 2 3 4 Years

CFsB -200,000 0 0 146,411 146,411


Time line:
0 k = 12% 1 2 10 Years
  
-50,000 6,000 6,000 6,000
Time lines:
Project A:
0 k = 12% 1 2 3 Periods

CFsA -5,000 2,000 2,500 2,250


Project B:
0 k = 14% 1 2 3 Periods

CFsB -5,000 3,000 2,600 2,900


0 1 2 3 4
| | | | |
CFsNew Tech -1,500 -315 -315 -315 -315 x x x
Time line:
0 1 2 3 Years
k = 14%

-42,000 14,280 16,200 11,400


TV = 18,120
29,520
Time line:
0 1 2 3 Years
k = 10%

-62,000 19,920 22,800 15,600


TV = 15,680
31,280
22
. Incremental cash flows Answer: d Diff: M
NPV
$

B
A

Expected increase in annual net income k


ARR = 0 10%
Initial (or average) investment IRRB IRRA

NPV
($)

Discount rate (%)


0 16% 17% 18% 30%

NPV
$
NPV ($)
P
C

k 10% k
10% 15% 20% %

NPV NPV
$

A A

B
B n
Sigma CF t
k k t=1
0 7% 12% 15% 0 7% 12% 14%
(1 + K)t
NPV
$

X
Crossover

k
10% IRRY 12% IRRX

0 k = 12% 1 2 3 4 5 6

CFsA -15,000 4,000 4,000 4,000 4,000 4,000 4,000


Project B:
0 k = ? 1 2 6 Years
  
CFsB -14,815 5,100 5,100 5,100
Time lines:
Project A:
0 1 2 3 4 Years
k = 12%

CFsA -25,000 13,000 13,000 13,000 13,000


NPVA = ? = 17,663 Terminal value = 5,000
CF 4 = 18,000
Project B:
0 1 2 3 4 Years
k = ?

CFsB -25,000 15,247 15,247 15,247 15,247


NPVA = NPVB = 17,663 Terminal value = 0
CF 4 = 15,247
Time line:
0 1 2 3 Years
k = 18%

-3,000 1,728 1,920 1,152


Time line:
0 1 2 3 4 5 Years
k = 12%

-45,000 7,800 10,680 7,560 5,880 -1,920


Time line:
0 k = 9% 1 2 3 4 5 Years

-40,000 9,800 11,720 9,640 8,520 15,320


Time lines:
Project A:
0 k = 14% 1 2 3 4 Years

CFsA -200,000 71,104 71,104 71,104 71,104


Project B:
0 k = 10% 1 2 3 4 Years

CFsB -200,000 0 0 146,411 146,411


Time line:
0 k = 12% 1 2 10 Years
  
-50,000 6,000 6,000 6,000
Time lines:
Project A:
0 k = 12% 1 2 3 Periods

CFsA -5,000 2,000 2,500 2,250


Project B:
0 k = 14% 1 2 3 Periods

CFsB -5,000 3,000 2,600 2,900


0 1 2 3 4
| | | | |
CFsNew Tech -1,500 -315 -315 -315 -315 x x x
Time line:
0 1 2 3 Years
k = 14%

-42,000 14,280 16,200 11,400


TV = 18,120
29,520
Time line:
0 1 2 3 Years
k = 10%

-62,000 19,920 22,800 15,600


TV = 15,680
31,280
23
.Answer (B) is correct. A tax shield is something that will protect income against taxation. Thus, a depreciation tax shield
is a reduction in income taxes due to a company's being allowed to deduct depreciation against otherwise taxable
income.
Answer (A) is incorrect because a tax shield is not a cash flow, but a means of reducing outflows for income taxes.
Answer (C) is incorrect because cash is not provided by recording depreciation; the shield is a result of deducting
depreciation from taxable revenues. Answer (D) is incorrect because depreciation is recognized as an expense even if it
has no tax benefit.

NPV
$

B
A

Expected increase in annual net income k


ARR = 0 10%
Initial (or average) investment IRRB IRRA

NPV
($)

Discount rate (%)


0 16% 17% 18% 30%
NPV
$
NPV ($)
P
C

k 10% k
10% 15% 20% %

NPV NPV
$

A A

B
B n
Sigma CF t
k k t=1
0 7% 12% 15% 0 7% 12% 14%
(1 + K)t
NPV
$

X
Crossover

k
10% IRRY 12% IRRX

0 k = 12% 1 2 3 4 5 6

CFsA -15,000 4,000 4,000 4,000 4,000 4,000 4,000


Project B:
0 k = ? 1 2 6 Years
  
CFsB -14,815 5,100 5,100 5,100
Time lines:
Project A:
0 1 2 3 4 Years
k = 12%

CFsA -25,000 13,000 13,000 13,000 13,000


NPVA = ? = 17,663 Terminal value = 5,000
CF 4 = 18,000
Project B:
0 1 2 3 4 Years
k = ?

CFsB -25,000 15,247 15,247 15,247 15,247


NPVA = NPVB = 17,663 Terminal value = 0
CF 4 = 15,247
Time line:
0 k = 18% 1 2 3 Years

-3,000 1,728 1,920 1,152


Time line:
0 1 2 3 4 5 Years
k = 12%

-45,000 7,800 10,680 7,560 5,880 -1,920


Time line:
0 1 2 3 4 5 Years
k = 9%

-40,000 9,800 11,720 9,640 8,520 15,320


Time lines:
Project A:
0 k = 14% 1 2 3 4 Years

CFsA -200,000 71,104 71,104 71,104 71,104


Project B:
0 k = 10% 1 2 3 4 Years

CFsB -200,000 0 0 146,411 146,411


Time line:
0 k = 12% 1 2 10 Years
  
-50,000 6,000 6,000 6,000
Time lines:
Project A:
0 k = 12% 1 2 3 Periods

CFsA -5,000 2,000 2,500 2,250


Project B:
0 k = 14% 1 2 3 Periods

CFsB -5,000 3,000 2,600 2,900


0 1 2 3 4
| | | | |
CFsNew Tech -1,500 -315 -315 -315 -315 x x x
Time line:
0 1 2 3 Years
k = 14%

-42,000 14,280 16,200 11,400


TV = 18,120
29,520
Time line:
0 1 2 3 Years
k = 10%

-62,000 19,920 22,800 15,600


TV = 15,680
31,280
24
.Answer (C) is correct. A depreciation deduction will reduce a firm's tax by the amount of the deduction times the
marginal tax rate. A dollar deducted is offset against the firm's last (marginal) dollar of income.
Answer (A) is incorrect because the savings will apply at the marginal rate. Without the deduction, income would be
higher and therefore subject to the marginal tax rate. Answer (B) is incorrect because the marginal rate is relevant to tax
savings from depreciation. Answer (D) is incorrect because one minus the firm's marginal tax rate times the
depreciation amount describes the effect on income, not taxes.

NPV
$

B
A

Expected increase in annual net income k


ARR = 0 10%
Initial (or average) investment IRRB IRRA

NPV
($)

Discount rate (%)


0 16% 17% 18% 30%

NPV
$
NPV ($)
P
C

k 10% k
10% 15% 20% %

NPV NPV
$

A A

B
B n
Sigma CF t
k k t=1
0 7% 12% 15% 0 7% 12% 14%
(1 + K)t
NPV
$

X
Crossover

k
10% IRRY 12% IRRX

0 k = 12% 1 2 3 4 5 6

CFsA -15,000 4,000 4,000 4,000 4,000 4,000 4,000


Project B:
0 k = ? 1 2 6 Years
  
CFsB -14,815 5,100 5,100 5,100
Time lines:
Project A:
0 1 2 3 4 Years
k = 12%

CFsA -25,000 13,000 13,000 13,000 13,000


NPVA = ? = 17,663 Terminal value = 5,000
CF 4 = 18,000
Project B:
0 1 2 3 4 Years
k = ?

CFsB -25,000 15,247 15,247 15,247 15,247


NPVA = NPVB = 17,663 Terminal value = 0
CF 4 = 15,247
Time line:
0 1 2 3 Years
k = 18%

-3,000 1,728 1,920 1,152


Time line:
0 1 2 3 4 5 Years
k = 12%

-45,000 7,800 10,680 7,560 5,880 -1,920


Time line:
0 k = 9% 1 2 3 4 5 Years

-40,000 9,800 11,720 9,640 8,520 15,320


Time lines:
Project A:
0 k = 14% 1 2 3 4 Years

CFsA -200,000 71,104 71,104 71,104 71,104


Project B:
0 k = 10% 1 2 3 4 Years

CFsB -200,000 0 0 146,411 146,411


Time line:
0 k = 12% 1 2 10 Years
  
-50,000 6,000 6,000 6,000
Time lines:
Project A:
0 k = 12% 1 2 3 Periods

CFsA -5,000 2,000 2,500 2,250


Project B:
0 k = 14% 1 2 3 Periods

CFsB -5,000 3,000 2,600 2,900


0 1 2 3 4
| | | | |
CFsNew Tech -1,500 -315 -315 -315 -315 x x x
Time line:
0 1 2 3 Years
k = 14%

-42,000 14,280 16,200 11,400


TV = 18,120
29,520
Time line:
0 1 2 3 Years
k = 10%

-62,000 19,920 22,800 15,600


TV = 15,680
31,280
25
.Answer (C) is correct. Accelerated depreciation results in greater depreciation in the early years of an asset's life
compared with the straight-line method. Thus, accelerated depreciation results in lower income tax expense in the early
years of a project and higher income tax expense in the later years. By effectively deferring taxes, the accelerated
method increases the present value of the depreciation tax shield.
Answer (A) is incorrect because the hurdle rate can be reached more easily as a result of the increased present value of
the depreciation tax shield. Answer (B) is incorrect because the greater depreciation tax shield increases the NPV.
Answer (D) is incorrect because greater initial depreciation reduces the cash outflows for the taxes, but has no effect on
the initial cash outflows.
NPV
$

B
A

Expected increase in annual net income k


ARR = 0 10%
Initial (or average) investment IRRB IRRA

NPV
($)

Discount rate (%)


0 16% 17% 18% 30%

NPV
$
NPV ($)
P
C

k 10% k
10% 15% 20% %

NPV NPV
$

A A

B
B n
Sigma CF t
k k t=1
0 7% 12% 15% 0 7% 12% 14%
(1 + K)t
NPV
$

X
Crossover

k
10% IRRY 12% IRRX

0 k = 12% 1 2 3 4 5 6

CFsA -15,000 4,000 4,000 4,000 4,000 4,000 4,000


Project B:
0 k = ? 1 2 6 Years
  
CFsB -14,815 5,100 5,100 5,100
Time lines:
Project A:
0 1 2 3 4 Years
k = 12%

CFsA -25,000 13,000 13,000 13,000 13,000


NPVA = ? = 17,663 Terminal value = 5,000
CF 4 = 18,000
Project B:
0 1 2 3 4 Years
k = ?

CFsB -25,000 15,247 15,247 15,247 15,247


NPVA = NPVB = 17,663 Terminal value = 0
CF 4 = 15,247
Time line:
0 1 2 3 Years
k = 18%

-3,000 1,728 1,920 1,152


Time line:
0 1 2 3 4 5 Years
k = 12%

-45,000 7,800 10,680 7,560 5,880 -1,920


Time line:
0 k = 9% 1 2 3 4 5 Years

-40,000 9,800 11,720 9,640 8,520 15,320


Time lines:
Project A:
0 k = 14% 1 2 3 4 Years

CFsA -200,000 71,104 71,104 71,104 71,104


Project B:
0 k = 10% 1 2 3 4 Years

CFsB -200,000 0 0 146,411 146,411


Time line:
0 k = 12% 1 2 10 Years
  
-50,000 6,000 6,000 6,000
Time lines:
Project A:
0 k = 12% 1 2 3 Periods

CFsA -5,000 2,000 2,500 2,250


Project B:
0 k = 14% 1 2 3 Periods

CFsB -5,000 3,000 2,600 2,900


0 1 2 3 4
| | | | |
CFsNew Tech -1,500 -315 -315 -315 -315 x x x
Time line:
0 1 2 3 Years
k = 14%

-42,000 14,280 16,200 11,400


TV = 18,120
29,520
Time line:
0 1 2 3 Years
k = 10%

-62,000 19,920 22,800 15,600


TV = 15,680
31,280
26
.Answer (D) is correct. According to microeconomic theory, a firm should produce until its marginal revenue equals its
marginal cost. In capital budgeting terms, marginal revenue is the marginal rate of return on investment, and marginal
cost is the company's marginal cost of capital (MCC). Hence, the firm should continue to invest until the cost of the last
investment equals the return.
Answer (A) is incorrect because the firm must balance cost and return. Minimizing MCC or average cost of capital (ACC
is minimized when it equals MCC) ignores possible returns. Answer (B) is incorrect because the firm must balance cost
and return. Minimizing MCC or average cost of capital (ACC is minimized when it equals MCC) ignores possible returns.
Answer (C) is incorrect because the rate of return on total assets is an average return. Setting MCC equal to this rate
may result in acceptance of poor investments.
NPV
$

B
A

Expected increase in annual net income k


ARR = 0 10%
Initial (or average) investment IRRB IRRA

NPV
($)

Discount rate (%)


0 16% 17% 18% 30%

NPV
$
NPV ($)
P
C

k 10% k
10% 15% 20% %

NPV NPV
$

A A

B
B n
Sigma CF t
k k t=1
0 7% 12% 15% 0 7% 12% 14%
(1 + K)t
NPV
$

X
Crossover

k
10% IRRY 12% IRRX

0 k = 12% 1 2 3 4 5 6

CFsA -15,000 4,000 4,000 4,000 4,000 4,000 4,000


Project B:
0 k = ? 1 2 6 Years
  
CFsB -14,815 5,100 5,100 5,100
Time lines:
Project A:
0 1 2 3 4 Years
k = 12%

CFsA -25,000 13,000 13,000 13,000 13,000


NPVA = ? = 17,663 Terminal value = 5,000
CF 4 = 18,000
Project B:
0 1 2 3 4 Years
k = ?

CFsB -25,000 15,247 15,247 15,247 15,247


NPVA = NPVB = 17,663 Terminal value = 0
CF 4 = 15,247
Time line:
0 1 2 3 Years
k = 18%

-3,000 1,728 1,920 1,152


Time line:
0 1 2 3 4 5 Years
k = 12%

-45,000 7,800 10,680 7,560 5,880 -1,920


Time line:
0 k = 9% 1 2 3 4 5 Years

-40,000 9,800 11,720 9,640 8,520 15,320


Time lines:
Project A:
0 k = 14% 1 2 3 4 Years

CFsA -200,000 71,104 71,104 71,104 71,104


Project B:
0 k = 10% 1 2 3 4 Years

CFsB -200,000 0 0 146,411 146,411


Time line:
0 k = 12% 1 2 10 Years
  
-50,000 6,000 6,000 6,000
Time lines:
Project A:
0 k = 12% 1 2 3 Periods

CFsA -5,000 2,000 2,500 2,250


Project B:
0 k = 14% 1 2 3 Periods

CFsB -5,000 3,000 2,600 2,900


0 1 2 3 4
| | | | |
CFsNew Tech -1,500 -315 -315 -315 -315 x x x
Time line:
0 1 2 3 Years
k = 14%

-42,000 14,280 16,200 11,400


TV = 18,120
29,520
Time line:
0 1 2 3 Years
k = 10%

-62,000 19,920 22,800 15,600


TV = 15,680
31,280
27
.REQUIRED: The determinant of the optimal capital budget.
DISCUSSION: (A) In economics, a basic principle is that a firm should increase output until marginal cost equals
marginal revenue. Similarly, the optimal capital budget is determined by calculating the point at which marginal cost of
capital (which increases as capital requirements increase) and marginal efficiency of investment (which decreases if the
most profitable projects are accepted first) intersect.
Answer (B) is incorrect because the intersection of average marginal cost with average projected rates of return when
the largest (not most profitable) projects are accepted first offers no meaningful capital budgeting conclusion. Answer
(C) is incorrect because the optimal capital budget may exclude profitable projects as lower cost capital goes first to
projects with higher rates of return. Answer (D) is incorrect because accepting projects with rates of return lower than
the cost of capital is not rational.
NPV
$

B
A

Expected increase in annual net income k


ARR = 0 10%
Initial (or average) investment IRRB IRRA

NPV
($)

Discount rate (%)


0 16% 17% 18% 30%

NPV
$
NPV ($)
P
C

k 10% k
10% 15% 20% %

NPV NPV
$

A A

B
B n
Sigma CF t
k k t=1
0 7% 12% 15% 0 7% 12% 14%
(1 + K)t
NPV
$

X
Crossover

k
10% IRRY 12% IRRX

0 k = 12% 1 2 3 4 5 6

CFsA -15,000 4,000 4,000 4,000 4,000 4,000 4,000


Project B:
0 k = ? 1 2 6 Years
  
CFsB -14,815 5,100 5,100 5,100
Time lines:
Project A:
0 1 2 3 4 Years
k = 12%

CFsA -25,000 13,000 13,000 13,000 13,000


NPVA = ? = 17,663 Terminal value = 5,000
CF 4 = 18,000
Project B:
0 1 2 3 4 Years
k = ?

CFsB -25,000 15,247 15,247 15,247 15,247


NPVA = NPVB = 17,663 Terminal value = 0
CF 4 = 15,247
Time line:
0 1 2 3 Years
k = 18%

-3,000 1,728 1,920 1,152


Time line:
0 1 2 3 4 5 Years
k = 12%

-45,000 7,800 10,680 7,560 5,880 -1,920


Time line:
0 k = 9% 1 2 3 4 5 Years

-40,000 9,800 11,720 9,640 8,520 15,320


Time lines:
Project A:
0 k = 14% 1 2 3 4 Years

CFsA -200,000 71,104 71,104 71,104 71,104


Project B:
0 k = 10% 1 2 3 4 Years

CFsB -200,000 0 0 146,411 146,411


Time line:
0 k = 12% 1 2 10 Years
  
-50,000 6,000 6,000 6,000
Time lines:
Project A:
0 k = 12% 1 2 3 Periods

CFsA -5,000 2,000 2,500 2,250


Project B:
0 k = 14% 1 2 3 Periods

CFsB -5,000 3,000 2,600 2,900


0 1 2 3 4
| | | | |
CFsNew Tech -1,500 -315 -315 -315 -315 x x x
Time line:
0 1 2 3 Years
k = 14%

-42,000 14,280 16,200 11,400


TV = 18,120
29,520
Time line:
0 1 2 3 Years
k = 10%

-62,000 19,920 22,800 15,600


TV = 15,680
31,280
28
.Find the WACCs using both John’s and Becky’s methods. (WACC = ks because there is no debt).

John’s WACC for Division B based on overall company’s beta:


k = kRF + RPM(b)
k = 5% + 5%(1.2)
k = 5% + 6%
k = 11%.

Therefore, John would only choose Project 1, because it is the only project whose IRR exceeds its cost of capital.
Consequently, the firm’s capital budget (based on John’s WACC) is only $400 million.

Becky’s WACC for Division B:


k = kRF + RPM(b)
k = 5% + 5%(0.9)
k = 5% + 4.5%
k = 9.5%.

Becky would choose projects 1, 2, 3, and 4 because all of these projects have an IRR that exceeds the Division’s 9.5
percent cost of capital. Based on Becky’s WACC, the firm’s capital budget would be $1,270 million ($400 + $300 + $250
+ $320). Therefore, the firm’s capital budget based on Becky’s WACC is $870 million ($1,270 - $400) larger than the one
based on John’s WACC.
NPV
$

B
A

Expected increase in annual net income k


ARR = 0 10%
Initial (or average) investment IRRB IRRA

NPV
($)

Discount rate (%)


0 16% 17% 18% 30%

NPV
$
NPV ($)
P
C

k 10% k
10% 15% 20% %

NPV NPV
$

A A

B
B n
Sigma CF t
k k t=1
0 7% 12% 15% 0 7% 12% 14%
(1 + K)t
NPV
$

X
Crossover

k
10% IRRY 12% IRRX

0 k = 12% 1 2 3 4 5 6

CFsA -15,000 4,000 4,000 4,000 4,000 4,000 4,000


Project B:
0 k = ? 1 2 6 Years
  
CFsB -14,815 5,100 5,100 5,100
Time lines:
Project A:
0 1 2 3 4 Years
k = 12%

CFsA -25,000 13,000 13,000 13,000 13,000


NPVA = ? = 17,663 Terminal value = 5,000
CF 4 = 18,000
Project B:
0 1 2 3 4 Years
k = ?

CFsB -25,000 15,247 15,247 15,247 15,247


NPVA = NPVB = 17,663 Terminal value = 0
CF 4 = 15,247
Time line:
0 1 2 3 Years
k = 18%

-3,000 1,728 1,920 1,152


Time line:
0 1 2 3 4 5 Years
k = 12%

-45,000 7,800 10,680 7,560 5,880 -1,920


Time line:
0 k = 9% 1 2 3 4 5 Years

-40,000 9,800 11,720 9,640 8,520 15,320


Time lines:
Project A:
0 k = 14% 1 2 3 4 Years

CFsA -200,000 71,104 71,104 71,104 71,104


Project B:
0 k = 10% 1 2 3 4 Years

CFsB -200,000 0 0 146,411 146,411


Time line:
0 k = 12% 1 2 10 Years
  
-50,000 6,000 6,000 6,000
Time lines:
Project A:
0 k = 12% 1 2 3 Periods

CFsA -5,000 2,000 2,500 2,250


Project B:
0 k = 14% 1 2 3 Periods

CFsB -5,000 3,000 2,600 2,900


0 1 2 3 4
| | | | |
CFsNew Tech -1,500 -315 -315 -315 -315 x x x
Time line:
0 1 2 3 Years
k = 14%

-42,000 14,280 16,200 11,400


TV = 18,120
29,520
Time line:
0 1 2 3 Years
k = 10%

-62,000 19,920 22,800 15,600


TV = 15,680
31,280
29
.The option to abandon will increase expected cash flow and decrease risk. If a firm has the option to abandon a project,
it will choose to do so only when things look bad (negative NPV). Thus, abandoning a project eliminates the
low/negative cash flows. Therefore, statement b is correct.

NPV
$

B
A

Expected increase in annual net income k


ARR = 0 10%
Initial (or average) investment IRRB IRRA

NPV
($)

Discount rate (%)


0 16% 17% 18% 30%
NPV
$
NPV ($)
P
C

k 10% k
10% 15% 20% %

NPV NPV
$

A A

B
B n
Sigma CF t
k k t=1
0 7% 12% 15% 0 7% 12% 14%
(1 + K)t
NPV
$

X
Crossover

k
10% IRRY 12% IRRX

0 k = 12% 1 2 3 4 5 6

CFsA -15,000 4,000 4,000 4,000 4,000 4,000 4,000


Project B:
0 k = ? 1 2 6 Years
  
CFsB -14,815 5,100 5,100 5,100
Time lines:
Project A:
0 1 2 3 4 Years
k = 12%

CFsA -25,000 13,000 13,000 13,000 13,000


NPVA = ? = 17,663 Terminal value = 5,000
CF 4 = 18,000
Project B:
0 1 2 3 4 Years
k = ?

CFsB -25,000 15,247 15,247 15,247 15,247


NPVA = NPVB = 17,663 Terminal value = 0
CF 4 = 15,247
Time line:
0 k = 18% 1 2 3 Years

-3,000 1,728 1,920 1,152


Time line:
0 1 2 3 4 5 Years
k = 12%

-45,000 7,800 10,680 7,560 5,880 -1,920


Time line:
0 1 2 3 4 5 Years
k = 9%

-40,000 9,800 11,720 9,640 8,520 15,320


Time lines:
Project A:
0 k = 14% 1 2 3 4 Years

CFsA -200,000 71,104 71,104 71,104 71,104


Project B:
0 k = 10% 1 2 3 4 Years

CFsB -200,000 0 0 146,411 146,411


Time line:
0 k = 12% 1 2 10 Years
  
-50,000 6,000 6,000 6,000
Time lines:
Project A:
0 k = 12% 1 2 3 Periods

CFsA -5,000 2,000 2,500 2,250


Project B:
0 k = 14% 1 2 3 Periods

CFsB -5,000 3,000 2,600 2,900


0 1 2 3 4
| | | | |
CFsNew Tech -1,500 -315 -315 -315 -315 x x x
Time line:
0 1 2 3 Years
k = 14%

-42,000 14,280 16,200 11,400


TV = 18,120
29,520
Time line:
0 1 2 3 Years
k = 10%

-62,000 19,920 22,800 15,600


TV = 15,680
31,280
30
.By having the ability to wait and see you reduce the risk of the project. Therefore, statement a is false. The greater the
uncertainty, the more value there is in waiting for additional information before going on with a project. Therefore,
statement b is false. Statement c is not necessarily true. By waiting to do a project you may lose strategic advantages
associated with being the first competitor to enter a new line of business, which may alter the cash flows. Since
statements a, b, and c are false, the correct choice is statement e.

NPV
$

B
A

Expected increase in annual net income k


ARR = 0 10%
Initial (or average) investment IRRB IRRA

NPV
($)

Discount rate (%)


0 16% 17% 18% 30%

NPV
$
NPV ($)
P
C

k 10% k
10% 15% 20% %

NPV NPV
$

A A

B
B n
Sigma CF t
k k t=1
0 7% 12% 15% 0 7% 12% 14%
(1 + K)t
NPV
$

X
Crossover

k
10% IRRY 12% IRRX

0 k = 12% 1 2 3 4 5 6

CFsA -15,000 4,000 4,000 4,000 4,000 4,000 4,000


Project B:
0 k = ? 1 2 6 Years
  
CFsB -14,815 5,100 5,100 5,100
Time lines:
Project A:
0 1 2 3 4 Years
k = 12%

CFsA -25,000 13,000 13,000 13,000 13,000


NPVA = ? = 17,663 Terminal value = 5,000
CF 4 = 18,000
Project B:
0 1 2 3 4 Years
k = ?

CFsB -25,000 15,247 15,247 15,247 15,247


NPVA = NPVB = 17,663 Terminal value = 0
CF 4 = 15,247
Time line:
0 1 2 3 Years
k = 18%

-3,000 1,728 1,920 1,152


Time line:
0 1 2 3 4 5 Years
k = 12%

-45,000 7,800 10,680 7,560 5,880 -1,920


Time line:
0 k = 9% 1 2 3 4 5 Years

-40,000 9,800 11,720 9,640 8,520 15,320


Time lines:
Project A:
0 k = 14% 1 2 3 4 Years

CFsA -200,000 71,104 71,104 71,104 71,104


Project B:
0 k = 10% 1 2 3 4 Years

CFsB -200,000 0 0 146,411 146,411


Time line:
0 k = 12% 1 2 10 Years
  
-50,000 6,000 6,000 6,000
Time lines:
Project A:
0 k = 12% 1 2 3 Periods

CFsA -5,000 2,000 2,500 2,250


Project B:
0 k = 14% 1 2 3 Periods

CFsB -5,000 3,000 2,600 2,900


0 1 2 3 4
| | | | |
CFsNew Tech -1,500 -315 -315 -315 -315 x x x
Time line:
0 1 2 3 Years
k = 14%

-42,000 14,280 16,200 11,400


TV = 18,120
29,520
Time line:
0 1 2 3 Years
k = 10%

-62,000 19,920 22,800 15,600


TV = 15,680
31,280
31
.Statements a, b, c, and d are all examples of different types of real options. A flexibility option permits the firm to alter
operations depending on how conditions change during the life of the project. Typically, either inputs or outputs, or both,
can be changed. Statement a is an example of an investment timing option, while statement b is an example of an
abandonment option. Statement c is an example of a flexibility option, while statement d is an example of a growth
option. Therefore, statement c is the correct choice.

NPV
$

B
A

Expected increase in annual net income k


ARR = 0 10%
Initial (or average) investment IRRB IRRA

NPV
($)

Discount rate (%)


0 16% 17% 18% 30%
NPV
$
NPV ($)
P
C

k 10% k
10% 15% 20% %

NPV NPV
$

A A

B
B n
Sigma CF t
k k t=1
0 7% 12% 15% 0 7% 12% 14%
(1 + K)t
NPV
$

X
Crossover

k
10% IRRY 12% IRRX

0 k = 12% 1 2 3 4 5 6

CFsA -15,000 4,000 4,000 4,000 4,000 4,000 4,000


Project B:
0 k = ? 1 2 6 Years
  
CFsB -14,815 5,100 5,100 5,100
Time lines:
Project A:
0 1 2 3 4 Years
k = 12%

CFsA -25,000 13,000 13,000 13,000 13,000


NPVA = ? = 17,663 Terminal value = 5,000
CF 4 = 18,000
Project B:
0 1 2 3 4 Years
k = ?

CFsB -25,000 15,247 15,247 15,247 15,247


NPVA = NPVB = 17,663 Terminal value = 0
CF 4 = 15,247
Time line:
0 k = 18% 1 2 3 Years

-3,000 1,728 1,920 1,152


Time line:
0 1 2 3 4 5 Years
k = 12%

-45,000 7,800 10,680 7,560 5,880 -1,920


Time line:
0 1 2 3 4 5 Years
k = 9%

-40,000 9,800 11,720 9,640 8,520 15,320


Time lines:
Project A:
0 k = 14% 1 2 3 4 Years

CFsA -200,000 71,104 71,104 71,104 71,104


Project B:
0 k = 10% 1 2 3 4 Years

CFsB -200,000 0 0 146,411 146,411


Time line:
0 k = 12% 1 2 10 Years
  
-50,000 6,000 6,000 6,000
Time lines:
Project A:
0 k = 12% 1 2 3 Periods

CFsA -5,000 2,000 2,500 2,250


Project B:
0 k = 14% 1 2 3 Periods

CFsB -5,000 3,000 2,600 2,900


0 1 2 3 4
| | | | |
CFsNew Tech -1,500 -315 -315 -315 -315 x x x
Time line:
0 1 2 3 Years
k = 14%

-42,000 14,280 16,200 11,400


TV = 18,120
29,520
Time line:
0 1 2 3 Years
k = 10%

-62,000 19,920 22,800 15,600


TV = 15,680
31,280
32
.By failing to consider both abandonment and growth options, the firm’s capital budget would be too small. In both
cases, the firm might reject what might otherwise be profitable projects if these options had been considered. Therefore,
the correct choice is statement a.
NPV
$

B
A

Expected increase in annual net income k


ARR = 0 10%
Initial (or average) investment IRRB IRRA

NPV
($)

Discount rate (%)


0 16% 17% 18% 30%

NPV
$
NPV ($)
P
C

k 10% k
10% 15% 20% %

NPV NPV
$

A A

B
B n
Sigma CF t
k k t=1
0 7% 12% 15% 0 7% 12% 14%
(1 + K)t
NPV
$

X
Crossover

k
10% IRRY 12% IRRX

0 k = 12% 1 2 3 4 5 6

CFsA -15,000 4,000 4,000 4,000 4,000 4,000 4,000


Project B:
0 k = ? 1 2 6 Years
  
CFsB -14,815 5,100 5,100 5,100
Time lines:
Project A:
0 1 2 3 4 Years
k = 12%

CFsA -25,000 13,000 13,000 13,000 13,000


NPVA = ? = 17,663 Terminal value = 5,000
CF 4 = 18,000
Project B:
0 1 2 3 4 Years
k = ?

CFsB -25,000 15,247 15,247 15,247 15,247


NPVA = NPVB = 17,663 Terminal value = 0
CF 4 = 15,247
Time line:
0 1 2 3 Years
k = 18%

-3,000 1,728 1,920 1,152


Time line:
0 1 2 3 4 5 Years
k = 12%

-45,000 7,800 10,680 7,560 5,880 -1,920


Time line:
0 k = 9% 1 2 3 4 5 Years

-40,000 9,800 11,720 9,640 8,520 15,320


Time lines:
Project A:
0 k = 14% 1 2 3 4 Years

CFsA -200,000 71,104 71,104 71,104 71,104


Project B:
0 k = 10% 1 2 3 4 Years

CFsB -200,000 0 0 146,411 146,411


Time line:
0 k = 12% 1 2 10 Years
  
-50,000 6,000 6,000 6,000
Time lines:
Project A:
0 k = 12% 1 2 3 Periods

CFsA -5,000 2,000 2,500 2,250


Project B:
0 k = 14% 1 2 3 Periods

CFsB -5,000 3,000 2,600 2,900


0 1 2 3 4
| | | | |
CFsNew Tech -1,500 -315 -315 -315 -315 x x x
Time line:
0 1 2 3 Years
k = 14%

-42,000 14,280 16,200 11,400


TV = 18,120
29,520
Time line:
0 1 2 3 Years
k = 10%

-62,000 19,920 22,800 15,600


TV = 15,680
31,280
33
.Answer (D) is correct. The accounting rate of return (unadjusted rate of return or book value rate of return) equals
accounting net income divided by the required initial or average investment. The accounting rate of return ignores the
time value of money.
Answer (A) is incorrect because the net present value is the sum of the present values of all the cash inflows and
outflows associated with an investment. Answer (B) is incorrect because the discounted payback method calculates the
payback period by determining the present values of the future cash flows. Answer (C) is incorrect because the internal
rate of return is the discount rate at which the NPV is zero.

NPV
$

B
A

Expected increase in annual net income k


ARR = 0 10%
Initial (or average) investment IRRB IRRA

NPV
($)

Discount rate (%)


0 16% 17% 18% 30%
NPV
$
NPV ($)
P
C

k 10% k
10% 15% 20% %

NPV NPV
$

A A

B
B n
Sigma CF t
k k t=1
0 7% 12% 15% 0 7% 12% 14%
(1 + K)t
NPV
$

X
Crossover

k
10% IRRY 12% IRRX

0 k = 12% 1 2 3 4 5 6

CFsA -15,000 4,000 4,000 4,000 4,000 4,000 4,000


Project B:
0 k = ? 1 2 6 Years
  
CFsB -14,815 5,100 5,100 5,100
Time lines:
Project A:
0 1 2 3 4 Years
k = 12%

CFsA -25,000 13,000 13,000 13,000 13,000


NPVA = ? = 17,663 Terminal value = 5,000
CF 4 = 18,000
Project B:
0 1 2 3 4 Years
k = ?

CFsB -25,000 15,247 15,247 15,247 15,247


NPVA = NPVB = 17,663 Terminal value = 0
CF 4 = 15,247
Time line:
0 k = 18% 1 2 3 Years

-3,000 1,728 1,920 1,152


Time line:
0 1 2 3 4 5 Years
k = 12%

-45,000 7,800 10,680 7,560 5,880 -1,920


Time line:
0 1 2 3 4 5 Years
k = 9%

-40,000 9,800 11,720 9,640 8,520 15,320


Time lines:
Project A:
0 k = 14% 1 2 3 4 Years

CFsA -200,000 71,104 71,104 71,104 71,104


Project B:
0 k = 10% 1 2 3 4 Years

CFsB -200,000 0 0 146,411 146,411


Time line:
0 k = 12% 1 2 10 Years
  
-50,000 6,000 6,000 6,000
Time lines:
Project A:
0 k = 12% 1 2 3 Periods

CFsA -5,000 2,000 2,500 2,250


Project B:
0 k = 14% 1 2 3 Periods

CFsB -5,000 3,000 2,600 2,900


0 1 2 3 4
| | | | |
CFsNew Tech -1,500 -315 -315 -315 -315 x x x
Time line:
0 1 2 3 Years
k = 14%

-42,000 14,280 16,200 11,400


TV = 18,120
29,520
Time line:
0 1 2 3 Years
k = 10%

-62,000 19,920 22,800 15,600


TV = 15,680
31,280
34
.Answer (D) is correct. The accounting rate of return (also called the unadjusted rate of return or book value rate of
return) measures investment performance by dividing the accounting net income by the average investment in the
project. This method ignores the time value of money.
Answer (A) is incorrect because the bail-out payback method measures the length of the payback period when the
periodic cash inflows are combined with the salvage value. Answer (B) is incorrect because the internal rate of return
method determines the rate at which the NPV is zero. Answer (C) is incorrect because the profitability index is the ratio
of the present value of future net cash inflows to the initial cash investment.

NPV
$

B
A

Expected increase in annual net income k


ARR = 0 10%
Initial (or average) investment IRRB IRRA

NPV
($)

Discount rate (%)


0 16% 17% 18% 30%

NPV
$
NPV ($)
P
C

k 10% k
10% 15% 20% %

NPV NPV
$

A A

B
B n
Sigma CF t
k k t=1
0 7% 12% 15% 0 7% 12% 14%
(1 + K)t
NPV
$

X
Crossover

k
10% IRRY 12% IRRX

0 k = 12% 1 2 3 4 5 6

CFsA -15,000 4,000 4,000 4,000 4,000 4,000 4,000


Project B:
0 k = ? 1 2 6 Years
  
CFsB -14,815 5,100 5,100 5,100
Time lines:
Project A:
0 1 2 3 4 Years
k = 12%

CFsA -25,000 13,000 13,000 13,000 13,000


NPVA = ? = 17,663 Terminal value = 5,000
CF 4 = 18,000
Project B:
0 1 2 3 4 Years
k = ?

CFsB -25,000 15,247 15,247 15,247 15,247


NPVA = NPVB = 17,663 Terminal value = 0
CF 4 = 15,247
Time line:
0 1 2 3 Years
k = 18%

-3,000 1,728 1,920 1,152


Time line:
0 1 2 3 4 5 Years
k = 12%

-45,000 7,800 10,680 7,560 5,880 -1,920


Time line:
0 k = 9% 1 2 3 4 5 Years

-40,000 9,800 11,720 9,640 8,520 15,320


Time lines:
Project A:
0 k = 14% 1 2 3 4 Years

CFsA -200,000 71,104 71,104 71,104 71,104


Project B:
0 k = 10% 1 2 3 4 Years

CFsB -200,000 0 0 146,411 146,411


Time line:
0 k = 12% 1 2 10 Years
  
-50,000 6,000 6,000 6,000
Time lines:
Project A:
0 k = 12% 1 2 3 Periods

CFsA -5,000 2,000 2,500 2,250


Project B:
0 k = 14% 1 2 3 Periods

CFsB -5,000 3,000 2,600 2,900


0 1 2 3 4
| | | | |
CFsNew Tech -1,500 -315 -315 -315 -315 x x x
Time line:
0 1 2 3 Years
k = 14%

-42,000 14,280 16,200 11,400


TV = 18,120
29,520
Time line:
0 1 2 3 Years
k = 10%

-62,000 19,920 22,800 15,600


TV = 15,680
31,280
35
.Answer (D) is correct. The accounting rate of return is calculated by dividing the annual after-tax net income from a
project by the book value of the investment in that project. The time value of money is ignored.
Answer (A) is incorrect because the average rate of return method does not divide by the average investment cost.
Answer (B) is incorrect because the internal rate of return incorporates the time value of money into the calculation. The
IRR is the discount rate that results in a net present value of zero. Answer (C) is incorrect because the capital asset
pricing model is a means of determining the cost of capital.

NPV
$

B
A

Expected increase in annual net income k


ARR = 0 10%
Initial (or average) investment IRRB IRRA

NPV
($)

Discount rate (%)


0 16% 17% 18% 30%
NPV
$
NPV ($)
P
C

k 10% k
10% 15% 20% %

NPV NPV
$

A A

B
B n
Sigma CF t
k k t=1
0 7% 12% 15% 0 7% 12% 14%
(1 + K)t
NPV
$

X
Crossover

k
10% IRRY 12% IRRX

0 k = 12% 1 2 3 4 5 6

CFsA -15,000 4,000 4,000 4,000 4,000 4,000 4,000


Project B:
0 k = ? 1 2 6 Years
  
CFsB -14,815 5,100 5,100 5,100
Time lines:
Project A:
0 1 2 3 4 Years
k = 12%

CFsA -25,000 13,000 13,000 13,000 13,000


NPVA = ? = 17,663 Terminal value = 5,000
CF 4 = 18,000
Project B:
0 1 2 3 4 Years
k = ?

CFsB -25,000 15,247 15,247 15,247 15,247


NPVA = NPVB = 17,663 Terminal value = 0
CF 4 = 15,247
Time line:
0 k = 18% 1 2 3 Years

-3,000 1,728 1,920 1,152


Time line:
0 1 2 3 4 5 Years
k = 12%

-45,000 7,800 10,680 7,560 5,880 -1,920


Time line:
0 1 2 3 4 5 Years
k = 9%

-40,000 9,800 11,720 9,640 8,520 15,320


Time lines:
Project A:
0 k = 14% 1 2 3 4 Years

CFsA -200,000 71,104 71,104 71,104 71,104


Project B:
0 k = 10% 1 2 3 4 Years

CFsB -200,000 0 0 146,411 146,411


Time line:
0 k = 12% 1 2 10 Years
  
-50,000 6,000 6,000 6,000
Time lines:
Project A:
0 k = 12% 1 2 3 Periods

CFsA -5,000 2,000 2,500 2,250


Project B:
0 k = 14% 1 2 3 Periods

CFsB -5,000 3,000 2,600 2,900


0 1 2 3 4
| | | | |
CFsNew Tech -1,500 -315 -315 -315 -315 x x x
Time line:
0 1 2 3 Years
k = 14%

-42,000 14,280 16,200 11,400


TV = 18,120
29,520
Time line:
0 1 2 3 Years
k = 10%

-62,000 19,920 22,800 15,600


TV = 15,680
31,280
36
.Answer (B) is correct. The accounting rate of return (also called the unadjusted rate of return or book value rate of
return) is calculated by dividing the increase in accounting net income by the required investment. Sometimes the
denominator is the average investment rather than the initial investment. This method ignores the time value of money
and focuses on income as opposed to cash flows.
Answer (A) is incorrect because the IRR is the rate at which the net present value is zero. Thus, it incorporates time
value of money concepts, whereas the accounting rate of return does not. Answer (C) is incorrect because the
accounting rate of return is similar to the divisional performance measure of return on investment. Answer (D) is
incorrect because the accounting rate of return ignores the time value of money.

NPV
$

B
A

Expected increase in annual net income k


ARR = 0 10%
Initial (or average) investment IRRB IRRA

NPV
($)

Discount rate (%)


0 16% 17% 18% 30%

NPV
$
NPV ($)
P
C

k 10% k
10% 15% 20% %

NPV NPV
$

A A

B
B n
Sigma CF t
k k t=1
0 7% 12% 15% 0 7% 12% 14%
(1 + K)t
NPV
$

X
Crossover

k
10% IRRY 12% IRRX

0 k = 12% 1 2 3 4 5 6

CFsA -15,000 4,000 4,000 4,000 4,000 4,000 4,000


Project B:
0 k = ? 1 2 6 Years
  
CFsB -14,815 5,100 5,100 5,100
Time lines:
Project A:
0 1 2 3 4 Years
k = 12%

CFsA -25,000 13,000 13,000 13,000 13,000


NPVA = ? = 17,663 Terminal value = 5,000
CF 4 = 18,000
Project B:
0 1 2 3 4 Years
k = ?

CFsB -25,000 15,247 15,247 15,247 15,247


NPVA = NPVB = 17,663 Terminal value = 0
CF 4 = 15,247
Time line:
0 1 2 3 Years
k = 18%

-3,000 1,728 1,920 1,152


Time line:
0 1 2 3 4 5 Years
k = 12%

-45,000 7,800 10,680 7,560 5,880 -1,920


Time line:
0 k = 9% 1 2 3 4 5 Years

-40,000 9,800 11,720 9,640 8,520 15,320


Time lines:
Project A:
0 k = 14% 1 2 3 4 Years

CFsA -200,000 71,104 71,104 71,104 71,104


Project B:
0 k = 10% 1 2 3 4 Years

CFsB -200,000 0 0 146,411 146,411


Time line:
0 k = 12% 1 2 10 Years
  
-50,000 6,000 6,000 6,000
Time lines:
Project A:
0 k = 12% 1 2 3 Periods

CFsA -5,000 2,000 2,500 2,250


Project B:
0 k = 14% 1 2 3 Periods

CFsB -5,000 3,000 2,600 2,900


0 1 2 3 4
| | | | |
CFsNew Tech -1,500 -315 -315 -315 -315 x x x
Time line:
0 1 2 3 Years
k = 14%

-42,000 14,280 16,200 11,400


TV = 18,120
29,520
Time line:
0 1 2 3 Years
k = 10%

-62,000 19,920 22,800 15,600


TV = 15,680
31,280
37
.Answer (D) is correct. The accounting rate of return (ARR) is based on financial statements prepared on the accrual
basis. The formula to compute the ARR is:
Expected increase in annual net income
ARR =
Initial (or average) investment
Both the revenue over life of project and depreciation expense are used in the calculation of the ARR. Depreciation
expense over the project’s life and other expenses directly associated with the project under consideration including
income tax effects are subtracted from revenue over life of the project to determine net income over life of project. Net
income over the project’s life is then divided by the economic life to determine annual net income, the numerator of the
ARR formula. This is a weakness of the ARR method because it does not consider actual cash flows or the time value
of money.
NPV
($)
NPV
$
A

B B
A

k Discount rate (%)


0 10% IRRB IRRA 0 16% 17% 18% 30%

NPV
$
NPV ($)
P
C

k 10% k
10% 15% 20% %

NPV NPV
$

A A

B
B n
Sigma CF t
k k t=1
0 7% 12% 15% 0 7% 12% 14%
(1 + K)t
NPV
$

X
Crossover

k
10% IRRY 12% IRRX

0 k = 12% 1 2 3 4 5 6

CFsA -15,000 4,000 4,000 4,000 4,000 4,000 4,000


Project B:
0 k = ? 1 2 6 Years
  
CFsB -14,815 5,100 5,100 5,100
Time lines:
Project A:
0 1 2 3 4 Years
k = 12%

CFsA -25,000 13,000 13,000 13,000 13,000


NPVA = ? = 17,663 Terminal value = 5,000
CF 4 = 18,000
Project B:
0 1 2 3 4 Years
k = ?

CFsB -25,000 15,247 15,247 15,247 15,247


NPVA = NPVB = 17,663 Terminal value = 0
CF 4 = 15,247
Time line:
0 1 2 3 Years
k = 18%

-3,000 1,728 1,920 1,152


Time line:
0 1 2 3 4 5 Years
k = 12%

-45,000 7,800 10,680 7,560 5,880 -1,920


Time line:
0 k = 9% 1 2 3 4 5 Years

-40,000 9,800 11,720 9,640 8,520 15,320


Time lines:
Project A:
0 k = 14% 1 2 3 4 Years

CFsA -200,000 71,104 71,104 71,104 71,104


Project B:
0 k = 10% 1 2 3 4 Years

CFsB -200,000 0 0 146,411 146,411


Time line:
0 k = 12% 1 2 10 Years
  
-50,000 6,000 6,000 6,000
Time lines:
Project A:
0 k = 12% 1 2 3 Periods

CFsA -5,000 2,000 2,500 2,250


Project B:
0 k = 14% 1 2 3 Periods

CFsB -5,000 3,000 2,600 2,900


0 1 2 3 4
| | | | |
CFsNew Tech -1,500 -315 -315 -315 -315 x x x
Time line:
0 1 2 3 Years
k = 14%

-42,000 14,280 16,200 11,400


TV = 18,120
29,520
Time line:
0 1 2 3 Years
k = 10%

-62,000 19,920 22,800 15,600


TV = 15,680
31,280
Expected increase in annual net income
ARR =
38 Initial (or average) investment .Answer (B) is correct. The accounting rate of return uses
undiscounted net income (not cash flows) to determine a rate of profitability. Annual after-tax net income is divided by
the average book value (or the initial value) of the investment in assets. Answer (C) is incorrect because the payback
period is the time required to complete the return of the original investment. This method gives no consideration to the
time value of money or to returns after the payback period.
Answer (A) is incorrect because the internal rate of return is the rate at which NPV is zero. The minimum desired rate of
return is not used in the discounting. Answer (D) is incorrect because the NPV method computes the discounted
present value of future cash inflows to determine whether it is greater than the initial cash outflow.

NPV
($)
NPV
$
A

B B
A

k Discount rate (%)


0 10% IRRB IRRA 0 16% 17% 18% 30%

NPV
$
NPV ($)
P
C

k 10% k
10% 15% 20% %
NPV NPV
$

A A

B
B n
Sigma CF t
k k t=1
0 7% 12% 15% 0 7% 12% 14%
(1 + K)t
NPV
$

X
Crossover

k
10% IRRY 12% IRRX

0 k = 12% 1 2 3 4 5 6

CFsA -15,000 4,000 4,000 4,000 4,000 4,000 4,000


Project B:
0 k = ? 1 2 6 Years
  
CFsB -14,815 5,100 5,100 5,100
Time lines:
Project A:
0 1 2 3 4 Years
k = 12%

CFsA -25,000 13,000 13,000 13,000 13,000


NPVA = ? = 17,663 Terminal value = 5,000
CF 4 = 18,000
Project B:
0 1 2 3 4 Years
k = ?

CFsB -25,000 15,247 15,247 15,247 15,247


NPVA = NPVB = 17,663 Terminal value = 0
CF 4 = 15,247
Time line:
0 k = 18% 1 2 3 Years

-3,000 1,728 1,920 1,152


Time line:
0 1 2 3 4 5 Years
k = 12%

-45,000 7,800 10,680 7,560 5,880 -1,920


Time line:
0 1 2 3 4 5 Years
k = 9%

-40,000 9,800 11,720 9,640 8,520 15,320


Time lines:
Project A:
0 k = 14% 1 2 3 4 Years

CFsA -200,000 71,104 71,104 71,104 71,104


Project B:
0 k = 10% 1 2 3 4 Years

CFsB -200,000 0 0 146,411 146,411


Time line:
0 k = 12% 1 2 10 Years
  
-50,000 6,000 6,000 6,000
Time lines:
Project A:
0 k = 12% 1 2 3 Periods

CFsA -5,000 2,000 2,500 2,250


Project B:
0 k = 14% 1 2 3 Periods

CFsB -5,000 3,000 2,600 2,900


0 1 2 3 4
| | | | |
CFsNew Tech -1,500 -315 -315 -315 -315 x x x
Time line:
0 1 2 3 Years
k = 14%

-42,000 14,280 16,200 11,400


TV = 18,120
29,520
Time line:
0 1 2 3 Years
k = 10%

-62,000 19,920 22,800 15,600


TV = 15,680
31,280
Expected increase in annual net income
ARR =
39 Initial (or average) investment .Answer (B) is correct. The payback method measures the
number of years required to complete the return of the original investment. This measure is computed by dividing the net
investment by the average expected cash inflows to be generated, resulting in the number of years required to recover
the original investment. The payback method gives no consideration to the time value of money, and there is no
consideration of returns after the payback period.
Answer (A) is incorrect because the discounted cash flow method computes a rate of return. Answer (C) is incorrect
because the net present value method is based on discounted cash flows; the length of time to recover an investment is
not the result. Answer (D) is incorrect because the net present value method is based on discounted cash flows; the
length of time to recover an investment is not the result.
NPV
($)
NPV
$
A

B B
A

k Discount rate (%)


0 10% IRRB IRRA 0 16% 17% 18% 30%

NPV
$
NPV ($)
P
C

k 10% k
10% 15% 20% %

NPV NPV
$

A A

B
B n
Sigma CF t
k k t=1
0 7% 12% 15% 0 7% 12% 14%
(1 + K)t
NPV
$

X
Crossover

k
10% IRRY 12% IRRX

0 k = 12% 1 2 3 4 5 6

CFsA -15,000 4,000 4,000 4,000 4,000 4,000 4,000


Project B:
0 k = ? 1 2 6 Years
  
CFsB -14,815 5,100 5,100 5,100
Time lines:
Project A:
0 1 2 3 4 Years
k = 12%

CFsA -25,000 13,000 13,000 13,000 13,000


NPVA = ? = 17,663 Terminal value = 5,000
CF 4 = 18,000
Project B:
0 1 2 3 4 Years
k = ?

CFsB -25,000 15,247 15,247 15,247 15,247


NPVA = NPVB = 17,663 Terminal value = 0
CF 4 = 15,247
Time line:
0 1 2 3 Years
k = 18%

-3,000 1,728 1,920 1,152


Time line:
0 1 2 3 4 5 Years
k = 12%

-45,000 7,800 10,680 7,560 5,880 -1,920


Time line:
0 k = 9% 1 2 3 4 5 Years

-40,000 9,800 11,720 9,640 8,520 15,320


Time lines:
Project A:
0 k = 14% 1 2 3 4 Years

CFsA -200,000 71,104 71,104 71,104 71,104


Project B:
0 k = 10% 1 2 3 4 Years

CFsB -200,000 0 0 146,411 146,411


Time line:
0 k = 12% 1 2 10 Years
  
-50,000 6,000 6,000 6,000
Time lines:
Project A:
0 k = 12% 1 2 3 Periods

CFsA -5,000 2,000 2,500 2,250


Project B:
0 k = 14% 1 2 3 Periods

CFsB -5,000 3,000 2,600 2,900


0 1 2 3 4
| | | | |
CFsNew Tech -1,500 -315 -315 -315 -315 x x x
Time line:
0 1 2 3 Years
k = 14%

-42,000 14,280 16,200 11,400


TV = 18,120
29,520
Time line:
0 1 2 3 Years
k = 10%

-62,000 19,920 22,800 15,600


TV = 15,680
31,280
Expected increase in annual net income
ARR =
40 Initial (or average) investment .Answer (B) is correct. The usual payback formula divides the
initial investment by the constant net annual cash inflow. The payback method is unsophisticated in that it ignores the
time value of money, but it is widely used because of its simplicity and emphasis on recovery of the initial investment.
Answer (A) is incorrect because the net present value method first discounts the future cash flows to their present value.
Answer (C) is incorrect because the profitability index method divides the present value of the future net cash inflows by
the initial investment. Answer (D) is incorrect because the accounting rate of return divides the annual net income by the
average investment in the project.

NPV
($)
NPV
$
A

B B
A

k Discount rate (%)


0 10% IRRB IRRA 0 16% 17% 18% 30%

NPV
$
NPV ($)
P
C

k 10% k
10% 15% 20% %
NPV NPV
$

A A

B
B n
Sigma CF t
k k t=1
0 7% 12% 15% 0 7% 12% 14%
(1 + K)t
NPV
$

X
Crossover

k
10% IRRY 12% IRRX

0 k = 12% 1 2 3 4 5 6

CFsA -15,000 4,000 4,000 4,000 4,000 4,000 4,000


Project B:
0 k = ? 1 2 6 Years
  
CFsB -14,815 5,100 5,100 5,100
Time lines:
Project A:
0 1 2 3 4 Years
k = 12%

CFsA -25,000 13,000 13,000 13,000 13,000


NPVA = ? = 17,663 Terminal value = 5,000
CF 4 = 18,000
Project B:
0 1 2 3 4 Years
k = ?

CFsB -25,000 15,247 15,247 15,247 15,247


NPVA = NPVB = 17,663 Terminal value = 0
CF 4 = 15,247
Time line:
0 k = 18% 1 2 3 Years

-3,000 1,728 1,920 1,152


Time line:
0 1 2 3 4 5 Years
k = 12%

-45,000 7,800 10,680 7,560 5,880 -1,920


Time line:
0 1 2 3 4 5 Years
k = 9%

-40,000 9,800 11,720 9,640 8,520 15,320


Time lines:
Project A:
0 k = 14% 1 2 3 4 Years

CFsA -200,000 71,104 71,104 71,104 71,104


Project B:
0 k = 10% 1 2 3 4 Years

CFsB -200,000 0 0 146,411 146,411


Time line:
0 k = 12% 1 2 10 Years
  
-50,000 6,000 6,000 6,000
Time lines:
Project A:
0 k = 12% 1 2 3 Periods

CFsA -5,000 2,000 2,500 2,250


Project B:
0 k = 14% 1 2 3 Periods

CFsB -5,000 3,000 2,600 2,900


0 1 2 3 4
| | | | |
CFsNew Tech -1,500 -315 -315 -315 -315 x x x
Time line:
0 1 2 3 Years
k = 14%

-42,000 14,280 16,200 11,400


TV = 18,120
29,520
Time line:
0 1 2 3 Years
k = 10%

-62,000 19,920 22,800 15,600


TV = 15,680
31,280
Expected increase in annual net income
ARR =
41 Initial (or average) investment .Answer (B) is correct. The payback method calculates the
number of years required to complete the return of the original investment. This measure is computed by dividing the net
investment required by the average expected cash flow to be generated, resulting in the number of years required to
recover the original investment. Payback is easy to calculate but has two principal problems: it ignores the time value of
money, and it gives no consideration to returns after the payback period. Thus, it ignores total project profitability.
Answer (A) is incorrect because the payback method does not incorporate the time value of money. Answer (C) is
incorrect because the payback method uses the net investment in the numerator of the calculation. Answer (D) is
incorrect because payback uses the net annual cash inflows in the denominator of the calculation.
NPV
($)
NPV
$
A

B B
A

k Discount rate (%)


0 10% IRRB IRRA 0 16% 17% 18% 30%

NPV
$
NPV ($)
P
C

k 10% k
10% 15% 20% %

NPV NPV
$

A A

B
B n
Sigma CF t
k k t=1
0 7% 12% 15% 0 7% 12% 14%
(1 + K)t
NPV
$

X
Crossover

k
10% IRRY 12% IRRX

0 k = 12% 1 2 3 4 5 6

CFsA -15,000 4,000 4,000 4,000 4,000 4,000 4,000


Project B:
0 k = ? 1 2 6 Years
  
CFsB -14,815 5,100 5,100 5,100
Time lines:
Project A:
0 1 2 3 4 Years
k = 12%

CFsA -25,000 13,000 13,000 13,000 13,000


NPVA = ? = 17,663 Terminal value = 5,000
CF 4 = 18,000
Project B:
0 1 2 3 4 Years
k = ?

CFsB -25,000 15,247 15,247 15,247 15,247


NPVA = NPVB = 17,663 Terminal value = 0
CF 4 = 15,247
Time line:
0 1 2 3 Years
k = 18%

-3,000 1,728 1,920 1,152


Time line:
0 1 2 3 4 5 Years
k = 12%

-45,000 7,800 10,680 7,560 5,880 -1,920


Time line:
0 k = 9% 1 2 3 4 5 Years

-40,000 9,800 11,720 9,640 8,520 15,320


Time lines:
Project A:
0 k = 14% 1 2 3 4 Years

CFsA -200,000 71,104 71,104 71,104 71,104


Project B:
0 k = 10% 1 2 3 4 Years

CFsB -200,000 0 0 146,411 146,411


Time line:
0 k = 12% 1 2 10 Years
  
-50,000 6,000 6,000 6,000
Time lines:
Project A:
0 k = 12% 1 2 3 Periods

CFsA -5,000 2,000 2,500 2,250


Project B:
0 k = 14% 1 2 3 Periods

CFsB -5,000 3,000 2,600 2,900


0 1 2 3 4
| | | | |
CFsNew Tech -1,500 -315 -315 -315 -315 x x x
Time line:
0 1 2 3 Years
k = 14%

-42,000 14,280 16,200 11,400


TV = 18,120
29,520
Time line:
0 1 2 3 Years
k = 10%

-62,000 19,920 22,800 15,600


TV = 15,680
31,280
Expected increase in annual net income
ARR =
42 Initial (or average) investment .Answer (A) is correct. The payback method calculates the
amount of time required to complete the return of the original investment, i.e., the time it takes for a new asset to pay for
itself. Although the payback method is easy to calculate, it has inherent problems. The time value of money and returns
after the payback period are not considered.
Answer (B) is incorrect because the payback method ignores cash flows after payback. Answer (C) is incorrect because
the payback method does not use discounted cash flow techniques. Answer (D) is incorrect because the payback
method may lead to different decisions.

NPV
($)
NPV
$
A

B B
A

k Discount rate (%)


0 10% IRRB IRRA 0 16% 17% 18% 30%

NPV
$
NPV ($)
P
C

k 10% k
10% 15% 20% %
NPV NPV
$

A A

B
B n
Sigma CF t
k k t=1
0 7% 12% 15% 0 7% 12% 14%
(1 + K)t
NPV
$

X
Crossover

k
10% IRRY 12% IRRX

0 k = 12% 1 2 3 4 5 6

CFsA -15,000 4,000 4,000 4,000 4,000 4,000 4,000


Project B:
0 k = ? 1 2 6 Years
  
CFsB -14,815 5,100 5,100 5,100
Time lines:
Project A:
0 1 2 3 4 Years
k = 12%

CFsA -25,000 13,000 13,000 13,000 13,000


NPVA = ? = 17,663 Terminal value = 5,000
CF 4 = 18,000
Project B:
0 1 2 3 4 Years
k = ?

CFsB -25,000 15,247 15,247 15,247 15,247


NPVA = NPVB = 17,663 Terminal value = 0
CF 4 = 15,247
Time line:
0 k = 18% 1 2 3 Years

-3,000 1,728 1,920 1,152


Time line:
0 1 2 3 4 5 Years
k = 12%

-45,000 7,800 10,680 7,560 5,880 -1,920


Time line:
0 1 2 3 4 5 Years
k = 9%

-40,000 9,800 11,720 9,640 8,520 15,320


Time lines:
Project A:
0 k = 14% 1 2 3 4 Years

CFsA -200,000 71,104 71,104 71,104 71,104


Project B:
0 k = 10% 1 2 3 4 Years

CFsB -200,000 0 0 146,411 146,411


Time line:
0 k = 12% 1 2 10 Years
  
-50,000 6,000 6,000 6,000
Time lines:
Project A:
0 k = 12% 1 2 3 Periods

CFsA -5,000 2,000 2,500 2,250


Project B:
0 k = 14% 1 2 3 Periods

CFsB -5,000 3,000 2,600 2,900


0 1 2 3 4
| | | | |
CFsNew Tech -1,500 -315 -315 -315 -315 x x x
Time line:
0 1 2 3 Years
k = 14%

-42,000 14,280 16,200 11,400


TV = 18,120
29,520
Time line:
0 1 2 3 Years
k = 10%

-62,000 19,920 22,800 15,600


TV = 15,680
31,280
Expected increase in annual net income
ARR =
43 Initial (or average) investment .Answer (D) is correct. The payback reciprocal (1 ÷ payback)
has been shown to approximate the internal rate of return (IRR) when the periodic cash flows are equal and the life of
the project is at least twice the payback period.
Answer (A) is incorrect because the payback reciprocal is not related to the profitability index. Answer (B) is incorrect
because the payback reciprocal approximates the IRR, which is the rate at which the NPV is $0. Answer (C) is incorrect
because the accounting rate of return is based on accrual-income based figures, not on discounted cash flows.
NPV
($)
NPV
$
A

B B
A

k Discount rate (%)


0 10% IRRB IRRA 0 16% 17% 18% 30%

NPV
$
NPV ($)
P
C

k 10% k
10% 15% 20% %

NPV NPV
$

A A

B
B n
Sigma CF t
k k t=1
0 7% 12% 15% 0 7% 12% 14%
(1 + K)t
NPV
$

X
Crossover

k
10% IRRY 12% IRRX

0 k = 12% 1 2 3 4 5 6

CFsA -15,000 4,000 4,000 4,000 4,000 4,000 4,000


Project B:
0 k = ? 1 2 6 Years
  
CFsB -14,815 5,100 5,100 5,100
Time lines:
Project A:
0 1 2 3 4 Years
k = 12%

CFsA -25,000 13,000 13,000 13,000 13,000


NPVA = ? = 17,663 Terminal value = 5,000
CF 4 = 18,000
Project B:
0 1 2 3 4 Years
k = ?

CFsB -25,000 15,247 15,247 15,247 15,247


NPVA = NPVB = 17,663 Terminal value = 0
CF 4 = 15,247
Time line:
0 1 2 3 Years
k = 18%

-3,000 1,728 1,920 1,152


Time line:
0 1 2 3 4 5 Years
k = 12%

-45,000 7,800 10,680 7,560 5,880 -1,920


Time line:
0 k = 9% 1 2 3 4 5 Years

-40,000 9,800 11,720 9,640 8,520 15,320


Time lines:
Project A:
0 k = 14% 1 2 3 4 Years

CFsA -200,000 71,104 71,104 71,104 71,104


Project B:
0 k = 10% 1 2 3 4 Years

CFsB -200,000 0 0 146,411 146,411


Time line:
0 k = 12% 1 2 10 Years
  
-50,000 6,000 6,000 6,000
Time lines:
Project A:
0 k = 12% 1 2 3 Periods

CFsA -5,000 2,000 2,500 2,250


Project B:
0 k = 14% 1 2 3 Periods

CFsB -5,000 3,000 2,600 2,900


0 1 2 3 4
| | | | |
CFsNew Tech -1,500 -315 -315 -315 -315 x x x
Time line:
0 1 2 3 Years
k = 14%

-42,000 14,280 16,200 11,400


TV = 18,120
29,520
Time line:
0 1 2 3 Years
k = 10%

-62,000 19,920 22,800 15,600


TV = 15,680
31,280
Expected increase in annual net income
ARR =
44 Initial (or average) investment .Answer (C) is correct. The payback period is computed by
dividing the initial investment by the annual net cash inflow. Depreciation expense is not subtracted from cash inflow;
ony the income taxes which are cause by the depreciation deduction are substracted. One of the weaknesses of the
payback period is that is ignores the time value of time.

NPV
($)
NPV
$
A

B B
A

k Discount rate (%)


0 10% IRRB IRRA 0 16% 17% 18% 30%

NPV
$
NPV ($)
P
C

k 10% k
10% 15% 20% %

NPV NPV
$

A A

B
B n
Sigma CF t
k k t=1
0 7% 12% 15% 0 7% 12% 14%
(1 + K)t
NPV
$

X
Crossover

k
10% IRRY 12% IRRX

0 k = 12% 1 2 3 4 5 6

CFsA -15,000 4,000 4,000 4,000 4,000 4,000 4,000


Project B:
0 k = ? 1 2 6 Years
  
CFsB -14,815 5,100 5,100 5,100
Time lines:
Project A:
0 1 2 3 4 Years
k = 12%

CFsA -25,000 13,000 13,000 13,000 13,000


NPVA = ? = 17,663 Terminal value = 5,000
CF 4 = 18,000
Project B:
0 1 2 3 4 Years
k = ?

CFsB -25,000 15,247 15,247 15,247 15,247


NPVA = NPVB = 17,663 Terminal value = 0
CF 4 = 15,247
Time line:
0 1 2 3 Years
k = 18%

-3,000 1,728 1,920 1,152


Time line:
0 1 2 3 4 5 Years
k = 12%

-45,000 7,800 10,680 7,560 5,880 -1,920


Time line:
0 k = 9% 1 2 3 4 5 Years

-40,000 9,800 11,720 9,640 8,520 15,320


Time lines:
Project A:
0 k = 14% 1 2 3 4 Years

CFsA -200,000 71,104 71,104 71,104 71,104


Project B:
0 k = 10% 1 2 3 4 Years

CFsB -200,000 0 0 146,411 146,411


Time line:
0 k = 12% 1 2 10 Years
  
-50,000 6,000 6,000 6,000
Time lines:
Project A:
0 k = 12% 1 2 3 Periods

CFsA -5,000 2,000 2,500 2,250


Project B:
0 k = 14% 1 2 3 Periods

CFsB -5,000 3,000 2,600 2,900


0 1 2 3 4
| | | | |
CFsNew Tech -1,500 -315 -315 -315 -315 x x x
Time line:
0 1 2 3 Years
k = 14%

-42,000 14,280 16,200 11,400


TV = 18,120
29,520
Time line:
0 1 2 3 Years
k = 10%

-62,000 19,920 22,800 15,600


TV = 15,680
31,280
Expected increase in annual net income
ARR =
45 Initial (or average) investment .Answer (B) is correct. The bailout payback period is the
length of time required for the sum of the cumulative net cash inflow from an investment and its salvage value to equal
the original investment. The bailout payback method measures the risk to the investor if the investment must be
abandoned. The shorter the period, the lower the risk.
Answer (A) is incorrect because the use of the bailout payback method is not limited to firms with federally insured
loans. Answer (C) is incorrect because the payback period is calculated by summing the net cash inflow and the
salvage value. Answer (D) is incorrect because the bailout payback method does not estimate short-term profitability.
NPV
($)
NPV
$
A

B B
A

k Discount rate (%)


0 10% IRRB IRRA 0 16% 17% 18% 30%

NPV
$
NPV ($)
P
C

k 10% k
10% 15% 20% %

NPV NPV
$

A A

B
B n
Sigma CF t
k k t=1
0 7% 12% 15% 0 7% 12% 14%
(1 + K)t
NPV
$

X
Crossover

k
10% IRRY 12% IRRX

0 k = 12% 1 2 3 4 5 6

CFsA -15,000 4,000 4,000 4,000 4,000 4,000 4,000


Project B:
0 k = ? 1 2 6 Years
  
CFsB -14,815 5,100 5,100 5,100
Time lines:
Project A:
0 1 2 3 4 Years
k = 12%

CFsA -25,000 13,000 13,000 13,000 13,000


NPVA = ? = 17,663 Terminal value = 5,000
CF 4 = 18,000
Project B:
0 1 2 3 4 Years
k = ?

CFsB -25,000 15,247 15,247 15,247 15,247


NPVA = NPVB = 17,663 Terminal value = 0
CF 4 = 15,247
Time line:
0 1 2 3 Years
k = 18%

-3,000 1,728 1,920 1,152


Time line:
0 1 2 3 4 5 Years
k = 12%

-45,000 7,800 10,680 7,560 5,880 -1,920


Time line:
0 k = 9% 1 2 3 4 5 Years

-40,000 9,800 11,720 9,640 8,520 15,320


Time lines:
Project A:
0 k = 14% 1 2 3 4 Years

CFsA -200,000 71,104 71,104 71,104 71,104


Project B:
0 k = 10% 1 2 3 4 Years

CFsB -200,000 0 0 146,411 146,411


Time line:
0 k = 12% 1 2 10 Years
  
-50,000 6,000 6,000 6,000
Time lines:
Project A:
0 k = 12% 1 2 3 Periods

CFsA -5,000 2,000 2,500 2,250


Project B:
0 k = 14% 1 2 3 Periods

CFsB -5,000 3,000 2,600 2,900


0 1 2 3 4
| | | | |
CFsNew Tech -1,500 -315 -315 -315 -315 x x x
Time line:
0 1 2 3 Years
k = 14%

-42,000 14,280 16,200 11,400


TV = 18,120
29,520
Time line:
0 1 2 3 Years
k = 10%

-62,000 19,920 22,800 15,600


TV = 15,680
31,280
Expected increase in annual net income
ARR =
46 Initial (or average) investment .Answer (D) is correct. The payback period equals the net
investment divided by the average expected cash flow, resulting in the number of years required to recover the original
investment. The bailout payback incorporates the salvage value of the asset into the calculation. It determines the length
of the payback period when the periodic cash inflows are combined with the salvage value. Hence, the method
measures risk. The longer the payback period, the more risky the investment.
Answer (A) is incorrect because the bailout payback method does not consider the time value of money. Answer (B) is
incorrect because the bailout payback includes salvage value as well as cash flow from operations. Answer (C) is
incorrect because the bailout payback incorporates the disposal value in the payback calculation.

NPV
($)
NPV
$
A

B B
A

k Discount rate (%)


0 10% IRRB IRRA 0 16% 17% 18% 30%

NPV
$
NPV ($)
P
C

k 10% k
10% 15% 20% %
NPV NPV
$

A A

B
B n
Sigma CF t
k k t=1
0 7% 12% 15% 0 7% 12% 14%
(1 + K)t
NPV
$

X
Crossover

k
10% IRRY 12% IRRX

0 k = 12% 1 2 3 4 5 6

CFsA -15,000 4,000 4,000 4,000 4,000 4,000 4,000


Project B:
0 k = ? 1 2 6 Years
  
CFsB -14,815 5,100 5,100 5,100
Time lines:
Project A:
0 1 2 3 4 Years
k = 12%

CFsA -25,000 13,000 13,000 13,000 13,000


NPVA = ? = 17,663 Terminal value = 5,000
CF 4 = 18,000
Project B:
0 1 2 3 4 Years
k = ?

CFsB -25,000 15,247 15,247 15,247 15,247


NPVA = NPVB = 17,663 Terminal value = 0
CF 4 = 15,247
Time line:
0 k = 18% 1 2 3 Years

-3,000 1,728 1,920 1,152


Time line:
0 1 2 3 4 5 Years
k = 12%

-45,000 7,800 10,680 7,560 5,880 -1,920


Time line:
0 1 2 3 4 5 Years
k = 9%

-40,000 9,800 11,720 9,640 8,520 15,320


Time lines:
Project A:
0 k = 14% 1 2 3 4 Years

CFsA -200,000 71,104 71,104 71,104 71,104


Project B:
0 k = 10% 1 2 3 4 Years

CFsB -200,000 0 0 146,411 146,411


Time line:
0 k = 12% 1 2 10 Years
  
-50,000 6,000 6,000 6,000
Time lines:
Project A:
0 k = 12% 1 2 3 Periods

CFsA -5,000 2,000 2,500 2,250


Project B:
0 k = 14% 1 2 3 Periods

CFsB -5,000 3,000 2,600 2,900


0 1 2 3 4
| | | | |
CFsNew Tech -1,500 -315 -315 -315 -315 x x x
Time line:
0 1 2 3 Years
k = 14%

-42,000 14,280 16,200 11,400


TV = 18,120
29,520
Time line:
0 1 2 3 Years
k = 10%

-62,000 19,920 22,800 15,600


TV = 15,680
31,280
Expected increase in annual net income
ARR =
47 Initial (or average) investment .Answer (A) is correct. Time value of money means that,
because of the interest factor, money held today is worth more than the same amount of money to be received in the
future. Interest is paid for the use of money, i.e., on debts, in a normal business transaction. This payment compensates
the lender for not being able to use the money for current consumption.
Answer (B) is incorrect because present value is the value today, net of the interest factor, of one or more payments to
be made in the future. Answer (C) is incorrect because future value is the value some time in the future of a deposit
today or of a series of deposits. Answer (D) is incorrect because an annuity is generally a series of equal payments at
equal intervals of time.
NPV
($)
NPV
$
A

B B
A

k Discount rate (%)


0 10% IRRB IRRA 0 16% 17% 18% 30%

NPV
$
NPV ($)
P
C

k 10% k
10% 15% 20% %

NPV NPV
$

A A

B
B n
Sigma CF t
k k t=1
0 7% 12% 15% 0 7% 12% 14%
(1 + K)t
NPV
$

X
Crossover

k
10% IRRY 12% IRRX

0 k = 12% 1 2 3 4 5 6

CFsA -15,000 4,000 4,000 4,000 4,000 4,000 4,000


Project B:
0 k = ? 1 2 6 Years
  
CFsB -14,815 5,100 5,100 5,100
Time lines:
Project A:
0 1 2 3 4 Years
k = 12%

CFsA -25,000 13,000 13,000 13,000 13,000


NPVA = ? = 17,663 Terminal value = 5,000
CF 4 = 18,000
Project B:
0 1 2 3 4 Years
k = ?

CFsB -25,000 15,247 15,247 15,247 15,247


NPVA = NPVB = 17,663 Terminal value = 0
CF 4 = 15,247
Time line:
0 1 2 3 Years
k = 18%

-3,000 1,728 1,920 1,152


Time line:
0 1 2 3 4 5 Years
k = 12%

-45,000 7,800 10,680 7,560 5,880 -1,920


Time line:
0 k = 9% 1 2 3 4 5 Years

-40,000 9,800 11,720 9,640 8,520 15,320


Time lines:
Project A:
0 k = 14% 1 2 3 4 Years

CFsA -200,000 71,104 71,104 71,104 71,104


Project B:
0 k = 10% 1 2 3 4 Years

CFsB -200,000 0 0 146,411 146,411


Time line:
0 k = 12% 1 2 10 Years
  
-50,000 6,000 6,000 6,000
Time lines:
Project A:
0 k = 12% 1 2 3 Periods

CFsA -5,000 2,000 2,500 2,250


Project B:
0 k = 14% 1 2 3 Periods

CFsB -5,000 3,000 2,600 2,900


0 1 2 3 4
| | | | |
CFsNew Tech -1,500 -315 -315 -315 -315 x x x
Time line:
0 1 2 3 Years
k = 14%

-42,000 14,280 16,200 11,400


TV = 18,120
29,520
Time line:
0 1 2 3 Years
k = 10%

-62,000 19,920 22,800 15,600


TV = 15,680
31,280
Expected increase in annual net income
ARR =
48 Initial (or average) investment .Answer (A) is correct. The time value of money is concerned
with two issues: (1) the investment value of money, and (2) the risk (uncertainty) inherent in any executory agreement.
Thus, a dollar today is worth more than a dollar in the future, and the longer one waits for a dollar, the more uncertain
the receipt is. The cost of capital involves a specific application of the time value of money principles. It is not a basic
concept thereof.
Answer (B) is incorrect because risk is a basic time value of money concept. Cost of capital is not. Answer (C) is
incorrect because the interest effect is a basic time value of money concept. Answer (D) is incorrect because the interest
effect and risk are basic time value of money concepts. Cost of capital is not.

NPV
($)
NPV
$
A

B B
A

k Discount rate (%)


0 10% IRRB IRRA 0 16% 17% 18% 30%

NPV
$
NPV ($)
P
C

k 10% k
10% 15% 20% %
NPV NPV
$

A A

B
B n
Sigma CF t
k k t=1
0 7% 12% 15% 0 7% 12% 14%
(1 + K)t
NPV
$

X
Crossover

k
10% IRRY 12% IRRX

0 k = 12% 1 2 3 4 5 6

CFsA -15,000 4,000 4,000 4,000 4,000 4,000 4,000


Project B:
0 k = ? 1 2 6 Years
  
CFsB -14,815 5,100 5,100 5,100
Time lines:
Project A:
0 1 2 3 4 Years
k = 12%

CFsA -25,000 13,000 13,000 13,000 13,000


NPVA = ? = 17,663 Terminal value = 5,000
CF 4 = 18,000
Project B:
0 1 2 3 4 Years
k = ?

CFsB -25,000 15,247 15,247 15,247 15,247


NPVA = NPVB = 17,663 Terminal value = 0
CF 4 = 15,247
Time line:
0 k = 18% 1 2 3 Years

-3,000 1,728 1,920 1,152


Time line:
0 1 2 3 4 5 Years
k = 12%

-45,000 7,800 10,680 7,560 5,880 -1,920


Time line:
0 1 2 3 4 5 Years
k = 9%

-40,000 9,800 11,720 9,640 8,520 15,320


Time lines:
Project A:
0 k = 14% 1 2 3 4 Years

CFsA -200,000 71,104 71,104 71,104 71,104


Project B:
0 k = 10% 1 2 3 4 Years

CFsB -200,000 0 0 146,411 146,411


Time line:
0 k = 12% 1 2 10 Years
  
-50,000 6,000 6,000 6,000
Time lines:
Project A:
0 k = 12% 1 2 3 Periods

CFsA -5,000 2,000 2,500 2,250


Project B:
0 k = 14% 1 2 3 Periods

CFsB -5,000 3,000 2,600 2,900


0 1 2 3 4
| | | | |
CFsNew Tech -1,500 -315 -315 -315 -315 x x x
Time line:
0 1 2 3 Years
k = 14%

-42,000 14,280 16,200 11,400


TV = 18,120
29,520
Time line:
0 1 2 3 Years
k = 10%

-62,000 19,920 22,800 15,600


TV = 15,680
31,280
Expected increase in annual net income
ARR =
49 Initial (or average) investment .Answer (A) is correct. The present value concept may be
applied both to dollars-in (inflows) and to dollars-out (outflows). Thus, individual cash inflows and cash outflows or a
series thereof (an annuity) may be discounted to time zero (the present). Net present value is the sum of discounted
cash inflows minus any discounted cash outflows. Net present value may be either positive or negative.
Answer (B) is incorrect because a present value may be calculated for discounted cash outflows. Answer (C) is incorrect
because a present value may be calculated for discounted cash inflows or a series thereof (an annuity). Answer (D) is
incorrect because a present value may be calculated for discounted cash inflows or outflows.
NPV
($)
NPV
$
A

B B
A

k Discount rate (%)


0 10% IRRB IRRA 0 16% 17% 18% 30%

NPV
$
NPV ($)
P
C

k 10% k
10% 15% 20% %

NPV NPV
$

A A

B
B n
Sigma CF t
k k t=1
0 7% 12% 15% 0 7% 12% 14%
(1 + K)t
NPV
$

X
Crossover

k
10% IRRY 12% IRRX

0 k = 12% 1 2 3 4 5 6

CFsA -15,000 4,000 4,000 4,000 4,000 4,000 4,000


Project B:
0 k = ? 1 2 6 Years
  
CFsB -14,815 5,100 5,100 5,100
Time lines:
Project A:
0 1 2 3 4 Years
k = 12%

CFsA -25,000 13,000 13,000 13,000 13,000


NPVA = ? = 17,663 Terminal value = 5,000
CF 4 = 18,000
Project B:
0 1 2 3 4 Years
k = ?

CFsB -25,000 15,247 15,247 15,247 15,247


NPVA = NPVB = 17,663 Terminal value = 0
CF 4 = 15,247
Time line:
0 1 2 3 Years
k = 18%

-3,000 1,728 1,920 1,152


Time line:
0 1 2 3 4 5 Years
k = 12%

-45,000 7,800 10,680 7,560 5,880 -1,920


Time line:
0 k = 9% 1 2 3 4 5 Years

-40,000 9,800 11,720 9,640 8,520 15,320


Time lines:
Project A:
0 k = 14% 1 2 3 4 Years

CFsA -200,000 71,104 71,104 71,104 71,104


Project B:
0 k = 10% 1 2 3 4 Years

CFsB -200,000 0 0 146,411 146,411


Time line:
0 k = 12% 1 2 10 Years
  
-50,000 6,000 6,000 6,000
Time lines:
Project A:
0 k = 12% 1 2 3 Periods

CFsA -5,000 2,000 2,500 2,250


Project B:
0 k = 14% 1 2 3 Periods

CFsB -5,000 3,000 2,600 2,900


0 1 2 3 4
| | | | |
CFsNew Tech -1,500 -315 -315 -315 -315 x x x
Time line:
0 1 2 3 Years
k = 14%

-42,000 14,280 16,200 11,400


TV = 18,120
29,520
Time line:
0 1 2 3 Years
k = 10%

-62,000 19,920 22,800 15,600


TV = 15,680
31,280
Expected increase in annual net income
ARR =
50 Initial (or average) investment .Answer (C) is correct. Discounted cash flow analysis, using
either the internal rate of return (IRR) or the net present value (NPV) method, is based on the time value of cash inflows
and outflows. All future operating cash savings are considered as well as the tax effects on cash flows of future
depreciation charges. The cash proceeds of future asset disposals are likewise a necessary consideration. Depreciation
expense is a consideration only to the extent that it affects the cash flows for taxes. Otherwise, depreciation is excluded
from the analysis because it is a noncash expense.
Answer (A) is incorrect because future operating cash savings is a consideration in discounted cash flow analysis.
Answer (B) is incorrect because the current asset disposal price is a consideration in discounted cash flow analysis.
Answer (D) is incorrect because the tax effects of future asset depreciation is a consideration in discounted cash flow
analysis.

NPV
($)
NPV
$
A

B B
A

k Discount rate (%)


0 10% IRRB IRRA 0 16% 17% 18% 30%

NPV
$
NPV ($)
P
C

k 10% k
10% 15% 20% %
NPV NPV
$

A A

B
B n
Sigma CF t
k k t=1
0 7% 12% 15% 0 7% 12% 14%
(1 + K)t
NPV
$

X
Crossover

k
10% IRRY 12% IRRX

0 k = 12% 1 2 3 4 5 6

CFsA -15,000 4,000 4,000 4,000 4,000 4,000 4,000


Project B:
0 k = ? 1 2 6 Years
  
CFsB -14,815 5,100 5,100 5,100
Time lines:
Project A:
0 1 2 3 4 Years
k = 12%

CFsA -25,000 13,000 13,000 13,000 13,000


NPVA = ? = 17,663 Terminal value = 5,000
CF 4 = 18,000
Project B:
0 1 2 3 4 Years
k = ?

CFsB -25,000 15,247 15,247 15,247 15,247


NPVA = NPVB = 17,663 Terminal value = 0
CF 4 = 15,247
Time line:
0 k = 18% 1 2 3 Years

-3,000 1,728 1,920 1,152


Time line:
0 1 2 3 4 5 Years
k = 12%

-45,000 7,800 10,680 7,560 5,880 -1,920


Time line:
0 1 2 3 4 5 Years
k = 9%

-40,000 9,800 11,720 9,640 8,520 15,320


Time lines:
Project A:
0 k = 14% 1 2 3 4 Years

CFsA -200,000 71,104 71,104 71,104 71,104


Project B:
0 k = 10% 1 2 3 4 Years

CFsB -200,000 0 0 146,411 146,411


Time line:
0 k = 12% 1 2 10 Years
  
-50,000 6,000 6,000 6,000
Time lines:
Project A:
0 k = 12% 1 2 3 Periods

CFsA -5,000 2,000 2,500 2,250


Project B:
0 k = 14% 1 2 3 Periods

CFsB -5,000 3,000 2,600 2,900


0 1 2 3 4
| | | | |
CFsNew Tech -1,500 -315 -315 -315 -315 x x x
Time line:
0 1 2 3 Years
k = 14%

-42,000 14,280 16,200 11,400


TV = 18,120
29,520
Time line:
0 1 2 3 Years
k = 10%

-62,000 19,920 22,800 15,600


TV = 15,680
31,280
Expected increase in annual net income
ARR =
51 Initial (or average) investment .Answer (D) is correct. The capital budgeting methods that
are generally considered the best for long-range decision making are the internal rate of return and net present value
methods. These are both discounted cash flow methods.
Answer (A) is incorrect because the payback method gives no consideration to the time value of money or to returns
after the payback period. Answer (B) is incorrect because the accounting rate of return does not consider the time value
of money. Answer (C) is incorrect because the unadjusted rate of return does not consider the time value of money.
NPV
($)
NPV
$
A

B B
A

k Discount rate (%)


0 10% IRRB IRRA 0 16% 17% 18% 30%

NPV
$
NPV ($)
P
C

k 10% k
10% 15% 20% %

NPV NPV
$

A A

B
B n
Sigma CF t
k k t=1
0 7% 12% 15% 0 7% 12% 14%
(1 + K)t
NPV
$

X
Crossover

k
10% IRRY 12% IRRX

0 k = 12% 1 2 3 4 5 6

CFsA -15,000 4,000 4,000 4,000 4,000 4,000 4,000


Project B:
0 k = ? 1 2 6 Years
  
CFsB -14,815 5,100 5,100 5,100
Time lines:
Project A:
0 1 2 3 4 Years
k = 12%

CFsA -25,000 13,000 13,000 13,000 13,000


NPVA = ? = 17,663 Terminal value = 5,000
CF 4 = 18,000
Project B:
0 1 2 3 4 Years
k = ?

CFsB -25,000 15,247 15,247 15,247 15,247


NPVA = NPVB = 17,663 Terminal value = 0
CF 4 = 15,247
Time line:
0 1 2 3 Years
k = 18%

-3,000 1,728 1,920 1,152


Time line:
0 1 2 3 4 5 Years
k = 12%

-45,000 7,800 10,680 7,560 5,880 -1,920


Time line:
0 k = 9% 1 2 3 4 5 Years

-40,000 9,800 11,720 9,640 8,520 15,320


Time lines:
Project A:
0 k = 14% 1 2 3 4 Years

CFsA -200,000 71,104 71,104 71,104 71,104


Project B:
0 k = 10% 1 2 3 4 Years

CFsB -200,000 0 0 146,411 146,411


Time line:
0 k = 12% 1 2 10 Years
  
-50,000 6,000 6,000 6,000
Time lines:
Project A:
0 k = 12% 1 2 3 Periods

CFsA -5,000 2,000 2,500 2,250


Project B:
0 k = 14% 1 2 3 Periods

CFsB -5,000 3,000 2,600 2,900


0 1 2 3 4
| | | | |
CFsNew Tech -1,500 -315 -315 -315 -315 x x x
Time line:
0 1 2 3 Years
k = 14%

-42,000 14,280 16,200 11,400


TV = 18,120
29,520
Time line:
0 1 2 3 Years
k = 10%

-62,000 19,920 22,800 15,600


TV = 15,680
31,280
Expected increase in annual net income
ARR =
52 Initial (or average) investment .DISCUSSION: (A) The time value of money is concerned
with two issues: (1) the investment value of money, and (2) the risk (uncertainty) inherent in any executory agreement.
Thus, a dollar today is worth more than a dollar in the future, and the longer one waits for a dollar, the more uncertain
the receipt is.
Answers (B), (C) and (D) are incorrect because risk and interest factors are concepts underlying the time value of
money.

NPV
($)
NPV
$
A

B B
A

k Discount rate (%)


0 10% IRRB IRRA 0 16% 17% 18% 30%

NPV
$
NPV ($)
P
C

k 10% k
10% 15% 20% %
NPV NPV
$

A A

B
B n
Sigma CF t
k k t=1
0 7% 12% 15% 0 7% 12% 14%
(1 + K)t
NPV
$

X
Crossover

k
10% IRRY 12% IRRX

0 k = 12% 1 2 3 4 5 6

CFsA -15,000 4,000 4,000 4,000 4,000 4,000 4,000


Project B:
0 k = ? 1 2 6 Years
  
CFsB -14,815 5,100 5,100 5,100
Time lines:
Project A:
0 1 2 3 4 Years
k = 12%

CFsA -25,000 13,000 13,000 13,000 13,000


NPVA = ? = 17,663 Terminal value = 5,000
CF 4 = 18,000
Project B:
0 1 2 3 4 Years
k = ?

CFsB -25,000 15,247 15,247 15,247 15,247


NPVA = NPVB = 17,663 Terminal value = 0
CF 4 = 15,247
Time line:
0 k = 18% 1 2 3 Years

-3,000 1,728 1,920 1,152


Time line:
0 1 2 3 4 5 Years
k = 12%

-45,000 7,800 10,680 7,560 5,880 -1,920


Time line:
0 1 2 3 4 5 Years
k = 9%

-40,000 9,800 11,720 9,640 8,520 15,320


Time lines:
Project A:
0 k = 14% 1 2 3 4 Years

CFsA -200,000 71,104 71,104 71,104 71,104


Project B:
0 k = 10% 1 2 3 4 Years

CFsB -200,000 0 0 146,411 146,411


Time line:
0 k = 12% 1 2 10 Years
  
-50,000 6,000 6,000 6,000
Time lines:
Project A:
0 k = 12% 1 2 3 Periods

CFsA -5,000 2,000 2,500 2,250


Project B:
0 k = 14% 1 2 3 Periods

CFsB -5,000 3,000 2,600 2,900


0 1 2 3 4
| | | | |
CFsNew Tech -1,500 -315 -315 -315 -315 x x x
Time line:
0 1 2 3 Years
k = 14%

-42,000 14,280 16,200 11,400


TV = 18,120
29,520
Time line:
0 1 2 3 Years
k = 10%

-62,000 19,920 22,800 15,600


TV = 15,680
31,280
Expected increase in annual net income
ARR =
53 Initial (or average) investment .Answer (C) is correct. Depreciation is a noncash expense
that is deductible for federal income tax purposes. Hence, it directly reduces the cash outlay for income taxes and is
explicitly incorporated in the capital budgeting model.
Answer (A) is incorrect because depreciation is not a cost of operations in the capital budgeting model. Also,
depreciation can be avoided by not making investments. Answer (B) is incorrect because depreciation is an allocation
of historical cost and as such is not a cash inflow, but it may reduce cash outflows for taxes. Answer (D) is incorrect
because periodic depreciation is determined by spreading the depreciation base, i.e., the cost of the asset minus
salvage value, not the initial cash outflow, over the life of the investment.
NPV
($)
NPV
$
A

B B
A

k Discount rate (%)


0 10% IRRB IRRA 0 16% 17% 18% 30%

NPV
$
NPV ($)
P
C

k 10% k
10% 15% 20% %

NPV NPV
$

A A

B
B n
Sigma CF t
k k t=1
0 7% 12% 15% 0 7% 12% 14%
(1 + K)t
NPV
$

X
Crossover

k
10% IRRY 12% IRRX

0 k = 12% 1 2 3 4 5 6

CFsA -15,000 4,000 4,000 4,000 4,000 4,000 4,000


Project B:
0 k = ? 1 2 6 Years
  
CFsB -14,815 5,100 5,100 5,100
Time lines:
Project A:
0 1 2 3 4 Years
k = 12%

CFsA -25,000 13,000 13,000 13,000 13,000


NPVA = ? = 17,663 Terminal value = 5,000
CF 4 = 18,000
Project B:
0 1 2 3 4 Years
k = ?

CFsB -25,000 15,247 15,247 15,247 15,247


NPVA = NPVB = 17,663 Terminal value = 0
CF 4 = 15,247
Time line:
0 1 2 3 Years
k = 18%

-3,000 1,728 1,920 1,152


Time line:
0 1 2 3 4 5 Years
k = 12%

-45,000 7,800 10,680 7,560 5,880 -1,920


Time line:
0 k = 9% 1 2 3 4 5 Years

-40,000 9,800 11,720 9,640 8,520 15,320


Time lines:
Project A:
0 k = 14% 1 2 3 4 Years

CFsA -200,000 71,104 71,104 71,104 71,104


Project B:
0 k = 10% 1 2 3 4 Years

CFsB -200,000 0 0 146,411 146,411


Time line:
0 k = 12% 1 2 10 Years
  
-50,000 6,000 6,000 6,000
Time lines:
Project A:
0 k = 12% 1 2 3 Periods

CFsA -5,000 2,000 2,500 2,250


Project B:
0 k = 14% 1 2 3 Periods

CFsB -5,000 3,000 2,600 2,900


0 1 2 3 4
| | | | |
CFsNew Tech -1,500 -315 -315 -315 -315 x x x
Time line:
0 1 2 3 Years
k = 14%

-42,000 14,280 16,200 11,400


TV = 18,120
29,520
Time line:
0 1 2 3 Years
k = 10%

-62,000 19,920 22,800 15,600


TV = 15,680
31,280
Expected increase in annual net income
ARR =
54 Initial (or average) investment .REQUIRED: The benchmark cost of capital.
DISCUSSION: (D) A weighted average of the costs of all financing sources should be used, with the weights
determined by the usual financing proportions. The terms of any financing raised at the time of initiating a particular
project do not represent the cost of capital for the firm. When a firm achieves its optimal capital structure, the weighted-
average cost of capital is minimized.
Answers (A), (B), and (C) are incorrect because the cost of capital is a composite, or weighted average, of all financing
sources in their usual proportions. The cost of capital should also be calculated on an after-tax basis.

NPV
($)
NPV
$
A

B B
A

k Discount rate (%)


0 10% IRRB IRRA 0 16% 17% 18% 30%

NPV
$
NPV ($)
P
C

k 10% k
10% 15% 20% %
NPV NPV
$

A A

B
B n
Sigma CF t
k k t=1
0 7% 12% 15% 0 7% 12% 14%
(1 + K)t
NPV
$

X
Crossover

k
10% IRRY 12% IRRX

0 k = 12% 1 2 3 4 5 6

CFsA -15,000 4,000 4,000 4,000 4,000 4,000 4,000


Project B:
0 k = ? 1 2 6 Years
  
CFsB -14,815 5,100 5,100 5,100
Time lines:
Project A:
0 1 2 3 4 Years
k = 12%

CFsA -25,000 13,000 13,000 13,000 13,000


NPVA = ? = 17,663 Terminal value = 5,000
CF 4 = 18,000
Project B:
0 1 2 3 4 Years
k = ?

CFsB -25,000 15,247 15,247 15,247 15,247


NPVA = NPVB = 17,663 Terminal value = 0
CF 4 = 15,247
Time line:
0 k = 18% 1 2 3 Years

-3,000 1,728 1,920 1,152


Time line:
0 1 2 3 4 5 Years
k = 12%

-45,000 7,800 10,680 7,560 5,880 -1,920


Time line:
0 1 2 3 4 5 Years
k = 9%

-40,000 9,800 11,720 9,640 8,520 15,320


Time lines:
Project A:
0 k = 14% 1 2 3 4 Years

CFsA -200,000 71,104 71,104 71,104 71,104


Project B:
0 k = 10% 1 2 3 4 Years

CFsB -200,000 0 0 146,411 146,411


Time line:
0 k = 12% 1 2 10 Years
  
-50,000 6,000 6,000 6,000
Time lines:
Project A:
0 k = 12% 1 2 3 Periods

CFsA -5,000 2,000 2,500 2,250


Project B:
0 k = 14% 1 2 3 Periods

CFsB -5,000 3,000 2,600 2,900


0 1 2 3 4
| | | | |
CFsNew Tech -1,500 -315 -315 -315 -315 x x x
Time line:
0 1 2 3 Years
k = 14%

-42,000 14,280 16,200 11,400


TV = 18,120
29,520
Time line:
0 1 2 3 Years
k = 10%

-62,000 19,920 22,800 15,600


TV = 15,680
31,280
Expected increase in annual net income
ARR =
55 Initial (or average) investment .REQUIRED: To determine when the discount rate (hurdle
rate) must be determined before a capital budgeting method can be used.
Answer (C) is correct. The net present value method calculates the expected net monetary gain or loss from a project
by discounting all expected future cash inflows and outflows to the present using some predetermined minimum desired
rate of return (hurdle rate).
Answer (A) is incorrect because the payback method measures the time it will take to recoup, in the form of cash inflows
from operations, the initial dollars invested in a project. The payback period does not consider the time value of money.
Answers (B) and (D) are incorrect. The time adjusted rate of return method is also called the internal rate of return
method. This method computes the rate of interest at which the present value of expected cash inflows from a project
equals the present value of expected cash outflows of the project. Here, the discount rate is not determined in advance
but is the end result of the calculation.
NPV
($)
NPV
$
A

B B
A

k Discount rate (%)


0 10% IRRB IRRA 0 16% 17% 18% 30%

NPV
$
NPV ($)
P
C

k 10% k
10% 15% 20% %

NPV NPV
$

A A

B
B n
Sigma CF t
k k t=1
0 7% 12% 15% 0 7% 12% 14%
(1 + K)t
NPV
$

X
Crossover

k
10% IRRY 12% IRRX

0 k = 12% 1 2 3 4 5 6

CFsA -15,000 4,000 4,000 4,000 4,000 4,000 4,000


Project B:
0 k = ? 1 2 6 Years
  
CFsB -14,815 5,100 5,100 5,100
Time lines:
Project A:
0 1 2 3 4 Years
k = 12%

CFsA -25,000 13,000 13,000 13,000 13,000


NPVA = ? = 17,663 Terminal value = 5,000
CF 4 = 18,000
Project B:
0 1 2 3 4 Years
k = ?

CFsB -25,000 15,247 15,247 15,247 15,247


NPVA = NPVB = 17,663 Terminal value = 0
CF 4 = 15,247
Time line:
0 1 2 3 Years
k = 18%

-3,000 1,728 1,920 1,152


Time line:
0 1 2 3 4 5 Years
k = 12%

-45,000 7,800 10,680 7,560 5,880 -1,920


Time line:
0 k = 9% 1 2 3 4 5 Years

-40,000 9,800 11,720 9,640 8,520 15,320


Time lines:
Project A:
0 k = 14% 1 2 3 4 Years

CFsA -200,000 71,104 71,104 71,104 71,104


Project B:
0 k = 10% 1 2 3 4 Years

CFsB -200,000 0 0 146,411 146,411


Time line:
0 k = 12% 1 2 10 Years
  
-50,000 6,000 6,000 6,000
Time lines:
Project A:
0 k = 12% 1 2 3 Periods

CFsA -5,000 2,000 2,500 2,250


Project B:
0 k = 14% 1 2 3 Periods

CFsB -5,000 3,000 2,600 2,900


0 1 2 3 4
| | | | |
CFsNew Tech -1,500 -315 -315 -315 -315 x x x
Time line:
0 1 2 3 Years
k = 14%

-42,000 14,280 16,200 11,400


TV = 18,120
29,520
Time line:
0 1 2 3 Years
k = 10%

-62,000 19,920 22,800 15,600


TV = 15,680
31,280
Expected increase in annual net income
ARR =
56 Initial (or average) investment .Answer (D) is correct. A hurdle rate is not necessary in
calculating the accounting rate of return. That return is calculated by dividing the net income from a project by the
investment in the project. Similarly, a company can calculate the internal rate of return (IRR) without knowing its hurdle
rate. The IRR is the discount rate at which the net present value is $0. However, the NPV cannot be calculated without
knowing the company's hurdle rate. The NPV method requires that future cash flows be discounted using the hurdle
rate.
Answer (A) is incorrect because the accounting rate of return and the IRR but not the NPV can be calculated without
knowing the hurdle rate. Answer (B) is incorrect because the accounting rate of return and the IRR but not the NPV can
be calculated without knowing the hurdle rate. Answer (C) is incorrect because the accounting rate of return and the IRR
but not the NPV can be calculated without knowing the hurdle rate.

NPV
($)
NPV
$
A

B B
A

k Discount rate (%)


0 10% IRRB IRRA 0 16% 17% 18% 30%

NPV
$
NPV ($)
P
C

k 10% k
10% 15% 20% %
NPV NPV
$

A A

B
B n
Sigma CF t
k k t=1
0 7% 12% 15% 0 7% 12% 14%
(1 + K)t
NPV
$

X
Crossover

k
10% IRRY 12% IRRX

0 k = 12% 1 2 3 4 5 6

CFsA -15,000 4,000 4,000 4,000 4,000 4,000 4,000


Project B:
0 k = ? 1 2 6 Years
  
CFsB -14,815 5,100 5,100 5,100
Time lines:
Project A:
0 1 2 3 4 Years
k = 12%

CFsA -25,000 13,000 13,000 13,000 13,000


NPVA = ? = 17,663 Terminal value = 5,000
CF 4 = 18,000
Project B:
0 1 2 3 4 Years
k = ?

CFsB -25,000 15,247 15,247 15,247 15,247


NPVA = NPVB = 17,663 Terminal value = 0
CF 4 = 15,247
Time line:
0 k = 18% 1 2 3 Years

-3,000 1,728 1,920 1,152


Time line:
0 1 2 3 4 5 Years
k = 12%

-45,000 7,800 10,680 7,560 5,880 -1,920


Time line:
0 1 2 3 4 5 Years
k = 9%

-40,000 9,800 11,720 9,640 8,520 15,320


Time lines:
Project A:
0 k = 14% 1 2 3 4 Years

CFsA -200,000 71,104 71,104 71,104 71,104


Project B:
0 k = 10% 1 2 3 4 Years

CFsB -200,000 0 0 146,411 146,411


Time line:
0 k = 12% 1 2 10 Years
  
-50,000 6,000 6,000 6,000
Time lines:
Project A:
0 k = 12% 1 2 3 Periods

CFsA -5,000 2,000 2,500 2,250


Project B:
0 k = 14% 1 2 3 Periods

CFsB -5,000 3,000 2,600 2,900


0 1 2 3 4
| | | | |
CFsNew Tech -1,500 -315 -315 -315 -315 x x x
Time line:
0 1 2 3 Years
k = 14%

-42,000 14,280 16,200 11,400


TV = 18,120
29,520
Time line:
0 1 2 3 Years
k = 10%

-62,000 19,920 22,800 15,600


TV = 15,680
31,280
Expected increase in annual net income
ARR =
57 Initial (or average) investment .Answer (D) is correct. Breakeven time evaluates the rapidity
of new product development. The usual calculation determines the period beginning with project approval that is
required for the discounted cumulative cash inflows to equal the discounted cumulative cash outflows. However, it may
also be calculated as the point at which discounted cumulative cash inflows on a project equal discounted total cash
outflows. The concept is similar to the payback period, but it is more sophisticated because it incorporates the time value
of money. It also differs from the payback method because the period covered begins at the outset of a project, not
when the initial cash outflow occurs.
Answer (A) is incorrect because it is related to breakeven point, not breakeven time. Answer (B) is incorrect because the
payback period equals investment divided by annual undiscounted net cash inflows. Answer (C) is incorrect because the
payback period is the period required for total undiscounted cash inflows to equal total undiscounted cash outflows.
NPV
($)
NPV
$
A

B B
A

k Discount rate (%)


0 10% IRRB IRRA 0 16% 17% 18% 30%

NPV
$
NPV ($)
P
C

k 10% k
10% 15% 20% %

NPV NPV
$

A A

B
B n
Sigma CF t
k k t=1
0 7% 12% 15% 0 7% 12% 14%
(1 + K)t
NPV
$

X
Crossover

k
10% IRRY 12% IRRX

0 k = 12% 1 2 3 4 5 6

CFsA -15,000 4,000 4,000 4,000 4,000 4,000 4,000


Project B:
0 k = ? 1 2 6 Years
  
CFsB -14,815 5,100 5,100 5,100
Time lines:
Project A:
0 1 2 3 4 Years
k = 12%

CFsA -25,000 13,000 13,000 13,000 13,000


NPVA = ? = 17,663 Terminal value = 5,000
CF 4 = 18,000
Project B:
0 1 2 3 4 Years
k = ?

CFsB -25,000 15,247 15,247 15,247 15,247


NPVA = NPVB = 17,663 Terminal value = 0
CF 4 = 15,247
Time line:
0 1 2 3 Years
k = 18%

-3,000 1,728 1,920 1,152


Time line:
0 1 2 3 4 5 Years
k = 12%

-45,000 7,800 10,680 7,560 5,880 -1,920


Time line:
0 k = 9% 1 2 3 4 5 Years

-40,000 9,800 11,720 9,640 8,520 15,320


Time lines:
Project A:
0 k = 14% 1 2 3 4 Years

CFsA -200,000 71,104 71,104 71,104 71,104


Project B:
0 k = 10% 1 2 3 4 Years

CFsB -200,000 0 0 146,411 146,411


Time line:
0 k = 12% 1 2 10 Years
  
-50,000 6,000 6,000 6,000
Time lines:
Project A:
0 k = 12% 1 2 3 Periods

CFsA -5,000 2,000 2,500 2,250


Project B:
0 k = 14% 1 2 3 Periods

CFsB -5,000 3,000 2,600 2,900


0 1 2 3 4
| | | | |
CFsNew Tech -1,500 -315 -315 -315 -315 x x x
Time line:
0 1 2 3 Years
k = 14%

-42,000 14,280 16,200 11,400


TV = 18,120
29,520
Time line:
0 1 2 3 Years
k = 10%

-62,000 19,920 22,800 15,600


TV = 15,680
31,280
Expected increase in annual net income
ARR =
58 Initial (or average) investment .Answer (C) is correct. The profitability index is another term
for the excess present value index. It measures the ratio of the present value of future net cash inflows to the original
investment. In organizations with unlimited capital funds, this index will produce no conflicts in the decision process. If
capital rationing is necessary, the index will be an insufficient determinant. The capital available as well as the dollar
amount of the net present value must both be considered.
Answer (A) is incorrect because capital rationing is not a technique but rather a condition that characterizes capital
budgeting when insufficient capital is available to finance all profitable investment opportunities. Answer (B) is incorrect
because the average rate of return method does not divide the future cash flows by the cost of the investment. Answer
(D) is incorrect because the accounting rate of return does not recognize the time value of money.

NPV
($)
NPV
$
A

B B
A

k Discount rate (%)


0 10% IRRB IRRA 0 16% 17% 18% 30%

NPV
$
NPV ($)
P
C

k 10% k
10% 15% 20% %
NPV NPV
$

A A

B
B n
Sigma CF t
k k t=1
0 7% 12% 15% 0 7% 12% 14%
(1 + K)t
NPV
$

X
Crossover

k
10% IRRY 12% IRRX

0 k = 12% 1 2 3 4 5 6

CFsA -15,000 4,000 4,000 4,000 4,000 4,000 4,000


Project B:
0 k = ? 1 2 6 Years
  
CFsB -14,815 5,100 5,100 5,100
Time lines:
Project A:
0 1 2 3 4 Years
k = 12%

CFsA -25,000 13,000 13,000 13,000 13,000


NPVA = ? = 17,663 Terminal value = 5,000
CF 4 = 18,000
Project B:
0 1 2 3 4 Years
k = ?

CFsB -25,000 15,247 15,247 15,247 15,247


NPVA = NPVB = 17,663 Terminal value = 0
CF 4 = 15,247
Time line:
0 k = 18% 1 2 3 Years

-3,000 1,728 1,920 1,152


Time line:
0 1 2 3 4 5 Years
k = 12%

-45,000 7,800 10,680 7,560 5,880 -1,920


Time line:
0 1 2 3 4 5 Years
k = 9%

-40,000 9,800 11,720 9,640 8,520 15,320


Time lines:
Project A:
0 k = 14% 1 2 3 4 Years

CFsA -200,000 71,104 71,104 71,104 71,104


Project B:
0 k = 10% 1 2 3 4 Years

CFsB -200,000 0 0 146,411 146,411


Time line:
0 k = 12% 1 2 10 Years
  
-50,000 6,000 6,000 6,000
Time lines:
Project A:
0 k = 12% 1 2 3 Periods

CFsA -5,000 2,000 2,500 2,250


Project B:
0 k = 14% 1 2 3 Periods

CFsB -5,000 3,000 2,600 2,900


0 1 2 3 4
| | | | |
CFsNew Tech -1,500 -315 -315 -315 -315 x x x
Time line:
0 1 2 3 Years
k = 14%

-42,000 14,280 16,200 11,400


TV = 18,120
29,520
Time line:
0 1 2 3 Years
k = 10%

-62,000 19,920 22,800 15,600


TV = 15,680
31,280
Expected increase in annual net income
ARR =
59 Initial (or average) investment .Answer (D) is correct. The profitability index, also known as
the excess present value index, is the ratio of the present value of future net cash inflows to the initial net cash
investment (discounted cash outflows). This tool is a variation of the NPV method that facilitates comparison of different-
sized investments.
Answer (A) is incorrect because the cash inflows are also discounted in the profitability index. Answer (B) is incorrect
because the numerator is the discounted net cash inflows. Answer (C) is incorrect because the profitability index is
based on cash flows, not profits.
NPV
($)
NPV
$
A

B B
A

k Discount rate (%)


0 10% IRRB IRRA 0 16% 17% 18% 30%

NPV
$
NPV ($)
P
C

k 10% k
10% 15% 20% %

NPV NPV
$

A A

B
B n
Sigma CF t
k k t=1
0 7% 12% 15% 0 7% 12% 14%
(1 + K)t
NPV
$

X
Crossover

k
10% IRRY 12% IRRX

0 k = 12% 1 2 3 4 5 6

CFsA -15,000 4,000 4,000 4,000 4,000 4,000 4,000


Project B:
0 k = ? 1 2 6 Years
  
CFsB -14,815 5,100 5,100 5,100
Time lines:
Project A:
0 1 2 3 4 Years
k = 12%

CFsA -25,000 13,000 13,000 13,000 13,000


NPVA = ? = 17,663 Terminal value = 5,000
CF 4 = 18,000
Project B:
0 1 2 3 4 Years
k = ?

CFsB -25,000 15,247 15,247 15,247 15,247


NPVA = NPVB = 17,663 Terminal value = 0
CF 4 = 15,247
Time line:
0 1 2 3 Years
k = 18%

-3,000 1,728 1,920 1,152


Time line:
0 1 2 3 4 5 Years
k = 12%

-45,000 7,800 10,680 7,560 5,880 -1,920


Time line:
0 k = 9% 1 2 3 4 5 Years

-40,000 9,800 11,720 9,640 8,520 15,320


Time lines:
Project A:
0 k = 14% 1 2 3 4 Years

CFsA -200,000 71,104 71,104 71,104 71,104


Project B:
0 k = 10% 1 2 3 4 Years

CFsB -200,000 0 0 146,411 146,411


Time line:
0 k = 12% 1 2 10 Years
  
-50,000 6,000 6,000 6,000
Time lines:
Project A:
0 k = 12% 1 2 3 Periods

CFsA -5,000 2,000 2,500 2,250


Project B:
0 k = 14% 1 2 3 Periods

CFsB -5,000 3,000 2,600 2,900


0 1 2 3 4
| | | | |
CFsNew Tech -1,500 -315 -315 -315 -315 x x x
Time line:
0 1 2 3 Years
k = 14%

-42,000 14,280 16,200 11,400


TV = 18,120
29,520
Time line:
0 1 2 3 Years
k = 10%

-62,000 19,920 22,800 15,600


TV = 15,680
31,280
Expected increase in annual net income
ARR =
60 Initial (or average) investment .Answer (B) is correct. The net present value (NPV) method
computes the discounted present value of future cash inflows to determine whether they are greater than the initial cash
outflow. The discount rate (cost of capital or hurdle rate) must be known to discount the future cash inflows. If the NPV is
positive (present value of future cash inflows exceeds initial cash outflow), the project should be accepted. If the NPV is
negative, the project should be rejected.
Answer (A) is incorrect because the accounting rate of return uses net income (not cash flows) to determine a rate of
profitability. Answer (C) is incorrect because the internal rate of return is the rate at which NPV is zero. The minimum
desired rate of return is not used. Answer (D) is incorrect because the payback method measures the time required to
complete the return of the original investment. It gives no consideration to the time value of money or to returns after the
payback period.

NPV
($)
NPV
$
A

B B
A

k Discount rate (%)


0 10% IRRB IRRA 0 16% 17% 18% 30%

NPV
$
NPV ($)
P
C

k 10% k
10% 15% 20% %
NPV NPV
$

A A

B
B n
Sigma CF t
k k t=1
0 7% 12% 15% 0 7% 12% 14%
(1 + K)t
NPV
$

X
Crossover

k
10% IRRY 12% IRRX

0 k = 12% 1 2 3 4 5 6

CFsA -15,000 4,000 4,000 4,000 4,000 4,000 4,000


Project B:
0 k = ? 1 2 6 Years
  
CFsB -14,815 5,100 5,100 5,100
Time lines:
Project A:
0 1 2 3 4 Years
k = 12%

CFsA -25,000 13,000 13,000 13,000 13,000


NPVA = ? = 17,663 Terminal value = 5,000
CF 4 = 18,000
Project B:
0 1 2 3 4 Years
k = ?

CFsB -25,000 15,247 15,247 15,247 15,247


NPVA = NPVB = 17,663 Terminal value = 0
CF 4 = 15,247
Time line:
0 k = 18% 1 2 3 Years

-3,000 1,728 1,920 1,152


Time line:
0 1 2 3 4 5 Years
k = 12%

-45,000 7,800 10,680 7,560 5,880 -1,920


Time line:
0 1 2 3 4 5 Years
k = 9%

-40,000 9,800 11,720 9,640 8,520 15,320


Time lines:
Project A:
0 k = 14% 1 2 3 4 Years

CFsA -200,000 71,104 71,104 71,104 71,104


Project B:
0 k = 10% 1 2 3 4 Years

CFsB -200,000 0 0 146,411 146,411


Time line:
0 k = 12% 1 2 10 Years
  
-50,000 6,000 6,000 6,000
Time lines:
Project A:
0 k = 12% 1 2 3 Periods

CFsA -5,000 2,000 2,500 2,250


Project B:
0 k = 14% 1 2 3 Periods

CFsB -5,000 3,000 2,600 2,900


0 1 2 3 4
| | | | |
CFsNew Tech -1,500 -315 -315 -315 -315 x x x
Time line:
0 1 2 3 Years
k = 14%

-42,000 14,280 16,200 11,400


TV = 18,120
29,520
Time line:
0 1 2 3 Years
k = 10%

-62,000 19,920 22,800 15,600


TV = 15,680
31,280
Expected increase in annual net income
ARR =
61 Initial (or average) investment .Answer (A) is correct. The net present value method
discounts future cash flows to the present value using some arbitrary rate of return, which is presumably the firm's cost
of capital. The initial cost of the project is then deducted from the present value. If the present value of the future cash
flows exceeds the cost, the investment is considered to be acceptable.
Answer (B) is incorrect because the payback method does not recognize the time value of money. Answer (C) is
incorrect because the average rate of return method does not use the firm's cost of capital as a discount rate. Answer
(D) is incorrect because the accounting rate of return method does not recognize the time value of money.
NPV
($)
NPV
$
A

B B
A

k Discount rate (%)


0 10% IRRB IRRA 0 16% 17% 18% 30%

NPV
$
NPV ($)
P
C

k 10% k
10% 15% 20% %

NPV NPV
$

A A

B
B n
Sigma CF t
k k t=1
0 7% 12% 15% 0 7% 12% 14%
(1 + K)t
NPV
$

X
Crossover

k
10% IRRY 12% IRRX

0 k = 12% 1 2 3 4 5 6

CFsA -15,000 4,000 4,000 4,000 4,000 4,000 4,000


Project B:
0 k = ? 1 2 6 Years
  
CFsB -14,815 5,100 5,100 5,100
Time lines:
Project A:
0 1 2 3 4 Years
k = 12%

CFsA -25,000 13,000 13,000 13,000 13,000


NPVA = ? = 17,663 Terminal value = 5,000
CF 4 = 18,000
Project B:
0 1 2 3 4 Years
k = ?

CFsB -25,000 15,247 15,247 15,247 15,247


NPVA = NPVB = 17,663 Terminal value = 0
CF 4 = 15,247
Time line:
0 1 2 3 Years
k = 18%

-3,000 1,728 1,920 1,152


Time line:
0 1 2 3 4 5 Years
k = 12%

-45,000 7,800 10,680 7,560 5,880 -1,920


Time line:
0 k = 9% 1 2 3 4 5 Years

-40,000 9,800 11,720 9,640 8,520 15,320


Time lines:
Project A:
0 k = 14% 1 2 3 4 Years

CFsA -200,000 71,104 71,104 71,104 71,104


Project B:
0 k = 10% 1 2 3 4 Years

CFsB -200,000 0 0 146,411 146,411


Time line:
0 k = 12% 1 2 10 Years
  
-50,000 6,000 6,000 6,000
Time lines:
Project A:
0 k = 12% 1 2 3 Periods

CFsA -5,000 2,000 2,500 2,250


Project B:
0 k = 14% 1 2 3 Periods

CFsB -5,000 3,000 2,600 2,900


0 1 2 3 4
| | | | |
CFsNew Tech -1,500 -315 -315 -315 -315 x x x
Time line:
0 1 2 3 Years
k = 14%

-42,000 14,280 16,200 11,400


TV = 18,120
29,520
Time line:
0 1 2 3 Years
k = 10%

-62,000 19,920 22,800 15,600


TV = 15,680
31,280
Expected increase in annual net income
ARR =
62 Initial (or average) investment .REQUIRED: The effect on the value of the firm and its stock
price of investment opportunities.
DISCUSSION: (A) Investments with present values in excess of their costs (positive NPVs) that can be identified or
created by the capital budgeting activities of the firm will have a positive impact on firm value and on the price of the
common stock of the firm. Accordingly, the more effective capital budgeting is, the higher the stock price.
Answer (B) is incorrect because positive NPV investments will increase, not decrease firm value and share price.
Answers (C) and (D) are incorrect because investments with present values equal to their costs have a zero NPV and
neither increase nor decrease firm value and share price.

NPV
($)
NPV
$
A

B B
A

k Discount rate (%)


0 10% IRRB IRRA 0 16% 17% 18% 30%

NPV
$
NPV ($)
P
C

k 10% k
10% 15% 20% %
NPV NPV
$

A A

B
B n
Sigma CF t
k k t=1
0 7% 12% 15% 0 7% 12% 14%
(1 + K)t
NPV
$

X
Crossover

k
10% IRRY 12% IRRX

0 k = 12% 1 2 3 4 5 6

CFsA -15,000 4,000 4,000 4,000 4,000 4,000 4,000


Project B:
0 k = ? 1 2 6 Years
  
CFsB -14,815 5,100 5,100 5,100
Time lines:
Project A:
0 1 2 3 4 Years
k = 12%

CFsA -25,000 13,000 13,000 13,000 13,000


NPVA = ? = 17,663 Terminal value = 5,000
CF 4 = 18,000
Project B:
0 1 2 3 4 Years
k = ?

CFsB -25,000 15,247 15,247 15,247 15,247


NPVA = NPVB = 17,663 Terminal value = 0
CF 4 = 15,247
Time line:
0 k = 18% 1 2 3 Years

-3,000 1,728 1,920 1,152


Time line:
0 1 2 3 4 5 Years
k = 12%

-45,000 7,800 10,680 7,560 5,880 -1,920


Time line:
0 1 2 3 4 5 Years
k = 9%

-40,000 9,800 11,720 9,640 8,520 15,320


Time lines:
Project A:
0 k = 14% 1 2 3 4 Years

CFsA -200,000 71,104 71,104 71,104 71,104


Project B:
0 k = 10% 1 2 3 4 Years

CFsB -200,000 0 0 146,411 146,411


Time line:
0 k = 12% 1 2 10 Years
  
-50,000 6,000 6,000 6,000
Time lines:
Project A:
0 k = 12% 1 2 3 Periods

CFsA -5,000 2,000 2,500 2,250


Project B:
0 k = 14% 1 2 3 Periods

CFsB -5,000 3,000 2,600 2,900


0 1 2 3 4
| | | | |
CFsNew Tech -1,500 -315 -315 -315 -315 x x x
Time line:
0 1 2 3 Years
k = 14%

-42,000 14,280 16,200 11,400


TV = 18,120
29,520
Time line:
0 1 2 3 Years
k = 10%

-62,000 19,920 22,800 15,600


TV = 15,680
31,280
Expected increase in annual net income
ARR =
63 Initial (or average) investment .Answer (A) is correct. The NPV method computes the
present value of future cash inflows to determine whether they are greater than the initial cash outflow. Future cash
inflows include any salvage value on facilities. Included in the initial investment are the cost of new equipment and other
facilities, and additional working capital needed for operations during the term of the project. The discount rate (cost of
capital or hurdle rate) must be known to discount the future cash inflows. If the NPV is positive, the project should be
accepted. The method of funding a project is a decision separate from that of whether to invest.
Answer (B) is incorrect because the initial costs of the project, including additional working capital needs, are necessary
to determine the NPV. Answer (C) is incorrect because the initial costs of the project, including additional working capital
needs, are necessary to determine the NPV. Answer (D) is incorrect because the project's salvage value is a future cash
inflow to be discounted.
NPV
($)
NPV
$
A

B B
A

k Discount rate (%)


0 10% IRRB IRRA 0 16% 17% 18% 30%

NPV
$
NPV ($)
P
C

k 10% k
10% 15% 20% %

NPV NPV
$

A A

B
B n
Sigma CF t
k k t=1
0 7% 12% 15% 0 7% 12% 14%
(1 + K)t
NPV
$

X
Crossover

k
10% IRRY 12% IRRX

0 k = 12% 1 2 3 4 5 6

CFsA -15,000 4,000 4,000 4,000 4,000 4,000 4,000


Project B:
0 k = ? 1 2 6 Years
  
CFsB -14,815 5,100 5,100 5,100
Time lines:
Project A:
0 1 2 3 4 Years
k = 12%

CFsA -25,000 13,000 13,000 13,000 13,000


NPVA = ? = 17,663 Terminal value = 5,000
CF 4 = 18,000
Project B:
0 1 2 3 4 Years
k = ?

CFsB -25,000 15,247 15,247 15,247 15,247


NPVA = NPVB = 17,663 Terminal value = 0
CF 4 = 15,247
Time line:
0 1 2 3 Years
k = 18%

-3,000 1,728 1,920 1,152


Time line:
0 1 2 3 4 5 Years
k = 12%

-45,000 7,800 10,680 7,560 5,880 -1,920


Time line:
0 k = 9% 1 2 3 4 5 Years

-40,000 9,800 11,720 9,640 8,520 15,320


Time lines:
Project A:
0 k = 14% 1 2 3 4 Years

CFsA -200,000 71,104 71,104 71,104 71,104


Project B:
0 k = 10% 1 2 3 4 Years

CFsB -200,000 0 0 146,411 146,411


Time line:
0 k = 12% 1 2 10 Years
  
-50,000 6,000 6,000 6,000
Time lines:
Project A:
0 k = 12% 1 2 3 Periods

CFsA -5,000 2,000 2,500 2,250


Project B:
0 k = 14% 1 2 3 Periods

CFsB -5,000 3,000 2,600 2,900


0 1 2 3 4
| | | | |
CFsNew Tech -1,500 -315 -315 -315 -315 x x x
Time line:
0 1 2 3 Years
k = 14%

-42,000 14,280 16,200 11,400


TV = 18,120
29,520
Time line:
0 1 2 3 Years
k = 10%

-62,000 19,920 22,800 15,600


TV = 15,680
31,280
Expected increase in annual net income
ARR =
64 Initial (or average) investment .REQUIRED: The variable(s) considered in the NPV
calculation.
DISCUSSION: (D)The NPV is the difference between the present value of the future cash flows from the project
discounted at an appropriate interest rate and the initial investment. If the NPV is zero or greater, the investment may
be economically rational. The method is a technique for ranking investment proposals. Consequently, the time value of
the cash flows over the life of the project is considered.
Answers (A), (B), and (C) are incorrect because the time value of the cash flows over the life of the project is
considered.

NPV
($)
NPV
$
A

B B
A

k Discount rate (%)


0 10% IRRB IRRA 0 16% 17% 18% 30%

NPV
$
NPV ($)
P
C

k 10% k
10% 15% 20% %
NPV NPV
$

A A

B
B n
Sigma CF t
k k t=1
0 7% 12% 15% 0 7% 12% 14%
(1 + K)t
NPV
$

X
Crossover

k
10% IRRY 12% IRRX

0 k = 12% 1 2 3 4 5 6

CFsA -15,000 4,000 4,000 4,000 4,000 4,000 4,000


Project B:
0 k = ? 1 2 6 Years
  
CFsB -14,815 5,100 5,100 5,100
Time lines:
Project A:
0 1 2 3 4 Years
k = 12%

CFsA -25,000 13,000 13,000 13,000 13,000


NPVA = ? = 17,663 Terminal value = 5,000
CF 4 = 18,000
Project B:
0 1 2 3 4 Years
k = ?

CFsB -25,000 15,247 15,247 15,247 15,247


NPVA = NPVB = 17,663 Terminal value = 0
CF 4 = 15,247
Time line:
0 k = 18% 1 2 3 Years

-3,000 1,728 1,920 1,152


Time line:
0 1 2 3 4 5 Years
k = 12%

-45,000 7,800 10,680 7,560 5,880 -1,920


Time line:
0 1 2 3 4 5 Years
k = 9%

-40,000 9,800 11,720 9,640 8,520 15,320


Time lines:
Project A:
0 k = 14% 1 2 3 4 Years

CFsA -200,000 71,104 71,104 71,104 71,104


Project B:
0 k = 10% 1 2 3 4 Years

CFsB -200,000 0 0 146,411 146,411


Time line:
0 k = 12% 1 2 10 Years
  
-50,000 6,000 6,000 6,000
Time lines:
Project A:
0 k = 12% 1 2 3 Periods

CFsA -5,000 2,000 2,500 2,250


Project B:
0 k = 14% 1 2 3 Periods

CFsB -5,000 3,000 2,600 2,900


0 1 2 3 4
| | | | |
CFsNew Tech -1,500 -315 -315 -315 -315 x x x
Time line:
0 1 2 3 Years
k = 14%

-42,000 14,280 16,200 11,400


TV = 18,120
29,520
Time line:
0 1 2 3 Years
k = 10%

-62,000 19,920 22,800 15,600


TV = 15,680
31,280
Expected increase in annual net income
ARR =
65 Initial (or average) investment .Answer (D) is correct. The effect of assuming cash flows
occur at the end of the year simply understates the present values of the future cash flows; in reality, they probably
occur on the average at mid-year.
Answer (A) is incorrect because cash flows in investment decisions do not all occur at the end of each year. Answer (B)
is incorrect because discounting cash flows approximately 6 months longer understates rather than overstates. Answer
(C) is incorrect because the effect of using the year-end assumption produces a slight conservatism in the model but
does not render the results unusable.
NPV
($)
NPV
$
A

B B
A

k Discount rate (%)


0 10% IRRB IRRA 0 16% 17% 18% 30%

NPV
$
NPV ($)
P
C

k 10% k
10% 15% 20% %

NPV NPV
$

A A

B
B n
Sigma CF t
k k t=1
0 7% 12% 15% 0 7% 12% 14%
(1 + K)t
NPV
$

X
Crossover

k
10% IRRY 12% IRRX

0 k = 12% 1 2 3 4 5 6

CFsA -15,000 4,000 4,000 4,000 4,000 4,000 4,000


Project B:
0 k = ? 1 2 6 Years
  
CFsB -14,815 5,100 5,100 5,100
Time lines:
Project A:
0 1 2 3 4 Years
k = 12%

CFsA -25,000 13,000 13,000 13,000 13,000


NPVA = ? = 17,663 Terminal value = 5,000
CF 4 = 18,000
Project B:
0 1 2 3 4 Years
k = ?

CFsB -25,000 15,247 15,247 15,247 15,247


NPVA = NPVB = 17,663 Terminal value = 0
CF 4 = 15,247
Time line:
0 1 2 3 Years
k = 18%

-3,000 1,728 1,920 1,152


Time line:
0 1 2 3 4 5 Years
k = 12%

-45,000 7,800 10,680 7,560 5,880 -1,920


Time line:
0 k = 9% 1 2 3 4 5 Years

-40,000 9,800 11,720 9,640 8,520 15,320


Time lines:
Project A:
0 k = 14% 1 2 3 4 Years

CFsA -200,000 71,104 71,104 71,104 71,104


Project B:
0 k = 10% 1 2 3 4 Years

CFsB -200,000 0 0 146,411 146,411


Time line:
0 k = 12% 1 2 10 Years
  
-50,000 6,000 6,000 6,000
Time lines:
Project A:
0 k = 12% 1 2 3 Periods

CFsA -5,000 2,000 2,500 2,250


Project B:
0 k = 14% 1 2 3 Periods

CFsB -5,000 3,000 2,600 2,900


0 1 2 3 4
| | | | |
CFsNew Tech -1,500 -315 -315 -315 -315 x x x
Time line:
0 1 2 3 Years
k = 14%

-42,000 14,280 16,200 11,400


TV = 18,120
29,520
Time line:
0 1 2 3 Years
k = 10%

-62,000 19,920 22,800 15,600


TV = 15,680
31,280
Expected increase in annual net income
ARR =
66 Initial (or average) investment .Answer (C) is correct. The NPV method is used when the
discount rate (cost of capital) is specified. It assumes that cash flows from the investment can be reinvested at the
particular project's cost of capital (the discount rate).
Answer (A) is incorrect because the internal rate of return method assumes that cash flows are reinvested at the internal
rate of return. Answer (B) is incorrect because the NPV method assumes that cash flows are reinvested at the NPV
discount rate. Answer (D) is incorrect because the NPV method assumes that cash flows are reinvested at the NPV
discount rate.

NPV
($)
NPV
$
A

B B
A

k Discount rate (%)


0 10% IRRB IRRA 0 16% 17% 18% 30%

NPV
$
NPV ($)
P
C

k 10% k
10% 15% 20% %
NPV NPV
$

A A

B
B n
Sigma CF t
k k t=1
0 7% 12% 15% 0 7% 12% 14%
(1 + K)t
NPV
$

X
Crossover

k
10% IRRY 12% IRRX

0 k = 12% 1 2 3 4 5 6

CFsA -15,000 4,000 4,000 4,000 4,000 4,000 4,000


Project B:
0 k = ? 1 2 6 Years
  
CFsB -14,815 5,100 5,100 5,100
Time lines:
Project A:
0 1 2 3 4 Years
k = 12%

CFsA -25,000 13,000 13,000 13,000 13,000


NPVA = ? = 17,663 Terminal value = 5,000
CF 4 = 18,000
Project B:
0 1 2 3 4 Years
k = ?

CFsB -25,000 15,247 15,247 15,247 15,247


NPVA = NPVB = 17,663 Terminal value = 0
CF 4 = 15,247
Time line:
0 k = 18% 1 2 3 Years

-3,000 1,728 1,920 1,152


Time line:
0 1 2 3 4 5 Years
k = 12%

-45,000 7,800 10,680 7,560 5,880 -1,920


Time line:
0 1 2 3 4 5 Years
k = 9%

-40,000 9,800 11,720 9,640 8,520 15,320


Time lines:
Project A:
0 k = 14% 1 2 3 4 Years

CFsA -200,000 71,104 71,104 71,104 71,104


Project B:
0 k = 10% 1 2 3 4 Years

CFsB -200,000 0 0 146,411 146,411


Time line:
0 k = 12% 1 2 10 Years
  
-50,000 6,000 6,000 6,000
Time lines:
Project A:
0 k = 12% 1 2 3 Periods

CFsA -5,000 2,000 2,500 2,250


Project B:
0 k = 14% 1 2 3 Periods

CFsB -5,000 3,000 2,600 2,900


0 1 2 3 4
| | | | |
CFsNew Tech -1,500 -315 -315 -315 -315 x x x
Time line:
0 1 2 3 Years
k = 14%

-42,000 14,280 16,200 11,400


TV = 18,120
29,520
Time line:
0 1 2 3 Years
k = 10%

-62,000 19,920 22,800 15,600


TV = 15,680
31,280
Expected increase in annual net income
ARR =
67 Initial (or average) investment .Answer (D) is correct. The NPV method assumes that
periodic cash inflows earned over the life of an investment are reinvested at the company's cost of capital (i.e., the
discount rate used in the analysis). This is contrary to the assumption under the internal rate of return method, which
assumes that cash inflows are reinvested at the internal rate of return. As a result of this difference, the two methods will
occasionally give different rankings of investment alternatives.
Answer (A) is incorrect because the NPV method assumes that cash inflows are reinvested at the discount rate used in
the NPV calculation. Answer (B) is incorrect because the NPV method assumes that cash inflows are reinvested at the
discount rate used in the NPV calculation. Answer (C) is incorrect because the internal rate of return method assumes a
reinvestment rate equal to the rate of return on the project.
NPV
($)
NPV
$
A

B B
A

k Discount rate (%)


0 10% IRRB IRRA 0 16% 17% 18% 30%

NPV
$
NPV ($)
P
C

k 10% k
10% 15% 20% %

NPV NPV
$

A A

B
B n
Sigma CF t
k k t=1
0 7% 12% 15% 0 7% 12% 14%
(1 + K)t
NPV
$

X
Crossover

k
10% IRRY 12% IRRX

0 k = 12% 1 2 3 4 5 6

CFsA -15,000 4,000 4,000 4,000 4,000 4,000 4,000


Project B:
0 k = ? 1 2 6 Years
  
CFsB -14,815 5,100 5,100 5,100
Time lines:
Project A:
0 1 2 3 4 Years
k = 12%

CFsA -25,000 13,000 13,000 13,000 13,000


NPVA = ? = 17,663 Terminal value = 5,000
CF 4 = 18,000
Project B:
0 1 2 3 4 Years
k = ?

CFsB -25,000 15,247 15,247 15,247 15,247


NPVA = NPVB = 17,663 Terminal value = 0
CF 4 = 15,247
Time line:
0 1 2 3 Years
k = 18%

-3,000 1,728 1,920 1,152


Time line:
0 1 2 3 4 5 Years
k = 12%

-45,000 7,800 10,680 7,560 5,880 -1,920


Time line:
0 k = 9% 1 2 3 4 5 Years

-40,000 9,800 11,720 9,640 8,520 15,320


Time lines:
Project A:
0 k = 14% 1 2 3 4 Years

CFsA -200,000 71,104 71,104 71,104 71,104


Project B:
0 k = 10% 1 2 3 4 Years

CFsB -200,000 0 0 146,411 146,411


Time line:
0 k = 12% 1 2 10 Years
  
-50,000 6,000 6,000 6,000
Time lines:
Project A:
0 k = 12% 1 2 3 Periods

CFsA -5,000 2,000 2,500 2,250


Project B:
0 k = 14% 1 2 3 Periods

CFsB -5,000 3,000 2,600 2,900


0 1 2 3 4
| | | | |
CFsNew Tech -1,500 -315 -315 -315 -315 x x x
Time line:
0 1 2 3 Years
k = 14%

-42,000 14,280 16,200 11,400


TV = 18,120
29,520
Time line:
0 1 2 3 Years
k = 10%

-62,000 19,920 22,800 15,600


TV = 15,680
31,280
Expected increase in annual net income
ARR =
68 Initial (or average) investment .Answer (B) is correct. The NPV method calculates the
present values of estimated future net cash inflows and compares the total with the net cost of the investment. The cost
of capital must be specified. If the NPV is positive, the project should be accepted. The IRR method computes the
interest rate at which the NPV is zero. The IRR method is relatively easy to use when cash inflows are the same from
one year to the next. However, when cash inflows differ from year to year, the IRR can be found only through the use of
trial and error. In such cases, the NPV method is usually easier to apply. Also, the NPV method can be used when the
rate of return required for each year varies. For example, a company might want to achieve a higher rate of return in
later years when risk might be greater. Only the NPV method can incorporate varying levels of rates of return.
Answer (A) is incorrect because the IRR method calculates a rate of return. Answer (C) is incorrect because the IRR is
the rate at which NPV is zero. Answer (D) is incorrect because both methods discount cash flows.

NPV
($)
NPV
$
A

B B
A

k Discount rate (%)


0 10% IRRB IRRA 0 16% 17% 18% 30%

NPV
$
NPV ($)
P
C

k 10% k
10% 15% 20% %
NPV NPV
$

A A

B
B n
Sigma CF t
k k t=1
0 7% 12% 15% 0 7% 12% 14%
(1 + K)t
NPV
$

X
Crossover

k
10% IRRY 12% IRRX

0 k = 12% 1 2 3 4 5 6

CFsA -15,000 4,000 4,000 4,000 4,000 4,000 4,000


Project B:
0 k = ? 1 2 6 Years
  
CFsB -14,815 5,100 5,100 5,100
Time lines:
Project A:
0 1 2 3 4 Years
k = 12%

CFsA -25,000 13,000 13,000 13,000 13,000


NPVA = ? = 17,663 Terminal value = 5,000
CF 4 = 18,000
Project B:
0 1 2 3 4 Years
k = ?

CFsB -25,000 15,247 15,247 15,247 15,247


NPVA = NPVB = 17,663 Terminal value = 0
CF 4 = 15,247
Time line:
0 k = 18% 1 2 3 Years

-3,000 1,728 1,920 1,152


Time line:
0 1 2 3 4 5 Years
k = 12%

-45,000 7,800 10,680 7,560 5,880 -1,920


Time line:
0 1 2 3 4 5 Years
k = 9%

-40,000 9,800 11,720 9,640 8,520 15,320


Time lines:
Project A:
0 k = 14% 1 2 3 4 Years

CFsA -200,000 71,104 71,104 71,104 71,104


Project B:
0 k = 10% 1 2 3 4 Years

CFsB -200,000 0 0 146,411 146,411


Time line:
0 k = 12% 1 2 10 Years
  
-50,000 6,000 6,000 6,000
Time lines:
Project A:
0 k = 12% 1 2 3 Periods

CFsA -5,000 2,000 2,500 2,250


Project B:
0 k = 14% 1 2 3 Periods

CFsB -5,000 3,000 2,600 2,900


0 1 2 3 4
| | | | |
CFsNew Tech -1,500 -315 -315 -315 -315 x x x
Time line:
0 1 2 3 Years
k = 14%

-42,000 14,280 16,200 11,400


TV = 18,120
29,520
Time line:
0 1 2 3 Years
k = 10%

-62,000 19,920 22,800 15,600


TV = 15,680
31,280
Expected increase in annual net income
ARR =
69 Initial (or average) investment .Answer (C) is correct. The NPV is broadly defined as the
excess of the present value of the estimated net cash inflows over the net cost of the investment. A discount rate has to
be stipulated by the person conducting the analysis. A disadvantage is that it does not provide the true rate of return for
an investment, only that the rate of return is higher than a stipulated discount rate (which may be the cost of capital).
Answer (A) is incorrect because the ability to perform sensitivity analysis is an advantage of the NPV method. Answer
(B) is incorrect because the NPV method does not compute the true interest rate. Answer (D) is incorrect because the
IRR method, not the NPV method, uses a trial-and-error approach when cash flows are not identical from year to year.
NPV
($)
NPV
$
A

B B
A

k Discount rate (%)


0 10% IRRB IRRA 0 16% 17% 18% 30%

NPV
$
NPV ($)
P
C

k 10% k
10% 15% 20% %

NPV NPV
$

A A

B
B n
Sigma CF t
k k t=1
0 7% 12% 15% 0 7% 12% 14%
(1 + K)t
NPV
$

X
Crossover

k
10% IRRY 12% IRRX

0 k = 12% 1 2 3 4 5 6

CFsA -15,000 4,000 4,000 4,000 4,000 4,000 4,000


Project B:
0 k = ? 1 2 6 Years
  
CFsB -14,815 5,100 5,100 5,100
Time lines:
Project A:
0 1 2 3 4 Years
k = 12%

CFsA -25,000 13,000 13,000 13,000 13,000


NPVA = ? = 17,663 Terminal value = 5,000
CF 4 = 18,000
Project B:
0 1 2 3 4 Years
k = ?

CFsB -25,000 15,247 15,247 15,247 15,247


NPVA = NPVB = 17,663 Terminal value = 0
CF 4 = 15,247
Time line:
0 1 2 3 Years
k = 18%

-3,000 1,728 1,920 1,152


Time line:
0 1 2 3 4 5 Years
k = 12%

-45,000 7,800 10,680 7,560 5,880 -1,920


Time line:
0 k = 9% 1 2 3 4 5 Years

-40,000 9,800 11,720 9,640 8,520 15,320


Time lines:
Project A:
0 k = 14% 1 2 3 4 Years

CFsA -200,000 71,104 71,104 71,104 71,104


Project B:
0 k = 10% 1 2 3 4 Years

CFsB -200,000 0 0 146,411 146,411


Time line:
0 k = 12% 1 2 10 Years
  
-50,000 6,000 6,000 6,000
Time lines:
Project A:
0 k = 12% 1 2 3 Periods

CFsA -5,000 2,000 2,500 2,250


Project B:
0 k = 14% 1 2 3 Periods

CFsB -5,000 3,000 2,600 2,900


0 1 2 3 4
| | | | |
CFsNew Tech -1,500 -315 -315 -315 -315 x x x
Time line:
0 1 2 3 Years
k = 14%

-42,000 14,280 16,200 11,400


TV = 18,120
29,520
Time line:
0 1 2 3 Years
k = 10%

-62,000 19,920 22,800 15,600


TV = 15,680
31,280
Expected increase in annual net income
ARR =
70 Initial (or average) investment .Answer (C) is correct. The discount rate most often used in
present value calculations is the minimum desired rate of return as set by management. The NPV arrived at in this
calculation is a first step in the decision process. It indicates how the project's return compares with the minimum
desired rate of return.
Answer (A) is incorrect because the Federal Reserve rate may be considered; however, the firm will set its minimum
desired rate of return in view of its needs. Answer (B) is incorrect because the Treasury bill rate may be considered;
however, the firm will set its minimum desired rate of return in view of its needs. Answer (D) is incorrect because the
prime rate may be considered; however, the firm will set its minimum desired rate of return in view of its needs.

NPV
($)
NPV
$
A

B B
A

k Discount rate (%)


0 10% IRRB IRRA 0 16% 17% 18% 30%

NPV
$
NPV ($)
P
C

k 10% k
10% 15% 20% %
NPV NPV
$

A A

B
B n
Sigma CF t
k k t=1
0 7% 12% 15% 0 7% 12% 14%
(1 + K)t
NPV
$

X
Crossover

k
10% IRRY 12% IRRX

0 k = 12% 1 2 3 4 5 6

CFsA -15,000 4,000 4,000 4,000 4,000 4,000 4,000


Project B:
0 k = ? 1 2 6 Years
  
CFsB -14,815 5,100 5,100 5,100
Time lines:
Project A:
0 1 2 3 4 Years
k = 12%

CFsA -25,000 13,000 13,000 13,000 13,000


NPVA = ? = 17,663 Terminal value = 5,000
CF 4 = 18,000
Project B:
0 1 2 3 4 Years
k = ?

CFsB -25,000 15,247 15,247 15,247 15,247


NPVA = NPVB = 17,663 Terminal value = 0
CF 4 = 15,247
Time line:
0 k = 18% 1 2 3 Years

-3,000 1,728 1,920 1,152


Time line:
0 1 2 3 4 5 Years
k = 12%

-45,000 7,800 10,680 7,560 5,880 -1,920


Time line:
0 1 2 3 4 5 Years
k = 9%

-40,000 9,800 11,720 9,640 8,520 15,320


Time lines:
Project A:
0 k = 14% 1 2 3 4 Years

CFsA -200,000 71,104 71,104 71,104 71,104


Project B:
0 k = 10% 1 2 3 4 Years

CFsB -200,000 0 0 146,411 146,411


Time line:
0 k = 12% 1 2 10 Years
  
-50,000 6,000 6,000 6,000
Time lines:
Project A:
0 k = 12% 1 2 3 Periods

CFsA -5,000 2,000 2,500 2,250


Project B:
0 k = 14% 1 2 3 Periods

CFsB -5,000 3,000 2,600 2,900


0 1 2 3 4
| | | | |
CFsNew Tech -1,500 -315 -315 -315 -315 x x x
Time line:
0 1 2 3 Years
k = 14%

-42,000 14,280 16,200 11,400


TV = 18,120
29,520
Time line:
0 1 2 3 Years
k = 10%

-62,000 19,920 22,800 15,600


TV = 15,680
31,280
Expected increase in annual net income
ARR =
71 Initial (or average) investment .Answer (A) is correct. The rate used to discount future cash
flows is sometimes called the cost of capital, the discount rate, the cutoff rate, or the hurdle rate. A risk-free rate is the
rate available on risk-free investments such as government bonds. The risk-free rate is not equivalent to the cost of
capital because the latter must incorporate a risk premium.
Answer (B) is incorrect because the rate used under the NPV method is the company's cost of capital. Answer (C) is
incorrect because the NPV method discounts future cash flows to their present values. Answer (D) is incorrect because
the cost of capital is often called a cutoff rate. Investments yielding less than the cost of capital should not be made.
NPV
($)
NPV
$
A

B B
A

k Discount rate (%)


0 10% IRRB IRRA 0 16% 17% 18% 30%

NPV
$
NPV ($)
P
C

k 10% k
10% 15% 20% %

NPV NPV
$

A A

B
B n
Sigma CF t
k k t=1
0 7% 12% 15% 0 7% 12% 14%
(1 + K)t
NPV
$

X
Crossover

k
10% IRRY 12% IRRX

0 k = 12% 1 2 3 4 5 6

CFsA -15,000 4,000 4,000 4,000 4,000 4,000 4,000


Project B:
0 k = ? 1 2 6 Years
  
CFsB -14,815 5,100 5,100 5,100
Time lines:
Project A:
0 1 2 3 4 Years
k = 12%

CFsA -25,000 13,000 13,000 13,000 13,000


NPVA = ? = 17,663 Terminal value = 5,000
CF 4 = 18,000
Project B:
0 1 2 3 4 Years
k = ?

CFsB -25,000 15,247 15,247 15,247 15,247


NPVA = NPVB = 17,663 Terminal value = 0
CF 4 = 15,247
Time line:
0 1 2 3 Years
k = 18%

-3,000 1,728 1,920 1,152


Time line:
0 1 2 3 4 5 Years
k = 12%

-45,000 7,800 10,680 7,560 5,880 -1,920


Time line:
0 k = 9% 1 2 3 4 5 Years

-40,000 9,800 11,720 9,640 8,520 15,320


Time lines:
Project A:
0 k = 14% 1 2 3 4 Years

CFsA -200,000 71,104 71,104 71,104 71,104


Project B:
0 k = 10% 1 2 3 4 Years

CFsB -200,000 0 0 146,411 146,411


Time line:
0 k = 12% 1 2 10 Years
  
-50,000 6,000 6,000 6,000
Time lines:
Project A:
0 k = 12% 1 2 3 Periods

CFsA -5,000 2,000 2,500 2,250


Project B:
0 k = 14% 1 2 3 Periods

CFsB -5,000 3,000 2,600 2,900


0 1 2 3 4
| | | | |
CFsNew Tech -1,500 -315 -315 -315 -315 x x x
Time line:
0 1 2 3 Years
k = 14%

-42,000 14,280 16,200 11,400


TV = 18,120
29,520
Time line:
0 1 2 3 Years
k = 10%

-62,000 19,920 22,800 15,600


TV = 15,680
31,280
Expected increase in annual net income
ARR =
72 Initial (or average) investment .Answer (D) is correct. The NPV is the excess of the present
values of the estimated cash inflows over the net cost of the investment. The discount rate used is sometimes the cost
of capital or other hurdle rate designated by management. This rate is also called the required rate of return. The
accounting rate of return is never used in NPV analysis because it ignores the time value of money; it is computed by
dividing the accounting net income by the investment.
Answer (A) is incorrect because cost of capital is a synonym for the rate used in NPV analysis. Answer (B) is incorrect
because hurdle rate is a synonym for the rate used in NPV analysis. Answer (C) is incorrect because discount rate is a
synonym for the rate used in NPV analysis.

NPV
($)
NPV
$
A

B B
A

k Discount rate (%)


0 10% IRRB IRRA 0 16% 17% 18% 30%

NPV
$
NPV ($)
P
C

k 10% k
10% 15% 20% %
NPV NPV
$

A A

B
B n
Sigma CF t
k k t=1
0 7% 12% 15% 0 7% 12% 14%
(1 + K)t
NPV
$

X
Crossover

k
10% IRRY 12% IRRX

0 k = 12% 1 2 3 4 5 6

CFsA -15,000 4,000 4,000 4,000 4,000 4,000 4,000


Project B:
0 k = ? 1 2 6 Years
  
CFsB -14,815 5,100 5,100 5,100
Time lines:
Project A:
0 1 2 3 4 Years
k = 12%

CFsA -25,000 13,000 13,000 13,000 13,000


NPVA = ? = 17,663 Terminal value = 5,000
CF 4 = 18,000
Project B:
0 1 2 3 4 Years
k = ?

CFsB -25,000 15,247 15,247 15,247 15,247


NPVA = NPVB = 17,663 Terminal value = 0
CF 4 = 15,247
Time line:
0 k = 18% 1 2 3 Years

-3,000 1,728 1,920 1,152


Time line:
0 1 2 3 4 5 Years
k = 12%

-45,000 7,800 10,680 7,560 5,880 -1,920


Time line:
0 1 2 3 4 5 Years
k = 9%

-40,000 9,800 11,720 9,640 8,520 15,320


Time lines:
Project A:
0 k = 14% 1 2 3 4 Years

CFsA -200,000 71,104 71,104 71,104 71,104


Project B:
0 k = 10% 1 2 3 4 Years

CFsB -200,000 0 0 146,411 146,411


Time line:
0 k = 12% 1 2 10 Years
  
-50,000 6,000 6,000 6,000
Time lines:
Project A:
0 k = 12% 1 2 3 Periods

CFsA -5,000 2,000 2,500 2,250


Project B:
0 k = 14% 1 2 3 Periods

CFsB -5,000 3,000 2,600 2,900


0 1 2 3 4
| | | | |
CFsNew Tech -1,500 -315 -315 -315 -315 x x x
Time line:
0 1 2 3 Years
k = 14%

-42,000 14,280 16,200 11,400


TV = 18,120
29,520
Time line:
0 1 2 3 Years
k = 10%

-62,000 19,920 22,800 15,600


TV = 15,680
31,280
Expected increase in annual net income
ARR =
73 Initial (or average) investment .REQUIRED: The matter affecting a project’s net present
value.
DISCUSSION: (A) To compute a project’s net present value, the initial investment is subtracted from the present value
of the after-tax cash flows. The proceeds from the sale of the asset to be replaced reduces the initial investment.
Answer (B) is incorrect because the carrying amount of the asset to be replaced affects the gain or loss on the sale.
Answer (C) is incorrect because the amount of annual depreciation on the asset to be replaced affects the carrying
value. Answer (D) is incorrect because annual depreciation of other assets, even if used directly, does not affect the
project’s net present value.
NPV
($)
NPV
$
A

B B
A

k Discount rate (%)


0 10% IRRB IRRA 0 16% 17% 18% 30%

NPV
$
NPV ($)
P
C

k 10% k
10% 15% 20% %

NPV NPV
$

A A

B
B n
Sigma CF t
k k t=1
0 7% 12% 15% 0 7% 12% 14%
(1 + K)t
NPV
$

X
Crossover

k
10% IRRY 12% IRRX

0 k = 12% 1 2 3 4 5 6

CFsA -15,000 4,000 4,000 4,000 4,000 4,000 4,000


Project B:
0 k = ? 1 2 6 Years
  
CFsB -14,815 5,100 5,100 5,100
Time lines:
Project A:
0 1 2 3 4 Years
k = 12%

CFsA -25,000 13,000 13,000 13,000 13,000


NPVA = ? = 17,663 Terminal value = 5,000
CF 4 = 18,000
Project B:
0 1 2 3 4 Years
k = ?

CFsB -25,000 15,247 15,247 15,247 15,247


NPVA = NPVB = 17,663 Terminal value = 0
CF 4 = 15,247
Time line:
0 1 2 3 Years
k = 18%

-3,000 1,728 1,920 1,152


Time line:
0 1 2 3 4 5 Years
k = 12%

-45,000 7,800 10,680 7,560 5,880 -1,920


Time line:
0 k = 9% 1 2 3 4 5 Years

-40,000 9,800 11,720 9,640 8,520 15,320


Time lines:
Project A:
0 k = 14% 1 2 3 4 Years

CFsA -200,000 71,104 71,104 71,104 71,104


Project B:
0 k = 10% 1 2 3 4 Years

CFsB -200,000 0 0 146,411 146,411


Time line:
0 k = 12% 1 2 10 Years
  
-50,000 6,000 6,000 6,000
Time lines:
Project A:
0 k = 12% 1 2 3 Periods

CFsA -5,000 2,000 2,500 2,250


Project B:
0 k = 14% 1 2 3 Periods

CFsB -5,000 3,000 2,600 2,900


0 1 2 3 4
| | | | |
CFsNew Tech -1,500 -315 -315 -315 -315 x x x
Time line:
0 1 2 3 Years
k = 14%

-42,000 14,280 16,200 11,400


TV = 18,120
29,520
Time line:
0 1 2 3 Years
k = 10%

-62,000 19,920 22,800 15,600


TV = 15,680
31,280
Expected increase in annual net income
ARR =
74 Initial (or average) investment .Answer (B) is correct. In an inflationary environment,
nominal future cash flows should increase to reflect the decrease in the value of the unit of measure. Also, the investor
should increase the discount rate to reflect the increased inflation premium arising from the additional uncertainty.
Lenders will require a higher interest rate in an inflationary environment.
Answer (A) is incorrect because future cash flows should also increase. Answer (C) is incorrect because the discount
rate should be increased to take into consideration future uncertainty and the risk premium that lenders will require in an
inflationary environment. Answer (D) is incorrect because cash flows should increase in an inflationary environment.

NPV
($)
NPV
$
A

B B
A

k Discount rate (%)


0 10% IRRB IRRA 0 16% 17% 18% 30%

NPV
$
NPV ($)
P
C

k 10% k
10% 15% 20% %
NPV NPV
$

A A

B
B n
Sigma CF t
k k t=1
0 7% 12% 15% 0 7% 12% 14%
(1 + K)t
NPV
$

X
Crossover

k
10% IRRY 12% IRRX

0 k = 12% 1 2 3 4 5 6

CFsA -15,000 4,000 4,000 4,000 4,000 4,000 4,000


Project B:
0 k = ? 1 2 6 Years
  
CFsB -14,815 5,100 5,100 5,100
Time lines:
Project A:
0 1 2 3 4 Years
k = 12%

CFsA -25,000 13,000 13,000 13,000 13,000


NPVA = ? = 17,663 Terminal value = 5,000
CF 4 = 18,000
Project B:
0 1 2 3 4 Years
k = ?

CFsB -25,000 15,247 15,247 15,247 15,247


NPVA = NPVB = 17,663 Terminal value = 0
CF 4 = 15,247
Time line:
0 k = 18% 1 2 3 Years

-3,000 1,728 1,920 1,152


Time line:
0 1 2 3 4 5 Years
k = 12%

-45,000 7,800 10,680 7,560 5,880 -1,920


Time line:
0 1 2 3 4 5 Years
k = 9%

-40,000 9,800 11,720 9,640 8,520 15,320


Time lines:
Project A:
0 k = 14% 1 2 3 4 Years

CFsA -200,000 71,104 71,104 71,104 71,104


Project B:
0 k = 10% 1 2 3 4 Years

CFsB -200,000 0 0 146,411 146,411


Time line:
0 k = 12% 1 2 10 Years
  
-50,000 6,000 6,000 6,000
Time lines:
Project A:
0 k = 12% 1 2 3 Periods

CFsA -5,000 2,000 2,500 2,250


Project B:
0 k = 14% 1 2 3 Periods

CFsB -5,000 3,000 2,600 2,900


0 1 2 3 4
| | | | |
CFsNew Tech -1,500 -315 -315 -315 -315 x x x
Time line:
0 1 2 3 Years
k = 14%

-42,000 14,280 16,200 11,400


TV = 18,120
29,520
Time line:
0 1 2 3 Years
k = 10%

-62,000 19,920 22,800 15,600


TV = 15,680
31,280
Expected increase in annual net income
ARR =
75 Initial (or average) investment .Answer (A) is correct. The internal rate of return (IRR) is the
discount rate at which the present value of the cash inflows equals the present values of the cash outflows (including the
original investment). Thus, the NPV of the project is zero at the IRR. The IRR is also the maximum borrowing cost the
firm can afford to pay for a specific project. The IRR is similar to the yield rate/effective rate quoted in the business
media.
Answer (B) is incorrect because the capital asset pricing model is a means of determining the cost of capital. Answer (C)
is incorrect because the profitability index is not an interest rate. Answer (D) is incorrect because the accounting rate of
return is not based on present values.
NPV
($)
NPV
$
A

B B
A

k Discount rate (%)


0 10% IRRB IRRA 0 16% 17% 18% 30%

NPV
$
NPV ($)
P
C

k 10% k
10% 15% 20% %

NPV NPV
$

A A

B
B n
Sigma CF t
k k t=1
0 7% 12% 15% 0 7% 12% 14%
(1 + K)t
NPV
$

X
Crossover

k
10% IRRY 12% IRRX

0 k = 12% 1 2 3 4 5 6

CFsA -15,000 4,000 4,000 4,000 4,000 4,000 4,000


Project B:
0 k = ? 1 2 6 Years
  
CFsB -14,815 5,100 5,100 5,100
Time lines:
Project A:
0 1 2 3 4 Years
k = 12%

CFsA -25,000 13,000 13,000 13,000 13,000


NPVA = ? = 17,663 Terminal value = 5,000
CF 4 = 18,000
Project B:
0 1 2 3 4 Years
k = ?

CFsB -25,000 15,247 15,247 15,247 15,247


NPVA = NPVB = 17,663 Terminal value = 0
CF 4 = 15,247
Time line:
0 1 2 3 Years
k = 18%

-3,000 1,728 1,920 1,152


Time line:
0 1 2 3 4 5 Years
k = 12%

-45,000 7,800 10,680 7,560 5,880 -1,920


Time line:
0 k = 9% 1 2 3 4 5 Years

-40,000 9,800 11,720 9,640 8,520 15,320


Time lines:
Project A:
0 k = 14% 1 2 3 4 Years

CFsA -200,000 71,104 71,104 71,104 71,104


Project B:
0 k = 10% 1 2 3 4 Years

CFsB -200,000 0 0 146,411 146,411


Time line:
0 k = 12% 1 2 10 Years
  
-50,000 6,000 6,000 6,000
Time lines:
Project A:
0 k = 12% 1 2 3 Periods

CFsA -5,000 2,000 2,500 2,250


Project B:
0 k = 14% 1 2 3 Periods

CFsB -5,000 3,000 2,600 2,900


0 1 2 3 4
| | | | |
CFsNew Tech -1,500 -315 -315 -315 -315 x x x
Time line:
0 1 2 3 Years
k = 14%

-42,000 14,280 16,200 11,400


TV = 18,120
29,520
Time line:
0 1 2 3 Years
k = 10%

-62,000 19,920 22,800 15,600


TV = 15,680
31,280
Expected increase in annual net income
ARR =
76 Initial (or average) investment .Answer (C) is correct. The IRR is the interest rate at which
the present value of the expected future cash inflows is equal to the present value of the cash outflows for a project.
Thus, the IRR is the interest rate that will produce a net present value (NPV) equal to zero. The IRR method assumes
that the cash flows will be reinvested at the internal rate of return.
Answer (A) is incorrect because the hurdle rate is a concept used to calculate the NPV of a project; it is determined by
management prior to the analysis. Answer (B) is incorrect because the IRR is the rate of interest at which the NPV is
zero. Answer (D) is incorrect because the IRR is a means of evaluating potential investment projects.

NPV
($)
NPV
$
A

B B
A

k Discount rate (%)


0 10% IRRB IRRA 0 16% 17% 18% 30%

NPV
$
NPV ($)
P
C

k 10% k
10% 15% 20% %
NPV NPV
$

A A

B
B n
Sigma CF t
k k t=1
0 7% 12% 15% 0 7% 12% 14%
(1 + K)t
NPV
$

X
Crossover

k
10% IRRY 12% IRRX

0 k = 12% 1 2 3 4 5 6

CFsA -15,000 4,000 4,000 4,000 4,000 4,000 4,000


Project B:
0 k = ? 1 2 6 Years
  
CFsB -14,815 5,100 5,100 5,100
Time lines:
Project A:
0 1 2 3 4 Years
k = 12%

CFsA -25,000 13,000 13,000 13,000 13,000


NPVA = ? = 17,663 Terminal value = 5,000
CF 4 = 18,000
Project B:
0 1 2 3 4 Years
k = ?

CFsB -25,000 15,247 15,247 15,247 15,247


NPVA = NPVB = 17,663 Terminal value = 0
CF 4 = 15,247
Time line:
0 k = 18% 1 2 3 Years

-3,000 1,728 1,920 1,152


Time line:
0 1 2 3 4 5 Years
k = 12%

-45,000 7,800 10,680 7,560 5,880 -1,920


Time line:
0 1 2 3 4 5 Years
k = 9%

-40,000 9,800 11,720 9,640 8,520 15,320


Time lines:
Project A:
0 k = 14% 1 2 3 4 Years

CFsA -200,000 71,104 71,104 71,104 71,104


Project B:
0 k = 10% 1 2 3 4 Years

CFsB -200,000 0 0 146,411 146,411


Time line:
0 k = 12% 1 2 10 Years
  
-50,000 6,000 6,000 6,000
Time lines:
Project A:
0 k = 12% 1 2 3 Periods

CFsA -5,000 2,000 2,500 2,250


Project B:
0 k = 14% 1 2 3 Periods

CFsB -5,000 3,000 2,600 2,900


0 1 2 3 4
| | | | |
CFsNew Tech -1,500 -315 -315 -315 -315 x x x
Time line:
0 1 2 3 Years
k = 14%

-42,000 14,280 16,200 11,400


TV = 18,120
29,520
Time line:
0 1 2 3 Years
k = 10%

-62,000 19,920 22,800 15,600


TV = 15,680
31,280
Expected increase in annual net income
ARR =
77 Initial (or average) investment .Answer (C) is correct. The IRR is a capital budgeting
technique that calculates the interest rate that yields a net present value equal to $0. It is the interest rate that will
discount the future cash flows to an amount equal to the initial cost of the project. Thus, the higher the IRR, the more
favorable the ranking of the project.
Answer (A) is incorrect because the cost of capital is not used in the calculation of the IRR. Answer (B) is incorrect
because the IRR can be determined regardless of the constancy of the cash flows. However, it is more difficult to
calculate when cash flows are not constant because a trial-and-error approach must be used. Answer (D) is incorrect
because there is no relationship between IRR and the profitability index.
NPV
($)
NPV
$
A

B B
A

k Discount rate (%)


0 10% IRRB IRRA 0 16% 17% 18% 30%

NPV
$
NPV ($)
P
C

k 10% k
10% 15% 20% %

NPV NPV
$

A A

B
B n
Sigma CF t
k k t=1
0 7% 12% 15% 0 7% 12% 14%
(1 + K)t
NPV
$

X
Crossover

k
10% IRRY 12% IRRX

0 k = 12% 1 2 3 4 5 6

CFsA -15,000 4,000 4,000 4,000 4,000 4,000 4,000


Project B:
0 k = ? 1 2 6 Years
  
CFsB -14,815 5,100 5,100 5,100
Time lines:
Project A:
0 1 2 3 4 Years
k = 12%

CFsA -25,000 13,000 13,000 13,000 13,000


NPVA = ? = 17,663 Terminal value = 5,000
CF 4 = 18,000
Project B:
0 1 2 3 4 Years
k = ?

CFsB -25,000 15,247 15,247 15,247 15,247


NPVA = NPVB = 17,663 Terminal value = 0
CF 4 = 15,247
Time line:
0 1 2 3 Years
k = 18%

-3,000 1,728 1,920 1,152


Time line:
0 1 2 3 4 5 Years
k = 12%

-45,000 7,800 10,680 7,560 5,880 -1,920


Time line:
0 k = 9% 1 2 3 4 5 Years

-40,000 9,800 11,720 9,640 8,520 15,320


Time lines:
Project A:
0 k = 14% 1 2 3 4 Years

CFsA -200,000 71,104 71,104 71,104 71,104


Project B:
0 k = 10% 1 2 3 4 Years

CFsB -200,000 0 0 146,411 146,411


Time line:
0 k = 12% 1 2 10 Years
  
-50,000 6,000 6,000 6,000
Time lines:
Project A:
0 k = 12% 1 2 3 Periods

CFsA -5,000 2,000 2,500 2,250


Project B:
0 k = 14% 1 2 3 Periods

CFsB -5,000 3,000 2,600 2,900


0 1 2 3 4
| | | | |
CFsNew Tech -1,500 -315 -315 -315 -315 x x x
Time line:
0 1 2 3 Years
k = 14%

-42,000 14,280 16,200 11,400


TV = 18,120
29,520
Time line:
0 1 2 3 Years
k = 10%

-62,000 19,920 22,800 15,600


TV = 15,680
31,280
Expected increase in annual net income
ARR =
78 Initial (or average) investment .REQUIRED: The true statement about internal rate of
return.
DISCUSSION: (D) The internal rate of return (IRR) is the discount rate at which the present value of the cash flows
equals the original investment. Thus, the NPV of the project is zero at the IRR. The IRR is also the maximum borrowing
cost the firm could afford to pay for a specific project. The IRR is similar to the yield rate/effective rate quoted in the
business media.
Answers (A), (B), and (C) are incorrect because the IRR is the discount rate at which the NPV of the cash flows is zero,
the breakeven borrowing rate for projects, and the yield rate/effective rate of interest quoted on long-term debt and other
instruments.

NPV
($)
NPV
$
A

B B
A

k Discount rate (%)


0 10% IRRB IRRA 0 16% 17% 18% 30%

NPV
$
NPV ($)
P
C

k 10% k
10% 15% 20% %
NPV NPV
$

A A

B
B n
Sigma CF t
k k t=1
0 7% 12% 15% 0 7% 12% 14%
(1 + K)t
NPV
$

X
Crossover

k
10% IRRY 12% IRRX

0 k = 12% 1 2 3 4 5 6

CFsA -15,000 4,000 4,000 4,000 4,000 4,000 4,000


Project B:
0 k = ? 1 2 6 Years
  
CFsB -14,815 5,100 5,100 5,100
Time lines:
Project A:
0 1 2 3 4 Years
k = 12%

CFsA -25,000 13,000 13,000 13,000 13,000


NPVA = ? = 17,663 Terminal value = 5,000
CF 4 = 18,000
Project B:
0 1 2 3 4 Years
k = ?

CFsB -25,000 15,247 15,247 15,247 15,247


NPVA = NPVB = 17,663 Terminal value = 0
CF 4 = 15,247
Time line:
0 k = 18% 1 2 3 Years

-3,000 1,728 1,920 1,152


Time line:
0 1 2 3 4 5 Years
k = 12%

-45,000 7,800 10,680 7,560 5,880 -1,920


Time line:
0 1 2 3 4 5 Years
k = 9%

-40,000 9,800 11,720 9,640 8,520 15,320


Time lines:
Project A:
0 k = 14% 1 2 3 4 Years

CFsA -200,000 71,104 71,104 71,104 71,104


Project B:
0 k = 10% 1 2 3 4 Years

CFsB -200,000 0 0 146,411 146,411


Time line:
0 k = 12% 1 2 10 Years
  
-50,000 6,000 6,000 6,000
Time lines:
Project A:
0 k = 12% 1 2 3 Periods

CFsA -5,000 2,000 2,500 2,250


Project B:
0 k = 14% 1 2 3 Periods

CFsB -5,000 3,000 2,600 2,900


0 1 2 3 4
| | | | |
CFsNew Tech -1,500 -315 -315 -315 -315 x x x
Time line:
0 1 2 3 Years
k = 14%

-42,000 14,280 16,200 11,400


TV = 18,120
29,520
Time line:
0 1 2 3 Years
k = 10%

-62,000 19,920 22,800 15,600


TV = 15,680
31,280
Expected increase in annual net income
ARR =
79 Initial (or average) investment .Answer (D) is correct. The internal rate of return (IRR) is the
discount rate at which the present value of the cash flows equals the original investment. Thus, the NPV of the project is
zero at the IRR. The IRR is also the maximum borrowing cost the firm could afford to pay for a specific project. The IRR
is similar to the yield rate/effective rate quoted in the business media.
Answer (A) is incorrect because the IRR is the discount rate at which the NPV of the cash flows is zero, the breakeven
borrowing rate for the project in question, the yield rate/effective rate of interest quoted on long-term debt and other
instruments, and favorable when it exceeds the hurdle rate. Answer (B) is incorrect because the IRR is the discount rate
at which the NPV of the cash flows is zero, the breakeven borrowing rate for the project in question, the yield
rate/effective rate of interest quoted on long-term debt and other instruments, and favorable when it exceeds the hurdle
rate. Answer (C) is incorrect because the IRR is the discount rate at which the NPV of the cash flows is zero, the
breakeven borrowing rate for the project in question, the yield rate/effective rate of interest quoted on long-term debt and
other instruments, and favorable when it exceeds the hurdle rate.
NPV
($)
NPV
$
A

B B
A

k Discount rate (%)


0 10% IRRB IRRA 0 16% 17% 18% 30%

NPV
$
NPV ($)
P
C

k 10% k
10% 15% 20% %

NPV NPV
$

A A

B
B n
Sigma CF t
k k t=1
0 7% 12% 15% 0 7% 12% 14%
(1 + K)t
NPV
$

X
Crossover

k
10% IRRY 12% IRRX

0 k = 12% 1 2 3 4 5 6

CFsA -15,000 4,000 4,000 4,000 4,000 4,000 4,000


Project B:
0 k = ? 1 2 6 Years
  
CFsB -14,815 5,100 5,100 5,100
Time lines:
Project A:
0 1 2 3 4 Years
k = 12%

CFsA -25,000 13,000 13,000 13,000 13,000


NPVA = ? = 17,663 Terminal value = 5,000
CF 4 = 18,000
Project B:
0 1 2 3 4 Years
k = ?

CFsB -25,000 15,247 15,247 15,247 15,247


NPVA = NPVB = 17,663 Terminal value = 0
CF 4 = 15,247
Time line:
0 1 2 3 Years
k = 18%

-3,000 1,728 1,920 1,152


Time line:
0 1 2 3 4 5 Years
k = 12%

-45,000 7,800 10,680 7,560 5,880 -1,920


Time line:
0 k = 9% 1 2 3 4 5 Years

-40,000 9,800 11,720 9,640 8,520 15,320


Time lines:
Project A:
0 k = 14% 1 2 3 4 Years

CFsA -200,000 71,104 71,104 71,104 71,104


Project B:
0 k = 10% 1 2 3 4 Years

CFsB -200,000 0 0 146,411 146,411


Time line:
0 k = 12% 1 2 10 Years
  
-50,000 6,000 6,000 6,000
Time lines:
Project A:
0 k = 12% 1 2 3 Periods

CFsA -5,000 2,000 2,500 2,250


Project B:
0 k = 14% 1 2 3 Periods

CFsB -5,000 3,000 2,600 2,900


0 1 2 3 4
| | | | |
CFsNew Tech -1,500 -315 -315 -315 -315 x x x
Time line:
0 1 2 3 Years
k = 14%

-42,000 14,280 16,200 11,400


TV = 18,120
29,520
Time line:
0 1 2 3 Years
k = 10%

-62,000 19,920 22,800 15,600


TV = 15,680
31,280
Expected increase in annual net income
ARR =
80 Initial (or average) investment .Answer (D) is correct. The internal rate of return of a
proposed project includes the residual sales value of a project but not the depreciation expense. This is true because
the residual sales value represents a future cash flow whereas depreciation expense (ignoring income tax
considerations) provides no cash inflow or outflow.
Answers (A), (B), and (C) are incorrect because they contain the wrong combination of responses.

NPV
($)
NPV
$
A

B B
A

k Discount rate (%)


0 10% IRRB IRRA 0 16% 17% 18% 30%

NPV
$
NPV ($)
P
C

k 10% k
10% 15% 20% %
NPV NPV
$

A A

B
B n
Sigma CF t
k k t=1
0 7% 12% 15% 0 7% 12% 14%
(1 + K)t
NPV
$

X
Crossover

k
10% IRRY 12% IRRX

0 k = 12% 1 2 3 4 5 6

CFsA -15,000 4,000 4,000 4,000 4,000 4,000 4,000


Project B:
0 k = ? 1 2 6 Years
  
CFsB -14,815 5,100 5,100 5,100
Time lines:
Project A:
0 1 2 3 4 Years
k = 12%

CFsA -25,000 13,000 13,000 13,000 13,000


NPVA = ? = 17,663 Terminal value = 5,000
CF 4 = 18,000
Project B:
0 1 2 3 4 Years
k = ?

CFsB -25,000 15,247 15,247 15,247 15,247


NPVA = NPVB = 17,663 Terminal value = 0
CF 4 = 15,247
Time line:
0 k = 18% 1 2 3 Years

-3,000 1,728 1,920 1,152


Time line:
0 1 2 3 4 5 Years
k = 12%

-45,000 7,800 10,680 7,560 5,880 -1,920


Time line:
0 1 2 3 4 5 Years
k = 9%

-40,000 9,800 11,720 9,640 8,520 15,320


Time lines:
Project A:
0 k = 14% 1 2 3 4 Years

CFsA -200,000 71,104 71,104 71,104 71,104


Project B:
0 k = 10% 1 2 3 4 Years

CFsB -200,000 0 0 146,411 146,411


Time line:
0 k = 12% 1 2 10 Years
  
-50,000 6,000 6,000 6,000
Time lines:
Project A:
0 k = 12% 1 2 3 Periods

CFsA -5,000 2,000 2,500 2,250


Project B:
0 k = 14% 1 2 3 Periods

CFsB -5,000 3,000 2,600 2,900


0 1 2 3 4
| | | | |
CFsNew Tech -1,500 -315 -315 -315 -315 x x x
Time line:
0 1 2 3 Years
k = 14%

-42,000 14,280 16,200 11,400


TV = 18,120
29,520
Time line:
0 1 2 3 Years
k = 10%

-62,000 19,920 22,800 15,600


TV = 15,680
31,280
Expected increase in annual net income
ARR =
81 Initial (or average) investment .Answer (C) is correct. Under the internal rate of return (IRR)
method, the interest rate is computed that will exactly match the present value of the future net inflows with the initial
cost of the investment. The IRR method assumes that cash flows will be reinvested at the IRR. Thus, if the project's
funds are not reinvested at the internal rate of return, the ranking calculations obtained under the IRR method may be in
error. The net present value method gives a better grasp of the problem in many decision situations because the
reinvestment is assumed to be at the cost of capital.
Answer (A) is incorrect because the IRR does calculate compounded interest rates. Answer (B) is incorrect because
both methods incorporate the time value of money. Answer (D) is incorrect because sensitivity analysis can be used
with NPV to handle multiple desired hurdle rates.
NPV
($)
NPV
$
A

B B
A

k Discount rate (%)


0 10% IRRB IRRA 0 16% 17% 18% 30%

NPV
$
NPV ($)
P
C

k 10% k
10% 15% 20% %

NPV NPV
$

A A

B
B n
Sigma CF t
k k t=1
0 7% 12% 15% 0 7% 12% 14%
(1 + K)t
NPV
$

X
Crossover

k
10% IRRY 12% IRRX

0 k = 12% 1 2 3 4 5 6

CFsA -15,000 4,000 4,000 4,000 4,000 4,000 4,000


Project B:
0 k = ? 1 2 6 Years
  
CFsB -14,815 5,100 5,100 5,100
Time lines:
Project A:
0 1 2 3 4 Years
k = 12%

CFsA -25,000 13,000 13,000 13,000 13,000


NPVA = ? = 17,663 Terminal value = 5,000
CF 4 = 18,000
Project B:
0 1 2 3 4 Years
k = ?

CFsB -25,000 15,247 15,247 15,247 15,247


NPVA = NPVB = 17,663 Terminal value = 0
CF 4 = 15,247
Time line:
0 1 2 3 Years
k = 18%

-3,000 1,728 1,920 1,152


Time line:
0 1 2 3 4 5 Years
k = 12%

-45,000 7,800 10,680 7,560 5,880 -1,920


Time line:
0 k = 9% 1 2 3 4 5 Years

-40,000 9,800 11,720 9,640 8,520 15,320


Time lines:
Project A:
0 k = 14% 1 2 3 4 Years

CFsA -200,000 71,104 71,104 71,104 71,104


Project B:
0 k = 10% 1 2 3 4 Years

CFsB -200,000 0 0 146,411 146,411


Time line:
0 k = 12% 1 2 10 Years
  
-50,000 6,000 6,000 6,000
Time lines:
Project A:
0 k = 12% 1 2 3 Periods

CFsA -5,000 2,000 2,500 2,250


Project B:
0 k = 14% 1 2 3 Periods

CFsB -5,000 3,000 2,600 2,900


0 1 2 3 4
| | | | |
CFsNew Tech -1,500 -315 -315 -315 -315 x x x
Time line:
0 1 2 3 Years
k = 14%

-42,000 14,280 16,200 11,400


TV = 18,120
29,520
Time line:
0 1 2 3 Years
k = 10%

-62,000 19,920 22,800 15,600


TV = 15,680
31,280
Expected increase in annual net income
ARR =
82 Initial (or average) investment .Answer (D) is correct. The IRR is the rate at which the
discounted future cash flows equal the net investment (NPV = 0). One disadvantage of the method is that inflows from
the early years are assumed to be reinvested at the IRR. This assumption may not be sound. Investments in the future
may not earn as high a rate as is currently available.
Answer (A) is incorrect because the IRR method considers the time value of money. Answer (B) is incorrect because the
IRR provides a straightforward decision criterion. Any project with an IRR greater than the cost of capital is acceptable.
Answer (C) is incorrect because the IRR method implicitly assumes reinvestment at the IRR; the NPV method implicitly
assumes reinvestment at the cost of capital.

NPV
($)
NPV
$
A

B B
A

k Discount rate (%)


0 10% IRRB IRRA 0 16% 17% 18% 30%

NPV
$
NPV ($)
P
C

k 10% k
10% 15% 20% %
NPV NPV
$

A A

B
B n
Sigma CF t
k k t=1
0 7% 12% 15% 0 7% 12% 14%
(1 + K)t
NPV
$

X
Crossover

k
10% IRRY 12% IRRX

0 k = 12% 1 2 3 4 5 6

CFsA -15,000 4,000 4,000 4,000 4,000 4,000 4,000


Project B:
0 k = ? 1 2 6 Years
  
CFsB -14,815 5,100 5,100 5,100
Time lines:
Project A:
0 1 2 3 4 Years
k = 12%

CFsA -25,000 13,000 13,000 13,000 13,000


NPVA = ? = 17,663 Terminal value = 5,000
CF 4 = 18,000
Project B:
0 1 2 3 4 Years
k = ?

CFsB -25,000 15,247 15,247 15,247 15,247


NPVA = NPVB = 17,663 Terminal value = 0
CF 4 = 15,247
Time line:
0 k = 18% 1 2 3 Years

-3,000 1,728 1,920 1,152


Time line:
0 1 2 3 4 5 Years
k = 12%

-45,000 7,800 10,680 7,560 5,880 -1,920


Time line:
0 1 2 3 4 5 Years
k = 9%

-40,000 9,800 11,720 9,640 8,520 15,320


Time lines:
Project A:
0 k = 14% 1 2 3 4 Years

CFsA -200,000 71,104 71,104 71,104 71,104


Project B:
0 k = 10% 1 2 3 4 Years

CFsB -200,000 0 0 146,411 146,411


Time line:
0 k = 12% 1 2 10 Years
  
-50,000 6,000 6,000 6,000
Time lines:
Project A:
0 k = 12% 1 2 3 Periods

CFsA -5,000 2,000 2,500 2,250


Project B:
0 k = 14% 1 2 3 Periods

CFsB -5,000 3,000 2,600 2,900


0 1 2 3 4
| | | | |
CFsNew Tech -1,500 -315 -315 -315 -315 x x x
Time line:
0 1 2 3 Years
k = 14%

-42,000 14,280 16,200 11,400


TV = 18,120
29,520
Time line:
0 1 2 3 Years
k = 10%

-62,000 19,920 22,800 15,600


TV = 15,680
31,280
Expected increase in annual net income
ARR =
83 Initial (or average) investment .REQUIRED: Advantage of the IRR over the accounting rate
of return.
DISCUSSION: (C) The IRR is the interest rate that equalizes the present value of future cash flows with the initial cost
of the investment. The accounting rate of return is calculated by dividing the increase in accounting net income by the
required investment. However, it ignores the time value of money and does not emphasize cash flows.
Answers (A), (B), and (D) are incorrect because both techniques recognize the project’s salvage value.
NPV
($)
NPV
$
A

B B
A

k Discount rate (%)


0 10% IRRB IRRA 0 16% 17% 18% 30%

NPV
$
NPV ($)
P
C

k 10% k
10% 15% 20% %

NPV NPV
$

A A

B
B n
Sigma CF t
k k t=1
0 7% 12% 15% 0 7% 12% 14%
(1 + K)t
NPV
$

X
Crossover

k
10% IRRY 12% IRRX

0 k = 12% 1 2 3 4 5 6

CFsA -15,000 4,000 4,000 4,000 4,000 4,000 4,000


Project B:
0 k = ? 1 2 6 Years
  
CFsB -14,815 5,100 5,100 5,100
Time lines:
Project A:
0 1 2 3 4 Years
k = 12%

CFsA -25,000 13,000 13,000 13,000 13,000


NPVA = ? = 17,663 Terminal value = 5,000
CF 4 = 18,000
Project B:
0 1 2 3 4 Years
k = ?

CFsB -25,000 15,247 15,247 15,247 15,247


NPVA = NPVB = 17,663 Terminal value = 0
CF 4 = 15,247
Time line:
0 1 2 3 Years
k = 18%

-3,000 1,728 1,920 1,152


Time line:
0 1 2 3 4 5 Years
k = 12%

-45,000 7,800 10,680 7,560 5,880 -1,920


Time line:
0 k = 9% 1 2 3 4 5 Years

-40,000 9,800 11,720 9,640 8,520 15,320


Time lines:
Project A:
0 k = 14% 1 2 3 4 Years

CFsA -200,000 71,104 71,104 71,104 71,104


Project B:
0 k = 10% 1 2 3 4 Years

CFsB -200,000 0 0 146,411 146,411


Time line:
0 k = 12% 1 2 10 Years
  
-50,000 6,000 6,000 6,000
Time lines:
Project A:
0 k = 12% 1 2 3 Periods

CFsA -5,000 2,000 2,500 2,250


Project B:
0 k = 14% 1 2 3 Periods

CFsB -5,000 3,000 2,600 2,900


0 1 2 3 4
| | | | |
CFsNew Tech -1,500 -315 -315 -315 -315 x x x
Time line:
0 1 2 3 Years
k = 14%

-42,000 14,280 16,200 11,400


TV = 18,120
29,520
Time line:
0 1 2 3 Years
k = 10%

-62,000 19,920 22,800 15,600


TV = 15,680
31,280
Expected increase in annual net income
ARR =
84 Initial (or average) investment .Statement c is correct; the other statements are false. MIRR
and NPV can conflict for mutually exclusive projects if the projects differ in size. NPV does not suffer from the multiple
IRR problem.

NPV
($)
NPV
$
A

B B
A

k Discount rate (%)


0 10% IRRB IRRA 0 16% 17% 18% 30%

NPV
$
NPV ($)
P
C

k 10% k
10% 15% 20% %

NPV NPV
$

A A

B
B n
Sigma CF t
k k t=1
0 7% 12% 15% 0 7% 12% 14%
(1 + K)t
NPV
$

X
Crossover

k
10% IRRY 12% IRRX

0 k = 12% 1 2 3 4 5 6

CFsA -15,000 4,000 4,000 4,000 4,000 4,000 4,000


Project B:
0 k = ? 1 2 6 Years
  
CFsB -14,815 5,100 5,100 5,100
Time lines:
Project A:
0 1 2 3 4 Years
k = 12%

CFsA -25,000 13,000 13,000 13,000 13,000


NPVA = ? = 17,663 Terminal value = 5,000
CF 4 = 18,000
Project B:
0 1 2 3 4 Years
k = ?

CFsB -25,000 15,247 15,247 15,247 15,247


NPVA = NPVB = 17,663 Terminal value = 0
CF 4 = 15,247
Time line:
0 1 2 3 Years
k = 18%

-3,000 1,728 1,920 1,152


Time line:
0 1 2 3 4 5 Years
k = 12%

-45,000 7,800 10,680 7,560 5,880 -1,920


Time line:
0 k = 9% 1 2 3 4 5 Years

-40,000 9,800 11,720 9,640 8,520 15,320


Time lines:
Project A:
0 k = 14% 1 2 3 4 Years

CFsA -200,000 71,104 71,104 71,104 71,104


Project B:
0 k = 10% 1 2 3 4 Years

CFsB -200,000 0 0 146,411 146,411


Time line:
0 k = 12% 1 2 10 Years
  
-50,000 6,000 6,000 6,000
Time lines:
Project A:
0 k = 12% 1 2 3 Periods

CFsA -5,000 2,000 2,500 2,250


Project B:
0 k = 14% 1 2 3 Periods

CFsB -5,000 3,000 2,600 2,900


0 1 2 3 4
| | | | |
CFsNew Tech -1,500 -315 -315 -315 -315 x x x
Time line:
0 1 2 3 Years
k = 14%

-42,000 14,280 16,200 11,400


TV = 18,120
29,520
Time line:
0 1 2 3 Years
k = 10%

-62,000 19,920 22,800 15,600


TV = 15,680
31,280
Expected increase in annual net income
ARR =
85 Initial (or average) investment .Answer (D) is correct. The profitability index is the ratio of
the present value of future net cash inflows to the initial net cash investment. It is a variation of the NPV method that
facilitates comparison of different-sized investments. A profitability index greater than 1.0 indicates a profitable
investment, or one that has a positive net present value.
Answer (A) is incorrect because the IRR is the discount rate at which the NPV is $0, which is also the rate at which the
profitability index is 1.0. The IRR cannot be determined solely from the index. Answer (B) is incorrect because, if the
index is 1.15 and the discount rate is the cost of capital, the NPV is positive, and the IRR must be higher than the cost of
capital. Answer (C) is incorrect because the IRR is a discount rate, whereas the NPV is an amount.
NPV
($)
NPV
$
A

B B
A

k Discount rate (%)


0 10% IRRB IRRA 0 16% 17% 18% 30%

NPV
$
NPV ($)
P
C

k 10% k
10% 15% 20% %

NPV NPV
$

A A

B
B n
Sigma CF t
k k t=1
0 7% 12% 15% 0 7% 12% 14%
(1 + K)t
NPV
$

X
Crossover

k
10% IRRY 12% IRRX

0 k = 12% 1 2 3 4 5 6

CFsA -15,000 4,000 4,000 4,000 4,000 4,000 4,000


Project B:
0 k = ? 1 2 6 Years
  
CFsB -14,815 5,100 5,100 5,100
Time lines:
Project A:
0 1 2 3 4 Years
k = 12%

CFsA -25,000 13,000 13,000 13,000 13,000


NPVA = ? = 17,663 Terminal value = 5,000
CF 4 = 18,000
Project B:
0 1 2 3 4 Years
k = ?

CFsB -25,000 15,247 15,247 15,247 15,247


NPVA = NPVB = 17,663 Terminal value = 0
CF 4 = 15,247
Time line:
0 1 2 3 Years
k = 18%

-3,000 1,728 1,920 1,152


Time line:
0 1 2 3 4 5 Years
k = 12%

-45,000 7,800 10,680 7,560 5,880 -1,920


Time line:
0 k = 9% 1 2 3 4 5 Years

-40,000 9,800 11,720 9,640 8,520 15,320


Time lines:
Project A:
0 k = 14% 1 2 3 4 Years

CFsA -200,000 71,104 71,104 71,104 71,104


Project B:
0 k = 10% 1 2 3 4 Years

CFsB -200,000 0 0 146,411 146,411


Time line:
0 k = 12% 1 2 10 Years
  
-50,000 6,000 6,000 6,000
Time lines:
Project A:
0 k = 12% 1 2 3 Periods

CFsA -5,000 2,000 2,500 2,250


Project B:
0 k = 14% 1 2 3 Periods

CFsB -5,000 3,000 2,600 2,900


0 1 2 3 4
| | | | |
CFsNew Tech -1,500 -315 -315 -315 -315 x x x
Time line:
0 1 2 3 Years
k = 14%

-42,000 14,280 16,200 11,400


TV = 18,120
29,520
Time line:
0 1 2 3 Years
k = 10%

-62,000 19,920 22,800 15,600


TV = 15,680
31,280
Expected increase in annual net income
ARR =
86 Initial (or average) investment .(d) The internal rate of return method determines the rate of
return at which the present value of the cash flows will exactly equal the investment outlay. It will indicate the rate of
return earned over the life of the project. The net present value method determines the present vale of all future cash
flows at a selected discount rate. If the NPV of the cash flows is positive, the return earned b the project is higher than
the selected rate. Both methods will provide the information needed to decide if a project’s rate of return will meet Polo
co.’s requirement.

NPV
($)
NPV
$
A

B B
A

k Discount rate (%)


0 10% IRRB IRRA 0 16% 17% 18% 30%

NPV
$
NPV ($)
P
C

k 10% k
10% 15% 20% %
NPV NPV
$

A A

B
B n
Sigma CF t
k k t=1
0 7% 12% 15% 0 7% 12% 14%
(1 + K)t
NPV
$

X
Crossover

k
10% IRRY 12% IRRX

0 k = 12% 1 2 3 4 5 6

CFsA -15,000 4,000 4,000 4,000 4,000 4,000 4,000


Project B:
0 k = ? 1 2 6 Years
  
CFsB -14,815 5,100 5,100 5,100
Time lines:
Project A:
0 1 2 3 4 Years
k = 12%

CFsA -25,000 13,000 13,000 13,000 13,000


NPVA = ? = 17,663 Terminal value = 5,000
CF 4 = 18,000
Project B:
0 1 2 3 4 Years
k = ?

CFsB -25,000 15,247 15,247 15,247 15,247


NPVA = NPVB = 17,663 Terminal value = 0
CF 4 = 15,247
Time line:
0 k = 18% 1 2 3 Years

-3,000 1,728 1,920 1,152


Time line:
0 1 2 3 4 5 Years
k = 12%

-45,000 7,800 10,680 7,560 5,880 -1,920


Time line:
0 1 2 3 4 5 Years
k = 9%

-40,000 9,800 11,720 9,640 8,520 15,320


Time lines:
Project A:
0 k = 14% 1 2 3 4 Years

CFsA -200,000 71,104 71,104 71,104 71,104


Project B:
0 k = 10% 1 2 3 4 Years

CFsB -200,000 0 0 146,411 146,411


Time line:
0 k = 12% 1 2 10 Years
  
-50,000 6,000 6,000 6,000
Time lines:
Project A:
0 k = 12% 1 2 3 Periods

CFsA -5,000 2,000 2,500 2,250


Project B:
0 k = 14% 1 2 3 Periods

CFsB -5,000 3,000 2,600 2,900


0 1 2 3 4
| | | | |
CFsNew Tech -1,500 -315 -315 -315 -315 x x x
Time line:
0 1 2 3 Years
k = 14%

-42,000 14,280 16,200 11,400


TV = 18,120
29,520
Time line:
0 1 2 3 Years
k = 10%

-62,000 19,920 22,800 15,600


TV = 15,680
31,280
Expected increase in annual net income
ARR =
87 Initial (or average) investment .Answer (D) is correct. The IRR is defined as the rate at
which the NPV is zero. Accordingly, if the NPV is positive at a cost of capital of 15%, the rate (the IRR) required to
reduce the NPV to zero must exceed 15%.
Answer (A) is incorrect because the accounting rate of return (net income ÷ investment) ignores the time value of
money. It is not determinable from the given information. Answer (B) is incorrect because the accounting rate of return
(net income ÷ investment) ignores the time value of money. It is not determinable from the given information. Answer
(C) is incorrect because the payback period ignores the time value of money. It is not determinable from the given
information.
NPV
($)
NPV
$
A

B B
A

k Discount rate (%)


0 10% IRRB IRRA 0 16% 17% 18% 30%

NPV
$
NPV ($)
P
C

k 10% k
10% 15% 20% %

NPV NPV
$

A A

B
B n
Sigma CF t
k k t=1
0 7% 12% 15% 0 7% 12% 14%
(1 + K)t
NPV
$

X
Crossover

k
10% IRRY 12% IRRX

0 k = 12% 1 2 3 4 5 6

CFsA -15,000 4,000 4,000 4,000 4,000 4,000 4,000


Project B:
0 k = ? 1 2 6 Years
  
CFsB -14,815 5,100 5,100 5,100
Time lines:
Project A:
0 1 2 3 4 Years
k = 12%

CFsA -25,000 13,000 13,000 13,000 13,000


NPVA = ? = 17,663 Terminal value = 5,000
CF 4 = 18,000
Project B:
0 1 2 3 4 Years
k = ?

CFsB -25,000 15,247 15,247 15,247 15,247


NPVA = NPVB = 17,663 Terminal value = 0
CF 4 = 15,247
Time line:
0 1 2 3 Years
k = 18%

-3,000 1,728 1,920 1,152


Time line:
0 1 2 3 4 5 Years
k = 12%

-45,000 7,800 10,680 7,560 5,880 -1,920


Time line:
0 k = 9% 1 2 3 4 5 Years

-40,000 9,800 11,720 9,640 8,520 15,320


Time lines:
Project A:
0 k = 14% 1 2 3 4 Years

CFsA -200,000 71,104 71,104 71,104 71,104


Project B:
0 k = 10% 1 2 3 4 Years

CFsB -200,000 0 0 146,411 146,411


Time line:
0 k = 12% 1 2 10 Years
  
-50,000 6,000 6,000 6,000
Time lines:
Project A:
0 k = 12% 1 2 3 Periods

CFsA -5,000 2,000 2,500 2,250


Project B:
0 k = 14% 1 2 3 Periods

CFsB -5,000 3,000 2,600 2,900


0 1 2 3 4
| | | | |
CFsNew Tech -1,500 -315 -315 -315 -315 x x x
Time line:
0 1 2 3 Years
k = 14%

-42,000 14,280 16,200 11,400


TV = 18,120
29,520
Time line:
0 1 2 3 Years
k = 10%

-62,000 19,920 22,800 15,600


TV = 15,680
31,280
Expected increase in annual net income
ARR =
88 Initial (or average) investment .Answer (C) is correct. The relationship between the NPV
method and the IRR method can be summarized as follows:
NPV IRR
NPV > 0 IRR > Discount rate
NPV = 0 IRR = Discount rate
NPV < 0 IRR < Discount rate
Since the problem states that Neu Co. has a positive net present value on the investment, then the internal rate of return
would be > 12%.

NPV
($)
NPV
$
A

B B
A

k Discount rate (%)


0 10% IRRB IRRA 0 16% 17% 18% 30%

NPV
$
NPV ($)
P
C

k 10% k
10% 15% 20% %
NPV NPV
$

A A

B
B n
Sigma CF t
k k t=1
0 7% 12% 15% 0 7% 12% 14%
(1 + K)t
NPV
$

X
Crossover

k
10% IRRY 12% IRRX

0 k = 12% 1 2 3 4 5 6

CFsA -15,000 4,000 4,000 4,000 4,000 4,000 4,000


Project B:
0 k = ? 1 2 6 Years
  
CFsB -14,815 5,100 5,100 5,100
Time lines:
Project A:
0 1 2 3 4 Years
k = 12%

CFsA -25,000 13,000 13,000 13,000 13,000


NPVA = ? = 17,663 Terminal value = 5,000
CF 4 = 18,000
Project B:
0 1 2 3 4 Years
k = ?

CFsB -25,000 15,247 15,247 15,247 15,247


NPVA = NPVB = 17,663 Terminal value = 0
CF 4 = 15,247
Time line:
0 k = 18% 1 2 3 Years

-3,000 1,728 1,920 1,152


Time line:
0 1 2 3 4 5 Years
k = 12%

-45,000 7,800 10,680 7,560 5,880 -1,920


Time line:
0 1 2 3 4 5 Years
k = 9%

-40,000 9,800 11,720 9,640 8,520 15,320


Time lines:
Project A:
0 k = 14% 1 2 3 4 Years

CFsA -200,000 71,104 71,104 71,104 71,104


Project B:
0 k = 10% 1 2 3 4 Years

CFsB -200,000 0 0 146,411 146,411


Time line:
0 k = 12% 1 2 10 Years
  
-50,000 6,000 6,000 6,000
Time lines:
Project A:
0 k = 12% 1 2 3 Periods

CFsA -5,000 2,000 2,500 2,250


Project B:
0 k = 14% 1 2 3 Periods

CFsB -5,000 3,000 2,600 2,900


0 1 2 3 4
| | | | |
CFsNew Tech -1,500 -315 -315 -315 -315 x x x
Time line:
0 1 2 3 Years
k = 14%

-42,000 14,280 16,200 11,400


TV = 18,120
29,520
Time line:
0 1 2 3 Years
k = 10%

-62,000 19,920 22,800 15,600


TV = 15,680
31,280
Expected increase in annual net income
ARR =
89 Initial (or average) investment .Answer (A) is correct. The higher the discount rate, the lower
the NPV. The IRR is the discount rate at which the NPV is zero. Consequently, if the NPV is negative, the discount rate
used must exceed the IRR.
Answer (B) is incorrect because, if the discount rate is less than the IRR, the NPV is positive. Answer (C) is incorrect
because the NPV measures the difference between a company's discount rate and the IRR. Answer (D) is incorrect
because the relationship between the discount rate and the risk-free rate is not a factor in investment analysis under the
NPV method.
NPV
($)
NPV
$
A

B B
A

k Discount rate (%)


0 10% IRRB IRRA 0 16% 17% 18% 30%

NPV
$
NPV ($)
P
C

k 10% k
10% 15% 20% %

NPV NPV
$

A A

B
B n
Sigma CF t
k k t=1
0 7% 12% 15% 0 7% 12% 14%
(1 + K)t
NPV
$

X
Crossover

k
10% IRRY 12% IRRX

0 k = 12% 1 2 3 4 5 6

CFsA -15,000 4,000 4,000 4,000 4,000 4,000 4,000


Project B:
0 k = ? 1 2 6 Years
  
CFsB -14,815 5,100 5,100 5,100
Time lines:
Project A:
0 1 2 3 4 Years
k = 12%

CFsA -25,000 13,000 13,000 13,000 13,000


NPVA = ? = 17,663 Terminal value = 5,000
CF 4 = 18,000
Project B:
0 1 2 3 4 Years
k = ?

CFsB -25,000 15,247 15,247 15,247 15,247


NPVA = NPVB = 17,663 Terminal value = 0
CF 4 = 15,247
Time line:
0 1 2 3 Years
k = 18%

-3,000 1,728 1,920 1,152


Time line:
0 1 2 3 4 5 Years
k = 12%

-45,000 7,800 10,680 7,560 5,880 -1,920


Time line:
0 k = 9% 1 2 3 4 5 Years

-40,000 9,800 11,720 9,640 8,520 15,320


Time lines:
Project A:
0 k = 14% 1 2 3 4 Years

CFsA -200,000 71,104 71,104 71,104 71,104


Project B:
0 k = 10% 1 2 3 4 Years

CFsB -200,000 0 0 146,411 146,411


Time line:
0 k = 12% 1 2 10 Years
  
-50,000 6,000 6,000 6,000
Time lines:
Project A:
0 k = 12% 1 2 3 Periods

CFsA -5,000 2,000 2,500 2,250


Project B:
0 k = 14% 1 2 3 Periods

CFsB -5,000 3,000 2,600 2,900


0 1 2 3 4
| | | | |
CFsNew Tech -1,500 -315 -315 -315 -315 x x x
Time line:
0 1 2 3 Years
k = 14%

-42,000 14,280 16,200 11,400


TV = 18,120
29,520
Time line:
0 1 2 3 Years
k = 10%

-62,000 19,920 22,800 15,600


TV = 15,680
31,280
Expected increase in annual net income
ARR =
90 Initial (or average) investment .Statement a is correct; the other statements are false. If the
projects are mutually exclusive, then project B may have a higher NPV even though Project A has a higher IRR. IRR is
calculated assuming cash flows are reinvested at the IRR, not the cost of capital.

NPV
($)
NPV
$
A

B B
A

k Discount rate (%)


0 10% IRRB IRRA 0 16% 17% 18% 30%

NPV
$
NPV ($)
P
C

k 10% k
10% 15% 20% %

NPV NPV
$

A A

B
B n
Sigma CF t
k k t=1
0 7% 12% 15% 0 7% 12% 14%
(1 + K)t
NPV
$

X
Crossover

k
10% IRRY 12% IRRX

0 k = 12% 1 2 3 4 5 6

CFsA -15,000 4,000 4,000 4,000 4,000 4,000 4,000


Project B:
0 k = ? 1 2 6 Years
  
CFsB -14,815 5,100 5,100 5,100
Time lines:
Project A:
0 1 2 3 4 Years
k = 12%

CFsA -25,000 13,000 13,000 13,000 13,000


NPVA = ? = 17,663 Terminal value = 5,000
CF 4 = 18,000
Project B:
0 1 2 3 4 Years
k = ?

CFsB -25,000 15,247 15,247 15,247 15,247


NPVA = NPVB = 17,663 Terminal value = 0
CF 4 = 15,247
Time line:
0 1 2 3 Years
k = 18%

-3,000 1,728 1,920 1,152


Time line:
0 1 2 3 4 5 Years
k = 12%

-45,000 7,800 10,680 7,560 5,880 -1,920


Time line:
0 k = 9% 1 2 3 4 5 Years

-40,000 9,800 11,720 9,640 8,520 15,320


Time lines:
Project A:
0 k = 14% 1 2 3 4 Years

CFsA -200,000 71,104 71,104 71,104 71,104


Project B:
0 k = 10% 1 2 3 4 Years

CFsB -200,000 0 0 146,411 146,411


Time line:
0 k = 12% 1 2 10 Years
  
-50,000 6,000 6,000 6,000
Time lines:
Project A:
0 k = 12% 1 2 3 Periods

CFsA -5,000 2,000 2,500 2,250


Project B:
0 k = 14% 1 2 3 Periods

CFsB -5,000 3,000 2,600 2,900


0 1 2 3 4
| | | | |
CFsNew Tech -1,500 -315 -315 -315 -315 x x x
Time line:
0 1 2 3 Years
k = 14%

-42,000 14,280 16,200 11,400


TV = 18,120
29,520
Time line:
0 1 2 3 Years
k = 10%

-62,000 19,920 22,800 15,600


TV = 15,680
31,280
Expected increase in annual net income
ARR =
91 Initial (or average) investment .The correct answer is a; the other statements are false. The
IRR is the discount rate at which a project’s NPV is zero. If a project’s IRR exceeds the firm’s cost of capital, then its
NPV must be positive, since NPV is calculated using the firm’s cost of capital to discount project cash flows.

NPV
($)
NPV
$
A

B B
A

k Discount rate (%)


0 10% IRRB IRRA 0 16% 17% 18% 30%

NPV
$
NPV ($)
P
C

k 10% k
10% 15% 20% %
NPV NPV
$

A A

B
B n
Sigma CF t
k k t=1
0 7% 12% 15% 0 7% 12% 14%
(1 + K)t
NPV
$

X
Crossover

k
10% IRRY 12% IRRX

0 k = 12% 1 2 3 4 5 6

CFsA -15,000 4,000 4,000 4,000 4,000 4,000 4,000


Project B:
0 k = ? 1 2 6 Years
  
CFsB -14,815 5,100 5,100 5,100
Time lines:
Project A:
0 1 2 3 4 Years
k = 12%

CFsA -25,000 13,000 13,000 13,000 13,000


NPVA = ? = 17,663 Terminal value = 5,000
CF 4 = 18,000
Project B:
0 1 2 3 4 Years
k = ?

CFsB -25,000 15,247 15,247 15,247 15,247


NPVA = NPVB = 17,663 Terminal value = 0
CF 4 = 15,247
Time line:
0 k = 18% 1 2 3 Years

-3,000 1,728 1,920 1,152


Time line:
0 1 2 3 4 5 Years
k = 12%

-45,000 7,800 10,680 7,560 5,880 -1,920


Time line:
0 1 2 3 4 5 Years
k = 9%

-40,000 9,800 11,720 9,640 8,520 15,320


Time lines:
Project A:
0 k = 14% 1 2 3 4 Years

CFsA -200,000 71,104 71,104 71,104 71,104


Project B:
0 k = 10% 1 2 3 4 Years

CFsB -200,000 0 0 146,411 146,411


Time line:
0 k = 12% 1 2 10 Years
  
-50,000 6,000 6,000 6,000
Time lines:
Project A:
0 k = 12% 1 2 3 Periods

CFsA -5,000 2,000 2,500 2,250


Project B:
0 k = 14% 1 2 3 Periods

CFsB -5,000 3,000 2,600 2,900


0 1 2 3 4
| | | | |
CFsNew Tech -1,500 -315 -315 -315 -315 x x x
Time line:
0 1 2 3 Years
k = 14%

-42,000 14,280 16,200 11,400


TV = 18,120
29,520
Time line:
0 1 2 3 Years
k = 10%

-62,000 19,920 22,800 15,600


TV = 15,680
31,280
Expected increase in annual net income
ARR =
92 Initial (or average) investment .Statement a is true; projects with IRRs greater than the cost
of capital will have a positive NPV. Statement b is false because you know nothing about the relative magnitudes of the
projects. Statement c is false because the IRR is independent of the cost of capital. Therefore, the correct choice is
statement a.
NPV
($)
NPV
$
A

B B
A

k Discount rate (%)


0 10% IRRB IRRA 0 16% 17% 18% 30%

NPV
$
NPV ($)
P
C

k 10% k
10% 15% 20% %

NPV NPV
$

A A

B
B n
Sigma CF t
k k t=1
0 7% 12% 15% 0 7% 12% 14%
(1 + K)t
NPV
$

X
Crossover

k
10% IRRY 12% IRRX

0 k = 12% 1 2 3 4 5 6

CFsA -15,000 4,000 4,000 4,000 4,000 4,000 4,000


Project B:
0 k = ? 1 2 6 Years
  
CFsB -14,815 5,100 5,100 5,100
Time lines:
Project A:
0 1 2 3 4 Years
k = 12%

CFsA -25,000 13,000 13,000 13,000 13,000


NPVA = ? = 17,663 Terminal value = 5,000
CF 4 = 18,000
Project B:
0 1 2 3 4 Years
k = ?

CFsB -25,000 15,247 15,247 15,247 15,247


NPVA = NPVB = 17,663 Terminal value = 0
CF 4 = 15,247
Time line:
0 1 2 3 Years
k = 18%

-3,000 1,728 1,920 1,152


Time line:
0 1 2 3 4 5 Years
k = 12%

-45,000 7,800 10,680 7,560 5,880 -1,920


Time line:
0 k = 9% 1 2 3 4 5 Years

-40,000 9,800 11,720 9,640 8,520 15,320


Time lines:
Project A:
0 k = 14% 1 2 3 4 Years

CFsA -200,000 71,104 71,104 71,104 71,104


Project B:
0 k = 10% 1 2 3 4 Years

CFsB -200,000 0 0 146,411 146,411


Time line:
0 k = 12% 1 2 10 Years
  
-50,000 6,000 6,000 6,000
Time lines:
Project A:
0 k = 12% 1 2 3 Periods

CFsA -5,000 2,000 2,500 2,250


Project B:
0 k = 14% 1 2 3 Periods

CFsB -5,000 3,000 2,600 2,900


0 1 2 3 4
| | | | |
CFsNew Tech -1,500 -315 -315 -315 -315 x x x
Time line:
0 1 2 3 Years
k = 14%

-42,000 14,280 16,200 11,400


TV = 18,120
29,520
Time line:
0 1 2 3 Years
k = 10%

-62,000 19,920 22,800 15,600


TV = 15,680
31,280
Expected increase in annual net income
ARR =
93 Initial (or average) investment .The correct statement is b; the other statements are false.
Since Project A’s IRR is 15%, at a WACC of 15% NPVA = 0; however, Project B would still have a positive NPV. Given
the information in a, we can’t conclude which project’s NPV is going to be greater. Since we are given no details about
each project’s cash flows we cannot conclude anything about payback. Finally, IRR is independent of the discount rate,
that is, IRR stays the same no matter what the WACC is.

NPV
($)
NPV
$
A

B B
A

k Discount rate (%)


0 10% IRRB IRRA 0 16% 17% 18% 30%

NPV
$
NPV ($)
P
C

k 10% k
10% 15% 20% %
NPV NPV
$

A A

B
B n
Sigma CF t
k k t=1
0 7% 12% 15% 0 7% 12% 14%
(1 + K)t
NPV
$

X
Crossover

k
10% IRRY 12% IRRX

0 k = 12% 1 2 3 4 5 6

CFsA -15,000 4,000 4,000 4,000 4,000 4,000 4,000


Project B:
0 k = ? 1 2 6 Years
  
CFsB -14,815 5,100 5,100 5,100
Time lines:
Project A:
0 1 2 3 4 Years
k = 12%

CFsA -25,000 13,000 13,000 13,000 13,000


NPVA = ? = 17,663 Terminal value = 5,000
CF 4 = 18,000
Project B:
0 1 2 3 4 Years
k = ?

CFsB -25,000 15,247 15,247 15,247 15,247


NPVA = NPVB = 17,663 Terminal value = 0
CF 4 = 15,247
Time line:
0 k = 18% 1 2 3 Years

-3,000 1,728 1,920 1,152


Time line:
0 1 2 3 4 5 Years
k = 12%

-45,000 7,800 10,680 7,560 5,880 -1,920


Time line:
0 1 2 3 4 5 Years
k = 9%

-40,000 9,800 11,720 9,640 8,520 15,320


Time lines:
Project A:
0 k = 14% 1 2 3 4 Years

CFsA -200,000 71,104 71,104 71,104 71,104


Project B:
0 k = 10% 1 2 3 4 Years

CFsB -200,000 0 0 146,411 146,411


Time line:
0 k = 12% 1 2 10 Years
  
-50,000 6,000 6,000 6,000
Time lines:
Project A:
0 k = 12% 1 2 3 Periods

CFsA -5,000 2,000 2,500 2,250


Project B:
0 k = 14% 1 2 3 Periods

CFsB -5,000 3,000 2,600 2,900


0 1 2 3 4
| | | | |
CFsNew Tech -1,500 -315 -315 -315 -315 x x x
Time line:
0 1 2 3 Years
k = 14%

-42,000 14,280 16,200 11,400


TV = 18,120
29,520
Time line:
0 1 2 3 Years
k = 10%

-62,000 19,920 22,800 15,600


TV = 15,680
31,280
Expected increase in annual net income
ARR =
94 Initial (or average) investment .Statement a is false. The projects could easily have different
NPVs based on different cash flows and costs of capital. Statement b is false. NPV is dependent upon the size of the
project. Think about the NPV of a $3 project versus the NPV of a $3 million project. Statement c is false. NPV is
dependent on a project’s risk. Therefore, the correct choice is statement e.
NPV
($)
NPV
$
A

B B
A

k Discount rate (%)


0 10% IRRB IRRA 0 16% 17% 18% 30%

NPV
$
NPV ($)
P
C

k 10% k
10% 15% 20% %

NPV NPV
$

A A

B
B n
Sigma CF t
k k t=1
0 7% 12% 15% 0 7% 12% 14%
(1 + K)t
NPV
$

X
Crossover

k
10% IRRY 12% IRRX

0 k = 12% 1 2 3 4 5 6

CFsA -15,000 4,000 4,000 4,000 4,000 4,000 4,000


Project B:
0 k = ? 1 2 6 Years
  
CFsB -14,815 5,100 5,100 5,100
Time lines:
Project A:
0 1 2 3 4 Years
k = 12%

CFsA -25,000 13,000 13,000 13,000 13,000


NPVA = ? = 17,663 Terminal value = 5,000
CF 4 = 18,000
Project B:
0 1 2 3 4 Years
k = ?

CFsB -25,000 15,247 15,247 15,247 15,247


NPVA = NPVB = 17,663 Terminal value = 0
CF 4 = 15,247
Time line:
0 1 2 3 Years
k = 18%

-3,000 1,728 1,920 1,152


Time line:
0 1 2 3 4 5 Years
k = 12%

-45,000 7,800 10,680 7,560 5,880 -1,920


Time line:
0 k = 9% 1 2 3 4 5 Years

-40,000 9,800 11,720 9,640 8,520 15,320


Time lines:
Project A:
0 k = 14% 1 2 3 4 Years

CFsA -200,000 71,104 71,104 71,104 71,104


Project B:
0 k = 10% 1 2 3 4 Years

CFsB -200,000 0 0 146,411 146,411


Time line:
0 k = 12% 1 2 10 Years
  
-50,000 6,000 6,000 6,000
Time lines:
Project A:
0 k = 12% 1 2 3 Periods

CFsA -5,000 2,000 2,500 2,250


Project B:
0 k = 14% 1 2 3 Periods

CFsB -5,000 3,000 2,600 2,900


0 1 2 3 4
| | | | |
CFsNew Tech -1,500 -315 -315 -315 -315 x x x
Time line:
0 1 2 3 Years
k = 14%

-42,000 14,280 16,200 11,400


TV = 18,120
29,520
Time line:
0 1 2 3 Years
k = 10%

-62,000 19,920 22,800 15,600


TV = 15,680
31,280
Expected increase in annual net income
ARR =
95 Initial (or average) investment .A project’s NPV increases as the cost of capital declines. A
project’s IRR is independent of its cost of capital, while a project’s MIRR is dependent on the cost of capital since the
terminal value in the MIRR equation is compounded at the cost of capital.

NPV
($)
NPV
$
A

B B
A

k Discount rate (%)


0 10% IRRB IRRA 0 16% 17% 18% 30%

NPV
$
NPV ($)
P
C

k 10% k
10% 15% 20% %

NPV NPV
$

A A

B
B n
Sigma CF t
k k t=1
0 7% 12% 15% 0 7% 12% 14%
(1 + K)t
NPV
$

X
Crossover

k
10% IRRY 12% IRRX

0 k = 12% 1 2 3 4 5 6

CFsA -15,000 4,000 4,000 4,000 4,000 4,000 4,000


Project B:
0 k = ? 1 2 6 Years
  
CFsB -14,815 5,100 5,100 5,100
Time lines:
Project A:
0 1 2 3 4 Years
k = 12%

CFsA -25,000 13,000 13,000 13,000 13,000


NPVA = ? = 17,663 Terminal value = 5,000
CF 4 = 18,000
Project B:
0 1 2 3 4 Years
k = ?

CFsB -25,000 15,247 15,247 15,247 15,247


NPVA = NPVB = 17,663 Terminal value = 0
CF 4 = 15,247
Time line:
0 1 2 3 Years
k = 18%

-3,000 1,728 1,920 1,152


Time line:
0 1 2 3 4 5 Years
k = 12%

-45,000 7,800 10,680 7,560 5,880 -1,920


Time line:
0 k = 9% 1 2 3 4 5 Years

-40,000 9,800 11,720 9,640 8,520 15,320


Time lines:
Project A:
0 k = 14% 1 2 3 4 Years

CFsA -200,000 71,104 71,104 71,104 71,104


Project B:
0 k = 10% 1 2 3 4 Years

CFsB -200,000 0 0 146,411 146,411


Time line:
0 k = 12% 1 2 10 Years
  
-50,000 6,000 6,000 6,000
Time lines:
Project A:
0 k = 12% 1 2 3 Periods

CFsA -5,000 2,000 2,500 2,250


Project B:
0 k = 14% 1 2 3 Periods

CFsB -5,000 3,000 2,600 2,900


0 1 2 3 4
| | | | |
CFsNew Tech -1,500 -315 -315 -315 -315 x x x
Time line:
0 1 2 3 Years
k = 14%

-42,000 14,280 16,200 11,400


TV = 18,120
29,520
Time line:
0 1 2 3 Years
k = 10%

-62,000 19,920 22,800 15,600


TV = 15,680
31,280
Expected increase in annual net income
ARR =
96 Initial (or average) investment .Statement a is the incorrect statement. NPV is positive if
IRR is greater than the cost of capital.

NPV
($)
NPV
$
A

B B
A

k Discount rate (%)


0 10% IRRB IRRA 0 16% 17% 18% 30%

NPV
$
NPV ($)
P
C

k 10% k
10% 15% 20% %
NPV NPV
$

A A

B
B n
Sigma CF t
k k t=1
0 7% 12% 15% 0 7% 12% 14%
(1 + K)t
NPV
$

X
Crossover

k
10% IRRY 12% IRRX

0 k = 12% 1 2 3 4 5 6

CFsA -15,000 4,000 4,000 4,000 4,000 4,000 4,000


Project B:
0 k = ? 1 2 6 Years
  
CFsB -14,815 5,100 5,100 5,100
Time lines:
Project A:
0 1 2 3 4 Years
k = 12%

CFsA -25,000 13,000 13,000 13,000 13,000


NPVA = ? = 17,663 Terminal value = 5,000
CF 4 = 18,000
Project B:
0 1 2 3 4 Years
k = ?

CFsB -25,000 15,247 15,247 15,247 15,247


NPVA = NPVB = 17,663 Terminal value = 0
CF 4 = 15,247
Time line:
0 k = 18% 1 2 3 Years

-3,000 1,728 1,920 1,152


Time line:
0 1 2 3 4 5 Years
k = 12%

-45,000 7,800 10,680 7,560 5,880 -1,920


Time line:
0 1 2 3 4 5 Years
k = 9%

-40,000 9,800 11,720 9,640 8,520 15,320


Time lines:
Project A:
0 k = 14% 1 2 3 4 Years

CFsA -200,000 71,104 71,104 71,104 71,104


Project B:
0 k = 10% 1 2 3 4 Years

CFsB -200,000 0 0 146,411 146,411


Time line:
0 k = 12% 1 2 10 Years
  
-50,000 6,000 6,000 6,000
Time lines:
Project A:
0 k = 12% 1 2 3 Periods

CFsA -5,000 2,000 2,500 2,250


Project B:
0 k = 14% 1 2 3 Periods

CFsB -5,000 3,000 2,600 2,900


0 1 2 3 4
| | | | |
CFsNew Tech -1,500 -315 -315 -315 -315 x x x
Time line:
0 1 2 3 Years
k = 14%

-42,000 14,280 16,200 11,400


TV = 18,120
29,520
Time line:
0 1 2 3 Years
k = 10%

-62,000 19,920 22,800 15,600


TV = 15,680
31,280
Expected increase in annual net income
ARR =
97 Initial (or average) investment .Statement b is correct; the other statements are incorrect.
Statement a is incorrect; if the NPV > 0, then the return must be > 12%. Statement c is incorrect; if NPV > 0, then MIRR
> WACC.
NPV
($)
NPV
$
A

B B
A

k Discount rate (%)


0 10% IRRB IRRA 0 16% 17% 18% 30%

NPV
$
NPV ($)
P
C

k 10% k
10% 15% 20% %

NPV NPV
$

A A

B
B n
Sigma CF t
k k t=1
0 7% 12% 15% 0 7% 12% 14%
(1 + K)t
NPV
$

X
Crossover

k
10% IRRY 12% IRRX

0 k = 12% 1 2 3 4 5 6

CFsA -15,000 4,000 4,000 4,000 4,000 4,000 4,000


Project B:
0 k = ? 1 2 6 Years
  
CFsB -14,815 5,100 5,100 5,100
Time lines:
Project A:
0 1 2 3 4 Years
k = 12%

CFsA -25,000 13,000 13,000 13,000 13,000


NPVA = ? = 17,663 Terminal value = 5,000
CF 4 = 18,000
Project B:
0 1 2 3 4 Years
k = ?

CFsB -25,000 15,247 15,247 15,247 15,247


NPVA = NPVB = 17,663 Terminal value = 0
CF 4 = 15,247
Time line:
0 1 2 3 Years
k = 18%

-3,000 1,728 1,920 1,152


Time line:
0 1 2 3 4 5 Years
k = 12%

-45,000 7,800 10,680 7,560 5,880 -1,920


Time line:
0 k = 9% 1 2 3 4 5 Years

-40,000 9,800 11,720 9,640 8,520 15,320


Time lines:
Project A:
0 k = 14% 1 2 3 4 Years

CFsA -200,000 71,104 71,104 71,104 71,104


Project B:
0 k = 10% 1 2 3 4 Years

CFsB -200,000 0 0 146,411 146,411


Time line:
0 k = 12% 1 2 10 Years
  
-50,000 6,000 6,000 6,000
Time lines:
Project A:
0 k = 12% 1 2 3 Periods

CFsA -5,000 2,000 2,500 2,250


Project B:
0 k = 14% 1 2 3 Periods

CFsB -5,000 3,000 2,600 2,900


0 1 2 3 4
| | | | |
CFsNew Tech -1,500 -315 -315 -315 -315 x x x
Time line:
0 1 2 3 Years
k = 14%

-42,000 14,280 16,200 11,400


TV = 18,120
29,520
Time line:
0 1 2 3 Years
k = 10%

-62,000 19,920 22,800 15,600


TV = 15,680
31,280
Expected increase in annual net income
ARR =
98 Initial (or average) investment .Statement e is correct; the other statements are incorrect.
Statement a is incorrect; the two projects’ NPV profiles could cross, consequently, a higher IRR doesn’t guarantee a
higher NPV. Statement b is incorrect; it the two projects’ NPV profiles cross, Y could have a higher NPV. Statement c is
incorrect; we don’t have enough information.

NPV
($)
NPV
$
A

B B
A

k Discount rate (%)


0 10% IRRB IRRA 0 16% 17% 18% 30%

NPV
$
NPV ($)
P
C

k 10% k
10% 15% 20% %
NPV NPV
$

A A

B
B n
Sigma CF t
k k t=1
0 7% 12% 15% 0 7% 12% 14%
(1 + K)t
NPV
$

X
Crossover

k
10% IRRY 12% IRRX

0 k = 12% 1 2 3 4 5 6

CFsA -15,000 4,000 4,000 4,000 4,000 4,000 4,000


Project B:
0 k = ? 1 2 6 Years
  
CFsB -14,815 5,100 5,100 5,100
Time lines:
Project A:
0 1 2 3 4 Years
k = 12%

CFsA -25,000 13,000 13,000 13,000 13,000


NPVA = ? = 17,663 Terminal value = 5,000
CF 4 = 18,000
Project B:
0 1 2 3 4 Years
k = ?

CFsB -25,000 15,247 15,247 15,247 15,247


NPVA = NPVB = 17,663 Terminal value = 0
CF 4 = 15,247
Time line:
0 k = 18% 1 2 3 Years

-3,000 1,728 1,920 1,152


Time line:
0 1 2 3 4 5 Years
k = 12%

-45,000 7,800 10,680 7,560 5,880 -1,920


Time line:
0 1 2 3 4 5 Years
k = 9%

-40,000 9,800 11,720 9,640 8,520 15,320


Time lines:
Project A:
0 k = 14% 1 2 3 4 Years

CFsA -200,000 71,104 71,104 71,104 71,104


Project B:
0 k = 10% 1 2 3 4 Years

CFsB -200,000 0 0 146,411 146,411


Time line:
0 k = 12% 1 2 10 Years
  
-50,000 6,000 6,000 6,000
Time lines:
Project A:
0 k = 12% 1 2 3 Periods

CFsA -5,000 2,000 2,500 2,250


Project B:
0 k = 14% 1 2 3 Periods

CFsB -5,000 3,000 2,600 2,900


0 1 2 3 4
| | | | |
CFsNew Tech -1,500 -315 -315 -315 -315 x x x
Time line:
0 1 2 3 Years
k = 14%

-42,000 14,280 16,200 11,400


TV = 18,120
29,520
Time line:
0 1 2 3 Years
k = 10%

-62,000 19,920 22,800 15,600


TV = 15,680
31,280
Expected increase in annual net income
ARR =
99 Initial (or average) investment .Statement a is true because the IRR exceeds the WACC.
Statement b is also true because the MIRR assumes that the inflows are reinvested at the WACC, which is less than the
IRR. Statement c is false. For a normal project, the discounted payback is always longer than the regular payback
because it takes longer for the discounted cash flows to cover the purchase price. So, statement d is the best choice.
NPV
($)
NPV
$
A

B B
A

k Discount rate (%)


0 10% IRRB IRRA 0 16% 17% 18% 30%

NPV
$
NPV ($)
P
C

k 10% k
10% 15% 20% %

NPV NPV
$

A A

B
B n
Sigma CF t
k k t=1
0 7% 12% 15% 0 7% 12% 14%
(1 + K)t
NPV
$

X
Crossover

k
10% IRRY 12% IRRX

0 k = 12% 1 2 3 4 5 6

CFsA -15,000 4,000 4,000 4,000 4,000 4,000 4,000


Project B:
0 k = ? 1 2 6 Years
  
CFsB -14,815 5,100 5,100 5,100
Time lines:
Project A:
0 1 2 3 4 Years
k = 12%

CFsA -25,000 13,000 13,000 13,000 13,000


NPVA = ? = 17,663 Terminal value = 5,000
CF 4 = 18,000
Project B:
0 1 2 3 4 Years
k = ?

CFsB -25,000 15,247 15,247 15,247 15,247


NPVA = NPVB = 17,663 Terminal value = 0
CF 4 = 15,247
Time line:
0 1 2 3 Years
k = 18%

-3,000 1,728 1,920 1,152


Time line:
0 1 2 3 4 5 Years
k = 12%

-45,000 7,800 10,680 7,560 5,880 -1,920


Time line:
0 k = 9% 1 2 3 4 5 Years

-40,000 9,800 11,720 9,640 8,520 15,320


Time lines:
Project A:
0 k = 14% 1 2 3 4 Years

CFsA -200,000 71,104 71,104 71,104 71,104


Project B:
0 k = 10% 1 2 3 4 Years

CFsB -200,000 0 0 146,411 146,411


Time line:
0 k = 12% 1 2 10 Years
  
-50,000 6,000 6,000 6,000
Time lines:
Project A:
0 k = 12% 1 2 3 Periods

CFsA -5,000 2,000 2,500 2,250


Project B:
0 k = 14% 1 2 3 Periods

CFsB -5,000 3,000 2,600 2,900


0 1 2 3 4
| | | | |
CFsNew Tech -1,500 -315 -315 -315 -315 x x x
Time line:
0 1 2 3 Years
k = 14%

-42,000 14,280 16,200 11,400


TV = 18,120
29,520
Time line:
0 1 2 3 Years
k = 10%

-62,000 19,920 22,800 15,600


TV = 15,680
31,280
Expected increase in annual net income
ARR =
100 Initial (or average) investment .Answer (A) is correct. If taxes are ignored, depreciation is
not a consideration in any of the methods based on cash flows because it is a non-cash expense. Thus, the internal rate
of return, net present value, and payback methods would not consider depreciation because these methods are based
on cash flows. However, the accounting rate of return is based on net income as calculated on an income statement.
Because depreciation is included in the determination of accrual accounting net income, it would affect the calculation of
the accounting rate of return.
Answer (B) is incorrect because the IRR and the payback period are based on cash flows. Depreciation is not needed in
their calculation. However, the accounting rate of return cannot be calculated without first deducting depreciation.
Answer (C) is incorrect because the IRR and the payback period are based on cash flows. Depreciation is not needed in
their calculation. However, the accounting rate of return cannot be calculated without first deducting depreciation.
Answer (D) is incorrect because the IRR and the payback period are based on cash flows. Depreciation is not needed in
their calculation. However, the accounting rate of return cannot be calculated without first deducting depreciation.

NPV
($)
NPV
$
A

B B
A

k Discount rate (%)


0 10% IRRB IRRA 0 16% 17% 18% 30%

NPV
$
NPV ($)
P
C

k 10% k
10% 15% 20% %
NPV NPV
$

A A

B
B n
Sigma CF t
k k t=1
0 7% 12% 15% 0 7% 12% 14%
(1 + K)t
NPV
$

X
Crossover

k
10% IRRY 12% IRRX

0 k = 12% 1 2 3 4 5 6

CFsA -15,000 4,000 4,000 4,000 4,000 4,000 4,000


Project B:
0 k = ? 1 2 6 Years
  
CFsB -14,815 5,100 5,100 5,100
Time lines:
Project A:
0 1 2 3 4 Years
k = 12%

CFsA -25,000 13,000 13,000 13,000 13,000


NPVA = ? = 17,663 Terminal value = 5,000
CF 4 = 18,000
Project B:
0 1 2 3 4 Years
k = ?

CFsB -25,000 15,247 15,247 15,247 15,247


NPVA = NPVB = 17,663 Terminal value = 0
CF 4 = 15,247
Time line:
0 k = 18% 1 2 3 Years

-3,000 1,728 1,920 1,152


Time line:
0 1 2 3 4 5 Years
k = 12%

-45,000 7,800 10,680 7,560 5,880 -1,920


Time line:
0 1 2 3 4 5 Years
k = 9%

-40,000 9,800 11,720 9,640 8,520 15,320


Time lines:
Project A:
0 k = 14% 1 2 3 4 Years

CFsA -200,000 71,104 71,104 71,104 71,104


Project B:
0 k = 10% 1 2 3 4 Years

CFsB -200,000 0 0 146,411 146,411


Time line:
0 k = 12% 1 2 10 Years
  
-50,000 6,000 6,000 6,000
Time lines:
Project A:
0 k = 12% 1 2 3 Periods

CFsA -5,000 2,000 2,500 2,250


Project B:
0 k = 14% 1 2 3 Periods

CFsB -5,000 3,000 2,600 2,900


0 1 2 3 4
| | | | |
CFsNew Tech -1,500 -315 -315 -315 -315 x x x
Time line:
0 1 2 3 Years
k = 14%

-42,000 14,280 16,200 11,400


TV = 18,120
29,520
Time line:
0 1 2 3 Years
k = 10%

-62,000 19,920 22,800 15,600


TV = 15,680
31,280
Expected increase in annual net income
ARR =
101 Initial (or average) investment .Answer (C) is correct. The profitability index is the ratio of
the present value of future net cash inflows to the initial net cash investment. It is a variation of the net present value
(NPV) method and facilitates the comparison of different-sized investments. Because it is based on the NPV method,
the profitability index will yield the same decision as the NPV for independent projects. However, decisions may differ for
mutually exclusive projects of different sizes.
Answer (A) is incorrect because the profitability index, like the NPV method, discounts cash flows based on the cost of
capital. Answer (B) is incorrect because the profitability index is cash based. Answer (D) is incorrect because the NPV
and the profitability index may yield different decisions if projects are mutually exclusive and of different sizes.
NPV
($)
NPV
$
A

B B
A

k Discount rate (%)


0 10% IRRB IRRA 0 16% 17% 18% 30%

NPV
$
NPV ($)
P
C

k 10% k
10% 15% 20% %

NPV NPV
$

A A

B
B n
Sigma CF t
k k t=1
0 7% 12% 15% 0 7% 12% 14%
(1 + K)t
NPV
$

X
Crossover

k
10% IRRY 12% IRRX

0 k = 12% 1 2 3 4 5 6

CFsA -15,000 4,000 4,000 4,000 4,000 4,000 4,000


Project B:
0 k = ? 1 2 6 Years
  
CFsB -14,815 5,100 5,100 5,100
Time lines:
Project A:
0 1 2 3 4 Years
k = 12%

CFsA -25,000 13,000 13,000 13,000 13,000


NPVA = ? = 17,663 Terminal value = 5,000
CF 4 = 18,000
Project B:
0 1 2 3 4 Years
k = ?

CFsB -25,000 15,247 15,247 15,247 15,247


NPVA = NPVB = 17,663 Terminal value = 0
CF 4 = 15,247
Time line:
0 1 2 3 Years
k = 18%

-3,000 1,728 1,920 1,152


Time line:
0 1 2 3 4 5 Years
k = 12%

-45,000 7,800 10,680 7,560 5,880 -1,920


Time line:
0 k = 9% 1 2 3 4 5 Years

-40,000 9,800 11,720 9,640 8,520 15,320


Time lines:
Project A:
0 k = 14% 1 2 3 4 Years

CFsA -200,000 71,104 71,104 71,104 71,104


Project B:
0 k = 10% 1 2 3 4 Years

CFsB -200,000 0 0 146,411 146,411


Time line:
0 k = 12% 1 2 10 Years
  
-50,000 6,000 6,000 6,000
Time lines:
Project A:
0 k = 12% 1 2 3 Periods

CFsA -5,000 2,000 2,500 2,250


Project B:
0 k = 14% 1 2 3 Periods

CFsB -5,000 3,000 2,600 2,900


0 1 2 3 4
| | | | |
CFsNew Tech -1,500 -315 -315 -315 -315 x x x
Time line:
0 1 2 3 Years
k = 14%

-42,000 14,280 16,200 11,400


TV = 18,120
29,520
Time line:
0 1 2 3 Years
k = 10%

-62,000 19,920 22,800 15,600


TV = 15,680
31,280
Expected increase in annual net income
ARR =
102 Initial (or average) investment .Answer (D) is correct. All three managers will reject the
project. Manager one will calculate a NPV of -$12,894 [-$100,000 + ($20,000 x 4.3553 PVIFA for six periods at 10%)].
Manager two will calculate a NPV of -$26,349 {- $100,000 + ($5,000 x .8772 PVIF for one period at 14%) + [($23,000 x
3.4331 PVIFA for five periods at 14%) x .8772 PVIF for one period at 14%]}. Manager three will calculate a net present
value of -$41,640 [-$100,000 + ($135,000 x .4323 PVIF for six periods at 15%)].
Answer (A) is incorrect because all three managers will calculate a negative NPV, and none will recommend
acceptance. Answer (B) is incorrect because all three managers will calculate a negative NPV, and none will
recommend acceptance. Answer (C) is incorrect because all three managers will calculate a negative NPV, and none
will recommend acceptance.

NPV
($)
NPV
$
A

B B
A

k Discount rate (%)


0 10% IRRB IRRA 0 16% 17% 18% 30%

NPV
$
NPV ($)
P
C

k 10% k
10% 15% 20% %
NPV NPV
$

A A

B
B n
Sigma CF t
k k t=1
0 7% 12% 15% 0 7% 12% 14%
(1 + K)t
NPV
$

X
Crossover

k
10% IRRY 12% IRRX

0 k = 12% 1 2 3 4 5 6

CFsA -15,000 4,000 4,000 4,000 4,000 4,000 4,000


Project B:
0 k = ? 1 2 6 Years
  
CFsB -14,815 5,100 5,100 5,100
Time lines:
Project A:
0 1 2 3 4 Years
k = 12%

CFsA -25,000 13,000 13,000 13,000 13,000


NPVA = ? = 17,663 Terminal value = 5,000
CF 4 = 18,000
Project B:
0 1 2 3 4 Years
k = ?

CFsB -25,000 15,247 15,247 15,247 15,247


NPVA = NPVB = 17,663 Terminal value = 0
CF 4 = 15,247
Time line:
0 k = 18% 1 2 3 Years

-3,000 1,728 1,920 1,152


Time line:
0 1 2 3 4 5 Years
k = 12%

-45,000 7,800 10,680 7,560 5,880 -1,920


Time line:
0 1 2 3 4 5 Years
k = 9%

-40,000 9,800 11,720 9,640 8,520 15,320


Time lines:
Project A:
0 k = 14% 1 2 3 4 Years

CFsA -200,000 71,104 71,104 71,104 71,104


Project B:
0 k = 10% 1 2 3 4 Years

CFsB -200,000 0 0 146,411 146,411


Time line:
0 k = 12% 1 2 10 Years
  
-50,000 6,000 6,000 6,000
Time lines:
Project A:
0 k = 12% 1 2 3 Periods

CFsA -5,000 2,000 2,500 2,250


Project B:
0 k = 14% 1 2 3 Periods

CFsB -5,000 3,000 2,600 2,900


0 1 2 3 4
| | | | |
CFsNew Tech -1,500 -315 -315 -315 -315 x x x
Time line:
0 1 2 3 Years
k = 14%

-42,000 14,280 16,200 11,400


TV = 18,120
29,520
Time line:
0 1 2 3 Years
k = 10%

-62,000 19,920 22,800 15,600


TV = 15,680
31,280
Expected increase in annual net income
ARR =
103 Initial (or average) investment .Answer (A) is correct. The payback period is the number of
periods it takes before the cash flows from the project repay the original investment outlay. This can be expressed as
net investment divided by the average expected cash flow. Manager one expects inflows of $20,000 per year, so it will
take exactly 5 years for the project to repay the original $100,000 invested. Manager two will calculate a payback period
of more than 5 years. Only $5,000 is expected at the end of year one, followed by inflows of $23,000 at the end of each
year in years two through six. At the end of year five, only $97,000 will have been received, based on these
expectations. Manager three will calculate a payback period of 6 years. She estimates one inflow of $135,000 at the end
of year six.
Answer (B) is incorrect because manager one will calculate a 5-year payback period, which is shorter than the periods
determined by managers two and three. Answer (C) is incorrect because manager one will calculate a 5-year payback
period, which is shorter than the periods determined by managers two and three. Answer (D) is incorrect because all
three managers will derive a different payback period for the project.
NPV
($)
NPV
$
A

B B
A

k Discount rate (%)


0 10% IRRB IRRA 0 16% 17% 18% 30%

NPV
$
NPV ($)
P
C

k 10% k
10% 15% 20% %

NPV NPV
$

A A

B
B n
Sigma CF t
k k t=1
0 7% 12% 15% 0 7% 12% 14%
(1 + K)t
NPV
$

X
Crossover

k
10% IRRY 12% IRRX

0 k = 12% 1 2 3 4 5 6

CFsA -15,000 4,000 4,000 4,000 4,000 4,000 4,000


Project B:
0 k = ? 1 2 6 Years
  
CFsB -14,815 5,100 5,100 5,100
Time lines:
Project A:
0 1 2 3 4 Years
k = 12%

CFsA -25,000 13,000 13,000 13,000 13,000


NPVA = ? = 17,663 Terminal value = 5,000
CF 4 = 18,000
Project B:
0 1 2 3 4 Years
k = ?

CFsB -25,000 15,247 15,247 15,247 15,247


NPVA = NPVB = 17,663 Terminal value = 0
CF 4 = 15,247
Time line:
0 1 2 3 Years
k = 18%

-3,000 1,728 1,920 1,152


Time line:
0 1 2 3 4 5 Years
k = 12%

-45,000 7,800 10,680 7,560 5,880 -1,920


Time line:
0 k = 9% 1 2 3 4 5 Years

-40,000 9,800 11,720 9,640 8,520 15,320


Time lines:
Project A:
0 k = 14% 1 2 3 4 Years

CFsA -200,000 71,104 71,104 71,104 71,104


Project B:
0 k = 10% 1 2 3 4 Years

CFsB -200,000 0 0 146,411 146,411


Time line:
0 k = 12% 1 2 10 Years
  
-50,000 6,000 6,000 6,000
Time lines:
Project A:
0 k = 12% 1 2 3 Periods

CFsA -5,000 2,000 2,500 2,250


Project B:
0 k = 14% 1 2 3 Periods

CFsB -5,000 3,000 2,600 2,900


0 1 2 3 4
| | | | |
CFsNew Tech -1,500 -315 -315 -315 -315 x x x
Time line:
0 1 2 3 Years
k = 14%

-42,000 14,280 16,200 11,400


TV = 18,120
29,520
Time line:
0 1 2 3 Years
k = 10%

-62,000 19,920 22,800 15,600


TV = 15,680
31,280
Expected increase in annual net income
ARR =
104 Initial (or average) investment .Answer (B) is correct. If the net present value (NPV) of an
investment is positive, the project should be accepted (unless projects are mutually exclusive). If the NPV is negative,
the investment should be rejected.
Answer (A) is incorrect because the present value of future net cash inflows must be compared with the initial cash
outlay to determine whether a project is acceptable. Answer (C) is incorrect because an IRR may be greater than zero
but less than a firm's cost of capital, in which case the project would not be profitable. Answer (D) is incorrect because
the accounting rate of return is not based on cash flows and is irrelevant to a company's hurdle rate.

NPV
($)
NPV
$
A

B B
A

k Discount rate (%)


0 10% IRRB IRRA 0 16% 17% 18% 30%

NPV
$
NPV ($)
P
C

k 10% k
10% 15% 20% %
NPV NPV
$

A A

B
B n
Sigma CF t
k k t=1
0 7% 12% 15% 0 7% 12% 14%
(1 + K)t
NPV
$

X
Crossover

k
10% IRRY 12% IRRX

0 k = 12% 1 2 3 4 5 6

CFsA -15,000 4,000 4,000 4,000 4,000 4,000 4,000


Project B:
0 k = ? 1 2 6 Years
  
CFsB -14,815 5,100 5,100 5,100
Time lines:
Project A:
0 1 2 3 4 Years
k = 12%

CFsA -25,000 13,000 13,000 13,000 13,000


NPVA = ? = 17,663 Terminal value = 5,000
CF 4 = 18,000
Project B:
0 1 2 3 4 Years
k = ?

CFsB -25,000 15,247 15,247 15,247 15,247


NPVA = NPVB = 17,663 Terminal value = 0
CF 4 = 15,247
Time line:
0 k = 18% 1 2 3 Years

-3,000 1,728 1,920 1,152


Time line:
0 1 2 3 4 5 Years
k = 12%

-45,000 7,800 10,680 7,560 5,880 -1,920


Time line:
0 1 2 3 4 5 Years
k = 9%

-40,000 9,800 11,720 9,640 8,520 15,320


Time lines:
Project A:
0 k = 14% 1 2 3 4 Years

CFsA -200,000 71,104 71,104 71,104 71,104


Project B:
0 k = 10% 1 2 3 4 Years

CFsB -200,000 0 0 146,411 146,411


Time line:
0 k = 12% 1 2 10 Years
  
-50,000 6,000 6,000 6,000
Time lines:
Project A:
0 k = 12% 1 2 3 Periods

CFsA -5,000 2,000 2,500 2,250


Project B:
0 k = 14% 1 2 3 Periods

CFsB -5,000 3,000 2,600 2,900


0 1 2 3 4
| | | | |
CFsNew Tech -1,500 -315 -315 -315 -315 x x x
Time line:
0 1 2 3 Years
k = 14%

-42,000 14,280 16,200 11,400


TV = 18,120
29,520
Time line:
0 1 2 3 Years
k = 10%

-62,000 19,920 22,800 15,600


TV = 15,680
31,280
Expected increase in annual net income
ARR =
105 Initial (or average) investment .Answer (D) is correct. Given unlimited funds, all projects with
a net present value greater than zero should be invested in. Thus, it would be profitable to invest in any company where
the rate of return is greater than the cost of capital.
Answer (A) is incorrect because neither the accounting rate of return nor the earnings as a percent of sales is useful in
capital budgeting. The accounting rate of return is accounting net income over the required investment; it ignores the
time value of money. Earnings as a percent of sales ignores the amount of required investment. Answer (B) is incorrect
because the payback criterion for capital budgeting is not efficient or effective. Answer (C) is incorrect because the
problem states that there are unlimited capital funds but does not indicate what the cost of capital is. Accordingly,
projects can only be invested in when the internal rate of return is greater than cost of capital, i.e., the net present value
is greater than zero.
NPV
($)
NPV
$
A

B B
A

k Discount rate (%)


0 10% IRRB IRRA 0 16% 17% 18% 30%

NPV
$
NPV ($)
P
C

k 10% k
10% 15% 20% %

NPV NPV
$

A A

B
B n
Sigma CF t
k k t=1
0 7% 12% 15% 0 7% 12% 14%
(1 + K)t
NPV
$

X
Crossover

k
10% IRRY 12% IRRX

0 k = 12% 1 2 3 4 5 6

CFsA -15,000 4,000 4,000 4,000 4,000 4,000 4,000


Project B:
0 k = ? 1 2 6 Years
  
CFsB -14,815 5,100 5,100 5,100
Time lines:
Project A:
0 1 2 3 4 Years
k = 12%

CFsA -25,000 13,000 13,000 13,000 13,000


NPVA = ? = 17,663 Terminal value = 5,000
CF 4 = 18,000
Project B:
0 1 2 3 4 Years
k = ?

CFsB -25,000 15,247 15,247 15,247 15,247


NPVA = NPVB = 17,663 Terminal value = 0
CF 4 = 15,247
Time line:
0 1 2 3 Years
k = 18%

-3,000 1,728 1,920 1,152


Time line:
0 1 2 3 4 5 Years
k = 12%

-45,000 7,800 10,680 7,560 5,880 -1,920


Time line:
0 k = 9% 1 2 3 4 5 Years

-40,000 9,800 11,720 9,640 8,520 15,320


Time lines:
Project A:
0 k = 14% 1 2 3 4 Years

CFsA -200,000 71,104 71,104 71,104 71,104


Project B:
0 k = 10% 1 2 3 4 Years

CFsB -200,000 0 0 146,411 146,411


Time line:
0 k = 12% 1 2 10 Years
  
-50,000 6,000 6,000 6,000
Time lines:
Project A:
0 k = 12% 1 2 3 Periods

CFsA -5,000 2,000 2,500 2,250


Project B:
0 k = 14% 1 2 3 Periods

CFsB -5,000 3,000 2,600 2,900


0 1 2 3 4
| | | | |
CFsNew Tech -1,500 -315 -315 -315 -315 x x x
Time line:
0 1 2 3 Years
k = 14%

-42,000 14,280 16,200 11,400


TV = 18,120
29,520
Time line:
0 1 2 3 Years
k = 10%

-62,000 19,920 22,800 15,600


TV = 15,680
31,280
Expected increase in annual net income
ARR =
106 Initial (or average) investment .Answer (D) is correct. A company should accept any
investment proposal, unless some are mutually exclusive, that has a positive net present value or an internal rate of
return greater than the company's cost of capital.
Answer (A) is incorrect because the mere availability of financing is not the only consideration; more important is the
cost of the financing, which must be less than the rate of return on the proposed investment. Answer (B) is incorrect
because an investment with positive cash flows may be a bad investment due to the time value of money; cash flows in
later years are not as valuable as those in earlier years. Answer (C) is incorrect because returns should exceed the
weighted-average cost of capital, which includes the cost of equity capital as well as the cost of debt capital.

NPV
($)
NPV
$
A

B B
A

k Discount rate (%)


0 10% IRRB IRRA 0 16% 17% 18% 30%

NPV
$
NPV ($)
P
C

k 10% k
10% 15% 20% %
NPV NPV
$

A A

B
B n
Sigma CF t
k k t=1
0 7% 12% 15% 0 7% 12% 14%
(1 + K)t
NPV
$

X
Crossover

k
10% IRRY 12% IRRX

0 k = 12% 1 2 3 4 5 6

CFsA -15,000 4,000 4,000 4,000 4,000 4,000 4,000


Project B:
0 k = ? 1 2 6 Years
  
CFsB -14,815 5,100 5,100 5,100
Time lines:
Project A:
0 1 2 3 4 Years
k = 12%

CFsA -25,000 13,000 13,000 13,000 13,000


NPVA = ? = 17,663 Terminal value = 5,000
CF 4 = 18,000
Project B:
0 1 2 3 4 Years
k = ?

CFsB -25,000 15,247 15,247 15,247 15,247


NPVA = NPVB = 17,663 Terminal value = 0
CF 4 = 15,247
Time line:
0 k = 18% 1 2 3 Years

-3,000 1,728 1,920 1,152


Time line:
0 1 2 3 4 5 Years
k = 12%

-45,000 7,800 10,680 7,560 5,880 -1,920


Time line:
0 1 2 3 4 5 Years
k = 9%

-40,000 9,800 11,720 9,640 8,520 15,320


Time lines:
Project A:
0 k = 14% 1 2 3 4 Years

CFsA -200,000 71,104 71,104 71,104 71,104


Project B:
0 k = 10% 1 2 3 4 Years

CFsB -200,000 0 0 146,411 146,411


Time line:
0 k = 12% 1 2 10 Years
  
-50,000 6,000 6,000 6,000
Time lines:
Project A:
0 k = 12% 1 2 3 Periods

CFsA -5,000 2,000 2,500 2,250


Project B:
0 k = 14% 1 2 3 Periods

CFsB -5,000 3,000 2,600 2,900


0 1 2 3 4
| | | | |
CFsNew Tech -1,500 -315 -315 -315 -315 x x x
Time line:
0 1 2 3 Years
k = 14%

-42,000 14,280 16,200 11,400


TV = 18,120
29,520
Time line:
0 1 2 3 Years
k = 10%

-62,000 19,920 22,800 15,600


TV = 15,680
31,280
Expected increase in annual net income
ARR =
107 Initial (or average) investment .REQUIRED: The true statement about the NPV and IRR
methods.
DISCUSSION: (A) The NPV criterion is that the NPV is positive, and the IRR criterion is that the cost of capital is less
than the IRR. When the cost of capital is less than the IRR, the NPV is positive. When it exceeds the IRR, the NPV is
negative. Accordingly, when two projects are independent, the NPV and IRR criteria will always lead to the same accept
or reject decision.
Answer (B) is incorrect because, if the second project’s IRR is higher than the first project’s, the organization would
accept the second project based on the IRR criterion. Answers (C) and (D) are incorrect because, if the projects are
independent, the NPV and IRR criteria indicates the same decision.
NPV
($)
NPV
$
A

B B
A

k Discount rate (%)


0 10% IRRB IRRA 0 16% 17% 18% 30%

NPV
$
NPV ($)
P
C

k 10% k
10% 15% 20% %

NPV NPV
$

A A

B
B n
Sigma CF t
k k t=1
0 7% 12% 15% 0 7% 12% 14%
(1 + K)t
NPV
$

X
Crossover

k
10% IRRY 12% IRRX

0 k = 12% 1 2 3 4 5 6

CFsA -15,000 4,000 4,000 4,000 4,000 4,000 4,000


Project B:
0 k = ? 1 2 6 Years
  
CFsB -14,815 5,100 5,100 5,100
Time lines:
Project A:
0 1 2 3 4 Years
k = 12%

CFsA -25,000 13,000 13,000 13,000 13,000


NPVA = ? = 17,663 Terminal value = 5,000
CF 4 = 18,000
Project B:
0 1 2 3 4 Years
k = ?

CFsB -25,000 15,247 15,247 15,247 15,247


NPVA = NPVB = 17,663 Terminal value = 0
CF 4 = 15,247
Time line:
0 1 2 3 Years
k = 18%

-3,000 1,728 1,920 1,152


Time line:
0 1 2 3 4 5 Years
k = 12%

-45,000 7,800 10,680 7,560 5,880 -1,920


Time line:
0 k = 9% 1 2 3 4 5 Years

-40,000 9,800 11,720 9,640 8,520 15,320


Time lines:
Project A:
0 k = 14% 1 2 3 4 Years

CFsA -200,000 71,104 71,104 71,104 71,104


Project B:
0 k = 10% 1 2 3 4 Years

CFsB -200,000 0 0 146,411 146,411


Time line:
0 k = 12% 1 2 10 Years
  
-50,000 6,000 6,000 6,000
Time lines:
Project A:
0 k = 12% 1 2 3 Periods

CFsA -5,000 2,000 2,500 2,250


Project B:
0 k = 14% 1 2 3 Periods

CFsB -5,000 3,000 2,600 2,900


0 1 2 3 4
| | | | |
CFsNew Tech -1,500 -315 -315 -315 -315 x x x
Time line:
0 1 2 3 Years
k = 14%

-42,000 14,280 16,200 11,400


TV = 18,120
29,520
Time line:
0 1 2 3 Years
k = 10%

-62,000 19,920 22,800 15,600


TV = 15,680
31,280
Expected increase in annual net income
ARR =
108 Initial (or average) investment .Answer (A) is correct. Although the NPV method and the
IRR method may rank projects differently, if a project is found acceptable under the NPV approach, it will also be
acceptable under the internal rate of return approach.
Answer (B) is incorrect because the two approaches may rank projects differently (the IRR assumes that reinvestment
will be at the discount rate, which is frequently not possible). Answer (C) is incorrect because the payback approach
does not consider the time value of money. Therefore, a project may be ranked differently than it would be under the
NPV approach or may be acceptable under the payback approach but not the NPV or IRR approaches. Answer (D) is
incorrect because the payback approach does not consider the time value of money. Therefore, a project may be ranked
differently than it would be under the NPV approach or may be acceptable under the payback approach but not the NPV
or IRR approaches.

NPV
($)
NPV
$
A

B B
A

k Discount rate (%)


0 10% IRRB IRRA 0 16% 17% 18% 30%

NPV
$
NPV ($)
P
C

k 10% k
10% 15% 20% %
NPV NPV
$

A A

B
B n
Sigma CF t
k k t=1
0 7% 12% 15% 0 7% 12% 14%
(1 + K)t
NPV
$

X
Crossover

k
10% IRRY 12% IRRX

0 k = 12% 1 2 3 4 5 6

CFsA -15,000 4,000 4,000 4,000 4,000 4,000 4,000


Project B:
0 k = ? 1 2 6 Years
  
CFsB -14,815 5,100 5,100 5,100
Time lines:
Project A:
0 1 2 3 4 Years
k = 12%

CFsA -25,000 13,000 13,000 13,000 13,000


NPVA = ? = 17,663 Terminal value = 5,000
CF 4 = 18,000
Project B:
0 1 2 3 4 Years
k = ?

CFsB -25,000 15,247 15,247 15,247 15,247


NPVA = NPVB = 17,663 Terminal value = 0
CF 4 = 15,247
Time line:
0 k = 18% 1 2 3 Years

-3,000 1,728 1,920 1,152


Time line:
0 1 2 3 4 5 Years
k = 12%

-45,000 7,800 10,680 7,560 5,880 -1,920


Time line:
0 1 2 3 4 5 Years
k = 9%

-40,000 9,800 11,720 9,640 8,520 15,320


Time lines:
Project A:
0 k = 14% 1 2 3 4 Years

CFsA -200,000 71,104 71,104 71,104 71,104


Project B:
0 k = 10% 1 2 3 4 Years

CFsB -200,000 0 0 146,411 146,411


Time line:
0 k = 12% 1 2 10 Years
  
-50,000 6,000 6,000 6,000
Time lines:
Project A:
0 k = 12% 1 2 3 Periods

CFsA -5,000 2,000 2,500 2,250


Project B:
0 k = 14% 1 2 3 Periods

CFsB -5,000 3,000 2,600 2,900


0 1 2 3 4
| | | | |
CFsNew Tech -1,500 -315 -315 -315 -315 x x x
Time line:
0 1 2 3 Years
k = 14%

-42,000 14,280 16,200 11,400


TV = 18,120
29,520
Time line:
0 1 2 3 Years
k = 10%

-62,000 19,920 22,800 15,600


TV = 15,680
31,280
Expected increase in annual net income
ARR =
109 Initial (or average) investment .Answer (D) is correct. The profitability (excess present
value) index facilitates the comparison of investments that have different initial costs. The profitability index equals the
present value of future net cash inflows divided by the initial cash investment. The investment with the greater
profitability index will be the preferred investment. However, if investments are mutually exclusive, the net present value
method may be the better way of ranking projects. The excess present value index indicates the best return per dollar
invested but does not consider the alternative possibilities for unused funds. Thus, the smaller of the mutually exclusive
projects may have the higher index, but the incremental investment in the larger project may make it the better choice.
For example, an $8,000,000 project may be a better use of funds than a combination of a $6,000,000 project with a
higher index and the best alternative use of the remaining $2,000,000.
Answer (A) is incorrect because the investment generating cash flows the longest may not have the best return. Answer
(B) is incorrect because, given a net present value of zero (a profitability index exactly equal to one), the investor would
be indifferent to the project. Answer (C) is incorrect because the accounting rate of return is not a good measure of
profitability. It ignores the time value of money.
NPV
($)
NPV
$
A

B B
A

k Discount rate (%)


0 10% IRRB IRRA 0 16% 17% 18% 30%

NPV
$
NPV ($)
P
C

k 10% k
10% 15% 20% %

NPV NPV
$

A A

B
B n
Sigma CF t
k k t=1
0 7% 12% 15% 0 7% 12% 14%
(1 + K)t
NPV
$

X
Crossover

k
10% IRRY 12% IRRX

0 k = 12% 1 2 3 4 5 6

CFsA -15,000 4,000 4,000 4,000 4,000 4,000 4,000


Project B:
0 k = ? 1 2 6 Years
  
CFsB -14,815 5,100 5,100 5,100
Time lines:
Project A:
0 1 2 3 4 Years
k = 12%

CFsA -25,000 13,000 13,000 13,000 13,000


NPVA = ? = 17,663 Terminal value = 5,000
CF 4 = 18,000
Project B:
0 1 2 3 4 Years
k = ?

CFsB -25,000 15,247 15,247 15,247 15,247


NPVA = NPVB = 17,663 Terminal value = 0
CF 4 = 15,247
Time line:
0 1 2 3 Years
k = 18%

-3,000 1,728 1,920 1,152


Time line:
0 1 2 3 4 5 Years
k = 12%

-45,000 7,800 10,680 7,560 5,880 -1,920


Time line:
0 k = 9% 1 2 3 4 5 Years

-40,000 9,800 11,720 9,640 8,520 15,320


Time lines:
Project A:
0 k = 14% 1 2 3 4 Years

CFsA -200,000 71,104 71,104 71,104 71,104


Project B:
0 k = 10% 1 2 3 4 Years

CFsB -200,000 0 0 146,411 146,411


Time line:
0 k = 12% 1 2 10 Years
  
-50,000 6,000 6,000 6,000
Time lines:
Project A:
0 k = 12% 1 2 3 Periods

CFsA -5,000 2,000 2,500 2,250


Project B:
0 k = 14% 1 2 3 Periods

CFsB -5,000 3,000 2,600 2,900


0 1 2 3 4
| | | | |
CFsNew Tech -1,500 -315 -315 -315 -315 x x x
Time line:
0 1 2 3 Years
k = 14%

-42,000 14,280 16,200 11,400


TV = 18,120
29,520
Time line:
0 1 2 3 Years
k = 10%

-62,000 19,920 22,800 15,600


TV = 15,680
31,280
Expected increase in annual net income
ARR =
110 Initial (or average) investment .Answer (C) is correct. Investment projects may be mutually
exclusive under conditions of capital rationing (limited capital). In other words, scarcity of resources will prevent an entity
from undertaking all available profitable activities. Under the IRR method, an interest rate is computed such that the
present value of the expected future cash flows equals the cost of the investment (NPV = 0). The IRR method assumes
that the cash flows will be reinvested at the IRR. The NPV is the excess of the present value of the estimated net cash
inflows over the net cost of the investment. The cost of capital must be specified in the NPV method. An assumption of
the NPV method is that cash flows from the investment will be reinvested at the particular project's cost of capital.
Because of the difference in the assumptions regarding the reinvestment of cash flows, the two methods will
occasionally give different answers regarding the ranking of mutually exclusive projects. Moreover, the IRR method may
rank several small, short-lived projects ahead of a large project with a lower rate of return but with a longer life span.
However, the large project might return more dollars to the company because of the larger amount invested and the
longer time span over which earnings will accrue. When faced with capital rationing, an investor will want to invest in
projects that generate the most dollars in relation to the limited resources available and the size and returns from the
possible investments. Thus, the NPV method should be used because it determines the aggregate present value for
each feasible combination of projects.
Answer (A) is incorrect because the IRR is a number computed based on the characteristics of a given project. Answer
(B) is incorrect because cash flows are discounted under the IRR method. Answer (D) is incorrect because an
accelerated depreciation method will generate larger net cash inflows in the early years of a project. To equate the
present value of these cash flows with the net investment will therefore require a higher discount rate (IRR).
NPV
($)
NPV
$
A

B B
A

k Discount rate (%)


0 10% IRRB IRRA 0 16% 17% 18% 30%

NPV
$
NPV ($)
P
C

k 10% k
10% 15% 20% %

NPV NPV
$

A A

B
B n
Sigma CF t
k k t=1
0 7% 12% 15% 0 7% 12% 14%
(1 + K)t
NPV
$

X
Crossover

k
10% IRRY 12% IRRX

0 k = 12% 1 2 3 4 5 6

CFsA -15,000 4,000 4,000 4,000 4,000 4,000 4,000


Project B:
0 k = ? 1 2 6 Years
  
CFsB -14,815 5,100 5,100 5,100
Time lines:
Project A:
0 1 2 3 4 Years
k = 12%

CFsA -25,000 13,000 13,000 13,000 13,000


NPVA = ? = 17,663 Terminal value = 5,000
CF 4 = 18,000
Project B:
0 1 2 3 4 Years
k = ?

CFsB -25,000 15,247 15,247 15,247 15,247


NPVA = NPVB = 17,663 Terminal value = 0
CF 4 = 15,247
Time line:
0 1 2 3 Years
k = 18%

-3,000 1,728 1,920 1,152


Time line:
0 1 2 3 4 5 Years
k = 12%

-45,000 7,800 10,680 7,560 5,880 -1,920


Time line:
0 k = 9% 1 2 3 4 5 Years

-40,000 9,800 11,720 9,640 8,520 15,320


Time lines:
Project A:
0 k = 14% 1 2 3 4 Years

CFsA -200,000 71,104 71,104 71,104 71,104


Project B:
0 k = 10% 1 2 3 4 Years

CFsB -200,000 0 0 146,411 146,411


Time line:
0 k = 12% 1 2 10 Years
  
-50,000 6,000 6,000 6,000
Time lines:
Project A:
0 k = 12% 1 2 3 Periods

CFsA -5,000 2,000 2,500 2,250


Project B:
0 k = 14% 1 2 3 Periods

CFsB -5,000 3,000 2,600 2,900


0 1 2 3 4
| | | | |
CFsNew Tech -1,500 -315 -315 -315 -315 x x x
Time line:
0 1 2 3 Years
k = 14%

-42,000 14,280 16,200 11,400


TV = 18,120
29,520
Time line:
0 1 2 3 Years
k = 10%

-62,000 19,920 22,800 15,600


TV = 15,680
31,280
Expected increase in annual net income
ARR =
111 Initial (or average) investment .Answer (D) is correct. The two methods ordinarily yield the
same results, but differences can occur when the duration of the projects and the initial investments differ. The reason is
that the IRR method assumes cash inflows from the early years will be reinvested at the internal rate of return. The NPV
method assumes that early cash inflows are reinvested at the cost of capital.
Answer (A) is incorrect because the two methods will give the same results if the lives and required investments are the
same. Answer (B) is incorrect because if the required rate of return equals the IRR (i.e., the cost of capital is equal to the
IRR), the two methods would yield the same decision. Answer (C) is incorrect because if the required rate of return is
higher than the IRR, both methods would yield a decision not to acquire the investment.

NPV
($)
NPV
$
A

B B
A

k Discount rate (%)


0 10% IRRB IRRA 0 16% 17% 18% 30%

NPV
$
NPV ($)
P
C

k 10% k
10% 15% 20% %
NPV NPV
$

A A

B
B n
Sigma CF t
k k t=1
0 7% 12% 15% 0 7% 12% 14%
(1 + K)t
NPV
$

X
Crossover

k
10% IRRY 12% IRRX

0 k = 12% 1 2 3 4 5 6

CFsA -15,000 4,000 4,000 4,000 4,000 4,000 4,000


Project B:
0 k = ? 1 2 6 Years
  
CFsB -14,815 5,100 5,100 5,100
Time lines:
Project A:
0 1 2 3 4 Years
k = 12%

CFsA -25,000 13,000 13,000 13,000 13,000


NPVA = ? = 17,663 Terminal value = 5,000
CF 4 = 18,000
Project B:
0 1 2 3 4 Years
k = ?

CFsB -25,000 15,247 15,247 15,247 15,247


NPVA = NPVB = 17,663 Terminal value = 0
CF 4 = 15,247
Time line:
0 k = 18% 1 2 3 Years

-3,000 1,728 1,920 1,152


Time line:
0 1 2 3 4 5 Years
k = 12%

-45,000 7,800 10,680 7,560 5,880 -1,920


Time line:
0 1 2 3 4 5 Years
k = 9%

-40,000 9,800 11,720 9,640 8,520 15,320


Time lines:
Project A:
0 k = 14% 1 2 3 4 Years

CFsA -200,000 71,104 71,104 71,104 71,104


Project B:
0 k = 10% 1 2 3 4 Years

CFsB -200,000 0 0 146,411 146,411


Time line:
0 k = 12% 1 2 10 Years
  
-50,000 6,000 6,000 6,000
Time lines:
Project A:
0 k = 12% 1 2 3 Periods

CFsA -5,000 2,000 2,500 2,250


Project B:
0 k = 14% 1 2 3 Periods

CFsB -5,000 3,000 2,600 2,900


0 1 2 3 4
| | | | |
CFsNew Tech -1,500 -315 -315 -315 -315 x x x
Time line:
0 1 2 3 Years
k = 14%

-42,000 14,280 16,200 11,400


TV = 18,120
29,520
Time line:
0 1 2 3 Years
k = 10%

-62,000 19,920 22,800 15,600


TV = 15,680
31,280
Expected increase in annual net income
ARR =
112 Initial (or average) investment .Answer (A) is correct. Project A's NPV is calculated as
follows:
$1,000 x 2.2832 $2,283.20
- Original cost (1,000.00)
NPV $1,283.20
The second project's NPV is:
$1,500 x (3.7845 - 2.2832) $2,251.95
- Original cost (1,000.00)
NPV $1,251.95
Since A has a slightly higher NPV, it should be selected.
Answer (B) is incorrect because Project A has a slightly higher NPV and IRR. Answer (C) is incorrect because Project A
has a slightly higher IRR. Answer (D) is incorrect because Project A has a slightly higher NPV.
NPV
($)
NPV
$
A

B B
A

k Discount rate (%)


0 10% IRRB IRRA 0 16% 17% 18% 30%

NPV
$
NPV ($)
P
C

k 10% k
10% 15% 20% %

NPV NPV
$

A A

B
B n
Sigma CF t
k k t=1
0 7% 12% 15% 0 7% 12% 14%
(1 + K)t
NPV
$

X
Crossover

k
10% IRRY 12% IRRX

0 k = 12% 1 2 3 4 5 6

CFsA -15,000 4,000 4,000 4,000 4,000 4,000 4,000


Project B:
0 k = ? 1 2 6 Years
  
CFsB -14,815 5,100 5,100 5,100
Time lines:
Project A:
0 1 2 3 4 Years
k = 12%

CFsA -25,000 13,000 13,000 13,000 13,000


NPVA = ? = 17,663 Terminal value = 5,000
CF 4 = 18,000
Project B:
0 1 2 3 4 Years
k = ?

CFsB -25,000 15,247 15,247 15,247 15,247


NPVA = NPVB = 17,663 Terminal value = 0
CF 4 = 15,247
Time line:
0 1 2 3 Years
k = 18%

-3,000 1,728 1,920 1,152


Time line:
0 1 2 3 4 5 Years
k = 12%

-45,000 7,800 10,680 7,560 5,880 -1,920


Time line:
0 k = 9% 1 2 3 4 5 Years

-40,000 9,800 11,720 9,640 8,520 15,320


Time lines:
Project A:
0 k = 14% 1 2 3 4 Years

CFsA -200,000 71,104 71,104 71,104 71,104


Project B:
0 k = 10% 1 2 3 4 Years

CFsB -200,000 0 0 146,411 146,411


Time line:
0 k = 12% 1 2 10 Years
  
-50,000 6,000 6,000 6,000
Time lines:
Project A:
0 k = 12% 1 2 3 Periods

CFsA -5,000 2,000 2,500 2,250


Project B:
0 k = 14% 1 2 3 Periods

CFsB -5,000 3,000 2,600 2,900


0 1 2 3 4
| | | | |
CFsNew Tech -1,500 -315 -315 -315 -315 x x x
Time line:
0 1 2 3 Years
k = 14%

-42,000 14,280 16,200 11,400


TV = 18,120
29,520
Time line:
0 1 2 3 Years
k = 10%

-62,000 19,920 22,800 15,600


TV = 15,680
31,280
Expected increase in annual net income
ARR =
113 Initial (or average) investment .Answer (D) is correct. The profitability index (PI) is often
used to decide among investment alternatives when more than one is acceptable. The profitability index is the ratio of
the present value of future net cash inflows to the initial net cash investment. The PI, although a variation of the net
present value method, facilitates comparison of different-sized investments.
Answer (A) is incorrect because the accounting rate of return is a poor technique. It ignores the time value of money.
Answer (B) is incorrect because the payback method ignores the time value of money and long-term profitability.
Answer (C) is incorrect because the internal rate of return is not effective when alternative investments have different
lives.

NPV
($)
NPV
$
A

B B
A

k Discount rate (%)


0 10% IRRB IRRA 0 16% 17% 18% 30%

NPV
$
NPV ($)
P
C

k 10% k
10% 15% 20% %
NPV NPV
$

A A

B
B n
Sigma CF t
k k t=1
0 7% 12% 15% 0 7% 12% 14%
(1 + K)t
NPV
$

X
Crossover

k
10% IRRY 12% IRRX

0 k = 12% 1 2 3 4 5 6

CFsA -15,000 4,000 4,000 4,000 4,000 4,000 4,000


Project B:
0 k = ? 1 2 6 Years
  
CFsB -14,815 5,100 5,100 5,100
Time lines:
Project A:
0 1 2 3 4 Years
k = 12%

CFsA -25,000 13,000 13,000 13,000 13,000


NPVA = ? = 17,663 Terminal value = 5,000
CF 4 = 18,000
Project B:
0 1 2 3 4 Years
k = ?

CFsB -25,000 15,247 15,247 15,247 15,247


NPVA = NPVB = 17,663 Terminal value = 0
CF 4 = 15,247
Time line:
0 k = 18% 1 2 3 Years

-3,000 1,728 1,920 1,152


Time line:
0 1 2 3 4 5 Years
k = 12%

-45,000 7,800 10,680 7,560 5,880 -1,920


Time line:
0 1 2 3 4 5 Years
k = 9%

-40,000 9,800 11,720 9,640 8,520 15,320


Time lines:
Project A:
0 k = 14% 1 2 3 4 Years

CFsA -200,000 71,104 71,104 71,104 71,104


Project B:
0 k = 10% 1 2 3 4 Years

CFsB -200,000 0 0 146,411 146,411


Time line:
0 k = 12% 1 2 10 Years
  
-50,000 6,000 6,000 6,000
Time lines:
Project A:
0 k = 12% 1 2 3 Periods

CFsA -5,000 2,000 2,500 2,250


Project B:
0 k = 14% 1 2 3 Periods

CFsB -5,000 3,000 2,600 2,900


0 1 2 3 4
| | | | |
CFsNew Tech -1,500 -315 -315 -315 -315 x x x
Time line:
0 1 2 3 Years
k = 14%

-42,000 14,280 16,200 11,400


TV = 18,120
29,520
Time line:
0 1 2 3 Years
k = 10%

-62,000 19,920 22,800 15,600


TV = 15,680
31,280
Expected increase in annual net income
ARR =
114 Initial (or average) investment .Answer (A) is correct. The profitability index is the ratio of the
present value of future net cash inflows to the initial cash investment; that is, the figures are those used to calculate the
net present value (NPV), but the numbers are divided rather than subtracted. This variation of the NPV method
facilitates comparison of different-sized investments. It provides an optimal ranking in the absence of capital rationing.
Answer (B) is incorrect because the profitability index method is a discounted cash flow method. Answer (C) is incorrect
because the payback method gives no consideration to the time value of money or to returns after the payback period.
Answer (D) is incorrect because the profitability index method and the NPV method are discounted cash flow methods.
However, the profitability index method is the variant that purports to calculate a return per dollar of investment.
NPV
($)
NPV
$
A

B B
A

k Discount rate (%)


0 10% IRRB IRRA 0 16% 17% 18% 30%

NPV
$
NPV ($)
P
C

k 10% k
10% 15% 20% %

NPV NPV
$

A A

B
B n
Sigma CF t
k k t=1
0 7% 12% 15% 0 7% 12% 14%
(1 + K)t
NPV
$

X
Crossover

k
10% IRRY 12% IRRX

0 k = 12% 1 2 3 4 5 6

CFsA -15,000 4,000 4,000 4,000 4,000 4,000 4,000


Project B:
0 k = ? 1 2 6 Years
  
CFsB -14,815 5,100 5,100 5,100
Time lines:
Project A:
0 1 2 3 4 Years
k = 12%

CFsA -25,000 13,000 13,000 13,000 13,000


NPVA = ? = 17,663 Terminal value = 5,000
CF 4 = 18,000
Project B:
0 1 2 3 4 Years
k = ?

CFsB -25,000 15,247 15,247 15,247 15,247


NPVA = NPVB = 17,663 Terminal value = 0
CF 4 = 15,247
Time line:
0 1 2 3 Years
k = 18%

-3,000 1,728 1,920 1,152


Time line:
0 1 2 3 4 5 Years
k = 12%

-45,000 7,800 10,680 7,560 5,880 -1,920


Time line:
0 k = 9% 1 2 3 4 5 Years

-40,000 9,800 11,720 9,640 8,520 15,320


Time lines:
Project A:
0 k = 14% 1 2 3 4 Years

CFsA -200,000 71,104 71,104 71,104 71,104


Project B:
0 k = 10% 1 2 3 4 Years

CFsB -200,000 0 0 146,411 146,411


Time line:
0 k = 12% 1 2 10 Years
  
-50,000 6,000 6,000 6,000
Time lines:
Project A:
0 k = 12% 1 2 3 Periods

CFsA -5,000 2,000 2,500 2,250


Project B:
0 k = 14% 1 2 3 Periods

CFsB -5,000 3,000 2,600 2,900


0 1 2 3 4
| | | | |
CFsNew Tech -1,500 -315 -315 -315 -315 x x x
Time line:
0 1 2 3 Years
k = 14%

-42,000 14,280 16,200 11,400


TV = 18,120
29,520
Time line:
0 1 2 3 Years
k = 10%

-62,000 19,920 22,800 15,600


TV = 15,680
31,280
Expected increase in annual net income
ARR =
115 Initial (or average) investment .Answer (C) is correct. The profitability index is the ratio of
the present value of future net cash inflows to the initial cash investment. This variation of the net present value method
facilitates comparison of different-sized investments. Were it not for this comparison feature, the profitability index would
be no better than the net present value method. Thus, it is the comparison, or ranking, advantage that makes the
profitability index different from the other capital budgeting tools.
Answer (A) is incorrect because the net present value (NPV > 0) is a capital budgeting tool that screens investments;
i.e., the investment must meet a certain standard to be acceptable. Answer (B) is incorrect because the time-adjusted
rate of return is a capital budgeting tool that screens investments; i.e., the investment must meet a certain standard (rate
of return) to be acceptable. Answer (D) is incorrect because the accounting rate of return is a capital budgeting tool that
screens investments; i.e., the investment must meet a certain standard (rate of return) to be acceptable.

NPV
($)
NPV
$
A

B B
A

k Discount rate (%)


0 10% IRRB IRRA 0 16% 17% 18% 30%

NPV
$
NPV ($)
P
C

k 10% k
10% 15% 20% %
NPV NPV
$

A A

B
B n
Sigma CF t
k k t=1
0 7% 12% 15% 0 7% 12% 14%
(1 + K)t
NPV
$

X
Crossover

k
10% IRRY 12% IRRX

0 k = 12% 1 2 3 4 5 6

CFsA -15,000 4,000 4,000 4,000 4,000 4,000 4,000


Project B:
0 k = ? 1 2 6 Years
  
CFsB -14,815 5,100 5,100 5,100
Time lines:
Project A:
0 1 2 3 4 Years
k = 12%

CFsA -25,000 13,000 13,000 13,000 13,000


NPVA = ? = 17,663 Terminal value = 5,000
CF 4 = 18,000
Project B:
0 1 2 3 4 Years
k = ?

CFsB -25,000 15,247 15,247 15,247 15,247


NPVA = NPVB = 17,663 Terminal value = 0
CF 4 = 15,247
Time line:
0 k = 18% 1 2 3 Years

-3,000 1,728 1,920 1,152


Time line:
0 1 2 3 4 5 Years
k = 12%

-45,000 7,800 10,680 7,560 5,880 -1,920


Time line:
0 1 2 3 4 5 Years
k = 9%

-40,000 9,800 11,720 9,640 8,520 15,320


Time lines:
Project A:
0 k = 14% 1 2 3 4 Years

CFsA -200,000 71,104 71,104 71,104 71,104


Project B:
0 k = 10% 1 2 3 4 Years

CFsB -200,000 0 0 146,411 146,411


Time line:
0 k = 12% 1 2 10 Years
  
-50,000 6,000 6,000 6,000
Time lines:
Project A:
0 k = 12% 1 2 3 Periods

CFsA -5,000 2,000 2,500 2,250


Project B:
0 k = 14% 1 2 3 Periods

CFsB -5,000 3,000 2,600 2,900


0 1 2 3 4
| | | | |
CFsNew Tech -1,500 -315 -315 -315 -315 x x x
Time line:
0 1 2 3 Years
k = 14%

-42,000 14,280 16,200 11,400


TV = 18,120
29,520
Time line:
0 1 2 3 Years
k = 10%

-62,000 19,920 22,800 15,600


TV = 15,680
31,280
Expected increase in annual net income
ARR =
116 Initial (or average) investment .Answer (B) is correct. The IRR is the discount rate at which
the net present value of a project is zero. Consequently, if the IRR exceeds the cost of capital, the NPV calculated at the
cost of capital must be positive. Projects with a positive NPV are expected to be profitable and should be considered.
Other factors being equal, projects with higher IRRs should be accepted before those with lower IRRs.
Answer (A) is incorrect because IRRs should exceed the cost of capital, and projects should be accepted in the
descending order of their IRRs. Answer (C) is incorrect because IRRs should exceed the cost of capital, and projects
should be accepted in the descending order of their IRRs. Answer (D) is incorrect because IRRs should exceed the cost
of capital, and projects should be accepted in the descending order of their IRRs.
NPV
($)
NPV
$
A

B B
A

k Discount rate (%)


0 10% IRRB IRRA 0 16% 17% 18% 30%

NPV
$
NPV ($)
P
C

k 10% k
10% 15% 20% %

NPV NPV
$

A A

B
B n
Sigma CF t
k k t=1
0 7% 12% 15% 0 7% 12% 14%
(1 + K)t
NPV
$

X
Crossover

k
10% IRRY 12% IRRX

0 k = 12% 1 2 3 4 5 6

CFsA -15,000 4,000 4,000 4,000 4,000 4,000 4,000


Project B:
0 k = ? 1 2 6 Years
  
CFsB -14,815 5,100 5,100 5,100
Time lines:
Project A:
0 1 2 3 4 Years
k = 12%

CFsA -25,000 13,000 13,000 13,000 13,000


NPVA = ? = 17,663 Terminal value = 5,000
CF 4 = 18,000
Project B:
0 1 2 3 4 Years
k = ?

CFsB -25,000 15,247 15,247 15,247 15,247


NPVA = NPVB = 17,663 Terminal value = 0
CF 4 = 15,247
Time line:
0 1 2 3 Years
k = 18%

-3,000 1,728 1,920 1,152


Time line:
0 1 2 3 4 5 Years
k = 12%

-45,000 7,800 10,680 7,560 5,880 -1,920


Time line:
0 k = 9% 1 2 3 4 5 Years

-40,000 9,800 11,720 9,640 8,520 15,320


Time lines:
Project A:
0 k = 14% 1 2 3 4 Years

CFsA -200,000 71,104 71,104 71,104 71,104


Project B:
0 k = 10% 1 2 3 4 Years

CFsB -200,000 0 0 146,411 146,411


Time line:
0 k = 12% 1 2 10 Years
  
-50,000 6,000 6,000 6,000
Time lines:
Project A:
0 k = 12% 1 2 3 Periods

CFsA -5,000 2,000 2,500 2,250


Project B:
0 k = 14% 1 2 3 Periods

CFsB -5,000 3,000 2,600 2,900


0 1 2 3 4
| | | | |
CFsNew Tech -1,500 -315 -315 -315 -315 x x x
Time line:
0 1 2 3 Years
k = 14%

-42,000 14,280 16,200 11,400


TV = 18,120
29,520
Time line:
0 1 2 3 Years
k = 10%

-62,000 19,920 22,800 15,600


TV = 15,680
31,280
Expected increase in annual net income
ARR =
117 Initial (or average) investment .Answer (C) is correct. Rational investors choose projects
that yield the best return given some level of risk. If an investor desires no risk, that is, an absolutely certain rate of
return, the risk-free rate is used in calculating net present value. The risk-free rate is the return on a risk-free investment
such as government bonds. Certainty equivalent adjustments involve a technique directly drawn from utility theory. It
forces the decision maker to specify at what point the firm is indifferent to the choice between a sum of money that is
certain and the expected value of a risky sum.
Answer (A) is incorrect because a risk-adjusted discount rate does not represent an absolutely certain rate of return. A
discount rate is adjusted upward as the investment becomes riskier. Answer (B) is incorrect because the cost of capital
has nothing to do with certainty equivalence. Answer (D) is incorrect because the cost of equity capital does not equate
to a certainty equivalent rate.

NPV
($)
NPV
$
A

B B
A

k Discount rate (%)


0 10% IRRB IRRA 0 16% 17% 18% 30%

NPV
$
NPV ($)
P
C

k 10% k
10% 15% 20% %
NPV NPV
$

A A

B
B n
Sigma CF t
k k t=1
0 7% 12% 15% 0 7% 12% 14%
(1 + K)t
NPV
$

X
Crossover

k
10% IRRY 12% IRRX

0 k = 12% 1 2 3 4 5 6

CFsA -15,000 4,000 4,000 4,000 4,000 4,000 4,000


Project B:
0 k = ? 1 2 6 Years
  
CFsB -14,815 5,100 5,100 5,100
Time lines:
Project A:
0 1 2 3 4 Years
k = 12%

CFsA -25,000 13,000 13,000 13,000 13,000


NPVA = ? = 17,663 Terminal value = 5,000
CF 4 = 18,000
Project B:
0 1 2 3 4 Years
k = ?

CFsB -25,000 15,247 15,247 15,247 15,247


NPVA = NPVB = 17,663 Terminal value = 0
CF 4 = 15,247
Time line:
0 k = 18% 1 2 3 Years

-3,000 1,728 1,920 1,152


Time line:
0 1 2 3 4 5 Years
k = 12%

-45,000 7,800 10,680 7,560 5,880 -1,920


Time line:
0 1 2 3 4 5 Years
k = 9%

-40,000 9,800 11,720 9,640 8,520 15,320


Time lines:
Project A:
0 k = 14% 1 2 3 4 Years

CFsA -200,000 71,104 71,104 71,104 71,104


Project B:
0 k = 10% 1 2 3 4 Years

CFsB -200,000 0 0 146,411 146,411


Time line:
0 k = 12% 1 2 10 Years
  
-50,000 6,000 6,000 6,000
Time lines:
Project A:
0 k = 12% 1 2 3 Periods

CFsA -5,000 2,000 2,500 2,250


Project B:
0 k = 14% 1 2 3 Periods

CFsB -5,000 3,000 2,600 2,900


0 1 2 3 4
| | | | |
CFsNew Tech -1,500 -315 -315 -315 -315 x x x
Time line:
0 1 2 3 Years
k = 14%

-42,000 14,280 16,200 11,400


TV = 18,120
29,520
Time line:
0 1 2 3 Years
k = 10%

-62,000 19,920 22,800 15,600


TV = 15,680
31,280
Expected increase in annual net income
ARR =
118 Initial (or average) investment .Answer (D) is correct. Under the certainty-equivalent
method, expected cash flows are multiplied by a certainty equivalent factor and discounted at the risk-free rate. Under
the risk-adjusted discount rate method, expected cash flows are discounted at the risk-adjusted discount rate.
Answer (A) is incorrect because the certainty-equivalent method uses the risk-free rate, not the cost of capital. Answer
(B) is incorrect because the risk-adjusted discount rate discounts expected cash flows at the risk-adjusted rate. Answer
(C) is incorrect because the certainty-equivalent method uses the risk-free rate, not the cost of capital.
NPV
($)
NPV
$
A

B B
A

k Discount rate (%)


0 10% IRRB IRRA 0 16% 17% 18% 30%

NPV
$
NPV ($)
P
C

k 10% k
10% 15% 20% %

NPV NPV
$

A A

B
B n
Sigma CF t
k k t=1
0 7% 12% 15% 0 7% 12% 14%
(1 + K)t
NPV
$

X
Crossover

k
10% IRRY 12% IRRX

0 k = 12% 1 2 3 4 5 6

CFsA -15,000 4,000 4,000 4,000 4,000 4,000 4,000


Project B:
0 k = ? 1 2 6 Years
  
CFsB -14,815 5,100 5,100 5,100
Time lines:
Project A:
0 1 2 3 4 Years
k = 12%

CFsA -25,000 13,000 13,000 13,000 13,000


NPVA = ? = 17,663 Terminal value = 5,000
CF 4 = 18,000
Project B:
0 1 2 3 4 Years
k = ?

CFsB -25,000 15,247 15,247 15,247 15,247


NPVA = NPVB = 17,663 Terminal value = 0
CF 4 = 15,247
Time line:
0 1 2 3 Years
k = 18%

-3,000 1,728 1,920 1,152


Time line:
0 1 2 3 4 5 Years
k = 12%

-45,000 7,800 10,680 7,560 5,880 -1,920


Time line:
0 k = 9% 1 2 3 4 5 Years

-40,000 9,800 11,720 9,640 8,520 15,320


Time lines:
Project A:
0 k = 14% 1 2 3 4 Years

CFsA -200,000 71,104 71,104 71,104 71,104


Project B:
0 k = 10% 1 2 3 4 Years

CFsB -200,000 0 0 146,411 146,411


Time line:
0 k = 12% 1 2 10 Years
  
-50,000 6,000 6,000 6,000
Time lines:
Project A:
0 k = 12% 1 2 3 Periods

CFsA -5,000 2,000 2,500 2,250


Project B:
0 k = 14% 1 2 3 Periods

CFsB -5,000 3,000 2,600 2,900


0 1 2 3 4
| | | | |
CFsNew Tech -1,500 -315 -315 -315 -315 x x x
Time line:
0 1 2 3 Years
k = 14%

-42,000 14,280 16,200 11,400


TV = 18,120
29,520
Time line:
0 1 2 3 Years
k = 10%

-62,000 19,920 22,800 15,600


TV = 15,680
31,280
Expected increase in annual net income
ARR =
119 Initial (or average) investment .Answer (D) is correct. Under the certainty-equivalent
approach, expected cash flows should be multiplied by certainty-equivalent factors and discounted at the risk-free rate.
Answer (A) is incorrect because the risk-free rate should be used rather than the cost of capital. Answer (B) is incorrect
because the risk-free rate should be used rather than the cost of capital. Answer (C) is incorrect because the risk-free
rate should be used rather than the cost of capital.

NPV
($)
NPV
$
A

B B
A

k Discount rate (%)


0 10% IRRB IRRA 0 16% 17% 18% 30%

NPV
$
NPV ($)
P
C

k 10% k
10% 15% 20% %
NPV NPV
$

A A

B
B n
Sigma CF t
k k t=1
0 7% 12% 15% 0 7% 12% 14%
(1 + K)t
NPV
$

X
Crossover

k
10% IRRY 12% IRRX

0 k = 12% 1 2 3 4 5 6

CFsA -15,000 4,000 4,000 4,000 4,000 4,000 4,000


Project B:
0 k = ? 1 2 6 Years
  
CFsB -14,815 5,100 5,100 5,100
Time lines:
Project A:
0 1 2 3 4 Years
k = 12%

CFsA -25,000 13,000 13,000 13,000 13,000


NPVA = ? = 17,663 Terminal value = 5,000
CF 4 = 18,000
Project B:
0 1 2 3 4 Years
k = ?

CFsB -25,000 15,247 15,247 15,247 15,247


NPVA = NPVB = 17,663 Terminal value = 0
CF 4 = 15,247
Time line:
0 k = 18% 1 2 3 Years

-3,000 1,728 1,920 1,152


Time line:
0 1 2 3 4 5 Years
k = 12%

-45,000 7,800 10,680 7,560 5,880 -1,920


Time line:
0 1 2 3 4 5 Years
k = 9%

-40,000 9,800 11,720 9,640 8,520 15,320


Time lines:
Project A:
0 k = 14% 1 2 3 4 Years

CFsA -200,000 71,104 71,104 71,104 71,104


Project B:
0 k = 10% 1 2 3 4 Years

CFsB -200,000 0 0 146,411 146,411


Time line:
0 k = 12% 1 2 10 Years
  
-50,000 6,000 6,000 6,000
Time lines:
Project A:
0 k = 12% 1 2 3 Periods

CFsA -5,000 2,000 2,500 2,250


Project B:
0 k = 14% 1 2 3 Periods

CFsB -5,000 3,000 2,600 2,900


0 1 2 3 4
| | | | |
CFsNew Tech -1,500 -315 -315 -315 -315 x x x
Time line:
0 1 2 3 Years
k = 14%

-42,000 14,280 16,200 11,400


TV = 18,120
29,520
Time line:
0 1 2 3 Years
k = 10%

-62,000 19,920 22,800 15,600


TV = 15,680
31,280
Expected increase in annual net income
ARR =
120 Initial (or average) investment .Answer (C) is correct. Uncertainty can be compensated for
by adjusting the desired rate of return. If projects have relatively uncertain returns, a higher rate should be required. A
lower rate of return may be acceptable given greater certainty. The concept is that with increased risk should come
increased rewards, i.e., a higher rate of return.
Answer (A) is incorrect because preparing an analysis of probability of outcomes is not a simple method of adjustment.
Answer (B) is incorrect because accelerated depreciation should probably be used for tax purposes in every capital
project to minimize taxes in early years. Answer (D) is incorrect because uniformly increasing the estimated cash flows
and/or ignoring salvage values introduces error into the capital budgeting analysis.
NPV
($)
NPV
$
A

B B
A

k Discount rate (%)


0 10% IRRB IRRA 0 16% 17% 18% 30%

NPV
$
NPV ($)
P
C

k 10% k
10% 15% 20% %

NPV NPV
$

A A

B
B n
Sigma CF t
k k t=1
0 7% 12% 15% 0 7% 12% 14%
(1 + K)t
NPV
$

X
Crossover

k
10% IRRY 12% IRRX

0 k = 12% 1 2 3 4 5 6

CFsA -15,000 4,000 4,000 4,000 4,000 4,000 4,000


Project B:
0 k = ? 1 2 6 Years
  
CFsB -14,815 5,100 5,100 5,100
Time lines:
Project A:
0 1 2 3 4 Years
k = 12%

CFsA -25,000 13,000 13,000 13,000 13,000


NPVA = ? = 17,663 Terminal value = 5,000
CF 4 = 18,000
Project B:
0 1 2 3 4 Years
k = ?

CFsB -25,000 15,247 15,247 15,247 15,247


NPVA = NPVB = 17,663 Terminal value = 0
CF 4 = 15,247
Time line:
0 1 2 3 Years
k = 18%

-3,000 1,728 1,920 1,152


Time line:
0 1 2 3 4 5 Years
k = 12%

-45,000 7,800 10,680 7,560 5,880 -1,920


Time line:
0 k = 9% 1 2 3 4 5 Years

-40,000 9,800 11,720 9,640 8,520 15,320


Time lines:
Project A:
0 k = 14% 1 2 3 4 Years

CFsA -200,000 71,104 71,104 71,104 71,104


Project B:
0 k = 10% 1 2 3 4 Years

CFsB -200,000 0 0 146,411 146,411


Time line:
0 k = 12% 1 2 10 Years
  
-50,000 6,000 6,000 6,000
Time lines:
Project A:
0 k = 12% 1 2 3 Periods

CFsA -5,000 2,000 2,500 2,250


Project B:
0 k = 14% 1 2 3 Periods

CFsB -5,000 3,000 2,600 2,900


0 1 2 3 4
| | | | |
CFsNew Tech -1,500 -315 -315 -315 -315 x x x
Time line:
0 1 2 3 Years
k = 14%

-42,000 14,280 16,200 11,400


TV = 18,120
29,520
Time line:
0 1 2 3 Years
k = 10%

-62,000 19,920 22,800 15,600


TV = 15,680
31,280
Expected increase in annual net income
ARR =
121 Initial (or average) investment .Answer (D) is correct. Risk analysis attempts to measure the
likelihood of the variability of future returns from the proposed investment. Risk can be incorporated into capital
budgeting decisions in a number of ways, one of which is to use a hurdle rate higher than the firm's cost of capital, that
is, a risk-adjusted discount rate. This technique adjusts the interest rate used for discounting upward as an investment
becomes riskier. The expected flow from the investment must be relatively larger or the increased discount rate will
generate a negative net present value, and the proposed acquisition will be rejected.
Answer (A) is incorrect because the nature of the funding may not be a sufficient reason to use a risk-adjusted rate. The
type of funding is just one factor affecting the risk of a project. Answer (B) is incorrect because a higher hurdle will result
in rejection of more projects. Answer (C) is incorrect because a risk-adjusted high hurdle rate is used for capital
investments with greater risk.

NPV
($)
NPV
$
A

B B
A

k Discount rate (%)


0 10% IRRB IRRA 0 16% 17% 18% 30%

NPV
$
NPV ($)
P
C

k 10% k
10% 15% 20% %
NPV NPV
$

A A

B
B n
Sigma CF t
k k t=1
0 7% 12% 15% 0 7% 12% 14%
(1 + K)t
NPV
$

X
Crossover

k
10% IRRY 12% IRRX

0 k = 12% 1 2 3 4 5 6

CFsA -15,000 4,000 4,000 4,000 4,000 4,000 4,000


Project B:
0 k = ? 1 2 6 Years
  
CFsB -14,815 5,100 5,100 5,100
Time lines:
Project A:
0 1 2 3 4 Years
k = 12%

CFsA -25,000 13,000 13,000 13,000 13,000


NPVA = ? = 17,663 Terminal value = 5,000
CF 4 = 18,000
Project B:
0 1 2 3 4 Years
k = ?

CFsB -25,000 15,247 15,247 15,247 15,247


NPVA = NPVB = 17,663 Terminal value = 0
CF 4 = 15,247
Time line:
0 k = 18% 1 2 3 Years

-3,000 1,728 1,920 1,152


Time line:
0 1 2 3 4 5 Years
k = 12%

-45,000 7,800 10,680 7,560 5,880 -1,920


Time line:
0 1 2 3 4 5 Years
k = 9%

-40,000 9,800 11,720 9,640 8,520 15,320


Time lines:
Project A:
0 k = 14% 1 2 3 4 Years

CFsA -200,000 71,104 71,104 71,104 71,104


Project B:
0 k = 10% 1 2 3 4 Years

CFsB -200,000 0 0 146,411 146,411


Time line:
0 k = 12% 1 2 10 Years
  
-50,000 6,000 6,000 6,000
Time lines:
Project A:
0 k = 12% 1 2 3 Periods

CFsA -5,000 2,000 2,500 2,250


Project B:
0 k = 14% 1 2 3 Periods

CFsB -5,000 3,000 2,600 2,900


0 1 2 3 4
| | | | |
CFsNew Tech -1,500 -315 -315 -315 -315 x x x
Time line:
0 1 2 3 Years
k = 14%

-42,000 14,280 16,200 11,400


TV = 18,120
29,520
Time line:
0 1 2 3 Years
k = 10%

-62,000 19,920 22,800 15,600


TV = 15,680
31,280
Expected increase in annual net income
ARR =
122 Initial (or average) investment .Answer (D) is correct. Risk-adjusted discount rates can be
used to evaluate capital investment options. If risks differ among various elements of the cash flows, then different
discount rates can be used for different flows.
Answer (A) is incorrect because the payback period ignores both the varying risk and the time value of money. Answer
(B) is incorrect because using the cost of capital as the discount rate does not make any adjustment for the risk
differentials among the various cash flows. Answer (C) is incorrect because risk has to be incorporated into the
company's hurdle rate to use the internal rate of return method with risk differentials.
NPV
($)
NPV
$
A

B B
A

k Discount rate (%)


0 10% IRRB IRRA 0 16% 17% 18% 30%

NPV
$
NPV ($)
P
C

k 10% k
10% 15% 20% %

NPV NPV
$

A A

B
B n
Sigma CF t
k k t=1
0 7% 12% 15% 0 7% 12% 14%
(1 + K)t
NPV
$

X
Crossover

k
10% IRRY 12% IRRX

0 k = 12% 1 2 3 4 5 6

CFsA -15,000 4,000 4,000 4,000 4,000 4,000 4,000


Project B:
0 k = ? 1 2 6 Years
  
CFsB -14,815 5,100 5,100 5,100
Time lines:
Project A:
0 1 2 3 4 Years
k = 12%

CFsA -25,000 13,000 13,000 13,000 13,000


NPVA = ? = 17,663 Terminal value = 5,000
CF 4 = 18,000
Project B:
0 1 2 3 4 Years
k = ?

CFsB -25,000 15,247 15,247 15,247 15,247


NPVA = NPVB = 17,663 Terminal value = 0
CF 4 = 15,247
Time line:
0 1 2 3 Years
k = 18%

-3,000 1,728 1,920 1,152


Time line:
0 1 2 3 4 5 Years
k = 12%

-45,000 7,800 10,680 7,560 5,880 -1,920


Time line:
0 k = 9% 1 2 3 4 5 Years

-40,000 9,800 11,720 9,640 8,520 15,320


Time lines:
Project A:
0 k = 14% 1 2 3 4 Years

CFsA -200,000 71,104 71,104 71,104 71,104


Project B:
0 k = 10% 1 2 3 4 Years

CFsB -200,000 0 0 146,411 146,411


Time line:
0 k = 12% 1 2 10 Years
  
-50,000 6,000 6,000 6,000
Time lines:
Project A:
0 k = 12% 1 2 3 Periods

CFsA -5,000 2,000 2,500 2,250


Project B:
0 k = 14% 1 2 3 Periods

CFsB -5,000 3,000 2,600 2,900


0 1 2 3 4
| | | | |
CFsNew Tech -1,500 -315 -315 -315 -315 x x x
Time line:
0 1 2 3 Years
k = 14%

-42,000 14,280 16,200 11,400


TV = 18,120
29,520
Time line:
0 1 2 3 Years
k = 10%

-62,000 19,920 22,800 15,600


TV = 15,680
31,280
Expected increase in annual net income
ARR =
123 Initial (or average) investment . Risk adjustment Answer: b Diff: E
NPV
($)
NPV
$
A

B B
A

k Discount rate (%)


0 10% IRRB IRRA 0 16% 17% 18% 30%

NPV
$
NPV ($)
P
C

k 10% k
10% 15% 20% %

NPV NPV
$

A A

B
B n
Sigma CF t
k k t=1
0 7% 12% 15% 0 7% 12% 14%
(1 + K)t
NPV
$

X
Crossover

k
10% IRRY 12% IRRX

0 k = 12% 1 2 3 4 5 6

CFsA -15,000 4,000 4,000 4,000 4,000 4,000 4,000


Project B:
0 k = ? 1 2 6 Years
  
CFsB -14,815 5,100 5,100 5,100
Time lines:
Project A:
0 1 2 3 4 Years
k = 12%

CFsA -25,000 13,000 13,000 13,000 13,000


NPVA = ? = 17,663 Terminal value = 5,000
CF 4 = 18,000
Project B:
0 1 2 3 4 Years
k = ?

CFsB -25,000 15,247 15,247 15,247 15,247


NPVA = NPVB = 17,663 Terminal value = 0
CF 4 = 15,247
Time line:
0 1 2 3 Years
k = 18%

-3,000 1,728 1,920 1,152


Time line:
0 1 2 3 4 5 Years
k = 12%

-45,000 7,800 10,680 7,560 5,880 -1,920


Time line:
0 k = 9% 1 2 3 4 5 Years

-40,000 9,800 11,720 9,640 8,520 15,320


Time lines:
Project A:
0 k = 14% 1 2 3 4 Years

CFsA -200,000 71,104 71,104 71,104 71,104


Project B:
0 k = 10% 1 2 3 4 Years

CFsB -200,000 0 0 146,411 146,411


Time line:
0 k = 12% 1 2 10 Years
  
-50,000 6,000 6,000 6,000
Time lines:
Project A:
0 k = 12% 1 2 3 Periods

CFsA -5,000 2,000 2,500 2,250


Project B:
0 k = 14% 1 2 3 Periods

CFsB -5,000 3,000 2,600 2,900


0 1 2 3 4
| | | | |
CFsNew Tech -1,500 -315 -315 -315 -315 x x x
Time line:
0 1 2 3 Years
k = 14%

-42,000 14,280 16,200 11,400


TV = 18,120
29,520
Time line:
0 1 2 3 Years
k = 10%

-62,000 19,920 22,800 15,600


TV = 15,680
31,280
Expected increase in annual net income
ARR =
124 Initial (or average) investment . Risk and project selection Answer: b Diff: E
NPV
($)
NPV
$
A

B B
A

k Discount rate (%)


0 10% IRRB IRRA 0 16% 17% 18% 30%

NPV
$
NPV ($)
P
C

k 10% k
10% 15% 20% %
NPV NPV
$

A A

B
B n
Sigma CF t
k k t=1
0 7% 12% 15% 0 7% 12% 14%
(1 + K)t
NPV
$

X
Crossover

k
10% IRRY 12% IRRX

0 k = 12% 1 2 3 4 5 6

CFsA -15,000 4,000 4,000 4,000 4,000 4,000 4,000


Project B:
0 k = ? 1 2 6 Years
  
CFsB -14,815 5,100 5,100 5,100
Time lines:
Project A:
0 1 2 3 4 Years
k = 12%

CFsA -25,000 13,000 13,000 13,000 13,000


NPVA = ? = 17,663 Terminal value = 5,000
CF 4 = 18,000
Project B:
0 1 2 3 4 Years
k = ?

CFsB -25,000 15,247 15,247 15,247 15,247


NPVA = NPVB = 17,663 Terminal value = 0
CF 4 = 15,247
Time line:
0 k = 18% 1 2 3 Years

-3,000 1,728 1,920 1,152


Time line:
0 1 2 3 4 5 Years
k = 12%

-45,000 7,800 10,680 7,560 5,880 -1,920


Time line:
0 1 2 3 4 5 Years
k = 9%

-40,000 9,800 11,720 9,640 8,520 15,320


Time lines:
Project A:
0 k = 14% 1 2 3 4 Years

CFsA -200,000 71,104 71,104 71,104 71,104


Project B:
0 k = 10% 1 2 3 4 Years

CFsB -200,000 0 0 146,411 146,411


Time line:
0 k = 12% 1 2 10 Years
  
-50,000 6,000 6,000 6,000
Time lines:
Project A:
0 k = 12% 1 2 3 Periods

CFsA -5,000 2,000 2,500 2,250


Project B:
0 k = 14% 1 2 3 Periods

CFsB -5,000 3,000 2,600 2,900


0 1 2 3 4
| | | | |
CFsNew Tech -1,500 -315 -315 -315 -315 x x x
Time line:
0 1 2 3 Years
k = 14%

-42,000 14,280 16,200 11,400


TV = 18,120
29,520
Time line:
0 1 2 3 Years
k = 10%

-62,000 19,920 22,800 15,600


TV = 15,680
31,280
Expected increase in annual net income
ARR =
125 Initial (or average) investment . Accepting risky projects Answer: e Diff: E
NPV
($)
NPV
$
A

B B
A

k Discount rate (%)


0 10% IRRB IRRA 0 16% 17% 18% 30%

NPV
$
NPV ($)
P
C

k 10% k
10% 15% 20% %

NPV NPV
$

A A

B
B n
Sigma CF t
k k t=1
0 7% 12% 15% 0 7% 12% 14%
(1 + K)t
NPV
$

X
Crossover

k
10% IRRY 12% IRRX

0 k = 12% 1 2 3 4 5 6

CFsA -15,000 4,000 4,000 4,000 4,000 4,000 4,000


Project B:
0 k = ? 1 2 6 Years
  
CFsB -14,815 5,100 5,100 5,100
Time lines:
Project A:
0 1 2 3 4 Years
k = 12%

CFsA -25,000 13,000 13,000 13,000 13,000


NPVA = ? = 17,663 Terminal value = 5,000
CF 4 = 18,000
Project B:
0 1 2 3 4 Years
k = ?

CFsB -25,000 15,247 15,247 15,247 15,247


NPVA = NPVB = 17,663 Terminal value = 0
CF 4 = 15,247
Time line:
0 1 2 3 Years
k = 18%

-3,000 1,728 1,920 1,152


Time line:
0 1 2 3 4 5 Years
k = 12%

-45,000 7,800 10,680 7,560 5,880 -1,920


Time line:
0 k = 9% 1 2 3 4 5 Years

-40,000 9,800 11,720 9,640 8,520 15,320


Time lines:
Project A:
0 k = 14% 1 2 3 4 Years

CFsA -200,000 71,104 71,104 71,104 71,104


Project B:
0 k = 10% 1 2 3 4 Years

CFsB -200,000 0 0 146,411 146,411


Time line:
0 k = 12% 1 2 10 Years
  
-50,000 6,000 6,000 6,000
Time lines:
Project A:
0 k = 12% 1 2 3 Periods

CFsA -5,000 2,000 2,500 2,250


Project B:
0 k = 14% 1 2 3 Periods

CFsB -5,000 3,000 2,600 2,900


0 1 2 3 4
| | | | |
CFsNew Tech -1,500 -315 -315 -315 -315 x x x
Time line:
0 1 2 3 Years
k = 14%

-42,000 14,280 16,200 11,400


TV = 18,120
29,520
Time line:
0 1 2 3 Years
k = 10%

-62,000 19,920 22,800 15,600


TV = 15,680
31,280
Expected increase in annual net income
ARR =
126 Initial (or average) investment . Risk and project selection

Answer: c Diff: E N

You have to find out what the required rate of return on each project is. Projects that are of high risk must have a
higher required rate of return than projects that are of low risk. The following table shows the required return for
each project on the basis of its risk level.

Required return for


Project Expected Return Risk the risk level
A 15% High 12%
B 12 Average 10%
C 11 High 12%
D 9 Low 8%
E 6 Low 8%

The company will accept all projects whose expected return exceeds its required return. Therefore, it will accept
Projects A, B, and D.
NPV
($)
NPV
$
A

B B
A

k Discount rate (%)


0 10% IRRB IRRA 0 16% 17% 18% 30%

NPV
$
NPV ($)
P
C

k 10% k
10% 15% 20% %

NPV NPV
$

A A

B
B n
Sigma CF t
k k t=1
0 7% 12% 15% 0 7% 12% 14%
(1 + K)t
NPV
$

X
Crossover

k
10% IRRY 12% IRRX

0 k = 12% 1 2 3 4 5 6

CFsA -15,000 4,000 4,000 4,000 4,000 4,000 4,000


Project B:
0 k = ? 1 2 6 Years
  
CFsB -14,815 5,100 5,100 5,100
Time lines:
Project A:
0 1 2 3 4 Years
k = 12%

CFsA -25,000 13,000 13,000 13,000 13,000


NPVA = ? = 17,663 Terminal value = 5,000
CF 4 = 18,000
Project B:
0 1 2 3 4 Years
k = ?

CFsB -25,000 15,247 15,247 15,247 15,247


NPVA = NPVB = 17,663 Terminal value = 0
CF 4 = 15,247
Time line:
0 1 2 3 Years
k = 18%

-3,000 1,728 1,920 1,152


Time line:
0 1 2 3 4 5 Years
k = 12%

-45,000 7,800 10,680 7,560 5,880 -1,920


Time line:
0 k = 9% 1 2 3 4 5 Years

-40,000 9,800 11,720 9,640 8,520 15,320


Time lines:
Project A:
0 k = 14% 1 2 3 4 Years

CFsA -200,000 71,104 71,104 71,104 71,104


Project B:
0 k = 10% 1 2 3 4 Years

CFsB -200,000 0 0 146,411 146,411


Time line:
0 k = 12% 1 2 10 Years
  
-50,000 6,000 6,000 6,000
Time lines:
Project A:
0 k = 12% 1 2 3 Periods

CFsA -5,000 2,000 2,500 2,250


Project B:
0 k = 14% 1 2 3 Periods

CFsB -5,000 3,000 2,600 2,900


0 1 2 3 4
| | | | |
CFsNew Tech -1,500 -315 -315 -315 -315 x x x
Time line:
0 1 2 3 Years
k = 14%

-42,000 14,280 16,200 11,400


TV = 18,120
29,520
Time line:
0 1 2 3 Years
k = 10%

-62,000 19,920 22,800 15,600


TV = 15,680
31,280
Expected increase in annual net income
ARR =
127 Initial (or average) investment .Answer (D) is correct. Risk analysis attempts to measure the
likelihood of the variability of future returns from the proposed investment. Risk can be incorporated into capital
budgeting decisions in a number of ways, one of which is to use a hurdle rate higher than the firm's cost of capital, that
is, a risk-adjusted discount rate. This technique adjusts the interest rate used for discounting upward as an investment
becomes riskier. The expected flow from the investment must be relatively larger, or the increased discount rate will
generate a negative net present value, and the proposed acquisition will be rejected. Accordingly, the IRR (the rate at
which the NPV is zero) for a rejected investment may exceed the cost of capital when the risk-adjusted rate is higher
than the IRR. Conversely, the IRR for an accepted investment may be less than the cost of capital when the risk-
adjusted rate is less than the IRR. In this case, the investment presumably has very little risk. Furthermore, risk-adjusted
rates may also reflect the differing degrees of risk, not only among investments, but by the same investments
undertaken by different organizational subunits.
Answer (A) is incorrect because discount rates may vary with the project or with the subunit of the organization. Answer
(B) is incorrect because the company may accept some projects with IRRs less than the cost of capital, or reject some
project with IRRs greater than the cost of capital. Answer (C) is incorrect because the company may accept some
projects with IRRs less than the cost of capital, or reject some project with IRRs greater than the cost of capital.

NPV
($)
NPV
$
A

B B
A

k Discount rate (%)


0 10% IRRB IRRA 0 16% 17% 18% 30%

NPV
$
NPV ($)
P
C

k 10% k
10% 15% 20% %
NPV NPV
$

A A

B
B n
Sigma CF t
k k t=1
0 7% 12% 15% 0 7% 12% 14%
(1 + K)t
NPV
$

X
Crossover

k
10% IRRY 12% IRRX

0 k = 12% 1 2 3 4 5 6

CFsA -15,000 4,000 4,000 4,000 4,000 4,000 4,000


Project B:
0 k = ? 1 2 6 Years
  
CFsB -14,815 5,100 5,100 5,100
Time lines:
Project A:
0 1 2 3 4 Years
k = 12%

CFsA -25,000 13,000 13,000 13,000 13,000


NPVA = ? = 17,663 Terminal value = 5,000
CF 4 = 18,000
Project B:
0 1 2 3 4 Years
k = ?

CFsB -25,000 15,247 15,247 15,247 15,247


NPVA = NPVB = 17,663 Terminal value = 0
CF 4 = 15,247
Time line:
0 k = 18% 1 2 3 Years

-3,000 1,728 1,920 1,152


Time line:
0 1 2 3 4 5 Years
k = 12%

-45,000 7,800 10,680 7,560 5,880 -1,920


Time line:
0 1 2 3 4 5 Years
k = 9%

-40,000 9,800 11,720 9,640 8,520 15,320


Time lines:
Project A:
0 k = 14% 1 2 3 4 Years

CFsA -200,000 71,104 71,104 71,104 71,104


Project B:
0 k = 10% 1 2 3 4 Years

CFsB -200,000 0 0 146,411 146,411


Time line:
0 k = 12% 1 2 10 Years
  
-50,000 6,000 6,000 6,000
Time lines:
Project A:
0 k = 12% 1 2 3 Periods

CFsA -5,000 2,000 2,500 2,250


Project B:
0 k = 14% 1 2 3 Periods

CFsB -5,000 3,000 2,600 2,900


0 1 2 3 4
| | | | |
CFsNew Tech -1,500 -315 -315 -315 -315 x x x
Time line:
0 1 2 3 Years
k = 14%

-42,000 14,280 16,200 11,400


TV = 18,120
29,520
Time line:
0 1 2 3 Years
k = 10%

-62,000 19,920 22,800 15,600


TV = 15,680
31,280
Expected increase in annual net income
ARR =
128 Initial (or average) investment .By Kemp not making the risk adjustment, it is true that the
company will accept more projects in the computer division, and fewer projects in the restaurant division. However, this
will make the company riskier overall, raising its cost of equity. Investors will discount their cash flows at a higher rate,
and the company’s value will fall. In addition, some of the computer projects might not exceed the appropriate risk-
adjusted hurdle rate, and will actually be negative NPV projects, further destroying value. Therefore, statement a is
false. Because fewer of the restaurant projects will be accepted, the restaurant division will become a smaller part of the
overall company. Therefore, statement b is false. As explained above, statement c is true.
NPV
($)
NPV
$
A

B B
A

k Discount rate (%)


0 10% IRRB IRRA 0 16% 17% 18% 30%

NPV
$
NPV ($)
P
C

k 10% k
10% 15% 20% %

NPV NPV
$

A A

B
B n
Sigma CF t
k k t=1
0 7% 12% 15% 0 7% 12% 14%
(1 + K)t
NPV
$

X
Crossover

k
10% IRRY 12% IRRX

0 k = 12% 1 2 3 4 5 6

CFsA -15,000 4,000 4,000 4,000 4,000 4,000 4,000


Project B:
0 k = ? 1 2 6 Years
  
CFsB -14,815 5,100 5,100 5,100
Time lines:
Project A:
0 1 2 3 4 Years
k = 12%

CFsA -25,000 13,000 13,000 13,000 13,000


NPVA = ? = 17,663 Terminal value = 5,000
CF 4 = 18,000
Project B:
0 1 2 3 4 Years
k = ?

CFsB -25,000 15,247 15,247 15,247 15,247


NPVA = NPVB = 17,663 Terminal value = 0
CF 4 = 15,247
Time line:
0 1 2 3 Years
k = 18%

-3,000 1,728 1,920 1,152


Time line:
0 1 2 3 4 5 Years
k = 12%

-45,000 7,800 10,680 7,560 5,880 -1,920


Time line:
0 k = 9% 1 2 3 4 5 Years

-40,000 9,800 11,720 9,640 8,520 15,320


Time lines:
Project A:
0 k = 14% 1 2 3 4 Years

CFsA -200,000 71,104 71,104 71,104 71,104


Project B:
0 k = 10% 1 2 3 4 Years

CFsB -200,000 0 0 146,411 146,411


Time line:
0 k = 12% 1 2 10 Years
  
-50,000 6,000 6,000 6,000
Time lines:
Project A:
0 k = 12% 1 2 3 Periods

CFsA -5,000 2,000 2,500 2,250


Project B:
0 k = 14% 1 2 3 Periods

CFsB -5,000 3,000 2,600 2,900


0 1 2 3 4
| | | | |
CFsNew Tech -1,500 -315 -315 -315 -315 x x x
Time line:
0 1 2 3 Years
k = 14%

-42,000 14,280 16,200 11,400


TV = 18,120
29,520
Time line:
0 1 2 3 Years
k = 10%

-62,000 19,920 22,800 15,600


TV = 15,680
31,280
Expected increase in annual net income
ARR =
129 Initial (or average) investment .By not risk adjusting the cost of capital, the firm will tend to
reject low-risk projects since their returns will be lower than the average cost of capital, and it will take on high-risk
projects since their returns will be higher than the average cost of capital.

NPV
($)
NPV
$
A

B B
A

k Discount rate (%)


0 10% IRRB IRRA 0 16% 17% 18% 30%

NPV
$
NPV ($)
P
C

k 10% k
10% 15% 20% %

NPV NPV
$

A A

B
B n
Sigma CF t
k k t=1
0 7% 12% 15% 0 7% 12% 14%
(1 + K)t
NPV
$

X
Crossover

k
10% IRRY 12% IRRX

0 k = 12% 1 2 3 4 5 6

CFsA -15,000 4,000 4,000 4,000 4,000 4,000 4,000


Project B:
0 k = ? 1 2 6 Years
  
CFsB -14,815 5,100 5,100 5,100
Time lines:
Project A:
0 1 2 3 4 Years
k = 12%

CFsA -25,000 13,000 13,000 13,000 13,000


NPVA = ? = 17,663 Terminal value = 5,000
CF 4 = 18,000
Project B:
0 1 2 3 4 Years
k = ?

CFsB -25,000 15,247 15,247 15,247 15,247


NPVA = NPVB = 17,663 Terminal value = 0
CF 4 = 15,247
Time line:
0 1 2 3 Years
k = 18%

-3,000 1,728 1,920 1,152


Time line:
0 1 2 3 4 5 Years
k = 12%

-45,000 7,800 10,680 7,560 5,880 -1,920


Time line:
0 k = 9% 1 2 3 4 5 Years

-40,000 9,800 11,720 9,640 8,520 15,320


Time lines:
Project A:
0 k = 14% 1 2 3 4 Years

CFsA -200,000 71,104 71,104 71,104 71,104


Project B:
0 k = 10% 1 2 3 4 Years

CFsB -200,000 0 0 146,411 146,411


Time line:
0 k = 12% 1 2 10 Years
  
-50,000 6,000 6,000 6,000
Time lines:
Project A:
0 k = 12% 1 2 3 Periods

CFsA -5,000 2,000 2,500 2,250


Project B:
0 k = 14% 1 2 3 Periods

CFsB -5,000 3,000 2,600 2,900


0 1 2 3 4
| | | | |
CFsNew Tech -1,500 -315 -315 -315 -315 x x x
Time line:
0 1 2 3 Years
k = 14%

-42,000 14,280 16,200 11,400


TV = 18,120
29,520
Time line:
0 1 2 3 Years
k = 10%

-62,000 19,920 22,800 15,600


TV = 15,680
31,280
Expected increase in annual net income
ARR =
130 Initial (or average) investment . Risk adjustment Answer: a Diff: M
kA = 13% - 3% = 10%.
If the cash flows are cost only outflows, and the analyst wants to correctly
reflect their risk, the discount rate should be adjusted downward (in this
case by subtracting 3 percentage points) to make the discounted flows
comparatively larger.
NPV
($)
NPV
$
A

B B
A

k Discount rate (%)


0 10% IRRB IRRA 0 16% 17% 18% 30%

NPV
$
NPV ($)
P
C

k 10% k
10% 15% 20% %

NPV NPV
$

A A

B
B n
Sigma CF t
k k t=1
0 7% 12% 15% 0 7% 12% 14%
(1 + K)t
NPV
$

X
Crossover

k
10% IRRY 12% IRRX

0 k = 12% 1 2 3 4 5 6

CFsA -15,000 4,000 4,000 4,000 4,000 4,000 4,000


Project B:
0 k = ? 1 2 6 Years
  
CFsB -14,815 5,100 5,100 5,100
Time lines:
Project A:
0 1 2 3 4 Years
k = 12%

CFsA -25,000 13,000 13,000 13,000 13,000


NPVA = ? = 17,663 Terminal value = 5,000
CF 4 = 18,000
Project B:
0 1 2 3 4 Years
k = ?

CFsB -25,000 15,247 15,247 15,247 15,247


NPVA = NPVB = 17,663 Terminal value = 0
CF 4 = 15,247
Time line:
0 1 2 3 Years
k = 18%

-3,000 1,728 1,920 1,152


Time line:
0 1 2 3 4 5 Years
k = 12%

-45,000 7,800 10,680 7,560 5,880 -1,920


Time line:
0 k = 9% 1 2 3 4 5 Years

-40,000 9,800 11,720 9,640 8,520 15,320


Time lines:
Project A:
0 k = 14% 1 2 3 4 Years

CFsA -200,000 71,104 71,104 71,104 71,104


Project B:
0 k = 10% 1 2 3 4 Years

CFsB -200,000 0 0 146,411 146,411


Time line:
0 k = 12% 1 2 10 Years
  
-50,000 6,000 6,000 6,000
Time lines:
Project A:
0 k = 12% 1 2 3 Periods

CFsA -5,000 2,000 2,500 2,250


Project B:
0 k = 14% 1 2 3 Periods

CFsB -5,000 3,000 2,600 2,900


0 1 2 3 4
| | | | |
CFsNew Tech -1,500 -315 -315 -315 -315 x x x
Time line:
0 1 2 3 Years
k = 14%

-42,000 14,280 16,200 11,400


TV = 18,120
29,520
Time line:
0 1 2 3 Years
k = 10%

-62,000 19,920 22,800 15,600


TV = 15,680
31,280
Expected increase in annual net income
ARR =
131 Initial (or average) investment . Risk analysis Answer: e Diff: E
NPV
($)
NPV
$
A

B B
A

k Discount rate (%)


0 10% IRRB IRRA 0 16% 17% 18% 30%

NPV
$
NPV ($)
P
C

k 10% k
10% 15% 20% %

NPV NPV
$

A A

B
B n
Sigma CF t
k k t=1
0 7% 12% 15% 0 7% 12% 14%
(1 + K)t
NPV
$

X
Crossover

k
10% IRRY 12% IRRX

0 k = 12% 1 2 3 4 5 6

CFsA -15,000 4,000 4,000 4,000 4,000 4,000 4,000


Project B:
0 k = ? 1 2 6 Years
  
CFsB -14,815 5,100 5,100 5,100
Time lines:
Project A:
0 1 2 3 4 Years
k = 12%

CFsA -25,000 13,000 13,000 13,000 13,000


NPVA = ? = 17,663 Terminal value = 5,000
CF 4 = 18,000
Project B:
0 1 2 3 4 Years
k = ?

CFsB -25,000 15,247 15,247 15,247 15,247


NPVA = NPVB = 17,663 Terminal value = 0
CF 4 = 15,247
Time line:
0 1 2 3 Years
k = 18%

-3,000 1,728 1,920 1,152


Time line:
0 1 2 3 4 5 Years
k = 12%

-45,000 7,800 10,680 7,560 5,880 -1,920


Time line:
0 k = 9% 1 2 3 4 5 Years

-40,000 9,800 11,720 9,640 8,520 15,320


Time lines:
Project A:
0 k = 14% 1 2 3 4 Years

CFsA -200,000 71,104 71,104 71,104 71,104


Project B:
0 k = 10% 1 2 3 4 Years

CFsB -200,000 0 0 146,411 146,411


Time line:
0 k = 12% 1 2 10 Years
  
-50,000 6,000 6,000 6,000
Time lines:
Project A:
0 k = 12% 1 2 3 Periods

CFsA -5,000 2,000 2,500 2,250


Project B:
0 k = 14% 1 2 3 Periods

CFsB -5,000 3,000 2,600 2,900


0 1 2 3 4
| | | | |
CFsNew Tech -1,500 -315 -315 -315 -315 x x x
Time line:
0 1 2 3 Years
k = 14%

-42,000 14,280 16,200 11,400


TV = 18,120
29,520
Time line:
0 1 2 3 Years
k = 10%

-62,000 19,920 22,800 15,600


TV = 15,680
31,280
Expected increase in annual net income
ARR =
132 Initial (or average) investment . Methods of analysis Answer: a Diff: M
NPV
($)
NPV
$
A

B B
A

k Discount rate (%)


0 10% IRRB IRRA 0 16% 17% 18% 30%

NPV
$
NPV ($)
P
C

k 10% k
10% 15% 20% %
NPV NPV
$

A A

B
B n
Sigma CF t
k k t=1
0 7% 12% 15% 0 7% 12% 14%
(1 + K)t
NPV
$

X
Crossover

k
10% IRRY 12% IRRX

0 k = 12% 1 2 3 4 5 6

CFsA -15,000 4,000 4,000 4,000 4,000 4,000 4,000


Project B:
0 k = ? 1 2 6 Years
  
CFsB -14,815 5,100 5,100 5,100
Time lines:
Project A:
0 1 2 3 4 Years
k = 12%

CFsA -25,000 13,000 13,000 13,000 13,000


NPVA = ? = 17,663 Terminal value = 5,000
CF 4 = 18,000
Project B:
0 1 2 3 4 Years
k = ?

CFsB -25,000 15,247 15,247 15,247 15,247


NPVA = NPVB = 17,663 Terminal value = 0
CF 4 = 15,247
Time line:
0 k = 18% 1 2 3 Years

-3,000 1,728 1,920 1,152


Time line:
0 1 2 3 4 5 Years
k = 12%

-45,000 7,800 10,680 7,560 5,880 -1,920


Time line:
0 1 2 3 4 5 Years
k = 9%

-40,000 9,800 11,720 9,640 8,520 15,320


Time lines:
Project A:
0 k = 14% 1 2 3 4 Years

CFsA -200,000 71,104 71,104 71,104 71,104


Project B:
0 k = 10% 1 2 3 4 Years

CFsB -200,000 0 0 146,411 146,411


Time line:
0 k = 12% 1 2 10 Years
  
-50,000 6,000 6,000 6,000
Time lines:
Project A:
0 k = 12% 1 2 3 Periods

CFsA -5,000 2,000 2,500 2,250


Project B:
0 k = 14% 1 2 3 Periods

CFsB -5,000 3,000 2,600 2,900


0 1 2 3 4
| | | | |
CFsNew Tech -1,500 -315 -315 -315 -315 x x x
Time line:
0 1 2 3 Years
k = 14%

-42,000 14,280 16,200 11,400


TV = 18,120
29,520
Time line:
0 1 2 3 Years
k = 10%

-62,000 19,920 22,800 15,600


TV = 15,680
31,280
Expected increase in annual net income
ARR =
133 Initial (or average) investment .REQUIRED: he technique used to evaluate cash flows from
the purchase of a machine.
DISCUSSION: (A) Simulation is a technique used to describe the behavior of a real-world system over time. This
technique usually employs a computer program to perform the simulation computations. Sensitivity analysis examines
how outcomes change as the model parameters change.
Answer (B) is incorrect because linear programming is a mathematical technique for optimizing a given objective
function subject to certain constraints. Answer (C) is incorrect because correlation analysis is a statistical procedure for
studying the relation between variables. Answer (D) is incorrect because differential analysis is used for decision
making that compares differences in costs (revenues) of two or more options.
NPV
($)
NPV
$
A

B B
A

k Discount rate (%)


0 10% IRRB IRRA 0 16% 17% 18% 30%

NPV
$
NPV ($)
P
C

k 10% k
10% 15% 20% %

NPV NPV
$

A A

B
B n
Sigma CF t
k k t=1
0 7% 12% 15% 0 7% 12% 14%
(1 + K)t
NPV
$

X
Crossover

k
10% IRRY 12% IRRX

0 k = 12% 1 2 3 4 5 6

CFsA -15,000 4,000 4,000 4,000 4,000 4,000 4,000


Project B:
0 k = ? 1 2 6 Years
  
CFsB -14,815 5,100 5,100 5,100
Time lines:
Project A:
0 1 2 3 4 Years
k = 12%

CFsA -25,000 13,000 13,000 13,000 13,000


NPVA = ? = 17,663 Terminal value = 5,000
CF 4 = 18,000
Project B:
0 1 2 3 4 Years
k = ?

CFsB -25,000 15,247 15,247 15,247 15,247


NPVA = NPVB = 17,663 Terminal value = 0
CF 4 = 15,247
Time line:
0 1 2 3 Years
k = 18%

-3,000 1,728 1,920 1,152


Time line:
0 1 2 3 4 5 Years
k = 12%

-45,000 7,800 10,680 7,560 5,880 -1,920


Time line:
0 k = 9% 1 2 3 4 5 Years

-40,000 9,800 11,720 9,640 8,520 15,320


Time lines:
Project A:
0 k = 14% 1 2 3 4 Years

CFsA -200,000 71,104 71,104 71,104 71,104


Project B:
0 k = 10% 1 2 3 4 Years

CFsB -200,000 0 0 146,411 146,411


Time line:
0 k = 12% 1 2 10 Years
  
-50,000 6,000 6,000 6,000
Time lines:
Project A:
0 k = 12% 1 2 3 Periods

CFsA -5,000 2,000 2,500 2,250


Project B:
0 k = 14% 1 2 3 Periods

CFsB -5,000 3,000 2,600 2,900


0 1 2 3 4
| | | | |
CFsNew Tech -1,500 -315 -315 -315 -315 x x x
Time line:
0 1 2 3 Years
k = 14%

-42,000 14,280 16,200 11,400


TV = 18,120
29,520
Time line:
0 1 2 3 Years
k = 10%

-62,000 19,920 22,800 15,600


TV = 15,680
31,280
Expected increase in annual net income
ARR =
134 Initial (or average) investment .Answer (C) is correct. Capital budgeting is the process of
planning expenditures for assets, the returns on which are expected to continue beyond 1 year. Simulation (Monte Carlo
simulation) as applied to capital budgeting is a technique for experimenting with logical/mathematical models using a
computer. The computer is used to generate many examples of results based upon various assumptions.
Answer (A) is incorrect because this is a method used to rank projects for capital-budgeting decision purposes. Answer
(B) is incorrect because this is a method used to rank projects for capital-budgeting decision purposes. Answer (D) is
incorrect because this is a method used to rank projects for capital-budgeting decision purposes.

NPV
($)
NPV
$
A

B B
A

k Discount rate (%)


0 10% IRRB IRRA 0 16% 17% 18% 30%

NPV
$
NPV ($)
P
C

k 10% k
10% 15% 20% %
NPV NPV
$

A A

B
B n
Sigma CF t
k k t=1
0 7% 12% 15% 0 7% 12% 14%
(1 + K)t
NPV
$

X
Crossover

k
10% IRRY 12% IRRX

0 k = 12% 1 2 3 4 5 6

CFsA -15,000 4,000 4,000 4,000 4,000 4,000 4,000


Project B:
0 k = ? 1 2 6 Years
  
CFsB -14,815 5,100 5,100 5,100
Time lines:
Project A:
0 1 2 3 4 Years
k = 12%

CFsA -25,000 13,000 13,000 13,000 13,000


NPVA = ? = 17,663 Terminal value = 5,000
CF 4 = 18,000
Project B:
0 1 2 3 4 Years
k = ?

CFsB -25,000 15,247 15,247 15,247 15,247


NPVA = NPVB = 17,663 Terminal value = 0
CF 4 = 15,247
Time line:
0 k = 18% 1 2 3 Years

-3,000 1,728 1,920 1,152


Time line:
0 1 2 3 4 5 Years
k = 12%

-45,000 7,800 10,680 7,560 5,880 -1,920


Time line:
0 1 2 3 4 5 Years
k = 9%

-40,000 9,800 11,720 9,640 8,520 15,320


Time lines:
Project A:
0 k = 14% 1 2 3 4 Years

CFsA -200,000 71,104 71,104 71,104 71,104


Project B:
0 k = 10% 1 2 3 4 Years

CFsB -200,000 0 0 146,411 146,411


Time line:
0 k = 12% 1 2 10 Years
  
-50,000 6,000 6,000 6,000
Time lines:
Project A:
0 k = 12% 1 2 3 Periods

CFsA -5,000 2,000 2,500 2,250


Project B:
0 k = 14% 1 2 3 Periods

CFsB -5,000 3,000 2,600 2,900


0 1 2 3 4
| | | | |
CFsNew Tech -1,500 -315 -315 -315 -315 x x x
Time line:
0 1 2 3 Years
k = 14%

-42,000 14,280 16,200 11,400


TV = 18,120
29,520
Time line:
0 1 2 3 Years
k = 10%

-62,000 19,920 22,800 15,600


TV = 15,680
31,280
Expected increase in annual net income
ARR =
135 Initial (or average) investment .Answer (B) is correct. The term "rates of return" suggests
the net present value and internal rate of return methods in this context, but simulation analysis can also be used.
Simulation is a mathematical modeling technique for performing a what-if analysis of estimated data. For instance, it
may determine how profitable a company will be if Machine A is purchased rather than Machine B.
Answer (A) is incorrect because regression analysis is based on past data and is often used to determine trends or
divide costs into their fixed and variable components. Answer (C) is incorrect because Markov chain analysis is used in
decision problems in which the probability of the occurrence of a future state depends only on the current state. A
characteristic of the Markov process is that the initial state matters less and less as times goes on, because the process
will eventually reach its steady state. Answer (D) is incorrect because Gantt charting involves drawing a bar chart
showing the progress of a project.
NPV
($)
NPV
$
A

B B
A

k Discount rate (%)


0 10% IRRB IRRA 0 16% 17% 18% 30%

NPV
$
NPV ($)
P
C

k 10% k
10% 15% 20% %

NPV NPV
$

A A

B
B n
Sigma CF t
k k t=1
0 7% 12% 15% 0 7% 12% 14%
(1 + K)t
NPV
$

X
Crossover

k
10% IRRY 12% IRRX

0 k = 12% 1 2 3 4 5 6

CFsA -15,000 4,000 4,000 4,000 4,000 4,000 4,000


Project B:
0 k = ? 1 2 6 Years
  
CFsB -14,815 5,100 5,100 5,100
Time lines:
Project A:
0 1 2 3 4 Years
k = 12%

CFsA -25,000 13,000 13,000 13,000 13,000


NPVA = ? = 17,663 Terminal value = 5,000
CF 4 = 18,000
Project B:
0 1 2 3 4 Years
k = ?

CFsB -25,000 15,247 15,247 15,247 15,247


NPVA = NPVB = 17,663 Terminal value = 0
CF 4 = 15,247
Time line:
0 1 2 3 Years
k = 18%

-3,000 1,728 1,920 1,152


Time line:
0 1 2 3 4 5 Years
k = 12%

-45,000 7,800 10,680 7,560 5,880 -1,920


Time line:
0 k = 9% 1 2 3 4 5 Years

-40,000 9,800 11,720 9,640 8,520 15,320


Time lines:
Project A:
0 k = 14% 1 2 3 4 Years

CFsA -200,000 71,104 71,104 71,104 71,104


Project B:
0 k = 10% 1 2 3 4 Years

CFsB -200,000 0 0 146,411 146,411


Time line:
0 k = 12% 1 2 10 Years
  
-50,000 6,000 6,000 6,000
Time lines:
Project A:
0 k = 12% 1 2 3 Periods

CFsA -5,000 2,000 2,500 2,250


Project B:
0 k = 14% 1 2 3 Periods

CFsB -5,000 3,000 2,600 2,900


0 1 2 3 4
| | | | |
CFsNew Tech -1,500 -315 -315 -315 -315 x x x
Time line:
0 1 2 3 Years
k = 14%

-42,000 14,280 16,200 11,400


TV = 18,120
29,520
Time line:
0 1 2 3 Years
k = 10%

-62,000 19,920 22,800 15,600


TV = 15,680
31,280
Expected increase in annual net income
ARR =
136 Initial (or average) investment . Sensitivity, scenario, and simulation analyses

Answer: c Diff: E N

Statement a is false; sensitivity analysis measures a project’s stand-alone risk. Statement b is false; sensitivity
analysis doesn’t take into account probabilities, while scenario analysis does. Statement c is correct.

NPV
($)
NPV
$
A

B B
A

k Discount rate (%)


0 10% IRRB IRRA 0 16% 17% 18% 30%

NPV
$
NPV ($)
P
C

k 10% k
10% 15% 20% %
NPV NPV
$

A A

B
B n
Sigma CF t
k k t=1
0 7% 12% 15% 0 7% 12% 14%
(1 + K)t
NPV
$

X
Crossover

k
10% IRRY 12% IRRX

0 k = 12% 1 2 3 4 5 6

CFsA -15,000 4,000 4,000 4,000 4,000 4,000 4,000


Project B:
0 k = ? 1 2 6 Years
  
CFsB -14,815 5,100 5,100 5,100
Time lines:
Project A:
0 1 2 3 4 Years
k = 12%

CFsA -25,000 13,000 13,000 13,000 13,000


NPVA = ? = 17,663 Terminal value = 5,000
CF 4 = 18,000
Project B:
0 1 2 3 4 Years
k = ?

CFsB -25,000 15,247 15,247 15,247 15,247


NPVA = NPVB = 17,663 Terminal value = 0
CF 4 = 15,247
Time line:
0 k = 18% 1 2 3 Years

-3,000 1,728 1,920 1,152


Time line:
0 1 2 3 4 5 Years
k = 12%

-45,000 7,800 10,680 7,560 5,880 -1,920


Time line:
0 1 2 3 4 5 Years
k = 9%

-40,000 9,800 11,720 9,640 8,520 15,320


Time lines:
Project A:
0 k = 14% 1 2 3 4 Years

CFsA -200,000 71,104 71,104 71,104 71,104


Project B:
0 k = 10% 1 2 3 4 Years

CFsB -200,000 0 0 146,411 146,411


Time line:
0 k = 12% 1 2 10 Years
  
-50,000 6,000 6,000 6,000
Time lines:
Project A:
0 k = 12% 1 2 3 Periods

CFsA -5,000 2,000 2,500 2,250


Project B:
0 k = 14% 1 2 3 Periods

CFsB -5,000 3,000 2,600 2,900


0 1 2 3 4
| | | | |
CFsNew Tech -1,500 -315 -315 -315 -315 x x x
Time line:
0 1 2 3 Years
k = 14%

-42,000 14,280 16,200 11,400


TV = 18,120
29,520
Time line:
0 1 2 3 Years
k = 10%

-62,000 19,920 22,800 15,600


TV = 15,680
31,280
Expected increase in annual net income
ARR =
137 Initial (or average) investment .Answer (C) is correct. After a problem has been formulated
into any mathematical model, it may be subjected to sensitivity analysis, which is a trial-and-error method used to
determine the sensitivity of the estimates used. For example, forecasts of many calculated NPVs under various
assumptions may be compared to determine how sensitive the NPV is to changing conditions. Changing the
assumptions about a certain variable or group of variables may drastically alter the NPV, suggesting that the risk of the
investment may be excessive.
Answer (A) is incorrect because sensitivity analysis is useful when cash flows, or other assumptions, are uncertain.
Answer (B) is incorrect because sensitivity analysis can be used with any of the capital budgeting methods. Answer (D)
is incorrect because sensitivity analysis is not a ranking technique; it calculates results under varying assumptions.
NPV
($)
NPV
$
A

B B
A

k Discount rate (%)


0 10% IRRB IRRA 0 16% 17% 18% 30%

NPV
$
NPV ($)
P
C

k 10% k
10% 15% 20% %

NPV NPV
$

A A

B
B n
Sigma CF t
k k t=1
0 7% 12% 15% 0 7% 12% 14%
(1 + K)t
NPV
$

X
Crossover

k
10% IRRY 12% IRRX

0 k = 12% 1 2 3 4 5 6

CFsA -15,000 4,000 4,000 4,000 4,000 4,000 4,000


Project B:
0 k = ? 1 2 6 Years
  
CFsB -14,815 5,100 5,100 5,100
Time lines:
Project A:
0 1 2 3 4 Years
k = 12%

CFsA -25,000 13,000 13,000 13,000 13,000


NPVA = ? = 17,663 Terminal value = 5,000
CF 4 = 18,000
Project B:
0 1 2 3 4 Years
k = ?

CFsB -25,000 15,247 15,247 15,247 15,247


NPVA = NPVB = 17,663 Terminal value = 0
CF 4 = 15,247
Time line:
0 1 2 3 Years
k = 18%

-3,000 1,728 1,920 1,152


Time line:
0 1 2 3 4 5 Years
k = 12%

-45,000 7,800 10,680 7,560 5,880 -1,920


Time line:
0 k = 9% 1 2 3 4 5 Years

-40,000 9,800 11,720 9,640 8,520 15,320


Time lines:
Project A:
0 k = 14% 1 2 3 4 Years

CFsA -200,000 71,104 71,104 71,104 71,104


Project B:
0 k = 10% 1 2 3 4 Years

CFsB -200,000 0 0 146,411 146,411


Time line:
0 k = 12% 1 2 10 Years
  
-50,000 6,000 6,000 6,000
Time lines:
Project A:
0 k = 12% 1 2 3 Periods

CFsA -5,000 2,000 2,500 2,250


Project B:
0 k = 14% 1 2 3 Periods

CFsB -5,000 3,000 2,600 2,900


0 1 2 3 4
| | | | |
CFsNew Tech -1,500 -315 -315 -315 -315 x x x
Time line:
0 1 2 3 Years
k = 14%

-42,000 14,280 16,200 11,400


TV = 18,120
29,520
Time line:
0 1 2 3 Years
k = 10%

-62,000 19,920 22,800 15,600


TV = 15,680
31,280
Expected increase in annual net income
ARR =
138 Initial (or average) investment .Answer (A) is correct. Sensitivity analysis is a technique to
evaluate a model in terms of the effect of changing the values of the parameters. It answers "what if" questions. In
capital budgeting models, sensitivity analysis is the examination of alternative outcomes under different assumptions.
Answer (B) is incorrect because probability (risk) analysis is used to examine the array of possible outcomes given
alternative parameters. Answer (C) is incorrect because cost behavior (variance) analysis concerns historical costs, not
predictions of future cash inflows and outflows. Answer (D) is incorrect because ROI analysis is appropriate for
determining the profitability of a company, segment, etc.

NPV
($)
NPV
$
A

B B
A

k Discount rate (%)


0 10% IRRB IRRA 0 16% 17% 18% 30%

NPV
$
NPV ($)
P
C

k 10% k
10% 15% 20% %
NPV NPV
$

A A

B
B n
Sigma CF t
k k t=1
0 7% 12% 15% 0 7% 12% 14%
(1 + K)t
NPV
$

X
Crossover

k
10% IRRY 12% IRRX

0 k = 12% 1 2 3 4 5 6

CFsA -15,000 4,000 4,000 4,000 4,000 4,000 4,000


Project B:
0 k = ? 1 2 6 Years
  
CFsB -14,815 5,100 5,100 5,100
Time lines:
Project A:
0 1 2 3 4 Years
k = 12%

CFsA -25,000 13,000 13,000 13,000 13,000


NPVA = ? = 17,663 Terminal value = 5,000
CF 4 = 18,000
Project B:
0 1 2 3 4 Years
k = ?

CFsB -25,000 15,247 15,247 15,247 15,247


NPVA = NPVB = 17,663 Terminal value = 0
CF 4 = 15,247
Time line:
0 k = 18% 1 2 3 Years

-3,000 1,728 1,920 1,152


Time line:
0 1 2 3 4 5 Years
k = 12%

-45,000 7,800 10,680 7,560 5,880 -1,920


Time line:
0 1 2 3 4 5 Years
k = 9%

-40,000 9,800 11,720 9,640 8,520 15,320


Time lines:
Project A:
0 k = 14% 1 2 3 4 Years

CFsA -200,000 71,104 71,104 71,104 71,104


Project B:
0 k = 10% 1 2 3 4 Years

CFsB -200,000 0 0 146,411 146,411


Time line:
0 k = 12% 1 2 10 Years
  
-50,000 6,000 6,000 6,000
Time lines:
Project A:
0 k = 12% 1 2 3 Periods

CFsA -5,000 2,000 2,500 2,250


Project B:
0 k = 14% 1 2 3 Periods

CFsB -5,000 3,000 2,600 2,900


0 1 2 3 4
| | | | |
CFsNew Tech -1,500 -315 -315 -315 -315 x x x
Time line:
0 1 2 3 Years
k = 14%

-42,000 14,280 16,200 11,400


TV = 18,120
29,520
Time line:
0 1 2 3 Years
k = 10%

-62,000 19,920 22,800 15,600


TV = 15,680
31,280
Expected increase in annual net income
ARR =
139 Initial (or average) investment .Answer (B) is correct. After a problem has been formulated
into any mathematical model, it may be subjected to sensitivity analysis, which is a trial-and-error method used to
determine the sensitivity of the estimates used. For example, forecasts of many calculated NPVs under various
assumptions may be compared to determine how sensitive the NPV is to changing conditions. Changing the
assumptions about a certain variable or group of variables may drastically alter the NPV, suggesting that the risk of the
investment may be excessive.
Answer (A) is incorrect because sensitivity analysis is a means of making several estimates of inputs into a capital
budgeting decision to determine the effect of changes in assumptions. Answer (C) is incorrect because sensitivity
analysis is not a simulation technique; it is simply a process of making more than one estimate of unknown variables.
Answer (D) is incorrect because sensitivity analysis would not identify the required market share; instead, it would be
used to make several estimates of market share to determine how sensitive the decision is to changes in market share.
NPV
($)
NPV
$
A

B B
A

k Discount rate (%)


0 10% IRRB IRRA 0 16% 17% 18% 30%

NPV
$
NPV ($)
P
C

k 10% k
10% 15% 20% %

NPV NPV
$

A A

B
B n
Sigma CF t
k k t=1
0 7% 12% 15% 0 7% 12% 14%
(1 + K)t
NPV
$

X
Crossover

k
10% IRRY 12% IRRX

0 k = 12% 1 2 3 4 5 6

CFsA -15,000 4,000 4,000 4,000 4,000 4,000 4,000


Project B:
0 k = ? 1 2 6 Years
  
CFsB -14,815 5,100 5,100 5,100
Time lines:
Project A:
0 1 2 3 4 Years
k = 12%

CFsA -25,000 13,000 13,000 13,000 13,000


NPVA = ? = 17,663 Terminal value = 5,000
CF 4 = 18,000
Project B:
0 1 2 3 4 Years
k = ?

CFsB -25,000 15,247 15,247 15,247 15,247


NPVA = NPVB = 17,663 Terminal value = 0
CF 4 = 15,247
Time line:
0 1 2 3 Years
k = 18%

-3,000 1,728 1,920 1,152


Time line:
0 1 2 3 4 5 Years
k = 12%

-45,000 7,800 10,680 7,560 5,880 -1,920


Time line:
0 k = 9% 1 2 3 4 5 Years

-40,000 9,800 11,720 9,640 8,520 15,320


Time lines:
Project A:
0 k = 14% 1 2 3 4 Years

CFsA -200,000 71,104 71,104 71,104 71,104


Project B:
0 k = 10% 1 2 3 4 Years

CFsB -200,000 0 0 146,411 146,411


Time line:
0 k = 12% 1 2 10 Years
  
-50,000 6,000 6,000 6,000
Time lines:
Project A:
0 k = 12% 1 2 3 Periods

CFsA -5,000 2,000 2,500 2,250


Project B:
0 k = 14% 1 2 3 Periods

CFsB -5,000 3,000 2,600 2,900


0 1 2 3 4
| | | | |
CFsNew Tech -1,500 -315 -315 -315 -315 x x x
Time line:
0 1 2 3 Years
k = 14%

-42,000 14,280 16,200 11,400


TV = 18,120
29,520
Time line:
0 1 2 3 Years
k = 10%

-62,000 19,920 22,800 15,600


TV = 15,680
31,280
Expected increase in annual net income
ARR =
140 Initial (or average) investment . Monte Carlo simulation Answer: e Diff: M
NPV
($)
NPV
$
A

B B
A

k Discount rate (%)


0 10% IRRB IRRA 0 16% 17% 18% 30%

NPV
$
NPV ($)
P
C

k 10% k
10% 15% 20% %

NPV NPV
$

A A

B
B n
Sigma CF t
k k t=1
0 7% 12% 15% 0 7% 12% 14%
(1 + K)t
NPV
$

X
Crossover

k
10% IRRY 12% IRRX

0 k = 12% 1 2 3 4 5 6

CFsA -15,000 4,000 4,000 4,000 4,000 4,000 4,000


Project B:
0 k = ? 1 2 6 Years
  
CFsB -14,815 5,100 5,100 5,100
Time lines:
Project A:
0 1 2 3 4 Years
k = 12%

CFsA -25,000 13,000 13,000 13,000 13,000


NPVA = ? = 17,663 Terminal value = 5,000
CF 4 = 18,000
Project B:
0 1 2 3 4 Years
k = ?

CFsB -25,000 15,247 15,247 15,247 15,247


NPVA = NPVB = 17,663 Terminal value = 0
CF 4 = 15,247
Time line:
0 1 2 3 Years
k = 18%

-3,000 1,728 1,920 1,152


Time line:
0 1 2 3 4 5 Years
k = 12%

-45,000 7,800 10,680 7,560 5,880 -1,920


Time line:
0 k = 9% 1 2 3 4 5 Years

-40,000 9,800 11,720 9,640 8,520 15,320


Time lines:
Project A:
0 k = 14% 1 2 3 4 Years

CFsA -200,000 71,104 71,104 71,104 71,104


Project B:
0 k = 10% 1 2 3 4 Years

CFsB -200,000 0 0 146,411 146,411


Time line:
0 k = 12% 1 2 10 Years
  
-50,000 6,000 6,000 6,000
Time lines:
Project A:
0 k = 12% 1 2 3 Periods

CFsA -5,000 2,000 2,500 2,250


Project B:
0 k = 14% 1 2 3 Periods

CFsB -5,000 3,000 2,600 2,900


0 1 2 3 4
| | | | |
CFsNew Tech -1,500 -315 -315 -315 -315 x x x
Time line:
0 1 2 3 Years
k = 14%

-42,000 14,280 16,200 11,400


TV = 18,120
29,520
Time line:
0 1 2 3 Years
k = 10%

-62,000 19,920 22,800 15,600


TV = 15,680
31,280
Expected increase in annual net income
ARR =
141 Initial (or average) investment .Answer (D) is correct. An increase in the discount rate would
lower the net present value, as would a decrease in cash flows or an increase in the initial investment.
Answer (A) is incorrect because a decrease in the tax rate would decrease tax expense, thus increasing cash flows and
the NPV. Answer (B) is incorrect because a decrease in the initial investment amount would increase the NPV. Answer
(C) is incorrect because an extension of the project life and associated cash inflows would increase the NPV.
NPV
($)
NPV
$
A

B B
A

k Discount rate (%)


0 10% IRRB IRRA 0 16% 17% 18% 30%

NPV
$
NPV ($)
P
C

k 10% k
10% 15% 20% %

NPV NPV
$

A A

B
B n
Sigma CF t
k k t=1
0 7% 12% 15% 0 7% 12% 14%
(1 + K)t
NPV
$

X
Crossover

k
10% IRRY 12% IRRX

0 k = 12% 1 2 3 4 5 6

CFsA -15,000 4,000 4,000 4,000 4,000 4,000 4,000


Project B:
0 k = ? 1 2 6 Years
  
CFsB -14,815 5,100 5,100 5,100
Time lines:
Project A:
0 1 2 3 4 Years
k = 12%

CFsA -25,000 13,000 13,000 13,000 13,000


NPVA = ? = 17,663 Terminal value = 5,000
CF 4 = 18,000
Project B:
0 1 2 3 4 Years
k = ?

CFsB -25,000 15,247 15,247 15,247 15,247


NPVA = NPVB = 17,663 Terminal value = 0
CF 4 = 15,247
Time line:
0 1 2 3 Years
k = 18%

-3,000 1,728 1,920 1,152


Time line:
0 1 2 3 4 5 Years
k = 12%

-45,000 7,800 10,680 7,560 5,880 -1,920


Time line:
0 k = 9% 1 2 3 4 5 Years

-40,000 9,800 11,720 9,640 8,520 15,320


Time lines:
Project A:
0 k = 14% 1 2 3 4 Years

CFsA -200,000 71,104 71,104 71,104 71,104


Project B:
0 k = 10% 1 2 3 4 Years

CFsB -200,000 0 0 146,411 146,411


Time line:
0 k = 12% 1 2 10 Years
  
-50,000 6,000 6,000 6,000
Time lines:
Project A:
0 k = 12% 1 2 3 Periods

CFsA -5,000 2,000 2,500 2,250


Project B:
0 k = 14% 1 2 3 Periods

CFsB -5,000 3,000 2,600 2,900


0 1 2 3 4
| | | | |
CFsNew Tech -1,500 -315 -315 -315 -315 x x x
Time line:
0 1 2 3 Years
k = 14%

-42,000 14,280 16,200 11,400


TV = 18,120
29,520
Time line:
0 1 2 3 Years
k = 10%

-62,000 19,920 22,800 15,600


TV = 15,680
31,280
Expected increase in annual net income
ARR =
142 Initial (or average) investment 12A-. NPV and depreciation Answer: c Diff: E
NPV
($)
NPV
$
A

B B
A

k Discount rate (%)


0 10% IRRB IRRA 0 16% 17% 18% 30%

NPV
$
NPV ($)
P
C

k 10% k
10% 15% 20% %

NPV NPV
$

A A

B
B n
Sigma CF t
k k t=1
0 7% 12% 15% 0 7% 12% 14%
(1 + K)t
NPV
$

X
Crossover

k
10% IRRY 12% IRRX

0 k = 12% 1 2 3 4 5 6

CFsA -15,000 4,000 4,000 4,000 4,000 4,000 4,000


Project B:
0 k = ? 1 2 6 Years
  
CFsB -14,815 5,100 5,100 5,100
Time lines:
Project A:
0 1 2 3 4 Years
k = 12%

CFsA -25,000 13,000 13,000 13,000 13,000


NPVA = ? = 17,663 Terminal value = 5,000
CF 4 = 18,000
Project B:
0 1 2 3 4 Years
k = ?

CFsB -25,000 15,247 15,247 15,247 15,247


NPVA = NPVB = 17,663 Terminal value = 0
CF 4 = 15,247
Time line:
0 1 2 3 Years
k = 18%

-3,000 1,728 1,920 1,152


Time line:
0 1 2 3 4 5 Years
k = 12%

-45,000 7,800 10,680 7,560 5,880 -1,920


Time line:
0 k = 9% 1 2 3 4 5 Years

-40,000 9,800 11,720 9,640 8,520 15,320


Time lines:
Project A:
0 k = 14% 1 2 3 4 Years

CFsA -200,000 71,104 71,104 71,104 71,104


Project B:
0 k = 10% 1 2 3 4 Years

CFsB -200,000 0 0 146,411 146,411


Time line:
0 k = 12% 1 2 10 Years
  
-50,000 6,000 6,000 6,000
Time lines:
Project A:
0 k = 12% 1 2 3 Periods

CFsA -5,000 2,000 2,500 2,250


Project B:
0 k = 14% 1 2 3 Periods

CFsB -5,000 3,000 2,600 2,900


0 1 2 3 4
| | | | |
CFsNew Tech -1,500 -315 -315 -315 -315 x x x
Time line:
0 1 2 3 Years
k = 14%

-42,000 14,280 16,200 11,400


TV = 18,120
29,520
Time line:
0 1 2 3 Years
k = 10%

-62,000 19,920 22,800 15,600


TV = 15,680
31,280
Expected increase in annual net income
ARR =
143 Initial (or average) investment .Statements a, b, c, and d are false. Statement e is correct
because you can think of a firm as a big project. If the stock is correctly priced, i.e., the stock market is efficient, the NPV
of this project should be zero.

NPV
($)
NPV
$
A

B B
A

k Discount rate (%)


0 10% IRRB IRRA 0 16% 17% 18% 30%

NPV
$
NPV ($)
P
C

k 10% k
10% 15% 20% %
NPV NPV
$

A A

B
B n
Sigma CF t
k k t=1
0 7% 12% 15% 0 7% 12% 14%
(1 + K)t
NPV
$

X
Crossover

k
10% IRRY 12% IRRX

0 k = 12% 1 2 3 4 5 6

CFsA -15,000 4,000 4,000 4,000 4,000 4,000 4,000


Project B:
0 k = ? 1 2 6 Years
  
CFsB -14,815 5,100 5,100 5,100
Time lines:
Project A:
0 1 2 3 4 Years
k = 12%

CFsA -25,000 13,000 13,000 13,000 13,000


NPVA = ? = 17,663 Terminal value = 5,000
CF 4 = 18,000
Project B:
0 1 2 3 4 Years
k = ?

CFsB -25,000 15,247 15,247 15,247 15,247


NPVA = NPVB = 17,663 Terminal value = 0
CF 4 = 15,247
Time line:
0 k = 18% 1 2 3 Years

-3,000 1,728 1,920 1,152


Time line:
0 1 2 3 4 5 Years
k = 12%

-45,000 7,800 10,680 7,560 5,880 -1,920


Time line:
0 1 2 3 4 5 Years
k = 9%

-40,000 9,800 11,720 9,640 8,520 15,320


Time lines:
Project A:
0 k = 14% 1 2 3 4 Years

CFsA -200,000 71,104 71,104 71,104 71,104


Project B:
0 k = 10% 1 2 3 4 Years

CFsB -200,000 0 0 146,411 146,411


Time line:
0 k = 12% 1 2 10 Years
  
-50,000 6,000 6,000 6,000
Time lines:
Project A:
0 k = 12% 1 2 3 Periods

CFsA -5,000 2,000 2,500 2,250


Project B:
0 k = 14% 1 2 3 Periods

CFsB -5,000 3,000 2,600 2,900


0 1 2 3 4
| | | | |
CFsNew Tech -1,500 -315 -315 -315 -315 x x x
Time line:
0 1 2 3 Years
k = 14%

-42,000 14,280 16,200 11,400


TV = 18,120
29,520
Time line:
0 1 2 3 Years
k = 10%

-62,000 19,920 22,800 15,600


TV = 15,680
31,280
Expected increase in annual net income
ARR =
144 Initial (or average) investment .REQUIRED: The true statement regarding the NPV profiles
of two mutually exclusive capital projects.
DISCUSSION: (A) The NPV is the excess of the present value of the estimated cash inflows over the net cost of the
investment. Thus, Project 2 has a higher internal rate of return. The internal rate of return is the cost of capital at which
the NPV is zero, that is, the cost of capital at which the NPV profile crosses the horizontal axis. The NPV profile for
Project 2 intersects the horizontal axis at a higher cost of capital percentage than does that for Project 1.
Answer (B) is incorrect because Project 1 has the lower internal rate of return. Answers (C) and (D) are incorrect
because the profiles of Projects 1 and 2 intersect. Neither project will have a higher NPV at all cost of capital
percentages. To the left of the intersection point, Project 1 has a higher NPV. To the right of the intersection point,
Project 2 has a higher NPV.
NPV
($)
NPV
$
A

B B
A

k Discount rate (%)


0 10% IRRB IRRA 0 16% 17% 18% 30%

NPV
$
NPV ($)
P
C

k 10% k
10% 15% 20% %

NPV NPV
$

A A

B
B n
Sigma CF t
k k t=1
0 7% 12% 15% 0 7% 12% 14%
(1 + K)t
NPV
$

X
Crossover

k
10% IRRY 12% IRRX

0 k = 12% 1 2 3 4 5 6

CFsA -15,000 4,000 4,000 4,000 4,000 4,000 4,000


Project B:
0 k = ? 1 2 6 Years
  
CFsB -14,815 5,100 5,100 5,100
Time lines:
Project A:
0 1 2 3 4 Years
k = 12%

CFsA -25,000 13,000 13,000 13,000 13,000


NPVA = ? = 17,663 Terminal value = 5,000
CF 4 = 18,000
Project B:
0 1 2 3 4 Years
k = ?

CFsB -25,000 15,247 15,247 15,247 15,247


NPVA = NPVB = 17,663 Terminal value = 0
CF 4 = 15,247
Time line:
0 1 2 3 Years
k = 18%

-3,000 1,728 1,920 1,152


Time line:
0 1 2 3 4 5 Years
k = 12%

-45,000 7,800 10,680 7,560 5,880 -1,920


Time line:
0 k = 9% 1 2 3 4 5 Years

-40,000 9,800 11,720 9,640 8,520 15,320


Time lines:
Project A:
0 k = 14% 1 2 3 4 Years

CFsA -200,000 71,104 71,104 71,104 71,104


Project B:
0 k = 10% 1 2 3 4 Years

CFsB -200,000 0 0 146,411 146,411


Time line:
0 k = 12% 1 2 10 Years
  
-50,000 6,000 6,000 6,000
Time lines:
Project A:
0 k = 12% 1 2 3 Periods

CFsA -5,000 2,000 2,500 2,250


Project B:
0 k = 14% 1 2 3 Periods

CFsB -5,000 3,000 2,600 2,900


0 1 2 3 4
| | | | |
CFsNew Tech -1,500 -315 -315 -315 -315 x x x
Time line:
0 1 2 3 Years
k = 14%

-42,000 14,280 16,200 11,400


TV = 18,120
29,520
Time line:
0 1 2 3 Years
k = 10%

-62,000 19,920 22,800 15,600


TV = 15,680
31,280
Expected increase in annual net income
ARR =
145 Initial (or average) investment .You can draw the NPV profiles to get an idea of what is
happening. (See the diagram below.) Statement a is false; Project B could have a higher NPV at some WACC if the
NPV profiles cross. Statement b is false; Project B could have a negative NPV when A’s NPV is positive. Statement c is
false; the IRR is unaffected by the WACC. Statement d is the correct choice.
NPV
$

B
A

k
0 10% IRRB IRRA
NPV
($)

Discount rate (%)


0 16% 17% 18% 30%

NPV
$
NPV ($)
P
C

k 10% k
10% 15% 20% %

NPV NPV
$

A A

B
B n
Sigma CF t
k k t=1
0 7% 12% 15% 0 7% 12% 14%
(1 + K)t
NPV
$

X
Crossover

k
10% IRRY 12% IRRX

0 k = 12% 1 2 3 4 5 6

CFsA -15,000 4,000 4,000 4,000 4,000 4,000 4,000


Project B:
0 k = ? 1 2 6 Years
  
CFsB -14,815 5,100 5,100 5,100
Time lines:
Project A:
0 1 2 3 4 Years
k = 12%

CFsA -25,000 13,000 13,000 13,000 13,000


NPVA = ? = 17,663 Terminal value = 5,000
CF 4 = 18,000
Project B:
0 1 2 3 4 Years
k = ?

CFsB -25,000 15,247 15,247 15,247 15,247


NPVA = NPVB = 17,663 Terminal value = 0
CF 4 = 15,247
Time line:
0 1 2 3 Years
k = 18%

-3,000 1,728 1,920 1,152


Time line:
0 1 2 3 4 5 Years
k = 12%

-45,000 7,800 10,680 7,560 5,880 -1,920


Time line:
0 k = 9% 1 2 3 4 5 Years

-40,000 9,800 11,720 9,640 8,520 15,320


Time lines:
Project A:
0 k = 14% 1 2 3 4 Years

CFsA -200,000 71,104 71,104 71,104 71,104


Project B:
0 k = 10% 1 2 3 4 Years

CFsB -200,000 0 0 146,411 146,411


Time line:
0 k = 12% 1 2 10 Years
  
-50,000 6,000 6,000 6,000
Time lines:
Project A:
0 k = 12% 1 2 3 Periods

CFsA -5,000 2,000 2,500 2,250


Project B:
0 k = 14% 1 2 3 Periods

CFsB -5,000 3,000 2,600 2,900


0 1 2 3 4
| | | | |
CFsNew Tech -1,500 -315 -315 -315 -315 x x x
Time line:
0 1 2 3 Years
k = 14%

-42,000 14,280 16,200 11,400


TV = 18,120
29,520
Time line:
0 1 2 3 Years
k = 10%

-62,000 19,920 22,800 15,600


TV = 15,680
31,280
NPV
$

B
A

Expected increase in annual net income k


ARR = 0 10%
Initial (or average) investment IRRB IRRA
146
.
NPV
($)

Discount rate (%)


0 16% 17% 18% 30%

Draw the NPV profiles using the information given in the problem. It is clear that Project A will have a higher NPV when
the cost of capital is 12 percent. Therefore, statement a is false. At a 17 percent cost of capital, Project B will have a
higher NPV than Project A. Therefore, statement b is true. If the cost of capital were 0, then the NPV of the projects
would be the simple sum of all the cash flows. In order for statement c to be correct, B’s NPV at a 0 cost of capital would
have to be higher than A’s. From the diagram we see that this is clearly incorrect. So, statement c is false.
NPV
$
NPV ($)
P
C

k 10% k
10% 15% 20% %

NPV NPV
$

A A

B
B n
Sigma CF t
k k t=1
0 7% 12% 15% 0 7% 12% 14%
(1 + K)t
NPV
$

X
Crossover

k
10% IRRY 12% IRRX

0 k = 12% 1 2 3 4 5 6

CFsA -15,000 4,000 4,000 4,000 4,000 4,000 4,000


Project B:
0 k = ? 1 2 6 Years
  
CFsB -14,815 5,100 5,100 5,100
Time lines:
Project A:
0 1 2 3 4 Years
k = 12%

CFsA -25,000 13,000 13,000 13,000 13,000


NPVA = ? = 17,663 Terminal value = 5,000
CF 4 = 18,000
Project B:
0 1 2 3 4 Years
k = ?

CFsB -25,000 15,247 15,247 15,247 15,247


NPVA = NPVB = 17,663 Terminal value = 0
CF 4 = 15,247
Time line:
0 k = 18% 1 2 3 Years

-3,000 1,728 1,920 1,152


Time line:
0 1 2 3 4 5 Years
k = 12%

-45,000 7,800 10,680 7,560 5,880 -1,920


Time line:
0 1 2 3 4 5 Years
k = 9%

-40,000 9,800 11,720 9,640 8,520 15,320


Time lines:
Project A:
0 k = 14% 1 2 3 4 Years

CFsA -200,000 71,104 71,104 71,104 71,104


Project B:
0 k = 10% 1 2 3 4 Years

CFsB -200,000 0 0 146,411 146,411


Time line:
0 k = 12% 1 2 10 Years
  
-50,000 6,000 6,000 6,000
Time lines:
Project A:
0 k = 12% 1 2 3 Periods

CFsA -5,000 2,000 2,500 2,250


Project B:
0 k = 14% 1 2 3 Periods

CFsB -5,000 3,000 2,600 2,900


0 1 2 3 4
| | | | |
CFsNew Tech -1,500 -315 -315 -315 -315 x x x
Time line:
0 1 2 3 Years
k = 14%

-42,000 14,280 16,200 11,400


TV = 18,120
29,520
Time line:
0 1 2 3 Years
k = 10%

-62,000 19,920 22,800 15,600


TV = 15,680
31,280
NPV
$

B
A

Expected increase in annual net income k


ARR = 0 10%
147 Initial (or average) investment IRRB IRRA

NPV
($)

Discount rate (%)


0 16% 17% 18% 30%
. NPV profiles

Answer: d Diff: M N

NPV
$

k
10% 15% 20% %
The diagram above can be drawn from the statements in this question. From the diagram drawn, statements a, b, and c
are correct; therefore, statement d is the correct choice.

NPV ($)
NPV

C
A
D

10% k k
0 7% 12% 15%

NPV
$

B n
Sigma CF t
k t=1
0 7% 12% 14%
(1 + K)t
NPV
$

X
Crossover

k
10% IRRY 12% IRRX

0 k = 12% 1 2 3 4 5 6

CFsA -15,000 4,000 4,000 4,000 4,000 4,000 4,000


Project B:
0 k = ? 1 2 6 Years
  
CFsB -14,815 5,100 5,100 5,100
Time lines:
Project A:
0 1 2 3 4 Years
k = 12%

CFsA -25,000 13,000 13,000 13,000 13,000


NPVA = ? = 17,663 Terminal value = 5,000
CF 4 = 18,000
Project B:
0 1 2 3 4 Years
k = ?

CFsB -25,000 15,247 15,247 15,247 15,247


NPVA = NPVB = 17,663 Terminal value = 0
CF 4 = 15,247
Time line:
0 k = 18% 1 2 3 Years

-3,000 1,728 1,920 1,152


Time line:
0 1 2 3 4 5 Years
k = 12%

-45,000 7,800 10,680 7,560 5,880 -1,920


Time line:
0 1 2 3 4 5 Years
k = 9%

-40,000 9,800 11,720 9,640 8,520 15,320


Time lines:
Project A:
0 k = 14% 1 2 3 4 Years

CFsA -200,000 71,104 71,104 71,104 71,104


Project B:
0 k = 10% 1 2 3 4 Years

CFsB -200,000 0 0 146,411 146,411


Time line:
0 k = 12% 1 2 10 Years
  
-50,000 6,000 6,000 6,000
Time lines:
Project A:
0 k = 12% 1 2 3 Periods

CFsA -5,000 2,000 2,500 2,250


Project B:
0 k = 14% 1 2 3 Periods

CFsB -5,000 3,000 2,600 2,900


0 1 2 3 4
| | | | |
CFsNew Tech -1,500 -315 -315 -315 -315 x x x
Time line:
0 1 2 3 Years
k = 14%

-42,000 14,280 16,200 11,400


TV = 18,120
29,520
Time line:
0 1 2 3 Years
k = 10%

-62,000 19,920 22,800 15,600


TV = 15,680
31,280
NPV
$

B
A

Expected increase in annual net income k


ARR = 0 10%
148 Initial (or average) investment IRRB IRRA

NPV
($)

Discount rate (%)


0 16% 17% 18% 30%
NPV
$

k
10% 15% 20% %
.

NPV ($)

10% k

First, draw the NPV profiles as shown above. Make sure the profiles cross at 10 percent because the projects have the
same NPV at a cost of capital of 10 percent. When WACC is less than 10 percent, C has a higher NPV, so C’s NPV
profile is above D’s NPV profile to the left of the crossover point (10%).

Statement a is true. IRR is always independent of the cost of capital, and from the diagram above, we can see that
D’s IRR is to the right of C’s where the two lines cross the X-axis. Statement b is false. IRR is independent of the
cost of capital, and from the diagram C’s IRR is always lower than D’s. Statement c is true. D’s MIRR will be
somewhere between the cost of capital and the IRR. Therefore, the correct choice is statement d.
NPV NPV
$

A A

B
B n
Sigma CF t
k k t=1
0 7% 12% 15% 0 7% 12% 14%
(1 + K)t
NPV
$

X
Crossover

k
10% IRRY 12% IRRX

0 k = 12% 1 2 3 4 5 6

CFsA -15,000 4,000 4,000 4,000 4,000 4,000 4,000


Project B:
0 k = ? 1 2 6 Years
  
CFsB -14,815 5,100 5,100 5,100
Time lines:
Project A:
0 1 2 3 4 Years
k = 12%

CFsA -25,000 13,000 13,000 13,000 13,000


NPVA = ? = 17,663 Terminal value = 5,000
CF 4 = 18,000
Project B:
0 1 2 3 4 Years
k = ?

CFsB -25,000 15,247 15,247 15,247 15,247


NPVA = NPVB = 17,663 Terminal value = 0
CF 4 = 15,247
Time line:
0 k = 18% 1 2 3 Years

-3,000 1,728 1,920 1,152


Time line:
0 1 2 3 4 5 Years
k = 12%

-45,000 7,800 10,680 7,560 5,880 -1,920


Time line:
0 1 2 3 4 5 Years
k = 9%

-40,000 9,800 11,720 9,640 8,520 15,320


Time lines:
Project A:
0 k = 14% 1 2 3 4 Years

CFsA -200,000 71,104 71,104 71,104 71,104


Project B:
0 k = 10% 1 2 3 4 Years

CFsB -200,000 0 0 146,411 146,411


Time line:
0 k = 12% 1 2 10 Years
  
-50,000 6,000 6,000 6,000
Time lines:
Project A:
0 k = 12% 1 2 3 Periods

CFsA -5,000 2,000 2,500 2,250


Project B:
0 k = 14% 1 2 3 Periods

CFsB -5,000 3,000 2,600 2,900


0 1 2 3 4
| | | | |
CFsNew Tech -1,500 -315 -315 -315 -315 x x x
Time line:
0 1 2 3 Years
k = 14%

-42,000 14,280 16,200 11,400


TV = 18,120
29,520
Time line:
0 1 2 3 Years
k = 10%

-62,000 19,920 22,800 15,600


TV = 15,680
31,280
NPV
$

B
A

Expected increase in annual net income k


ARR = 0 10%
149 Initial (or average) investment IRRB IRRA

NPV
($)

Discount rate (%)


0 16% 17% 18% 30%

NPV
$
NPV ($)
P
C

k 10% k
10% 15% 20% %
.
NPV

k
0 7% 12% 15%

Since both projects have an IRR greater than the cost of capital, both will have a positive NPV. Therefore, statement a is
true. At 6 percent, the cost of capital is less than the crossover rate and Project A has a higher NPV than B. Therefore,
statement b is false. If the cost of capital is 13 percent, then the cost of capital is greater than the crossover rate and B
would have a higher NPV than A. Therefore, statement c is true. Since statements a and c are both true, the correct
choice is statement e.
NPV
$

B n
Sigma CF t
k t=1
0 7% 12% 14%
(1 + K)t
NPV
$

X
Crossover

k
10% IRRY 12% IRRX

0 k = 12% 1 2 3 4 5 6

CFsA -15,000 4,000 4,000 4,000 4,000 4,000 4,000


Project B:
0 k = ? 1 2 6 Years
  
CFsB -14,815 5,100 5,100 5,100
Time lines:
Project A:
0 1 2 3 4 Years
k = 12%

CFsA -25,000 13,000 13,000 13,000 13,000


NPVA = ? = 17,663 Terminal value = 5,000
CF 4 = 18,000
Project B:
0 1 2 3 4 Years
k = ?

CFsB -25,000 15,247 15,247 15,247 15,247


NPVA = NPVB = 17,663 Terminal value = 0
CF 4 = 15,247
Time line:
0 1 2 3 Years
k = 18%

-3,000 1,728 1,920 1,152


Time line:
0 1 2 3 4 5 Years
k = 12%

-45,000 7,800 10,680 7,560 5,880 -1,920


Time line:
0 k = 9% 1 2 3 4 5 Years

-40,000 9,800 11,720 9,640 8,520 15,320


Time lines:
Project A:
0 k = 14% 1 2 3 4 Years

CFsA -200,000 71,104 71,104 71,104 71,104


Project B:
0 k = 10% 1 2 3 4 Years

CFsB -200,000 0 0 146,411 146,411


Time line:
0 k = 12% 1 2 10 Years
  
-50,000 6,000 6,000 6,000
Time lines:
Project A:
0 k = 12% 1 2 3 Periods

CFsA -5,000 2,000 2,500 2,250


Project B:
0 k = 14% 1 2 3 Periods

CFsB -5,000 3,000 2,600 2,900


0 1 2 3 4
| | | | |
CFsNew Tech -1,500 -315 -315 -315 -315 x x x
Time line:
0 1 2 3 Years
k = 14%

-42,000 14,280 16,200 11,400


TV = 18,120
29,520
Time line:
0 1 2 3 Years
k = 10%

-62,000 19,920 22,800 15,600


TV = 15,680
31,280
NPV
$

B
A

Expected increase in annual net income k


ARR = 0 10%
150 Initial (or average) investment IRRB IRRA

NPV
($)

Discount rate (%)


0 16% 17% 18% 30%

NPV
$
NPV ($)
P
C

k 10% k
10% 15% 20% %

NPV

k
0 7% 12% 15%
.
NPV
$

k
0 7% 12% 14%

Statement a is correct, because at any point to the right of the crossover point B will have a higher NPV than A.
Statement b is correct for the same reason that statement a is true; at any point to the right of the crossover point, B will
have a higher NPV than A. Statement c is correct. If B’s cost of capital is 9 percent, when MIRR is calculated the cash
flows are being reinvested at 9 percent. When IRR is used, the IRR calculation assumes that cash flows are being
reinvested at the IRR (which is higher than the cost of capital.) Statement e is the correct choice.

NPV
$

X
Crossover

n
Sigma CF t k
t=1
10% IRRY 12% IRRX
t
(1 + K)
0 k = 12% 1 2 3 4 5 6

CFsA -15,000 4,000 4,000 4,000 4,000 4,000 4,000


Project B:
0 k = ? 1 2 6 Years
  
CFsB -14,815 5,100 5,100 5,100
Time lines:
Project A:
0 1 2 3 4 Years
k = 12%

CFsA -25,000 13,000 13,000 13,000 13,000


NPVA = ? = 17,663 Terminal value = 5,000
CF 4 = 18,000
Project B:
0 1 2 3 4 Years
k = ?

CFsB -25,000 15,247 15,247 15,247 15,247


NPVA = NPVB = 17,663 Terminal value = 0
CF 4 = 15,247
Time line:
0 k = 18% 1 2 3 Years

-3,000 1,728 1,920 1,152


Time line:
0 1 2 3 4 5 Years
k = 12%

-45,000 7,800 10,680 7,560 5,880 -1,920


Time line:
0 k = 9% 1 2 3 4 5 Years

-40,000 9,800 11,720 9,640 8,520 15,320


Time lines:
Project A:
0 k = 14% 1 2 3 4 Years

CFsA -200,000 71,104 71,104 71,104 71,104


Project B:
0 k = 10% 1 2 3 4 Years

CFsB -200,000 0 0 146,411 146,411


Time line:
0 k = 12% 1 2 10 Years
  
-50,000 6,000 6,000 6,000
Time lines:
Project A:
0 k = 12% 1 2 3 Periods

CFsA -5,000 2,000 2,500 2,250


Project B:
0 k = 14% 1 2 3 Periods

CFsB -5,000 3,000 2,600 2,900


0 1 2 3 4
| | | | |
CFsNew Tech -1,500 -315 -315 -315 -315 x x x
Time line:
0 1 2 3 Years
k = 14%

-42,000 14,280 16,200 11,400


TV = 18,120
29,520
Time line:
0 1 2 3 Years
k = 10%

-62,000 19,920 22,800 15,600


TV = 15,680
31,280
NPV
$

B
A

Expected increase in annual net income k


ARR = 0 10%
151 Initial (or average) investment IRRB IRRA

NPV
($)

Discount rate (%)


0 16% 17% 18% 30%

NPV
$
NPV ($)
P
C

k 10% k
10% 15% 20% %

NPV NPV
$

A A

B
B

k k
0 7% 12% 15% 0 7% 12% 14%

.REQUIRED: The true statement about the IRR.


DISCUSSION: (B) the IRR is the discount rate at which the net present value is zero. Because the present value of a
dollar is higher the sooner it is received, projects with later cash flows will have lower net present values for any given
discount rate than will projects with earlier cash flows, if other factors are constant. Hence, projects with later cash flows
will have a lower IRR.
Answer (A) is incorrect because the present value of the cash inflows is inversely related to the discount rate, that is, if
the discount rate is higher, the present value of the cash inflows is lower. If the investment cost is lower, a higher
discount rate (the IRR) will be required to set the net present value equal to zero. Answer (C) is incorrect because the
larger the cash inflows, the higher the IRR will be. Higher cash inflows have a higher present value at any given
discount rate. A higher discount rate will be required to set the net present value equal to zero. Answer (D) is incorrect
because projects with shorter payback periods have higher cash inflow early in the life of the project. Projects with
earlier cash inflows have the higher IRRs.

NPV
$

X
Crossover

n
Sigma CF t k
t=1
10% IRRY 12% IRRX
(1 + K)t
0 k = 12% 1 2 3 4 5 6

CFsA -15,000 4,000 4,000 4,000 4,000 4,000 4,000


Project B:
0 k = ? 1 2 6 Years
  
CFsB -14,815 5,100 5,100 5,100
Time lines:
Project A:
0 1 2 3 4 Years
k = 12%

CFsA -25,000 13,000 13,000 13,000 13,000


NPVA = ? = 17,663 Terminal value = 5,000
CF 4 = 18,000
Project B:
0 1 2 3 4 Years
k = ?

CFsB -25,000 15,247 15,247 15,247 15,247


NPVA = NPVB = 17,663 Terminal value = 0
CF 4 = 15,247
Time line:
0 1 2 3 Years
k = 18%

-3,000 1,728 1,920 1,152


Time line:
0 1 2 3 4 5 Years
k = 12%

-45,000 7,800 10,680 7,560 5,880 -1,920


Time line:
0 k = 9% 1 2 3 4 5 Years

-40,000 9,800 11,720 9,640 8,520 15,320


Time lines:
Project A:
0 k = 14% 1 2 3 4 Years

CFsA -200,000 71,104 71,104 71,104 71,104


Project B:
0 k = 10% 1 2 3 4 Years

CFsB -200,000 0 0 146,411 146,411


Time line:
0 k = 12% 1 2 10 Years
  
-50,000 6,000 6,000 6,000
Time lines:
Project A:
0 k = 12% 1 2 3 Periods

CFsA -5,000 2,000 2,500 2,250


Project B:
0 k = 14% 1 2 3 Periods

CFsB -5,000 3,000 2,600 2,900


0 1 2 3 4
| | | | |
CFsNew Tech -1,500 -315 -315 -315 -315 x x x
Time line:
0 1 2 3 Years
k = 14%

-42,000 14,280 16,200 11,400


TV = 18,120
29,520
Time line:
0 1 2 3 Years
k = 10%

-62,000 19,920 22,800 15,600


TV = 15,680
31,280
NPV
$

B
A

Expected increase in annual net income k


ARR = 0 10%
152 Initial (or average) investment IRRB IRRA

NPV
($)

Discount rate (%)


0 16% 17% 18% 30%

NPV
$
NPV ($)
P
C

k 10% k
10% 15% 20% %

NPV NPV
$

A A

B
B

k k
0 7% 12% 15% 0 7% 12% 14%
.Answer (A)
is correct. Investments with present values in excess of their costs (positive NPVs) that can be identified or created by
the capital budgeting activities of the firm will have a positive impact on firm value and on the price of the common
shares of the firm. Accordingly, the more effective capital budgeting is, the higher the share price.
Answer (B) is incorrect because positive NPV investments will increase, not decrease, firm value and share price.
Answer (C) is incorrect because investments with present values equal to their costs have a zero NPV and neither
increase nor decrease firm value and share price. Answer (D) is incorrect because investments with present values
equal to their costs have a zero NPV and neither increase nor decrease firm value and share price.

NPV
$

X
Crossover

n
Sigma CF t k
t=1
10% IRRY 12% IRRX
(1 + K)t
0 k = 12% 1 2 3 4 5 6

CFsA -15,000 4,000 4,000 4,000 4,000 4,000 4,000


Project B:
0 k = ? 1 2 6 Years
  
CFsB -14,815 5,100 5,100 5,100
Time lines:
Project A:
0 1 2 3 4 Years
k = 12%

CFsA -25,000 13,000 13,000 13,000 13,000


NPVA = ? = 17,663 Terminal value = 5,000
CF 4 = 18,000
Project B:
0 1 2 3 4 Years
k = ?

CFsB -25,000 15,247 15,247 15,247 15,247


NPVA = NPVB = 17,663 Terminal value = 0
CF 4 = 15,247
Time line:
0 1 2 3 Years
k = 18%

-3,000 1,728 1,920 1,152


Time line:
0 1 2 3 4 5 Years
k = 12%

-45,000 7,800 10,680 7,560 5,880 -1,920


Time line:
0 k = 9% 1 2 3 4 5 Years

-40,000 9,800 11,720 9,640 8,520 15,320


Time lines:
Project A:
0 k = 14% 1 2 3 4 Years

CFsA -200,000 71,104 71,104 71,104 71,104


Project B:
0 k = 10% 1 2 3 4 Years

CFsB -200,000 0 0 146,411 146,411


Time line:
0 k = 12% 1 2 10 Years
  
-50,000 6,000 6,000 6,000
Time lines:
Project A:
0 k = 12% 1 2 3 Periods

CFsA -5,000 2,000 2,500 2,250


Project B:
0 k = 14% 1 2 3 Periods

CFsB -5,000 3,000 2,600 2,900


0 1 2 3 4
| | | | |
CFsNew Tech -1,500 -315 -315 -315 -315 x x x
Time line:
0 1 2 3 Years
k = 14%

-42,000 14,280 16,200 11,400


TV = 18,120
29,520
Time line:
0 1 2 3 Years
k = 10%

-62,000 19,920 22,800 15,600


TV = 15,680
31,280
NPV
$

B
A

Expected increase in annual net income k


ARR = 0 10%
153 Initial (or average) investment IRRB IRRA

NPV
($)

Discount rate (%)


0 16% 17% 18% 30%
NPV
$
NPV ($)
P
C

k 10% k
10% 15% 20% %

NPV NPV
$

A A

B
B

k k
0 7% 12% 15% 0 7% 12% 14%
.Answer (A)
is correct. The value of the firm is the present value of the expected cash flows, which is given by the following
expression:
n
Sigma CF t
t=1
(1 + K)t
If CF is net cash flow, K is the discount rate (cost of capital), and t is time, value will rise as CF increases.
Answer (B) is incorrect because an increase in systematic (market or undiversifiable) risk will increase the overall cost of
capital and thereby increase K, the discount rate. As a result, the value of the firm will fall. Answer (C) is incorrect
because an increase in unsystematic (or firm-specific) risk will have no effect on the value of the firm. The total risk is
the sum of systematic and unsystematic risk. By definition, the latter is the risk that can be eliminated by diversification.
Answer (D) is incorrect because an increase in the discount rate will reduce the value of the firm.

NPV
$

X
Crossover

k
10% IRRY 12% IRRX

0 k = 12% 1 2 3 4 5 6

CFsA -15,000 4,000 4,000 4,000 4,000 4,000 4,000


Project B:
0 k = ? 1 2 6 Years
  
CFsB -14,815 5,100 5,100 5,100
Time lines:
Project A:
0 1 2 3 4 Years
k = 12%

CFsA -25,000 13,000 13,000 13,000 13,000


NPVA = ? = 17,663 Terminal value = 5,000
CF 4 = 18,000
Project B:
0 1 2 3 4 Years
k = ?

CFsB -25,000 15,247 15,247 15,247 15,247


NPVA = NPVB = 17,663 Terminal value = 0
CF 4 = 15,247
Time line:
0 1 2 3 Years
k = 18%

-3,000 1,728 1,920 1,152


Time line:
0 1 2 3 4 5 Years
k = 12%

-45,000 7,800 10,680 7,560 5,880 -1,920


Time line:
0 k = 9% 1 2 3 4 5 Years

-40,000 9,800 11,720 9,640 8,520 15,320


Time lines:
Project A:
0 k = 14% 1 2 3 4 Years

CFsA -200,000 71,104 71,104 71,104 71,104


Project B:
0 k = 10% 1 2 3 4 Years

CFsB -200,000 0 0 146,411 146,411


Time line:
0 k = 12% 1 2 10 Years
  
-50,000 6,000 6,000 6,000
Time lines:
Project A:
0 k = 12% 1 2 3 Periods

CFsA -5,000 2,000 2,500 2,250


Project B:
0 k = 14% 1 2 3 Periods

CFsB -5,000 3,000 2,600 2,900


0 1 2 3 4
| | | | |
CFsNew Tech -1,500 -315 -315 -315 -315 x x x
Time line:
0 1 2 3 Years
k = 14%

-42,000 14,280 16,200 11,400


TV = 18,120
29,520
Time line:
0 1 2 3 Years
k = 10%

-62,000 19,920 22,800 15,600


TV = 15,680
31,280
NPV
$

B
A

Expected increase in annual net income k


ARR = 0 10%
154 Initial (or average) investment IRRB IRRA

NPV
($)

Discount rate (%)


0 16% 17% 18% 30%
NPV
$
NPV ($)
P
C

k 10% k
10% 15% 20% %

NPV NPV
$

A A

B
B n
Sigma CF t
k k t=1
0 7% 12% 15% 0 7% 12% 14%
(1 + K)t

. Risk analysis Answer: c Diff: E N

Statement a is false. Stand-alone risk is measured by standard deviation. Therefore, since Y’s standard deviation is
higher than X’s, Y has higher stand-alone risk than X. Statement b is false. Corporate risk is measured by the
correlation of project cash flows with other company cash flows. Therefore, since Y’s cash flows are highly
correlated with the cash flows of existing projects, while X’s are not, Y has higher corporate risk than X. Market
risk is measured by beta. Therefore, since X’s beta is greater than Y’s, statement c is true.
NPV
$

X
Crossover

k
10% IRRY 12% IRRX

0 k = 12% 1 2 3 4 5 6

CFsA -15,000 4,000 4,000 4,000 4,000 4,000 4,000


Project B:
0 k = ? 1 2 6 Years
  
CFsB -14,815 5,100 5,100 5,100
Time lines:
Project A:
0 1 2 3 4 Years
k = 12%

CFsA -25,000 13,000 13,000 13,000 13,000


NPVA = ? = 17,663 Terminal value = 5,000
CF 4 = 18,000
Project B:
0 1 2 3 4 Years
k = ?

CFsB -25,000 15,247 15,247 15,247 15,247


NPVA = NPVB = 17,663 Terminal value = 0
CF 4 = 15,247
Time line:
0 1 2 3 Years
k = 18%

-3,000 1,728 1,920 1,152


Time line:
0 1 2 3 4 5 Years
k = 12%

-45,000 7,800 10,680 7,560 5,880 -1,920


Time line:
0 k = 9% 1 2 3 4 5 Years

-40,000 9,800 11,720 9,640 8,520 15,320


Time lines:
Project A:
0 k = 14% 1 2 3 4 Years

CFsA -200,000 71,104 71,104 71,104 71,104


Project B:
0 k = 10% 1 2 3 4 Years

CFsB -200,000 0 0 146,411 146,411


Time line:
0 k = 12% 1 2 10 Years
  
-50,000 6,000 6,000 6,000
Time lines:
Project A:
0 k = 12% 1 2 3 Periods

CFsA -5,000 2,000 2,500 2,250


Project B:
0 k = 14% 1 2 3 Periods

CFsB -5,000 3,000 2,600 2,900


0 1 2 3 4
| | | | |
CFsNew Tech -1,500 -315 -315 -315 -315 x x x
Time line:
0 1 2 3 Years
k = 14%

-42,000 14,280 16,200 11,400


TV = 18,120
29,520
Time line:
0 1 2 3 Years
k = 10%

-62,000 19,920 22,800 15,600


TV = 15,680
31,280
NPV
$

B
A

Expected increase in annual net income k


ARR = 0 10%
155 Initial (or average) investment IRRB IRRA

NPV
($)

Discount rate (%)


0 16% 17% 18% 30%
NPV
$
NPV ($)
P
C

k 10% k
10% 15% 20% %

NPV NPV
$

A A

B
B

k k
0 7% 12% 15% 0 7% 12% 14%
n
Sigma CF t
t=1
(1 + K)t .Statement a is correct. The IRR’s of both exceed the cost of capital. Statement b is incorrect. We
cannot determine this without knowing the NPV’s of the projects. Statement c is correct. To see why draw the NPV
profiles. Statement d is incorrect. Therefore, statement e is the correct answer.

NPV
$

X
Crossover

k
10% IRRY 12% IRRX

0 k = 12% 1 2 3 4 5 6

CFsA -15,000 4,000 4,000 4,000 4,000 4,000 4,000


Project B:
0 k = ? 1 2 6 Years
  
CFsB -14,815 5,100 5,100 5,100
Time lines:
Project A:
0 1 2 3 4 Years
k = 12%

CFsA -25,000 13,000 13,000 13,000 13,000


NPVA = ? = 17,663 Terminal value = 5,000
CF 4 = 18,000
Project B:
0 1 2 3 4 Years
k = ?

CFsB -25,000 15,247 15,247 15,247 15,247


NPVA = NPVB = 17,663 Terminal value = 0
CF 4 = 15,247
Time line:
0 k = 18% 1 2 3 Years

-3,000 1,728 1,920 1,152


Time line:
0 1 2 3 4 5 Years
k = 12%

-45,000 7,800 10,680 7,560 5,880 -1,920


Time line:
0 1 2 3 4 5 Years
k = 9%

-40,000 9,800 11,720 9,640 8,520 15,320


Time lines:
Project A:
0 k = 14% 1 2 3 4 Years

CFsA -200,000 71,104 71,104 71,104 71,104


Project B:
0 k = 10% 1 2 3 4 Years

CFsB -200,000 0 0 146,411 146,411


Time line:
0 k = 12% 1 2 10 Years
  
-50,000 6,000 6,000 6,000
Time lines:
Project A:
0 k = 12% 1 2 3 Periods

CFsA -5,000 2,000 2,500 2,250


Project B:
0 k = 14% 1 2 3 Periods

CFsB -5,000 3,000 2,600 2,900


0 1 2 3 4
| | | | |
CFsNew Tech -1,500 -315 -315 -315 -315 x x x
Time line:
0 1 2 3 Years
k = 14%

-42,000 14,280 16,200 11,400


TV = 18,120
29,520
Time line:
0 1 2 3 Years
k = 10%

-62,000 19,920 22,800 15,600


TV = 15,680
31,280
NPV
$

B
A

Expected increase in annual net income k


ARR = 0 10%
156 Initial (or average) investment IRRB IRRA

NPV
($)

Discount rate (%)


0 16% 17% 18% 30%

NPV
$
NPV ($)
P
C

k 10% k
10% 15% 20% %

NPV NPV
$

A A

B
B

k k
0 7% 12% 15% 0 7% 12% 14%
n
Sigma CF t
t=1
(1 + K)t .This is the only project with either a positive NPV or an IRR which exceeds the cost of capital.

NPV
$

X
Crossover

k
10% IRRY 12% IRRX

0 k = 12% 1 2 3 4 5 6

CFsA -15,000 4,000 4,000 4,000 4,000 4,000 4,000


Project B:
0 k = ? 1 2 6 Years
  
CFsB -14,815 5,100 5,100 5,100
Time lines:
Project A:
0 1 2 3 4 Years
k = 12%

CFsA -25,000 13,000 13,000 13,000 13,000


NPVA = ? = 17,663 Terminal value = 5,000
CF 4 = 18,000
Project B:
0 1 2 3 4 Years
k = ?

CFsB -25,000 15,247 15,247 15,247 15,247


NPVA = NPVB = 17,663 Terminal value = 0
CF 4 = 15,247
Time line:
0 1 2 3 Years
k = 18%

-3,000 1,728 1,920 1,152


Time line:
0 1 2 3 4 5 Years
k = 12%

-45,000 7,800 10,680 7,560 5,880 -1,920


Time line:
0 k = 9% 1 2 3 4 5 Years

-40,000 9,800 11,720 9,640 8,520 15,320


Time lines:
Project A:
0 k = 14% 1 2 3 4 Years

CFsA -200,000 71,104 71,104 71,104 71,104


Project B:
0 k = 10% 1 2 3 4 Years

CFsB -200,000 0 0 146,411 146,411


Time line:
0 k = 12% 1 2 10 Years
  
-50,000 6,000 6,000 6,000
Time lines:
Project A:
0 k = 12% 1 2 3 Periods

CFsA -5,000 2,000 2,500 2,250


Project B:
0 k = 14% 1 2 3 Periods

CFsB -5,000 3,000 2,600 2,900


0 1 2 3 4
| | | | |
CFsNew Tech -1,500 -315 -315 -315 -315 x x x
Time line:
0 1 2 3 Years
k = 14%

-42,000 14,280 16,200 11,400


TV = 18,120
29,520
Time line:
0 1 2 3 Years
k = 10%

-62,000 19,920 22,800 15,600


TV = 15,680
31,280
NPV
$

B
A

Expected increase in annual net income k


ARR = 0 10%
157 Initial (or average) investment IRRB IRRA

NPV
($)

Discount rate (%)


0 16% 17% 18% 30%
NPV
$
NPV ($)
P
C

k 10% k
10% 15% 20% %

NPV NPV
$

A A

B
B

k k
0 7% 12% 15% 0 7% 12% 14%
n
Sigma CF t
t=1
(1 + K)t .Draw out the NPV profiles of these two projects. As B’s NPV declines more rapidly with an increase in
discount rates, this implies that more of the cash flows are coming later on. Therefore, Project A has a faster payback
than Project B.

NPV
$

X
Crossover

k
10% IRRY 12% IRRX

0 k = 12% 1 2 3 4 5 6

CFsA -15,000 4,000 4,000 4,000 4,000 4,000 4,000


Project B:
0 k = ? 1 2 6 Years
  
CFsB -14,815 5,100 5,100 5,100
Time lines:
Project A:
0 1 2 3 4 Years
k = 12%

CFsA -25,000 13,000 13,000 13,000 13,000


NPVA = ? = 17,663 Terminal value = 5,000
CF 4 = 18,000
Project B:
0 1 2 3 4 Years
k = ?

CFsB -25,000 15,247 15,247 15,247 15,247


NPVA = NPVB = 17,663 Terminal value = 0
CF 4 = 15,247
Time line:
0 k = 18% 1 2 3 Years

-3,000 1,728 1,920 1,152


Time line:
0 1 2 3 4 5 Years
k = 12%

-45,000 7,800 10,680 7,560 5,880 -1,920


Time line:
0 1 2 3 4 5 Years
k = 9%

-40,000 9,800 11,720 9,640 8,520 15,320


Time lines:
Project A:
0 k = 14% 1 2 3 4 Years

CFsA -200,000 71,104 71,104 71,104 71,104


Project B:
0 k = 10% 1 2 3 4 Years

CFsB -200,000 0 0 146,411 146,411


Time line:
0 k = 12% 1 2 10 Years
  
-50,000 6,000 6,000 6,000
Time lines:
Project A:
0 k = 12% 1 2 3 Periods

CFsA -5,000 2,000 2,500 2,250


Project B:
0 k = 14% 1 2 3 Periods

CFsB -5,000 3,000 2,600 2,900


0 1 2 3 4
| | | | |
CFsNew Tech -1,500 -315 -315 -315 -315 x x x
Time line:
0 1 2 3 Years
k = 14%

-42,000 14,280 16,200 11,400


TV = 18,120
29,520
Time line:
0 1 2 3 Years
k = 10%

-62,000 19,920 22,800 15,600


TV = 15,680
31,280
NPV
$

B
A

Expected increase in annual net income k


ARR = 0 10%
158 Initial (or average) investment IRRB IRRA

NPV
($)

Discount rate (%)


0 16% 17% 18% 30%

NPV
$
NPV ($)
P
C

k 10% k
10% 15% 20% %

NPV NPV
$

A A

B
B

k k
0 7% 12% 15% 0 7% 12% 14%
n
Sigma CF t
t=1
(1 + K)t .
NPV
$

X
Crossover

k
10% IRRY 12% IRRX

If IRRX is greater than MIRRX, then its IRR must be higher than the cost of capital. (Remember that the MIRR will be
somewhere between the cost of capital and the IRR). Therefore, statement a must be true. Similarly, if IRR Y is less than
MIRRY, then its IRR must be lower than the cost of capital. Therefore, statement b must be true. At a cost of capital of 10
percent they have the same NPV, so this is the crossover rate. From statements a and b we know that IRR X must be
greater than IRRY, so to the right of the crossover rate NPV X will be larger than NPVY. Consequently, to the left of the
crossover rate NPVX must be smaller than NPVY. Therefore, statement c is also true. Since statements a, b, and c are all
true, the correct choice is statement d.

0 k = 12% 1 2 3 4 5 6

CFsA -15,000 4,000 4,000 4,000 4,000 4,000 4,000


Project B:
0 k = ? 1 2 6 Years
  
CFsB -14,815 5,100 5,100 5,100
Time lines:
Project A:
0 1 2 3 4 Years
k = 12%

CFsA -25,000 13,000 13,000 13,000 13,000


NPVA = ? = 17,663 Terminal value = 5,000
CF 4 = 18,000
Project B:
0 1 2 3 4 Years
k = ?

CFsB -25,000 15,247 15,247 15,247 15,247


NPVA = NPVB = 17,663 Terminal value = 0
CF 4 = 15,247
Time line:
0 k = 18% 1 2 3 Years

-3,000 1,728 1,920 1,152


Time line:
0 1 2 3 4 5 Years
k = 12%

-45,000 7,800 10,680 7,560 5,880 -1,920


Time line:
0 1 2 3 4 5 Years
k = 9%

-40,000 9,800 11,720 9,640 8,520 15,320


Time lines:
Project A:
0 k = 14% 1 2 3 4 Years

CFsA -200,000 71,104 71,104 71,104 71,104


Project B:
0 k = 10% 1 2 3 4 Years

CFsB -200,000 0 0 146,411 146,411


Time line:
0 k = 12% 1 2 10 Years
  
-50,000 6,000 6,000 6,000
Time lines:
Project A:
0 k = 12% 1 2 3 Periods

CFsA -5,000 2,000 2,500 2,250


Project B:
0 k = 14% 1 2 3 Periods

CFsB -5,000 3,000 2,600 2,900


0 1 2 3 4
| | | | |
CFsNew Tech -1,500 -315 -315 -315 -315 x x x
Time line:
0 1 2 3 Years
k = 14%

-42,000 14,280 16,200 11,400


TV = 18,120
29,520
Time line:
0 1 2 3 Years
k = 10%

-62,000 19,920 22,800 15,600


TV = 15,680
31,280
NPV
$

B
A

Expected increase in annual net income k


ARR = 0 10%
159 Initial (or average) investment IRRB IRRA

NPV
($)

Discount rate (%)


0 16% 17% 18% 30%

NPV
$
NPV ($)
P
C

k 10% k
10% 15% 20% %

NPV NPV
$

A A

B
B

k k
0 7% 12% 15% 0 7% 12% 14%
NPV
$

X
Crossover

n
Sigma CF t k
t=1
10% IRRY 12% IRRX
t
(1 + K) .This statement reflects exactly the difference
between the NPV and IRR methods.

0 k = 12% 1 2 3 4 5 6

CFsA -15,000 4,000 4,000 4,000 4,000 4,000 4,000


Project B:
0 k = ? 1 2 6 Years
  
CFsB -14,815 5,100 5,100 5,100
Time lines:
Project A:
0 1 2 3 4 Years
k = 12%

CFsA -25,000 13,000 13,000 13,000 13,000


NPVA = ? = 17,663 Terminal value = 5,000
CF 4 = 18,000
Project B:
0 1 2 3 4 Years
k = ?

CFsB -25,000 15,247 15,247 15,247 15,247


NPVA = NPVB = 17,663 Terminal value = 0
CF 4 = 15,247
Time line:
0 k = 18% 1 2 3 Years

-3,000 1,728 1,920 1,152


Time line:
0 1 2 3 4 5 Years
k = 12%

-45,000 7,800 10,680 7,560 5,880 -1,920


Time line:
0 1 2 3 4 5 Years
k = 9%

-40,000 9,800 11,720 9,640 8,520 15,320


Time lines:
Project A:
0 k = 14% 1 2 3 4 Years

CFsA -200,000 71,104 71,104 71,104 71,104


Project B:
0 k = 10% 1 2 3 4 Years

CFsB -200,000 0 0 146,411 146,411


Time line:
0 k = 12% 1 2 10 Years
  
-50,000 6,000 6,000 6,000
Time lines:
Project A:
0 k = 12% 1 2 3 Periods

CFsA -5,000 2,000 2,500 2,250


Project B:
0 k = 14% 1 2 3 Periods

CFsB -5,000 3,000 2,600 2,900


0 1 2 3 4
| | | | |
CFsNew Tech -1,500 -315 -315 -315 -315 x x x
Time line:
0 1 2 3 Years
k = 14%

-42,000 14,280 16,200 11,400


TV = 18,120
29,520
Time line:
0 1 2 3 Years
k = 10%

-62,000 19,920 22,800 15,600


TV = 15,680
31,280
NPV
$

B
A

Expected increase in annual net income k


ARR = 0 10%
160 Initial (or average) investment IRRB IRRA

NPV
($)

Discount rate (%)


0 16% 17% 18% 30%
NPV
$
NPV ($)
P
C

k 10% k
10% 15% 20% %

NPV NPV
$

A A

B
B

k k
0 7% 12% 15% 0 7% 12% 14%

NPV
$

X
Crossover

n
Sigma CF t k
t=1
10% IRRY 12% IRRX
t
(1 + K) .Both statements a and c are correct;
therefore, statement d is the correct choice. Due to reinvestment rate assumptions, NPV and IRR can lead to conflicts;
however, there will be no conflict between NPV and MIRR if the projects are equal in size (which is one of the
assumptions in this question).

0 k = 12% 1 2 3 4 5 6

CFsA -15,000 4,000 4,000 4,000 4,000 4,000 4,000


Project B:
0 k = ? 1 2 6 Years
  
CFsB -14,815 5,100 5,100 5,100
Time lines:
Project A:
0 1 2 3 4 Years
k = 12%

CFsA -25,000 13,000 13,000 13,000 13,000


NPVA = ? = 17,663 Terminal value = 5,000
CF 4 = 18,000
Project B:
0 1 2 3 4 Years
k = ?

CFsB -25,000 15,247 15,247 15,247 15,247


NPVA = NPVB = 17,663 Terminal value = 0
CF 4 = 15,247
Time line:
0 k = 18% 1 2 3 Years

-3,000 1,728 1,920 1,152


Time line:
0 1 2 3 4 5 Years
k = 12%

-45,000 7,800 10,680 7,560 5,880 -1,920


Time line:
0 1 2 3 4 5 Years
k = 9%

-40,000 9,800 11,720 9,640 8,520 15,320


Time lines:
Project A:
0 k = 14% 1 2 3 4 Years

CFsA -200,000 71,104 71,104 71,104 71,104


Project B:
0 k = 10% 1 2 3 4 Years

CFsB -200,000 0 0 146,411 146,411


Time line:
0 k = 12% 1 2 10 Years
  
-50,000 6,000 6,000 6,000
Time lines:
Project A:
0 k = 12% 1 2 3 Periods

CFsA -5,000 2,000 2,500 2,250


Project B:
0 k = 14% 1 2 3 Periods

CFsB -5,000 3,000 2,600 2,900


0 1 2 3 4
| | | | |
CFsNew Tech -1,500 -315 -315 -315 -315 x x x
Time line:
0 1 2 3 Years
k = 14%

-42,000 14,280 16,200 11,400


TV = 18,120
29,520
Time line:
0 1 2 3 Years
k = 10%

-62,000 19,920 22,800 15,600


TV = 15,680
31,280
NPV
$

B
A

Expected increase in annual net income k


ARR = 0 10%
161 Initial (or average) investment IRRB IRRA

NPV
($)

Discount rate (%)


0 16% 17% 18% 30%

NPV
$
NPV ($)
P
C

k 10% k
10% 15% 20% %

NPV NPV
$

A A

B
B

k k
0 7% 12% 15% 0 7% 12% 14%
NPV
$

X
Crossover

n
Sigma CF t k
t=1
10% IRRY 12% IRRX
t
(1 + K) .Statement a is true. To see this, sketch out a
NPV profile for a normal, independent project, which means that only one NPV profile will appear on the graph. If WACC
< IRR, then IRR says accept. But in that case, NPV > 0, so NPV will also say accept. Statement d is false. Here is the
reasoning:
1. For the NPV profiles to cross, then one project must have a higher NPV at k = 0 than the other project, that is, their
vertical axis intercepts will be different.
2. A second condition for NPV profiles to cross is that one have a higher IRR than the other.
3. The third condition necessary for profiles to cross is that the project with the higher NPV at k = 0 will have the lower
IRR.
One can sketch out two NPV profiles on a graph to see that these three conditions are indeed required.
4. The project with the higher NPV at k = 0 must have more cash inflows, because it has the higher NPV when cash
flows are not discounted, which is the situation if k = 0.
5. If the project with more total cash inflows also had its cash flows come in earlier, it would dominate the other
project--its NPV would be higher at all discount rates, and its IRR would also be higher, so the profiles would not
cross. The only way the profiles can cross is for the project with more total cash inflows to get a relatively high
percentage of those inflows in distant years, so that their PVs are low when discounted at high rates. Most students
either grasp this intuitively or else just guess at the question!

0 k = 12% 1 2 3 4 5 6

CFsA -15,000 4,000 4,000 4,000 4,000 4,000 4,000


Project B:
0 k = ? 1 2 6 Years
  
CFsB -14,815 5,100 5,100 5,100
Time lines:
Project A:
0 1 2 3 4 Years
k = 12%

CFsA -25,000 13,000 13,000 13,000 13,000


NPVA = ? = 17,663 Terminal value = 5,000
CF 4 = 18,000
Project B:
0 1 2 3 4 Years
k = ?

CFsB -25,000 15,247 15,247 15,247 15,247


NPVA = NPVB = 17,663 Terminal value = 0
CF 4 = 15,247
Time line:
0 1 2 3 Years
k = 18%

-3,000 1,728 1,920 1,152


Time line:
0 1 2 3 4 5 Years
k = 12%

-45,000 7,800 10,680 7,560 5,880 -1,920


Time line:
0 k = 9% 1 2 3 4 5 Years

-40,000 9,800 11,720 9,640 8,520 15,320


Time lines:
Project A:
0 k = 14% 1 2 3 4 Years

CFsA -200,000 71,104 71,104 71,104 71,104


Project B:
0 k = 10% 1 2 3 4 Years

CFsB -200,000 0 0 146,411 146,411


Time line:
0 k = 12% 1 2 10 Years
  
-50,000 6,000 6,000 6,000
Time lines:
Project A:
0 k = 12% 1 2 3 Periods

CFsA -5,000 2,000 2,500 2,250


Project B:
0 k = 14% 1 2 3 Periods

CFsB -5,000 3,000 2,600 2,900


0 1 2 3 4
| | | | |
CFsNew Tech -1,500 -315 -315 -315 -315 x x x
Time line:
0 1 2 3 Years
k = 14%

-42,000 14,280 16,200 11,400


TV = 18,120
29,520
Time line:
0 1 2 3 Years
k = 10%

-62,000 19,920 22,800 15,600


TV = 15,680
31,280
NPV
$

B
A

Expected increase in annual net income k


ARR = 0 10%
162 Initial (or average) investment IRRB IRRA

NPV
($)

Discount rate (%)


0 16% 17% 18% 30%

NPV
$
NPV ($)
P
C

k 10% k
10% 15% 20% %

NPV NPV
$

A A

B
B

k k
0 7% 12% 15% 0 7% 12% 14%
NPV
$

X
Crossover

n
Sigma CF t k
t=1
10% IRRY 12% IRRX
t
(1 + K) .Answer (A) is correct. If the economic results
of alternative capital investments were known with certainty or with minimal risk, the quantitative analyses would reveal
the absolute best investment options. However, if the economic results are highly uncertain, using a decision-making
process that combines rational analysis with intuition is appropriate. Moreover, nonquantifiable variables may be
involved.
Answer (B) is incorrect because the decision-making process described combines rational quantitative analysis with
intuition. In addition, research has shown that intuition can improve decision making. Answer (C) is incorrect because
knowing with certainty which investment is the most profitable is not possible. Answer (D) is incorrect because the term
bounded rationality refers to the inability to perceive all aspects of a situation and the tendency to simplify, not to intuitive
decision making.

0 k = 12% 1 2 3 4 5 6

CFsA -15,000 4,000 4,000 4,000 4,000 4,000 4,000


Project B:
0 k = ? 1 2 6 Years
  
CFsB -14,815 5,100 5,100 5,100
Time lines:
Project A:
0 1 2 3 4 Years
k = 12%

CFsA -25,000 13,000 13,000 13,000 13,000


NPVA = ? = 17,663 Terminal value = 5,000
CF 4 = 18,000
Project B:
0 1 2 3 4 Years
k = ?

CFsB -25,000 15,247 15,247 15,247 15,247


NPVA = NPVB = 17,663 Terminal value = 0
CF 4 = 15,247
Time line:
0 k = 18% 1 2 3 Years

-3,000 1,728 1,920 1,152


Time line:
0 1 2 3 4 5 Years
k = 12%

-45,000 7,800 10,680 7,560 5,880 -1,920


Time line:
0 1 2 3 4 5 Years
k = 9%

-40,000 9,800 11,720 9,640 8,520 15,320


Time lines:
Project A:
0 k = 14% 1 2 3 4 Years

CFsA -200,000 71,104 71,104 71,104 71,104


Project B:
0 k = 10% 1 2 3 4 Years

CFsB -200,000 0 0 146,411 146,411


Time line:
0 k = 12% 1 2 10 Years
  
-50,000 6,000 6,000 6,000
Time lines:
Project A:
0 k = 12% 1 2 3 Periods

CFsA -5,000 2,000 2,500 2,250


Project B:
0 k = 14% 1 2 3 Periods

CFsB -5,000 3,000 2,600 2,900


0 1 2 3 4
| | | | |
CFsNew Tech -1,500 -315 -315 -315 -315 x x x
Time line:
0 1 2 3 Years
k = 14%

-42,000 14,280 16,200 11,400


TV = 18,120
29,520
Time line:
0 1 2 3 Years
k = 10%

-62,000 19,920 22,800 15,600


TV = 15,680
31,280
NPV
$

B
A

Expected increase in annual net income k


ARR = 0 10%
163 Initial (or average) investment IRRB IRRA

NPV
($)

Discount rate (%)


0 16% 17% 18% 30%

NPV
$
NPV ($)
P
C

k 10% k
10% 15% 20% %

NPV NPV
$

A A

B
B

k k
0 7% 12% 15% 0 7% 12% 14%
NPV
$

X
Crossover

n
Sigma CF t k
t=1
10% IRRY 12% IRRX
t
(1 + K) .Statement e is correct; the other statements
are false. IRR can lead to conflicting decisions with NPV even with normal cash flows if the projects are mutually
exclusive. Cash outflows are discounted at the cost of capital with the MIRR method, while cash inflows are
compounded at the cost of capital. Conflicts between NPV and IRR arise when the cost of capital is below the
crossover point. The discounted payback method does correct the problem of ignoring the time value of money, but it
still does not account for cash flows beyond the payback period.

0 k = 12% 1 2 3 4 5 6

CFsA -15,000 4,000 4,000 4,000 4,000 4,000 4,000


Project B:
0 k = ? 1 2 6 Years
  
CFsB -14,815 5,100 5,100 5,100
Time lines:
Project A:
0 1 2 3 4 Years
k = 12%

CFsA -25,000 13,000 13,000 13,000 13,000


NPVA = ? = 17,663 Terminal value = 5,000
CF 4 = 18,000
Project B:
0 1 2 3 4 Years
k = ?

CFsB -25,000 15,247 15,247 15,247 15,247


NPVA = NPVB = 17,663 Terminal value = 0
CF 4 = 15,247
Time line:
0 k = 18% 1 2 3 Years

-3,000 1,728 1,920 1,152


Time line:
0 1 2 3 4 5 Years
k = 12%

-45,000 7,800 10,680 7,560 5,880 -1,920


Time line:
0 1 2 3 4 5 Years
k = 9%

-40,000 9,800 11,720 9,640 8,520 15,320


Time lines:
Project A:
0 k = 14% 1 2 3 4 Years

CFsA -200,000 71,104 71,104 71,104 71,104


Project B:
0 k = 10% 1 2 3 4 Years

CFsB -200,000 0 0 146,411 146,411


Time line:
0 k = 12% 1 2 10 Years
  
-50,000 6,000 6,000 6,000
Time lines:
Project A:
0 k = 12% 1 2 3 Periods

CFsA -5,000 2,000 2,500 2,250


Project B:
0 k = 14% 1 2 3 Periods

CFsB -5,000 3,000 2,600 2,900


0 1 2 3 4
| | | | |
CFsNew Tech -1,500 -315 -315 -315 -315 x x x
Time line:
0 1 2 3 Years
k = 14%

-42,000 14,280 16,200 11,400


TV = 18,120
29,520
Time line:
0 1 2 3 Years
k = 10%

-62,000 19,920 22,800 15,600


TV = 15,680
31,280
NPV
$

B
A

Expected increase in annual net income k


ARR = 0 10%
164 Initial (or average) investment IRRB IRRA

NPV
($)

Discount rate (%)


0 16% 17% 18% 30%
NPV
$
NPV ($)
P
C

k 10% k
10% 15% 20% %

NPV NPV
$

A A

B
B

k k
0 7% 12% 15% 0 7% 12% 14%

NPV
$

X
Crossover

n
Sigma CF t k
t=1
10% IRRY 12% IRRX
t
(1 + K) .Statements a and c are correct; therefore,
statement d is the correct choice. The discounted payback method still ignores cash flows after the payback period.

0 k = 12% 1 2 3 4 5 6

CFsA -15,000 4,000 4,000 4,000 4,000 4,000 4,000


Project B:
0 k = ? 1 2 6 Years
  
CFsB -14,815 5,100 5,100 5,100
Time lines:
Project A:
0 1 2 3 4 Years
k = 12%

CFsA -25,000 13,000 13,000 13,000 13,000


NPVA = ? = 17,663 Terminal value = 5,000
CF 4 = 18,000
Project B:
0 1 2 3 4 Years
k = ?

CFsB -25,000 15,247 15,247 15,247 15,247


NPVA = NPVB = 17,663 Terminal value = 0
CF 4 = 15,247
Time line:
0 k = 18% 1 2 3 Years

-3,000 1,728 1,920 1,152


Time line:
0 1 2 3 4 5 Years
k = 12%

-45,000 7,800 10,680 7,560 5,880 -1,920


Time line:
0 1 2 3 4 5 Years
k = 9%

-40,000 9,800 11,720 9,640 8,520 15,320


Time lines:
Project A:
0 k = 14% 1 2 3 4 Years

CFsA -200,000 71,104 71,104 71,104 71,104


Project B:
0 k = 10% 1 2 3 4 Years

CFsB -200,000 0 0 146,411 146,411


Time line:
0 k = 12% 1 2 10 Years
  
-50,000 6,000 6,000 6,000
Time lines:
Project A:
0 k = 12% 1 2 3 Periods

CFsA -5,000 2,000 2,500 2,250


Project B:
0 k = 14% 1 2 3 Periods

CFsB -5,000 3,000 2,600 2,900


0 1 2 3 4
| | | | |
CFsNew Tech -1,500 -315 -315 -315 -315 x x x
Time line:
0 1 2 3 Years
k = 14%

-42,000 14,280 16,200 11,400


TV = 18,120
29,520
Time line:
0 1 2 3 Years
k = 10%

-62,000 19,920 22,800 15,600


TV = 15,680
31,280
NPV
$

B
A

Expected increase in annual net income k


ARR = 0 10%
165 Initial (or average) investment IRRB IRRA

NPV
($)

Discount rate (%)


0 16% 17% 18% 30%

NPV
$
NPV ($)
P
C

k 10% k
10% 15% 20% %

NPV NPV
$

A A

B
B

k k
0 7% 12% 15% 0 7% 12% 14%
NPV
$

X
Crossover

n
Sigma CF t k
t=1
10% IRRY 12% IRRX
t
(1 + K) .Statement a is correct; the other statements
are false. Multiple IRRs can occur only for projects with nonnormal cash flows. Mutually exclusive projects imply that
only one project should be chosen. The project with the highest NPV should be chosen.

0 k = 12% 1 2 3 4 5 6

CFsA -15,000 4,000 4,000 4,000 4,000 4,000 4,000


Project B:
0 k = ? 1 2 6 Years
  
CFsB -14,815 5,100 5,100 5,100
Time lines:
Project A:
0 1 2 3 4 Years
k = 12%

CFsA -25,000 13,000 13,000 13,000 13,000


NPVA = ? = 17,663 Terminal value = 5,000
CF 4 = 18,000
Project B:
0 1 2 3 4 Years
k = ?

CFsB -25,000 15,247 15,247 15,247 15,247


NPVA = NPVB = 17,663 Terminal value = 0
CF 4 = 15,247
Time line:
0 1 2 3 Years
k = 18%

-3,000 1,728 1,920 1,152


Time line:
0 1 2 3 4 5 Years
k = 12%

-45,000 7,800 10,680 7,560 5,880 -1,920


Time line:
0 k = 9% 1 2 3 4 5 Years

-40,000 9,800 11,720 9,640 8,520 15,320


Time lines:
Project A:
0 k = 14% 1 2 3 4 Years

CFsA -200,000 71,104 71,104 71,104 71,104


Project B:
0 k = 10% 1 2 3 4 Years

CFsB -200,000 0 0 146,411 146,411


Time line:
0 k = 12% 1 2 10 Years
  
-50,000 6,000 6,000 6,000
Time lines:
Project A:
0 k = 12% 1 2 3 Periods

CFsA -5,000 2,000 2,500 2,250


Project B:
0 k = 14% 1 2 3 Periods

CFsB -5,000 3,000 2,600 2,900


0 1 2 3 4
| | | | |
CFsNew Tech -1,500 -315 -315 -315 -315 x x x
Time line:
0 1 2 3 Years
k = 14%

-42,000 14,280 16,200 11,400


TV = 18,120
29,520
Time line:
0 1 2 3 Years
k = 10%

-62,000 19,920 22,800 15,600


TV = 15,680
31,280
NPV
$

B
A

Expected increase in annual net income k


ARR = 0 10%
166 Initial (or average) investment IRRB IRRA

NPV
($)

Discount rate (%)


0 16% 17% 18% 30%
NPV
$
NPV ($)
P
C

k 10% k
10% 15% 20% %

NPV NPV
$

A A

B
B

k k
0 7% 12% 15% 0 7% 12% 14%

NPV
$

X
Crossover

n
Sigma CF t k
t=1
10% IRRY 12% IRRX
t
(1 + K) .Statement a is correct; the other statements
are false. Sketch the profiles. From the information given, D has the higher IRR. The project’s scale cannot be
determined from the information given. As C’s NPV declines more rapidly with an increase in rates, this implies that
more of the cash flows are coming later on. So C would have a slower payback than D.

0 k = 12% 1 2 3 4 5 6

CFsA -15,000 4,000 4,000 4,000 4,000 4,000 4,000


Project B:
0 k = ? 1 2 6 Years
  
CFsB -14,815 5,100 5,100 5,100
Time lines:
Project A:
0 1 2 3 4 Years
k = 12%

CFsA -25,000 13,000 13,000 13,000 13,000


NPVA = ? = 17,663 Terminal value = 5,000
CF 4 = 18,000
Project B:
0 1 2 3 4 Years
k = ?

CFsB -25,000 15,247 15,247 15,247 15,247


NPVA = NPVB = 17,663 Terminal value = 0
CF 4 = 15,247
Time line:
0 k = 18% 1 2 3 Years

-3,000 1,728 1,920 1,152


Time line:
0 1 2 3 4 5 Years
k = 12%

-45,000 7,800 10,680 7,560 5,880 -1,920


Time line:
0 1 2 3 4 5 Years
k = 9%

-40,000 9,800 11,720 9,640 8,520 15,320


Time lines:
Project A:
0 k = 14% 1 2 3 4 Years

CFsA -200,000 71,104 71,104 71,104 71,104


Project B:
0 k = 10% 1 2 3 4 Years

CFsB -200,000 0 0 146,411 146,411


Time line:
0 k = 12% 1 2 10 Years
  
-50,000 6,000 6,000 6,000
Time lines:
Project A:
0 k = 12% 1 2 3 Periods

CFsA -5,000 2,000 2,500 2,250


Project B:
0 k = 14% 1 2 3 Periods

CFsB -5,000 3,000 2,600 2,900


0 1 2 3 4
| | | | |
CFsNew Tech -1,500 -315 -315 -315 -315 x x x
Time line:
0 1 2 3 Years
k = 14%

-42,000 14,280 16,200 11,400


TV = 18,120
29,520
Time line:
0 1 2 3 Years
k = 10%

-62,000 19,920 22,800 15,600


TV = 15,680
31,280
NPV
$

B
A

Expected increase in annual net income k


ARR = 0 10%
167 Initial (or average) investment IRRB IRRA

NPV
($)

Discount rate (%)


0 16% 17% 18% 30%

NPV
$
NPV ($)
P
C

k 10% k
10% 15% 20% %

NPV NPV
$

A A

B
B

k k
0 7% 12% 15% 0 7% 12% 14%
NPV
$

X
Crossover

n
Sigma CF t k
t=1
10% IRRY 12% IRRX
t
(1 + K) .Answer (D) is correct. MACRS is an
accelerated method of depreciation under which depreciation expense will be greater during the early years of an
asset's life. Thus, the outflows for income taxes will be less in the early years, but greater in the later years, and the NPV
(present value of net cash inflows - investment) will be increased. The profitability index (present value of net cash
inflows ÷ the investment) must increase if the NPV increases.
Answer (A) is incorrect because the NPV will increase. The present value of the net inflows will increase with no change
in the investment. Answer (B) is incorrect because the IRR will increase. Deferring expenses to later years increases the
discount rate needed to reduce the NPV to $0. Answer (C) is incorrect because the payback period will be shortened.
Switching to MACRS defers expenses and increases cash flows early in the project's life.

0 k = 12% 1 2 3 4 5 6

CFsA -15,000 4,000 4,000 4,000 4,000 4,000 4,000


Project B:
0 k = ? 1 2 6 Years
  
CFsB -14,815 5,100 5,100 5,100
Time lines:
Project A:
0 1 2 3 4 Years
k = 12%

CFsA -25,000 13,000 13,000 13,000 13,000


NPVA = ? = 17,663 Terminal value = 5,000
CF 4 = 18,000
Project B:
0 1 2 3 4 Years
k = ?

CFsB -25,000 15,247 15,247 15,247 15,247


NPVA = NPVB = 17,663 Terminal value = 0
CF 4 = 15,247
Time line:
0 1 2 3 Years
k = 18%

-3,000 1,728 1,920 1,152


Time line:
0 1 2 3 4 5 Years
k = 12%

-45,000 7,800 10,680 7,560 5,880 -1,920


Time line:
0 k = 9% 1 2 3 4 5 Years

-40,000 9,800 11,720 9,640 8,520 15,320


Time lines:
Project A:
0 k = 14% 1 2 3 4 Years

CFsA -200,000 71,104 71,104 71,104 71,104


Project B:
0 k = 10% 1 2 3 4 Years

CFsB -200,000 0 0 146,411 146,411


Time line:
0 k = 12% 1 2 10 Years
  
-50,000 6,000 6,000 6,000
Time lines:
Project A:
0 k = 12% 1 2 3 Periods

CFsA -5,000 2,000 2,500 2,250


Project B:
0 k = 14% 1 2 3 Periods

CFsB -5,000 3,000 2,600 2,900


0 1 2 3 4
| | | | |
CFsNew Tech -1,500 -315 -315 -315 -315 x x x
Time line:
0 1 2 3 Years
k = 14%

-42,000 14,280 16,200 11,400


TV = 18,120
29,520
Time line:
0 1 2 3 Years
k = 10%

-62,000 19,920 22,800 15,600


TV = 15,680
31,280
NPV
$

B
A

Expected increase in annual net income k


ARR = 0 10%
168 Initial (or average) investment IRRB IRRA

NPV
($)

Discount rate (%)


0 16% 17% 18% 30%

NPV
$
NPV ($)
P
C

k 10% k
10% 15% 20% %

NPV NPV
$

A A

B
B n
Sigma CF t
k k t=1
0 7% 12% 15% 0 7% 12% 14%
(1 + K)t
NPV
$

X
Crossover

k
10% IRRY 12% IRRX
12A-. Depreciation cash flows Answer: c Diff: M
0 k = 12% 1 2 3 4 5 6

CFsA -15,000 4,000 4,000 4,000 4,000 4,000 4,000


Project B:
0 k = ? 1 2 6 Years
  
CFsB -14,815 5,100 5,100 5,100
Time lines:
Project A:
0 1 2 3 4 Years
k = 12%

CFsA -25,000 13,000 13,000 13,000 13,000


NPVA = ? = 17,663 Terminal value = 5,000
CF 4 = 18,000
Project B:
0 1 2 3 4 Years
k = ?

CFsB -25,000 15,247 15,247 15,247 15,247


NPVA = NPVB = 17,663 Terminal value = 0
CF 4 = 15,247
Time line:
0 1 2 3 Years
k = 18%

-3,000 1,728 1,920 1,152


Time line:
0 1 2 3 4 5 Years
k = 12%

-45,000 7,800 10,680 7,560 5,880 -1,920


Time line:
0 k = 9% 1 2 3 4 5 Years

-40,000 9,800 11,720 9,640 8,520 15,320


Time lines:
Project A:
0 k = 14% 1 2 3 4 Years

CFsA -200,000 71,104 71,104 71,104 71,104


Project B:
0 k = 10% 1 2 3 4 Years

CFsB -200,000 0 0 146,411 146,411


Time line:
0 k = 12% 1 2 10 Years
  
-50,000 6,000 6,000 6,000
Time lines:
Project A:
0 k = 12% 1 2 3 Periods

CFsA -5,000 2,000 2,500 2,250


Project B:
0 k = 14% 1 2 3 Periods

CFsB -5,000 3,000 2,600 2,900


0 1 2 3 4
| | | | |
CFsNew Tech -1,500 -315 -315 -315 -315 x x x
Time line:
0 1 2 3 Years
k = 14%

-42,000 14,280 16,200 11,400


TV = 18,120
29,520
Time line:
0 1 2 3 Years
k = 10%

-62,000 19,920 22,800 15,600


TV = 15,680
31,280
NPV
$

B
A

Expected increase in annual net income k


ARR = 0 10%
169 Initial (or average) investment IRRB IRRA

NPV
($)

Discount rate (%)


0 16% 17% 18% 30%
NPV
$
NPV ($)
P
C

k 10% k
10% 15% 20% %

NPV NPV
$

A A

B
B

k k
0 7% 12% 15% 0 7% 12% 14%

NPV
$

X
Crossover

n
Sigma CF t k
t=1
10% IRRY 12% IRRX
t
(1 + K) .Answer (D) is correct. MACRS for assets with
lives of 10 years or less is based on the 200% declining-balance method of depreciation. Thus, an asset with a 3-year
life would have a straight-line rate of 33-1/3%, or a double-declining-balance rate of 66-2/3%.
Answer (A) is incorrect because the straight-line method uses the same percentage each year during an asset's life, but
MACRS uses various percentages. Answer (B) is incorrect because MACRS is unrelated to the units-of-production
method. Answer (C) is incorrect because MACRS is unrelated to SYD depreciation.

0 k = 12% 1 2 3 4 5 6

CFsA -15,000 4,000 4,000 4,000 4,000 4,000 4,000


Project B:
0 k = ? 1 2 6 Years
  
CFsB -14,815 5,100 5,100 5,100
Time lines:
Project A:
0 1 2 3 4 Years
k = 12%

CFsA -25,000 13,000 13,000 13,000 13,000


NPVA = ? = 17,663 Terminal value = 5,000
CF 4 = 18,000
Project B:
0 1 2 3 4 Years
k = ?

CFsB -25,000 15,247 15,247 15,247 15,247


NPVA = NPVB = 17,663 Terminal value = 0
CF 4 = 15,247
Time line:
0 k = 18% 1 2 3 Years

-3,000 1,728 1,920 1,152


Time line:
0 1 2 3 4 5 Years
k = 12%

-45,000 7,800 10,680 7,560 5,880 -1,920


Time line:
0 1 2 3 4 5 Years
k = 9%

-40,000 9,800 11,720 9,640 8,520 15,320


Time lines:
Project A:
0 k = 14% 1 2 3 4 Years

CFsA -200,000 71,104 71,104 71,104 71,104


Project B:
0 k = 10% 1 2 3 4 Years

CFsB -200,000 0 0 146,411 146,411


Time line:
0 k = 12% 1 2 10 Years
  
-50,000 6,000 6,000 6,000
Time lines:
Project A:
0 k = 12% 1 2 3 Periods

CFsA -5,000 2,000 2,500 2,250


Project B:
0 k = 14% 1 2 3 Periods

CFsB -5,000 3,000 2,600 2,900


0 1 2 3 4
| | | | |
CFsNew Tech -1,500 -315 -315 -315 -315 x x x
Time line:
0 1 2 3 Years
k = 14%

-42,000 14,280 16,200 11,400


TV = 18,120
29,520
Time line:
0 1 2 3 Years
k = 10%

-62,000 19,920 22,800 15,600


TV = 15,680
31,280
NPV
$

B
A

Expected increase in annual net income k


ARR = 0 10%
170 Initial (or average) investment IRRB IRRA

NPV
($)

Discount rate (%)


0 16% 17% 18% 30%

NPV
$
NPV ($)
P
C

k 10% k
10% 15% 20% %

NPV NPV
$

A A

B
B

k k
0 7% 12% 15% 0 7% 12% 14%
NPV
$

X
Crossover

n
Sigma CF t k
t=1
10% IRRY 12% IRRX
t
(1 + K) .Answer (A) is correct. For tax purposes,
straight-line depreciation is an alternative to the MACRS method. Both methods will result in the same total depreciation
over the life of the asset; however, MACRS will result in greater depreciation in the early years of the asset's life
because it is an accelerated method. Given that MACRS results in larger depreciation deductions in the early years,
taxes will be lower in the early years and higher in the later years. Because the incremental benefits will be discounted
over a shorter period than the incremental depreciation costs, MACRS is preferable to the straight-line method.
Answer (B) is incorrect because both methods will produce the same total depreciation over the life of the asset. Answer
(C) is incorrect because both methods will produce the same total tax payments (assuming rates do not change).
However, given that the tax payments will be lower in the early years under MACRS, discounting for the time value of
money makes the straight-line alternative less advantageous. Answer (D) is incorrect because both methods will
produce the same total depreciation over the life of the asset.

0 k = 12% 1 2 3 4 5 6

CFsA -15,000 4,000 4,000 4,000 4,000 4,000 4,000


Project B:
0 k = ? 1 2 6 Years
  
CFsB -14,815 5,100 5,100 5,100
Time lines:
Project A:
0 1 2 3 4 Years
k = 12%

CFsA -25,000 13,000 13,000 13,000 13,000


NPVA = ? = 17,663 Terminal value = 5,000
CF 4 = 18,000
Project B:
0 1 2 3 4 Years
k = ?

CFsB -25,000 15,247 15,247 15,247 15,247


NPVA = NPVB = 17,663 Terminal value = 0
CF 4 = 15,247
Time line:
0 1 2 3 Years
k = 18%

-3,000 1,728 1,920 1,152


Time line:
0 1 2 3 4 5 Years
k = 12%

-45,000 7,800 10,680 7,560 5,880 -1,920


Time line:
0 k = 9% 1 2 3 4 5 Years

-40,000 9,800 11,720 9,640 8,520 15,320


Time lines:
Project A:
0 k = 14% 1 2 3 4 Years

CFsA -200,000 71,104 71,104 71,104 71,104


Project B:
0 k = 10% 1 2 3 4 Years

CFsB -200,000 0 0 146,411 146,411


Time line:
0 k = 12% 1 2 10 Years
  
-50,000 6,000 6,000 6,000
Time lines:
Project A:
0 k = 12% 1 2 3 Periods

CFsA -5,000 2,000 2,500 2,250


Project B:
0 k = 14% 1 2 3 Periods

CFsB -5,000 3,000 2,600 2,900


0 1 2 3 4
| | | | |
CFsNew Tech -1,500 -315 -315 -315 -315 x x x
Time line:
0 1 2 3 Years
k = 14%

-42,000 14,280 16,200 11,400


TV = 18,120
29,520
Time line:
0 1 2 3 Years
k = 10%

-62,000 19,920 22,800 15,600


TV = 15,680
31,280
NPV
$

B
A

Expected increase in annual net income k


ARR = 0 10%
171 Initial (or average) investment IRRB IRRA

NPV
($)

Discount rate (%)


0 16% 17% 18% 30%
NPV
$
NPV ($)
P
C

k 10% k
10% 15% 20% %

NPV NPV
$

A A

B
B

k k
0 7% 12% 15% 0 7% 12% 14%
NPV
$

X
Crossover

n
Sigma CF t k
t=1
10% IRRY 12% IRRX
t
(1 + K) .Taxes on gain on saleAnswer: b Diff:

E N

When the machine is sold the total accumulated depreciation on it is: (0.2 + 0.32 + 0.19)  $1,000,000 = $710,000.
The book value of the equipment is: $1,000,000 - $710,000 = $290,000. The machine is sold for $400,000, so the
gain is $400,000 - $290,000 = $110,000. Taxes are calculated as $110,000  0.4 = $44,000.
0 k = 12% 1 2 3 4 5 6

CFsA -15,000 4,000 4,000 4,000 4,000 4,000 4,000


Project B:
0 k = ? 1 2 6 Years
  
CFsB -14,815 5,100 5,100 5,100
Time lines:
Project A:
0 1 2 3 4 Years
k = 12%

CFsA -25,000 13,000 13,000 13,000 13,000


NPVA = ? = 17,663 Terminal value = 5,000
CF 4 = 18,000
Project B:
0 1 2 3 4 Years
k = ?

CFsB -25,000 15,247 15,247 15,247 15,247


NPVA = NPVB = 17,663 Terminal value = 0
CF 4 = 15,247
Time line:
0 1 2 3 Years
k = 18%

-3,000 1,728 1,920 1,152


Time line:
0 1 2 3 4 5 Years
k = 12%

-45,000 7,800 10,680 7,560 5,880 -1,920


Time line:
0 k = 9% 1 2 3 4 5 Years

-40,000 9,800 11,720 9,640 8,520 15,320


Time lines:
Project A:
0 k = 14% 1 2 3 4 Years

CFsA -200,000 71,104 71,104 71,104 71,104


Project B:
0 k = 10% 1 2 3 4 Years

CFsB -200,000 0 0 146,411 146,411


Time line:
0 k = 12% 1 2 10 Years
  
-50,000 6,000 6,000 6,000
Time lines:
Project A:
0 k = 12% 1 2 3 Periods

CFsA -5,000 2,000 2,500 2,250


Project B:
0 k = 14% 1 2 3 Periods

CFsB -5,000 3,000 2,600 2,900


0 1 2 3 4
| | | | |
CFsNew Tech -1,500 -315 -315 -315 -315 x x x
Time line:
0 1 2 3 Years
k = 14%

-42,000 14,280 16,200 11,400


TV = 18,120
29,520
Time line:
0 1 2 3 Years
k = 10%

-62,000 19,920 22,800 15,600


TV = 15,680
31,280
NPV
$

B
A

Expected increase in annual net income k


ARR = 0 10%
172 Initial (or average) investment IRRB IRRA

NPV
($)

Discount rate (%)


0 16% 17% 18% 30%
NPV
$
NPV ($)
P
C

k 10% k
10% 15% 20% %

NPV NPV
$

A A

B
B n
Sigma CF t
k k t=1
0 7% 12% 15% 0 7% 12% 14%
(1 + K)t
NPV
$

X
Crossover

k
10% IRRY 12% IRRX

. Risk-adjusted discount rate Answer: c Diff: E


ks = 10% + (16% - 10%)1.5 = 10% + 9% = 19%.
Original IRR = 21%. 21% - Risk adjustment 1% = 20%.
Risk adjusted IRR = 20% > ks = 19%.
0 k = 12% 1 2 3 4 5 6

CFsA -15,000 4,000 4,000 4,000 4,000 4,000 4,000


Project B:
0 k = ? 1 2 6 Years
  
CFsB -14,815 5,100 5,100 5,100
Time lines:
Project A:
0 1 2 3 4 Years
k = 12%

CFsA -25,000 13,000 13,000 13,000 13,000


NPVA = ? = 17,663 Terminal value = 5,000
CF 4 = 18,000
Project B:
0 1 2 3 4 Years
k = ?

CFsB -25,000 15,247 15,247 15,247 15,247


NPVA = NPVB = 17,663 Terminal value = 0
CF 4 = 15,247
Time line:
0 k = 18% 1 2 3 Years

-3,000 1,728 1,920 1,152


Time line:
0 1 2 3 4 5 Years
k = 12%

-45,000 7,800 10,680 7,560 5,880 -1,920


Time line:
0 1 2 3 4 5 Years
k = 9%

-40,000 9,800 11,720 9,640 8,520 15,320


Time lines:
Project A:
0 k = 14% 1 2 3 4 Years

CFsA -200,000 71,104 71,104 71,104 71,104


Project B:
0 k = 10% 1 2 3 4 Years

CFsB -200,000 0 0 146,411 146,411


Time line:
0 k = 12% 1 2 10 Years
  
-50,000 6,000 6,000 6,000
Time lines:
Project A:
0 k = 12% 1 2 3 Periods

CFsA -5,000 2,000 2,500 2,250


Project B:
0 k = 14% 1 2 3 Periods

CFsB -5,000 3,000 2,600 2,900


0 1 2 3 4
| | | | |
CFsNew Tech -1,500 -315 -315 -315 -315 x x x
Time line:
0 1 2 3 Years
k = 14%

-42,000 14,280 16,200 11,400


TV = 18,120
29,520
Time line:
0 1 2 3 Years
k = 10%

-62,000 19,920 22,800 15,600


TV = 15,680
31,280
NPV
$

B
A

Expected increase in annual net income k


ARR = 0 10%
173 Initial (or average) investment IRRB IRRA

NPV
($)

Discount rate (%)


0 16% 17% 18% 30%

NPV
$
NPV ($)
P
C

k 10% k
10% 15% 20% %

NPV NPV
$

A A

B
B n
Sigma CF t
k k t=1
0 7% 12% 15% 0 7% 12% 14%
(1 + K)t
NPV
$

X
Crossover

k
10% IRRY 12% IRRX

. Risk-adjusted discount rate Answer: b Diff: M


Time lines:
Project A:
0 k = 12% 1 2 3 4 5 6

CFsA -15,000 4,000 4,000 4,000 4,000 4,000 4,000


NPVA = ? = 3,978.60 5,000 Salvage value
Terminal CF = 9,000
Project B:
0 k = ? 1 2 6 Years
  
CFsB -14,815 5,100 5,100 5,100
NPVB = NPVA = 3,978.60 0 Salvage value
Terminal CF = 5,100

Tabular solution:
Solve for the NPV of Project A, which is also the NPV of Project B at some k
= ?
NPVA = -$15,000 + $4,000(PVIFA12%,6) + $5,000(PVIF12%,6)
= -$15,000 + $4,000(4.1114) + $5,000(0.5066) = $3,978.60.
Solve for kB
NPVB = $3,978.60 = -$14,815 + $5,100(PVIFAk,6)
$18,793.60 = $5,100(PVIFAk,6)
PVIFAk,6 = 3.68502.
Look across the row for 6 years in the PVIFA table. The factor for 16 percent
is 3.6847; therefore, the risk-adjusted rate for Project B is approximately
16 percent.

Financial calculator solution:


A: Inputs: CF0 = -15,000; CF1 = 4,000; Nj = 5; CF2 = 9,000; I = 12.
Output: NPV = $3,978.78.
B: Inputs: CF0 = -18,793.78; CF1 = 5,100; Nj = 6.
Output: IRR = 15.997%  16%.
Time lines:
Project A:
0 1 2 3 4 Years
k = 12%

CFsA -25,000 13,000 13,000 13,000 13,000


NPVA = ? = 17,663 Terminal value = 5,000
CF 4 = 18,000
Project B:
0 1 2 3 4 Years
k = ?

CFsB -25,000 15,247 15,247 15,247 15,247


NPVA = NPVB = 17,663 Terminal value = 0
CF 4 = 15,247
Time line:
0 k = 18% 1 2 3 Years

-3,000 1,728 1,920 1,152


Time line:
0 1 2 3 4 5 Years
k = 12%

-45,000 7,800 10,680 7,560 5,880 -1,920


Time line:
0 1 2 3 4 5 Years
k = 9%

-40,000 9,800 11,720 9,640 8,520 15,320


Time lines:
Project A:
0 k = 14% 1 2 3 4 Years

CFsA -200,000 71,104 71,104 71,104 71,104


Project B:
0 k = 10% 1 2 3 4 Years

CFsB -200,000 0 0 146,411 146,411


Time line:
0 k = 12% 1 2 10 Years
  
-50,000 6,000 6,000 6,000
Time lines:
Project A:
0 k = 12% 1 2 3 Periods

CFsA -5,000 2,000 2,500 2,250


Project B:
0 k = 14% 1 2 3 Periods

CFsB -5,000 3,000 2,600 2,900


0 1 2 3 4
| | | | |
CFsNew Tech -1,500 -315 -315 -315 -315 x x x
Time line:
0 1 2 3 Years
k = 14%

-42,000 14,280 16,200 11,400


TV = 18,120
29,520
Time line:
0 1 2 3 Years
k = 10%

-62,000 19,920 22,800 15,600


TV = 15,680
31,280
NPV
$

B
A

Expected increase in annual net income k


ARR = 0 10%
174 Initial (or average) investment IRRB IRRA

NPV
($)

Discount rate (%)


0 16% 17% 18% 30%

NPV
$
NPV ($)
P
C

k 10% k
10% 15% 20% %

NPV NPV
$

A A

B
B n
Sigma CF t
k k t=1
0 7% 12% 15% 0 7% 12% 14%
(1 + K)t
NPV
$

X
Crossover

k
10% IRRY 12% IRRX

0 k = 12% 1 2 3 4 5 6

CFsA -15,000 4,000 4,000 4,000 4,000 4,000 4,000


Project B:
0 k = ? 1 2 6 Years
  
CFsB -14,815 5,100 5,100 5,100 . Risk-
adjusted discount rate Answer: e Diff: T
Time lines:
Project A:
0 1 2 3 4 Years
k = 12%

CFsA -25,000 13,000 13,000 13,000 13,000


NPVA = ? = 17,663 Terminal value = 5,000
CF 4 = 18,000
Project B:
0 1 2 3 4 Years
k = ?

CFsB -25,000 15,247 15,247 15,247 15,247


NPVA = NPVB = 17,663 Terminal value = 0
CF 4 = 15,247

Tabular solution:
NPVA = -$25,000 + $13,000(PVIFA12%,3) + $18,000(PVIF12%,4)
= -$25,000 + $13,000(2.4018) + $18,000(0.6355) = $17,662.40.
NPVB = $17,662.40 = -$25,000 + $15,247(PVIFAk,4)
$42,662.40 = $15,247(PVIFAk,4)
(PVIFAk,4) = 2.79808
k  16%.

Financial calculator solution:


A: Inputs: CF0 = -25,000; CF1 = 13,000; Nj = 3; CF2 = 18,000; I = 12.
Output: NPVA = 17,663.13.
B: Inputs: CF0 = -42,663.13; CF1 = 15,247; Nj = 4.
Output: IRR = 16.0% = k.
Time line:
0 k = 18% 1 2 3 Years

-3,000 1,728 1,920 1,152


Time line:
0 1 2 3 4 5 Years
k = 12%

-45,000 7,800 10,680 7,560 5,880 -1,920


Time line:
0 k = 9% 1 2 3 4 5 Years

-40,000 9,800 11,720 9,640 8,520 15,320


Time lines:
Project A:
0 k = 14% 1 2 3 4 Years

CFsA -200,000 71,104 71,104 71,104 71,104


Project B:
0 k = 10% 1 2 3 4 Years

CFsB -200,000 0 0 146,411 146,411


Time line:
0 k = 12% 1 2 10 Years
  
-50,000 6,000 6,000 6,000
Time lines:
Project A:
0 k = 12% 1 2 3 Periods

CFsA -5,000 2,000 2,500 2,250


Project B:
0 k = 14% 1 2 3 Periods

CFsB -5,000 3,000 2,600 2,900


0 1 2 3 4
| | | | |
CFsNew Tech -1,500 -315 -315 -315 -315 x x x
Time line:
0 1 2 3 Years
k = 14%

-42,000 14,280 16,200 11,400


TV = 18,120
29,520
Time line:
0 1 2 3 Years
k = 10%

-62,000 19,920 22,800 15,600


TV = 15,680
31,280
NPV
$

B
A

Expected increase in annual net income k


ARR = 0 10%
175 Initial (or average) investment IRRB IRRA

NPV
($)

Discount rate (%)


0 16% 17% 18% 30%

NPV
$
NPV ($)
P
C

k 10% k
10% 15% 20% %

NPV NPV
$

A A

B
B n
Sigma CF t
k k t=1
0 7% 12% 15% 0 7% 12% 14%
(1 + K)t
NPV
$

X
Crossover

k
10% IRRY 12% IRRX

0 k = 12% 1 2 3 4 5 6

CFsA -15,000 4,000 4,000 4,000 4,000 4,000 4,000


Project B:
0 k = ? 1 2 6 Years
  
CFsB -14,815 5,100 5,100 5,100
Time lines:
Project A:
0 1 2 3 4 Years
k = 12%

CFsA -25,000 13,000 13,000 13,000 13,000


NPVA = ? = 17,663 Terminal value = 5,000
CF 4 = 18,000
Project B:
0 1 2 3 4 Years
k = ?

CFsB -25,000 15,247 15,247 15,247 15,247


NPVA = NPVB = 17,663 Terminal value = 0
CF 4 = 15,247

. New project NPVAnswer: e Diff: M


Time line:
0 1 2 3 Years
k = 18%

-3,000 1,728 1,920 1,152


NPV = ?

Project analysis worksheet:


I Initial outlay
1) Cost ($4,000)
2) Decrease in NOWC 1,000
3) Total net investment ($3,000)
II Operating flows: Year: 0 1 2 3
4) EBT and depreciation $2,000 $2,000 $2,000
5) Earnings after taxes
(line 4  0.6) 1,200 1,200 1,200
6) Depreciation (from table) 1,320 1,800 600
7) Tax savings from
depreciation (5  0.4) 528 720 240
8) Net operating flows
(line 5 + 7) $1,728 $1,920 $1,440
III Terminal year cash flows:
9) Estimated salvage value $1,000
10) Tax on salvage value
((1,000 - 280)  0.4) (288)
11) Return of NOWC (1,000)
12) Total termination CFs ($ 288)
IV Net cash flows:
13) Total net cash flows ($3,000) $1,728 $1,920 $1,152

Tabular solution:
NPV = -$3,000 + $1,728(PVIF18%,1) + $1,920(PVIF18%,2) + $1,152(PVIF18%,3)
= -$3,000 + $1,728(0.8475) + $1,920(0.7182) + $1,152(0.6086) = $544.53.

Financial calculator solution:


Inputs: CF0 = -3000; CF1 = 1728; CF2 = 1920; CF3 = 1152; I = 18.
Output: NPV = $544.46  $544.
Time line:
0 1 2 3 4 5 Years
k = 12%

-45,000 7,800 10,680 7,560 5,880 -1,920


Time line:
0 1 2 3 4 5 Years
k = 9%

-40,000 9,800 11,720 9,640 8,520 15,320


Time lines:
Project A:
0 k = 14% 1 2 3 4 Years

CFsA -200,000 71,104 71,104 71,104 71,104


Project B:
0 k = 10% 1 2 3 4 Years

CFsB -200,000 0 0 146,411 146,411


Time line:
0 k = 12% 1 2 10 Years
  
-50,000 6,000 6,000 6,000
Time lines:
Project A:
0 k = 12% 1 2 3 Periods

CFsA -5,000 2,000 2,500 2,250


Project B:
0 k = 14% 1 2 3 Periods

CFsB -5,000 3,000 2,600 2,900


0 1 2 3 4
| | | | |
CFsNew Tech -1,500 -315 -315 -315 -315 x x x
Time line:
0 1 2 3 Years
k = 14%

-42,000 14,280 16,200 11,400


TV = 18,120
29,520
Time line:
0 1 2 3 Years
k = 10%

-62,000 19,920 22,800 15,600


TV = 15,680
31,280
NPV
$

B
A

Expected increase in annual net income k


ARR = 0 10%
176 Initial (or average) investment IRRB IRRA

NPV
($)

Discount rate (%)


0 16% 17% 18% 30%

NPV
$
NPV ($)
P
C

k 10% k
10% 15% 20% %

NPV NPV
$

A A

B
B n
Sigma CF t
k k t=1
0 7% 12% 15% 0 7% 12% 14%
(1 + K)t
NPV
$

X
Crossover

k
10% IRRY 12% IRRX

0 k = 12% 1 2 3 4 5 6

CFsA -15,000 4,000 4,000 4,000 4,000 4,000 4,000


Project B:
0 k = ? 1 2 6 Years
  
CFsB -14,815 5,100 5,100 5,100
Time lines:
Project A:
0 1 2 3 4 Years
k = 12%

CFsA -25,000 13,000 13,000 13,000 13,000


NPVA = ? = 17,663 Terminal value = 5,000
CF 4 = 18,000
Project B:
0 1 2 3 4 Years
k = ?

CFsB -25,000 15,247 15,247 15,247 15,247


NPVA = NPVB = 17,663 Terminal value = 0
CF 4 = 15,247
Time line:
0 1 2 3 Years
k = 18%

-3,000 1,728 1,920 1,152

. New project NPV Answer: d Diff: M


Time line:
0 1 2 3 4 5 Years
k = 12%

-45,000 7,800 10,680 7,560 5,880 -1,920


NPV = ?

Depreciation cash flows:


MACRS Depreciable Annual
Year Percent Basis Depreciation
1 0.20 $60,000 $12,000
2 0.32 60,000 19,200
3 0.19 60,000 11,400
4 0.12 60,000 7,200
5 0.11 60,000 6,600
6 0.06 60,000 3,600
$60,000

Project analysis worksheet:


I Initial outlay
1) Machine cost ($60,000)
2) Decrease in NOWC 15,000
3) Total net inv. ($45,000)
II Operating cash flows
Year: 0 1 2 3 4 5
4) Reduction in cost $ 5,000 $ 5,000 $ 5,000 $ 5,000 $ 5,000
5) After-tax dec. in cost 3,000 3,000 3,000 3,000 3,000
6) Deprec. (from table) 12,000 19,200 11,400 7,200 6,600
7) Tax savings deprec.
(line 6  0.4) 4,800 7,680 4,560 2,880 2,640
8) Net operating CFs
(line 5 + 7) $ 7,800 $10,680 $ 7,560 $ 5,880 $ 5,640
III Terminal year CFs
9) Estimated salvage value $10,000
10) Tax on salvage value
(10,000 - 3,600)(0.4) (2,560)
11) Return of NOWC (15,000)
12) Total termination CFs (7,560)
IV Net CFs
13) Total Net CFs ($45,000) $ 7,800 $10,680 $ 7,560 $ 5,880 ($ 1,920)

Tabular solution:
NPV = -$45,000 + $7,800(PVIF12%,1) + $10,680(PVIF12%,2) + $7,560(PVIF12%,3)
+ $5,880(PVIF12%,4) - $1,920(PVIF12%,5)
= -$45,000 + $7,800(0.8929) + $10,680(0.7972) + $7,560(0.7118)
+ $5,880(0.6355) - $1,920(0.5674) = -$21,492.74  -$21,493.

Financial calculator solution:


Inputs: CF0 = -45,000; CF1 = 7,800; CF2 = 10,680; CF3 = 7,560;
CF4 = 5,880; CF5 = -1,920; I = 12.
Output: NPV = -$21,493.24  $21,493.
Time line:
0 k = 9% 1 2 3 4 5 Years

-40,000 9,800 11,720 9,640 8,520 15,320


Time lines:
Project A:
0 k = 14% 1 2 3 4 Years

CFsA -200,000 71,104 71,104 71,104 71,104


Project B:
0 k = 10% 1 2 3 4 Years

CFsB -200,000 0 0 146,411 146,411


Time line:
0 k = 12% 1 2 10 Years
  
-50,000 6,000 6,000 6,000
Time lines:
Project A:
0 k = 12% 1 2 3 Periods

CFsA -5,000 2,000 2,500 2,250


Project B:
0 k = 14% 1 2 3 Periods

CFsB -5,000 3,000 2,600 2,900


0 1 2 3 4
| | | | |
CFsNew Tech -1,500 -315 -315 -315 -315 x x x
Time line:
0 1 2 3 Years
k = 14%

-42,000 14,280 16,200 11,400


TV = 18,120
29,520
Time line:
0 1 2 3 Years
k = 10%

-62,000 19,920 22,800 15,600


TV = 15,680
31,280
NPV
$

B
A

Expected increase in annual net income k


ARR = 0 10%
177 Initial (or average) investment IRRB IRRA

NPV
($)

Discount rate (%)


0 16% 17% 18% 30%

NPV
$
NPV ($)
P
C

k 10% k
10% 15% 20% %

NPV NPV
$

A A

B
B n
Sigma CF t
k k t=1
0 7% 12% 15% 0 7% 12% 14%
(1 + K)t
NPV
$

X
Crossover

k
10% IRRY 12% IRRX

0 k = 12% 1 2 3 4 5 6

CFsA -15,000 4,000 4,000 4,000 4,000 4,000 4,000


Project B:
0 k = ? 1 2 6 Years
  
CFsB -14,815 5,100 5,100 5,100
Time lines:
Project A:
0 1 2 3 4 Years
k = 12%

CFsA -25,000 13,000 13,000 13,000 13,000


NPVA = ? = 17,663 Terminal value = 5,000
CF 4 = 18,000
Project B:
0 1 2 3 4 Years
k = ?

CFsB -25,000 15,247 15,247 15,247 15,247


NPVA = NPVB = 17,663 Terminal value = 0
CF 4 = 15,247
Time line:
0 1 2 3 Years
k = 18%

-3,000 1,728 1,920 1,152


Time line:
0 1 2 3 4 5 Years
k = 12%

-45,000 7,800 10,680 7,560 5,880 -1,920

. New project NPVAnswer: b Diff: M


Time line:
0 1 2 3 4 5 Years
k = 9%

-40,000 9,800 11,720 9,640 8,520 15,320 NPV


= ?

Depreciation cash flows:


MACRS Depreciable Annual
Year Percent Basis Depreciation
1 0.20 $40,000 $ 8,000
2 0.32 40,000 12,800
3 0.19 40,000 7,600
4 0.12 40,000 4,800
5 0.11 40,000 4,400
6 0.06 40,000 2,400
$40,000
Project analysis worksheet:
I Initial outlay
1) Machine cost ($40,000)
2) Decrease in NOWC --
3) Total net inv. ($40,000)
II Operating cash flows
Year: 0 1 2 3 4 5
4) Inc. in earnings
before deprec. & tax $ 6,000 $ 6,000 $ 6,000 $ 6,000 $ 6,000
5) After-tax increase in
earnings (line 4  0.6) 3,600 3,600 3,600 3,600 3,600
6) Before tax reduction
in cost 5,000 5,000 5,000 5,000 5,000
7) After tax reduction
in cost (line 6  0.4) 3,000 3,000 3,000 3,000 3,000
8) Deprec. (from table) 8,000 12,800 7,600 4,800 4,400
9) Deprec. tax savings
(line 8  0.4) 3,200 5,120 3,040 1,920 1,760
10) Net operating CFs _______ _______ _______ _______ _______
(line 5 + 7 + 9) $ 9,800 $11,720 $ 9,640 $ 8,520 $ 8,360
III Terminal year CFs
11) Estimated salvage value $10,000
12) Tax on salvage value
(10,000 - 2,400)(0.4) (3,040)
13) Return of NOWC --
14) Total termination CFs 6,960
IV Net CFs
15) Total Net CFs ($40,000) $ 9,800 $11,720 $ 9,640 $ 8,520 $15,320

Tabular solution:
NPV = -$40,000 + $9,800(PVIF9%,1) + $11,720(PVIF9%,2) + $9,640(PVIF9%,3)
+ $8,520(PVIF9%,4) + $15,320(PVIF9%,5)
= -$40,000 + $9,800(0.9174) + $11,720(0.8417) + $9,640(0.7722)
+ $8,520(0.7084) + $15,320(0.6499) = $2,291.29  $2,292.

Financial calculator solution:


Inputs: CF0 = -40,000; CF1 = 9,800; CF2 = 11,720; CF3 = 9,640;
CF4 = 8,520; CF5 = 15,320; I = 9.
Output: NPV = $2,291.90  $2,292.
Time lines:
Project A:
0 k = 14% 1 2 3 4 Years

CFsA -200,000 71,104 71,104 71,104 71,104


Project B:
0 k = 10% 1 2 3 4 Years

CFsB -200,000 0 0 146,411 146,411


Time line:
0 k = 12% 1 2 10 Years
  
-50,000 6,000 6,000 6,000
Time lines:
Project A:
0 k = 12% 1 2 3 Periods

CFsA -5,000 2,000 2,500 2,250


Project B:
0 k = 14% 1 2 3 Periods

CFsB -5,000 3,000 2,600 2,900


0 1 2 3 4
| | | | |
CFsNew Tech -1,500 -315 -315 -315 -315 x x x
Time line:
0 1 2 3 Years
k = 14%

-42,000 14,280 16,200 11,400


TV = 18,120
29,520
Time line:
0 1 2 3 Years
k = 10%

-62,000 19,920 22,800 15,600


TV = 15,680
31,280
NPV
$

B
A

Expected increase in annual net income k


ARR = 0 10%
178 Initial (or average) investment IRRB IRRA

NPV
($)

Discount rate (%)


0 16% 17% 18% 30%
NPV
$
NPV ($)
P
C

k 10% k
10% 15% 20% %

NPV NPV
$

A A

B
B n
Sigma CF t
k k t=1
0 7% 12% 15% 0 7% 12% 14%
(1 + K)t
NPV
$

X
Crossover

k
10% IRRY 12% IRRX

0 k = 12% 1 2 3 4 5 6

CFsA -15,000 4,000 4,000 4,000 4,000 4,000 4,000

Project B:
0 k = ? 1 2 6 Years
  
CFsB -14,815 5,100 5,100 5,100

Time lines:
Project A:
0 1 2 3 4 Years
k = 12%

CFsA -25,000 13,000 13,000 13,000 13,000


NPVA = ? = 17,663 Terminal value = 5,000
CF 4 = 18,000

Project B:
0 1 2 3 4 Years
k = ?

CFsB -25,000 15,247 15,247 15,247 15,247


NPVA = NPVB = 17,663 Terminal value = 0
CF 4 = 15,247
Time line:
0 k = 18% 1 2 3 Years

-3,000 1,728 1,920 1,152

Time line:
0 1 2 3 4 5 Years
k = 12%

-45,000 7,800 10,680 7,560 5,880 -1,920

Time line:
0 k = 9% 1 2 3 4 5 Years

-40,000 9,800 11,720 9,640 8,520 15,320 .New project

NPV Answer: a Diff: M N

0 1 2 3
Equipment purchase -$600,000
NOWC -50,000

Sales increase $2,000,000 $2,000,000 $2,000,000


Operating costs 1,400,000 1,400,000 1,400,000
Operating income $ 600,000 $ 600,000 $ 600,000
Depreciation 200,000 200,000 200,000
EBIT $ 400,000 $ 400,000 $ 400,000
Taxes (35%) 140,000 140,000 140,000
EBIT(1 - T) $ 260,000 $ 260,000 $ 260,000
+Depreciation 200,000 200,000 200,000
Operating cash flow $ 460,000 $ 460,000 $ 460,000
Recovery of NOWC 50,000
Equipment sale +100,000
Taxes on sale _________ __________ __________ -35,000
Net CF -$650,000 $ 460,000 $ 460,000 $ 575,000

NPV = -$650,000 + $460,000/1.12 + $460,000/(1.12)2 + $575,000/(1.12)3


= -$650,000 + $410,714.29 + $366,709.18 + $409,273.64
= $536,697.11  $536,697.
Time lines:
Project A:
0 k = 14% 1 2 3 4 Years

CFsA -200,000 71,104 71,104 71,104 71,104


Project B:
0 k = 10% 1 2 3 4 Years

CFsB -200,000 0 0 146,411 146,411


Time line:
0 k = 12% 1 2 10 Years
  
-50,000 6,000 6,000 6,000
Time lines:
Project A:
0 k = 12% 1 2 3 Periods

CFsA -5,000 2,000 2,500 2,250


Project B:
0 k = 14% 1 2 3 Periods

CFsB -5,000 3,000 2,600 2,900


0 1 2 3 4
| | | | |
CFsNew Tech -1,500 -315 -315 -315 -315 x x x
Time line:
0 1 2 3 Years
k = 14%

-42,000 14,280 16,200 11,400


TV = 18,120
29,520
Time line:
0 1 2 3 Years
k = 10%

-62,000 19,920 22,800 15,600


TV = 15,680
31,280
NPV
$

B
A

Expected increase in annual net income k


ARR = 0 10%
179 Initial (or average) investment IRRB IRRA

NPV
($)

Discount rate (%)


0 16% 17% 18% 30%
NPV
$
NPV ($)
P
C

k 10% k
10% 15% 20% %

NPV NPV
$

A A

B
B n
Sigma CF t
k k t=1
0 7% 12% 15% 0 7% 12% 14%
(1 + K)t
NPV
$

X
Crossover

k
10% IRRY 12% IRRX

0 k = 12% 1 2 3 4 5 6

CFsA -15,000 4,000 4,000 4,000 4,000 4,000 4,000


Project B:
0 k = ? 1 2 6 Years
  
CFsB -14,815 5,100 5,100 5,100
Time lines:
Project A:
0 1 2 3 4 Years
k = 12%

CFsA -25,000 13,000 13,000 13,000 13,000


NPVA = ? = 17,663 Terminal value = 5,000
CF 4 = 18,000
Project B:
0 1 2 3 4 Years
k = ?

CFsB -25,000 15,247 15,247 15,247 15,247


NPVA = NPVB = 17,663 Terminal value = 0
CF 4 = 15,247
Time line:
0 k = 18% 1 2 3 Years

-3,000 1,728 1,920 1,152


Time line:
0 1 2 3 4 5 Years
k = 12%

-45,000 7,800 10,680 7,560 5,880 -1,920


Time line:
0 1 2 3 4 5 Years
k = 9%

-40,000 9,800 11,720 9,640 8,520 15,320


. New project NPV Answer: b Diff: M N

Year 0 1 2 3 4
Project cost -5,000,000
NOWC* -300,000

Sales $3,000,000 $4,000,000 $5,000,000 $2,000,000


Operating costs (75%) 2,250,000 3,000,000 3,750,000 1,500,000
Operating income before deprec. $ 750,000 $1,000,000 $1,250,000 $ 500,000
Depreciation 1,650,000 2,250,000 750,000 350,000
EBIT -$ 900,000 -$1,250,000 $ 500,000 $ 150,000
Taxes (40%) -360,000 -500,000 200,000 60,000
EBIT(1 - T) -$ 540,000 -$ 750,000 $ 300,000 $ 90,000
Plus: Depreciation 1,650,000 2,250,000 750,000 350,000
Operating CF $1,110,000 $1,500,000 $1,050,000 $ 440,000
Recovery of NOWC 300,000
Net CF -$5,300,000 $1,110,000 $1,500,000 $1,050,000 $ 740,000

*An increase in inventories is a use of funds for the company, and an increase in accounts payable is a source of
funds for the company. Thus, the change in net operating working capital will be $200,000 - $500,000 = -$300,000
at time 0.

NPV = -$5,300,000 + $1,110,000/1.10 + $1,500,000/(1.10)2 + $1,050,000/(1.10)3 + $740,000/(1.10)4


NPV = -$5,300,000 + $1,009,091 + $1,239,669 + $788,881 + $505,430
NPV = -$1,756,929.

Time lines:
Project A:
0 k = 14% 1 2 3 4 Years

CFsA -200,000 71,104 71,104 71,104 71,104


Project B:
0 k = 10% 1 2 3 4 Years

CFsB -200,000 0 0 146,411 146,411


Time line:
0 k = 12% 1 2 10 Years
  
-50,000 6,000 6,000 6,000
Time lines:
Project A:
0 k = 12% 1 2 3 Periods

CFsA -5,000 2,000 2,500 2,250


Project B:
0 k = 14% 1 2 3 Periods

CFsB -5,000 3,000 2,600 2,900


0 1 2 3 4
| | | | |
CFsNew Tech -1,500 -315 -315 -315 -315 x x x
Time line:
0 1 2 3 Years
k = 14%

-42,000 14,280 16,200 11,400


TV = 18,120
29,520
Time line:
0 1 2 3 Years
k = 10%

-62,000 19,920 22,800 15,600


TV = 15,680
31,280
NPV
$

B
A

Expected increase in annual net income k


ARR = 0 10%
180 Initial (or average) investment IRRB IRRA

NPV
($)

Discount rate (%)


0 16% 17% 18% 30%
NPV
$
NPV ($)
P
C

k 10% k
10% 15% 20% %

NPV NPV
$

A A

B
B

k k
0 7% 12% 15% 0 7% 12% 14%
NPV
$

X
Crossover

n
Sigma CF t k
t=1
10% IRRY 12% IRRX
t
(1 + K)

0 k = 12% 1 2 3 4 5 6

CFsA -15,000 4,000 4,000 4,000 4,000 4,000 4,000

Project B:
0 k = ? 1 2 6 Years
  
CFsB -14,815 5,100 5,100 5,100

Time lines:
Project A:
0 1 2 3 4 Years
k = 12%

CFsA -25,000 13,000 13,000 13,000 13,000


NPVA = ? = 17,663 Terminal value = 5,000
CF 4 = 18,000

Project B:
0 1 2 3 4 Years
k = ?

CFsB -25,000 15,247 15,247 15,247 15,247


NPVA = NPVB = 17,663 Terminal value = 0
CF 4 = 15,247
Time line:
0 k = 18% 1 2 3 Years

-3,000 1,728 1,920 1,152

Time line:
0 1 2 3 4 5 Years
k = 12%

-45,000 7,800 10,680 7,560 5,880 -1,920

Time line:
0 k = 9% 1 2 3 4 5 Years

-40,000 9,800 11,720 9,640 8,520 15,320 .New

project NPV Answer: a Diff: M N

0 1 2 3 4
Project cost ($500,000)
NOWC (40,000)

Sales $800,000 $800,000 $500,000 $500,000


Operating costs 480,000 480,000 300,000 300,000
Depreciation ________ 165,000 225,000 75,000 35,000
EBIT $155,000 $ 95,000 $125,000 $165,000
Taxes (40%) 62,000 38,000 50,000 66,000
EBIT(1 - T) $ 93,000 $ 57,000 $ 75,000 $ 99,000
Plus: Depreciation 165,000 225,000 75,000 35,000
Recovery of NOWC ________ ________ ________ ________ 40,000
Net CF ($540,000) $258,000 $282,000 $150,000 $174,000

NPV = -$540,000 + $258,000/1.10 + $282,000/(1.10)2 + $150,000/(1.10)3 + $174,000/(1.10)4


= -$540,000 + $234,545.45 + $233,057.85 + $112,697.22 + $118,844.34
= $159,144.86  $159,145.

Time lines:
Project A:
0 k = 14% 1 2 3 4 Years

CFsA -200,000 71,104 71,104 71,104 71,104


Project B:
0 k = 10% 1 2 3 4 Years

CFsB -200,000 0 0 146,411 146,411


Time line:
0 k = 12% 1 2 10 Years
  
-50,000 6,000 6,000 6,000
Time lines:
Project A:
0 k = 12% 1 2 3 Periods

CFsA -5,000 2,000 2,500 2,250


Project B:
0 k = 14% 1 2 3 Periods

CFsB -5,000 3,000 2,600 2,900


0 1 2 3 4
| | | | |
CFsNew Tech -1,500 -315 -315 -315 -315 x x x
Time line:
0 1 2 3 Years
k = 14%

-42,000 14,280 16,200 11,400


TV = 18,120
29,520
Time line:
0 1 2 3 Years
k = 10%

-62,000 19,920 22,800 15,600


TV = 15,680
31,280
NPV
$

B
A

Expected increase in annual net income k


ARR = 0 10%
181 Initial (or average) investment IRRB IRRA

NPV
($)

Discount rate (%)


0 16% 17% 18% 30%

NPV
$
NPV ($)
P
C

k 10% k
10% 15% 20% %
NPV NPV
$

A A

B
B n
Sigma CF t
k k t=1
0 7% 12% 15% 0 7% 12% 14%
(1 + K)t
NPV
$

X
Crossover

k
10% IRRY 12% IRRX

0 k = 12% 1 2 3 4 5 6

CFsA -15,000 4,000 4,000 4,000 4,000 4,000 4,000


Project B:
0 k = ? 1 2 6 Years
  
CFsB -14,815 5,100 5,100 5,100
Time lines:
Project A:
0 1 2 3 4 Years
k = 12%

CFsA -25,000 13,000 13,000 13,000 13,000


NPVA = ? = 17,663 Terminal value = 5,000
CF 4 = 18,000
Project B:
0 1 2 3 4 Years
k = ?

CFsB -25,000 15,247 15,247 15,247 15,247


NPVA = NPVB = 17,663 Terminal value = 0
CF 4 = 15,247
Time line:
0 k = 18% 1 2 3 Years

-3,000 1,728 1,920 1,152


Time line:
0 1 2 3 4 5 Years
k = 12%

-45,000 7,800 10,680 7,560 5,880 -1,920


Time line:
0 1 2 3 4 5 Years
k = 9%

-40,000 9,800 11,720 9,640 8,520 15,320 . New


project NPV Answer: d Diff: T
Step 1: Calculate depreciation:
Dep 1 = 100,000(0.33) = 33,000.
Dep 2 = 100,000(0.45) = 45,000.
Dep 3 = 100,000(0.15) = 15,000.
Dep 4 = 100,000(0.07) = 7,000.

Step 2: Calculate cash flows:


CF 0 = -100,000 - 5,000 = -105,000.
CF 1 = 50,000 + 33,000 = 83,000.
CF 2 = 60,000 + 45,000 = 105,000.
CF 3 = 70,000 + 15,000 = 85,000.
CF 4 = 60,000 + 7,000 + 5,000 + 15,000 = 87,000.

Step 3: Calculate NPV:


Use CF key on calculator. Enter cash flows shown above. Enter I/YR =
12%. Solve for NPV = $168,604.

Time lines:
Project A:
0 k = 14% 1 2 3 4 Years

CFsA -200,000 71,104 71,104 71,104 71,104


Project B:
0 k = 10% 1 2 3 4 Years

CFsB -200,000 0 0 146,411 146,411


Time line:
0 k = 12% 1 2 10 Years
  
-50,000 6,000 6,000 6,000
Time lines:
Project A:
0 k = 12% 1 2 3 Periods

CFsA -5,000 2,000 2,500 2,250


Project B:
0 k = 14% 1 2 3 Periods

CFsB -5,000 3,000 2,600 2,900


0 1 2 3 4
| | | | |
CFsNew Tech -1,500 -315 -315 -315 -315 x x x
Time line:
0 1 2 3 Years
k = 14%

-42,000 14,280 16,200 11,400


TV = 18,120
29,520
Time line:
0 1 2 3 Years
k = 10%

-62,000 19,920 22,800 15,600


TV = 15,680
31,280
NPV
$

B
A

Expected increase in annual net income k


ARR = 0 10%
182 Initial (or average) investment IRRB IRRA

NPV
($)

Discount rate (%)


0 16% 17% 18% 30%

NPV
$
NPV ($)
P
C

k 10% k
10% 15% 20% %
NPV NPV
$

A A

B
B n
Sigma CF t
k k t=1
0 7% 12% 15% 0 7% 12% 14%
(1 + K)t
NPV
$

X
Crossover

k
10% IRRY 12% IRRX

0 k = 12% 1 2 3 4 5 6

CFsA -15,000 4,000 4,000 4,000 4,000 4,000 4,000


Project B:
0 k = ? 1 2 6 Years
  
CFsB -14,815 5,100 5,100 5,100
Time lines:
Project A:
0 1 2 3 4 Years
k = 12%

CFsA -25,000 13,000 13,000 13,000 13,000


NPVA = ? = 17,663 Terminal value = 5,000
CF 4 = 18,000
Project B:
0 1 2 3 4 Years
k = ?

CFsB -25,000 15,247 15,247 15,247 15,247


NPVA = NPVB = 17,663 Terminal value = 0
CF 4 = 15,247
Time line:
0 k = 18% 1 2 3 Years

-3,000 1,728 1,920 1,152


Time line:
0 1 2 3 4 5 Years
k = 12%

-45,000 7,800 10,680 7,560 5,880 -1,920


Time line:
0 1 2 3 4 5 Years
k = 9%

-40,000 9,800 11,720 9,640 8,520 15,320 . New


project NPV Answer: d Diff: T
First, find the after-tax CFs associated with the project. This is
accomplished by subtracting the depreciation expense from the raw CF,
reducing this net CF by taxes and then adding back the depreciation expense.

For t = 1: ($45,000 - $33,000)(1 - 0.4) + $33,000 = $40,200.


Similarly, the after-tax CFs for t = 2, t = 3, and t = 4 are $45,000,
$33,000, and $29,800, respectively.

Now, enter these CFs along with the cost of the equipment to find the pre-
salvage NPV (note that the salvage value is not yet accounted for in these
CFs). The appropriate discount rate for these CFs is 11 percent. This yields
a pre-salvage NPV of $16,498.72.

Finally, the salvage value must be discounted. The PV of the salvage value
is: N = 4; I = 12; PMT = 0; FV = -10,000; and PV = $6,355.18. Adding the PV
of the salvage amount to the pre-salvage NPV yields the project NPV of
$22,853.90.
Time lines:
Project A:
0 k = 14% 1 2 3 4 Years

CFsA -200,000 71,104 71,104 71,104 71,104


Project B:
0 k = 10% 1 2 3 4 Years

CFsB -200,000 0 0 146,411 146,411


Time line:
0 k = 12% 1 2 10 Years
  
-50,000 6,000 6,000 6,000
Time lines:
Project A:
0 k = 12% 1 2 3 Periods

CFsA -5,000 2,000 2,500 2,250


Project B:
0 k = 14% 1 2 3 Periods

CFsB -5,000 3,000 2,600 2,900


0 1 2 3 4
| | | | |
CFsNew Tech -1,500 -315 -315 -315 -315 x x x
Time line:
0 1 2 3 Years
k = 14%

-42,000 14,280 16,200 11,400


TV = 18,120
29,520
Time line:
0 1 2 3 Years
k = 10%

-62,000 19,920 22,800 15,600


TV = 15,680
31,280
NPV
$

B
A

Expected increase in annual net income k


ARR = 0 10%
183 Initial (or average) investment IRRB IRRA

NPV
($)

Discount rate (%)


0 16% 17% 18% 30%

NPV
$
NPV ($)
P
C

k 10% k
10% 15% 20% %

NPV NPV
$

A A

B
B n
Sigma CF t
k k t=1
0 7% 12% 15% 0 7% 12% 14%
(1 + K)t
NPV
$

X
Crossover

k
10% IRRY 12% IRRX

0 k = 12% 1 2 3 4 5 6

CFsA -15,000 4,000 4,000 4,000 4,000 4,000 4,000


Project B:
0 k = ? 1 2 6 Years
  
CFsB -14,815 5,100 5,100 5,100
Time lines:
Project A:
0 1 2 3 4 Years
k = 12%

CFsA -25,000 13,000 13,000 13,000 13,000


NPVA = ? = 17,663 Terminal value = 5,000
CF 4 = 18,000
Project B:
0 1 2 3 4 Years
k = ?

CFsB -25,000 15,247 15,247 15,247 15,247


NPVA = NPVB = 17,663 Terminal value = 0
CF 4 = 15,247
Time line:
0 1 2 3 Years
k = 18%

-3,000 1,728 1,920 1,152


Time line:
0 1 2 3 4 5 Years
k = 12%

-45,000 7,800 10,680 7,560 5,880 -1,920


Time line:
0 k = 9% 1 2 3 4 5 Years

-40,000 9,800 11,720 9,640 8,520 15,320


. New project NPVAnswer: d Diff: T
The cash flows for each of the years are as follows:
0 -100,000
1 [90,000 - 50,000 - (100,000)(0.20)](1 - 0.4) + (100,000)(0.20) = 32,000
2 [90,000 - 50,000 - (100,000)(0.32)](1 - 0.4) + (100,000)(0.32) = 36,800
3 [90,000 - 50,000 - (100,000)(0.19)](1 - 0.4) + (100,000)(0.19) = 31,600
4 [90,000 - 50,000 - (100,000)(0.12)](1 - 0.4) + (100,000)(0.12) = 28,800
5 [90,000 - 50,000 - (100,000)(0.11)](1 - 0.4) + (100,000)(0.11) = 28,400
6 [90,000 - 50,000 - (100,000)(0.06)](1 - 0.4) + (100,000)(0.06) +
(10,000)(1 - 0.4) = 32,400

Enter the cash flows and solve for the NPV = $38,839.56.
Time lines:
Project A:
0 k = 14% 1 2 3 4 Years

CFsA -200,000 71,104 71,104 71,104 71,104


Project B:
0 k = 10% 1 2 3 4 Years

CFsB -200,000 0 0 146,411 146,411


Time line:
0 k = 12% 1 2 10 Years
  
-50,000 6,000 6,000 6,000
Time lines:
Project A:
0 k = 12% 1 2 3 Periods

CFsA -5,000 2,000 2,500 2,250


Project B:
0 k = 14% 1 2 3 Periods

CFsB -5,000 3,000 2,600 2,900


0 1 2 3 4
| | | | |
CFsNew Tech -1,500 -315 -315 -315 -315 x x x
Time line:
0 1 2 3 Years
k = 14%

-42,000 14,280 16,200 11,400


TV = 18,120
29,520
Time line:
0 1 2 3 Years
k = 10%

-62,000 19,920 22,800 15,600


TV = 15,680
31,280
NPV
$

B
A

Expected increase in annual net income k


ARR = 0 10%
184 Initial (or average) investment IRRB IRRA

NPV
($)

Discount rate (%)


0 16% 17% 18% 30%

NPV
$
NPV ($)
P
C

k 10% k
10% 15% 20% %

NPV NPV
$

A A

B
B n
Sigma CF t
k k t=1
0 7% 12% 15% 0 7% 12% 14%
(1 + K)t
NPV
$

X
Crossover

k
10% IRRY 12% IRRX

0 k = 12% 1 2 3 4 5 6

CFsA -15,000 4,000 4,000 4,000 4,000 4,000 4,000


Project B:
0 k = ? 1 2 6 Years
  
CFsB -14,815 5,100 5,100 5,100
Time lines:
Project A:
0 1 2 3 4 Years
k = 12%

CFsA -25,000 13,000 13,000 13,000 13,000


NPVA = ? = 17,663 Terminal value = 5,000
CF 4 = 18,000
Project B:
0 1 2 3 4 Years
k = ?

CFsB -25,000 15,247 15,247 15,247 15,247


NPVA = NPVB = 17,663 Terminal value = 0
CF 4 = 15,247
Time line:
0 1 2 3 Years
k = 18%

-3,000 1,728 1,920 1,152


Time line:
0 1 2 3 4 5 Years
k = 12%

-45,000 7,800 10,680 7,560 5,880 -1,920


Time line:
0 k = 9% 1 2 3 4 5 Years

-40,000 9,800 11,720 9,640 8,520 15,320


. New project NPVAnswer: c Diff: T
Get the depreciation using the MACRS table provided in the question.
0 1 2 3 4
Cost (500,000)
Inventory ( 50,000)
Accounts Payable 10,000
Sales 600,000 600,000 600,000 600,000
Operating Cost 400,000 400,000 400,000 400,000
Depreciation 165,000 225,000 75,000 35,000
EBIT 35,000 ( 25,000) 125,000 165,000
Taxes 14,000 ( 10,000) 50,000 66,000
EBIT(1 - T) 21,000 ( 15,000) 75,000 99,000
After-tax salvage value 30,000
Return of NOWC 40,000
+ Depreciation ________ 165,000 225,000 75,000 35,000
Net CF ($540,000) $186,000 $210,000 $150,000 $204,000

Note in year 4 your $40,000 of net operating working capital is recovered


plus the after tax salvage value of $30,000.

Enter the cash flows into the cash flow register and solve for the NPV using
the WACC of 10%. NPV = $54,676.59.
Time lines:
Project A:
0 k = 14% 1 2 3 4 Years

CFsA -200,000 71,104 71,104 71,104 71,104


Project B:
0 k = 10% 1 2 3 4 Years

CFsB -200,000 0 0 146,411 146,411


Time line:
0 k = 12% 1 2 10 Years
  
-50,000 6,000 6,000 6,000
Time lines:
Project A:
0 k = 12% 1 2 3 Periods

CFsA -5,000 2,000 2,500 2,250


Project B:
0 k = 14% 1 2 3 Periods

CFsB -5,000 3,000 2,600 2,900


0 1 2 3 4
| | | | |
CFsNew Tech -1,500 -315 -315 -315 -315 x x x
Time line:
0 1 2 3 Years
k = 14%

-42,000 14,280 16,200 11,400


TV = 18,120
29,520
Time line:
0 1 2 3 Years
k = 10%

-62,000 19,920 22,800 15,600


TV = 15,680
31,280
NPV
$

B
A

Expected increase in annual net income k


ARR = 0 10%
185 Initial (or average) investment IRRB IRRA

NPV
($)

Discount rate (%)


0 16% 17% 18% 30%

NPV
$
NPV ($)
P
C

k 10% k
10% 15% 20% %

NPV NPV
$

A A

B
B n
Sigma CF t
k k t=1
0 7% 12% 15% 0 7% 12% 14%
(1 + K)t
NPV
$

X
Crossover

k
10% IRRY 12% IRRX

0 k = 12% 1 2 3 4 5 6

CFsA -15,000 4,000 4,000 4,000 4,000 4,000 4,000


Project B:
0 k = ? 1 2 6 Years
  
CFsB -14,815 5,100 5,100 5,100
Time lines:
Project A:
0 1 2 3 4 Years
k = 12%

CFsA -25,000 13,000 13,000 13,000 13,000


NPVA = ? = 17,663 Terminal value = 5,000
CF 4 = 18,000
Project B:
0 1 2 3 4 Years
k = ?

CFsB -25,000 15,247 15,247 15,247 15,247


NPVA = NPVB = 17,663 Terminal value = 0
CF 4 = 15,247
Time line:
0 1 2 3 Years
k = 18%

-3,000 1,728 1,920 1,152


Time line:
0 1 2 3 4 5 Years
k = 12%

-45,000 7,800 10,680 7,560 5,880 -1,920


Time line:
0 k = 9% 1 2 3 4 5 Years

-40,000 9,800 11,720 9,640 8,520 15,320 . Risk-


adjusted NPV Answer: a Diff: M
Time lines:
Project A:
0 k = 14% 1 2 3 4 Years

CFsA -200,000 71,104 71,104 71,104 71,104


NPVA = ?

Project B:
0 k = 10% 1 2 3 4 Years

CFsB -200,000 0 0 146,411 146,411


NPVB = ?
Calculate required returns on A and B:
Project A High risk kRisk adjusted = 12% + 2% = 14%.
Project B Low risk kRisk adjusted = 12% - 2% = 10%.

Tabular solution:
NPVA = $71,104(PVIFA14%,4) - $200,000
= $71,104(2.9137) - $200,000 = $7,175.72.
NPVB = $146,411(PVIF10%,3) + $146,411(PVIF10%,4) - $200,000
= $146,411(0.7513) + $146,411(0.6830) - $200,000 = $9,997.30.
Project B has the higher NPV. Since they are mutually exclusive, select
Project B.

Financial calculator solution:


A Inputs: CF0 = -200,000; CF1 = 71,104; Nj = 4; I = 14.
Output: NPVA = $7,176.60  $7,177.
B Inputs: CF0 = -200,000; CF1 = 0; Nj = 2; CF2 = 146,411; Nj = 2; I = 10.
Output: NPVB = $10,001.43  $10,001.
Note: The difference in the NPVB between the tabular solution and financial
calculator cash flow solution of $4.13 is due to rounding of PVIF factors to
four significant decimals. Greater precision in the PVIF factors produces
identical answers.
Time line:
0 k = 12% 1 2 10 Years
  
-50,000 6,000 6,000 6,000
Time lines:
Project A:
0 k = 12% 1 2 3 Periods

CFsA -5,000 2,000 2,500 2,250


Project B:
0 k = 14% 1 2 3 Periods

CFsB -5,000 3,000 2,600 2,900


0 1 2 3 4
| | | | |
CFsNew Tech -1,500 -315 -315 -315 -315 x x x
Time line:
0 1 2 3 Years
k = 14%

-42,000 14,280 16,200 11,400


TV = 18,120
29,520
Time line:
0 1 2 3 Years
k = 10%

-62,000 19,920 22,800 15,600


TV = 15,680
31,280
NPV
$

B
A

Expected increase in annual net income k


ARR = 0 10%
186 Initial (or average) investment IRRB IRRA

NPV
($)

Discount rate (%)


0 16% 17% 18% 30%

NPV
$
NPV ($)
P
C

k 10% k
10% 15% 20% %

NPV NPV
$

A A

B
B n
Sigma CF t
k k t=1
0 7% 12% 15% 0 7% 12% 14%
(1 + K)t
NPV
$

X
Crossover

k
10% IRRY 12% IRRX

0 k = 12% 1 2 3 4 5 6

CFsA -15,000 4,000 4,000 4,000 4,000 4,000 4,000


Project B:
0 k = ? 1 2 6 Years
  
CFsB -14,815 5,100 5,100 5,100
Time lines:
Project A:
0 1 2 3 4 Years
k = 12%

CFsA -25,000 13,000 13,000 13,000 13,000


NPVA = ? = 17,663 Terminal value = 5,000
CF 4 = 18,000
Project B:
0 1 2 3 4 Years
k = ?

CFsB -25,000 15,247 15,247 15,247 15,247


NPVA = NPVB = 17,663 Terminal value = 0
CF 4 = 15,247
Time line:
0 1 2 3 Years
k = 18%

-3,000 1,728 1,920 1,152


Time line:
0 1 2 3 4 5 Years
k = 12%

-45,000 7,800 10,680 7,560 5,880 -1,920


Time line:
0 k = 9% 1 2 3 4 5 Years

-40,000 9,800 11,720 9,640 8,520 15,320


Time lines:
Project A:
0 k = 14% 1 2 3 4 Years

CFsA -200,000 71,104 71,104 71,104 71,104


Project B:
0 k = 10% 1 2 3 4 Years

CFsB -200,000 0 0 146,411 146,411 . Risk-adjusted


NPV Answer: e Diff: M
Time line:
0 k = 12% 1 2 10 Years
  
-50,000 6,000 6,000 6,000
NPV = ? Salvage value 10,000
CF10 = 16,000
kProject = 6% + 4%(1.5) = 12%.

Tabular solution:
NPV = -$50,000 + $6,000(PVIFA12%,10) + $10,000(PVIF12%,10)
= -$50,000 + $6,000(5.6502) + $10,000(0.3220)
= -$12,878.80  -$12,879.

Financial calculator solution:


Inputs: CF0 = -50,000; CF1 = 6,000; Nj = 9; CF10 = 16,000; I = 12%.
Output: NPV = -$12,878.93  -$12,879.

Time lines:
Project A:
0 k = 12% 1 2 3 Periods

CFsA -5,000 2,000 2,500 2,250


Project B:
0 k = 14% 1 2 3 Periods

CFsB -5,000 3,000 2,600 2,900


0 1 2 3 4
| | | | |
CFsNew Tech -1,500 -315 -315 -315 -315 x x x
Time line:
0 1 2 3 Years
k = 14%

-42,000 14,280 16,200 11,400


TV = 18,120
29,520
Time line:
0 1 2 3 Years
k = 10%

-62,000 19,920 22,800 15,600


TV = 15,680
31,280
NPV
$

B
A

Expected increase in annual net income k


ARR = 0 10%
187 Initial (or average) investment IRRB IRRA

NPV
($)

Discount rate (%)


0 16% 17% 18% 30%

NPV
$
NPV ($)
P
C

k 10% k
10% 15% 20% %

NPV NPV
$

A A

B
B n
Sigma CF t
k k t=1
0 7% 12% 15% 0 7% 12% 14%
(1 + K)t
NPV
$

X
Crossover

k
10% IRRY 12% IRRX

0 k = 12% 1 2 3 4 5 6

CFsA -15,000 4,000 4,000 4,000 4,000 4,000 4,000


Project B:
0 k = ? 1 2 6 Years
  
CFsB -14,815 5,100 5,100 5,100
Time lines:
Project A:
0 1 2 3 4 Years
k = 12%

CFsA -25,000 13,000 13,000 13,000 13,000


NPVA = ? = 17,663 Terminal value = 5,000
CF 4 = 18,000
Project B:
0 1 2 3 4 Years
k = ?

CFsB -25,000 15,247 15,247 15,247 15,247


NPVA = NPVB = 17,663 Terminal value = 0
CF 4 = 15,247
Time line:
0 1 2 3 Years
k = 18%

-3,000 1,728 1,920 1,152


Time line:
0 1 2 3 4 5 Years
k = 12%

-45,000 7,800 10,680 7,560 5,880 -1,920


Time line:
0 k = 9% 1 2 3 4 5 Years

-40,000 9,800 11,720 9,640 8,520 15,320


Time lines:
Project A:
0 k = 14% 1 2 3 4 Years

CFsA -200,000 71,104 71,104 71,104 71,104


Project B:
0 k = 10% 1 2 3 4 Years

CFsB -200,000 0 0 146,411 146,411


Time line:
0 k = 12% 1 2 10 Years
  
-50,000 6,000 6,000 6,000 . Risk-adjusted
NPV Answer: c Diff: M
Time lines:
Project A:
0 k = 12% 1 2 3 Periods

CFsA -5,000 2,000 2,500 2,250 NPVA


= ?

Project B:
0 k = 14% 1 2 3 Periods

CFsB -5,000 3,000 2,600 2,900 NPVB


= ?

Project A: kAverage risk = 12%.


Project B: kHigh risk = 12% + 2% = 14%.

Tabular solution:
NPVA = $2,000(PVIF12%,1) + $2,500(PVIF12%,2) + $2,250(PVIF12%,3) - $5,000
= $2,000(0.8929) + $2,500(0.7972) + $2,250(0.7118) - $5,000
= $380.35  $380.
NPVB = $3,000(PVIF14%,1) + $2,600(PVIF14%,2) + $2,900(PVIF14%,3) - $5,000
= $3,000(0.8772) + $2,600(0.7695) + $2,900(0.6750) - $5,000
= $1,589.80  $1,590.

Financial calculator solution:


A: Inputs: CF0 = -5,000; CF1 = 2,000; CF2 = 2,500; CF3 = 2,250; I% = 12.
Output: NPV = $380.20  $380.
B: Inputs: CF0 = -5,000; CF1 = 3,000; CF2 = 2,600; CF3 = 2,900; I% = 14.
Output: NPV = $1,589.61  $1,590.
0 1 2 3 4
| | | | |
CFsNew Tech -1,500 -315 -315 -315 -315 x x x
Time line:
0 1 2 3 Years
k = 14%

-42,000 14,280 16,200 11,400


TV = 18,120
29,520
Time line:
0 1 2 3 Years
k = 10%

-62,000 19,920 22,800 15,600


TV = 15,680
31,280
NPV
$

B
A

Expected increase in annual net income k


ARR = 0 10%
188 Initial (or average) investment IRRB IRRA

NPV
($)

Discount rate (%)


0 16% 17% 18% 30%

NPV
$
NPV ($)
P
C

k 10% k
10% 15% 20% %

NPV NPV
$

A A

B
B n
Sigma CF t
k k t=1
0 7% 12% 15% 0 7% 12% 14%
(1 + K)t
NPV
$

X
Crossover

k
10% IRRY 12% IRRX

0 k = 12% 1 2 3 4 5 6

CFsA -15,000 4,000 4,000 4,000 4,000 4,000 4,000


Project B:
0 k = ? 1 2 6 Years
  
CFsB -14,815 5,100 5,100 5,100
Time lines:
Project A:
0 1 2 3 4 Years
k = 12%

CFsA -25,000 13,000 13,000 13,000 13,000


NPVA = ? = 17,663 Terminal value = 5,000
CF 4 = 18,000
Project B:
0 1 2 3 4 Years
k = ?

CFsB -25,000 15,247 15,247 15,247 15,247


NPVA = NPVB = 17,663 Terminal value = 0
CF 4 = 15,247
Time line:
0 1 2 3 Years
k = 18%

-3,000 1,728 1,920 1,152


Time line:
0 1 2 3 4 5 Years
k = 12%

-45,000 7,800 10,680 7,560 5,880 -1,920


Time line:
0 k = 9% 1 2 3 4 5 Years

-40,000 9,800 11,720 9,640 8,520 15,320


Time lines:
Project A:
0 k = 14% 1 2 3 4 Years

CFsA -200,000 71,104 71,104 71,104 71,104


Project B:
0 k = 10% 1 2 3 4 Years

CFsB -200,000 0 0 146,411 146,411


Time line:
0 k = 12% 1 2 10 Years
  
-50,000 6,000 6,000 6,000
Time lines:
Project A:
0 k = 12% 1 2 3 Periods

CFsA -5,000 2,000 2,500 2,250


Project B:
0 k = 14% 1 2 3 Periods

CFsB -5,000 3,000 2,600 2,900

. NPV and risk-adjusted discount rate Answer: e Diff: T N

The following table shows the cash flows:

0 1 2 3 4 5
Initial invest. outlay -$30.0
Sales $20.0 $20.0 $20.0 $20.0 $20.0
Oper. cost 12.0 12.0 12.0 12.0 12.0
Deprec. 10.0 10.0 10.0 0.0 0.0
EBIT -$ 2.0 -$ 2.0 -$ 2.0 $ 8.0 $ 8.0
Less: Taxes -0.8 -0.8 -0.8 3.2 3.2
EBIT(1 - T) -$ 1.2 -$ 1.2 -$ 1.2 $ 4.8 $ 4.8
Add back: Deprec. 10.0 10.0 10.0 0.0 0.0
NCF -$30.0 $ 8.8 $ 8.8 $ 8.8 $ 4.8 $ 4.8

Step 1: Determine the NPV of net cash flows:


NPV = -$30 + $8.8/1.10 + $8.8/(1.10)2 + $8.8/(1.10)3 + $4.8/(1.10)4 + $4.8/(1.10)5
= -$30 + $8 + $7.2727 + $6.6116 + $3.2785 + $2.9804
= -$1.8568 million.

Step 2: Determine the NPV of the project’s AT salvage value:


$1.2/(1.12)5 = $0.6809 million.

Step 3: Determine the project’s NPV:


Add the PV of the salvage value to the NPV of the cash flows to get the project’s NPV.
NPV = -$1.8568 + $0.6809 = -$1.1759 million  -$1.18 million.

Financial calculator solution:


Step 1: Determine the NPV of net cash flows:
Enter the following inputs in the calculator:
CF0 = -30, CF1-3 = 8.8, CF4-5 = 4.8, I = 10, and then solve for NPV = -$1.8568 million.

Step 2: Determine the NPV of the project’s AT salvage value:


Enter the following inputs in the calculator:
CF0 = 0, CF1-4 = 0, CF5 = 1.2, I = 12, and then solve for NPV = $0.6809 million.

Step 3: Determine the project’s NPV:


Add the PV of the salvage value to the NPV of the cash flows to get the project’s NPV.
-$1.8568 + $0.6809 = -$1.1759 million  $-1.18 million.

0 1 2 3 4
| | | | |
CFsNew Tech -1,500 -315 -315 -315 -315 x x x
Time line:
0 1 2 3 Years
k = 14%

-42,000 14,280 16,200 11,400


TV = 18,120
29,520
Time line:
0 1 2 3 Years
k = 10%

-62,000 19,920 22,800 15,600


TV = 15,680
31,280
NPV
$

B
A

Expected increase in annual net income k


ARR = 0 10%
189 Initial (or average) investment IRRB IRRA

NPV
($)

Discount rate (%)


0 16% 17% 18% 30%
NPV
$
NPV ($)
P
C

k 10% k
10% 15% 20% %

NPV NPV
$

A A

B
B n
Sigma CF t
k k t=1
0 7% 12% 15% 0 7% 12% 14%
(1 + K)t
NPV
$

X
Crossover

k
10% IRRY 12% IRRX

0 k = 12% 1 2 3 4 5 6

CFsA -15,000 4,000 4,000 4,000 4,000 4,000 4,000


Project B:
0 k = ? 1 2 6 Years
  
CFsB -14,815 5,100 5,100 5,100
Time lines:
Project A:
0 1 2 3 4 Years
k = 12%

CFsA -25,000 13,000 13,000 13,000 13,000


NPVA = ? = 17,663 Terminal value = 5,000
CF 4 = 18,000
Project B:
0 1 2 3 4 Years
k = ?

CFsB -25,000 15,247 15,247 15,247 15,247


NPVA = NPVB = 17,663 Terminal value = 0
CF 4 = 15,247
Time line:
0 k = 18% 1 2 3 Years

-3,000 1,728 1,920 1,152


Time line:
0 1 2 3 4 5 Years
k = 12%

-45,000 7,800 10,680 7,560 5,880 -1,920


Time line:
0 1 2 3 4 5 Years
k = 9%

-40,000 9,800 11,720 9,640 8,520 15,320


Time lines:
Project A:
0 k = 14% 1 2 3 4 Years

CFsA -200,000 71,104 71,104 71,104 71,104


Project B:
0 k = 10% 1 2 3 4 Years

CFsB -200,000 0 0 146,411 146,411


Time line:
0 k = 12% 1 2 10 Years
  
-50,000 6,000 6,000 6,000
Time lines:
Project A:
0 k = 12% 1 2 3 Periods

CFsA -5,000 2,000 2,500 2,250


Project B:
0 k = 14% 1 2 3 Periods

CFsB -5,000 3,000 2,600 2,900


. Discounting risky outflows Answer: e Diff: M
Time line:
0 1 2 3 4
| | | | |
CFsNew Tech -1,500 -315 -315 -315 -315
NPVNew Tech = ?
CFsOld Tech -600 -600 -600 -600 -600
NPVOld Tech = ?
Recognize that (1) risk outflows must be discounted at lower rates, and (2)
since Project New Tech is risky, it must be discounted at a rate of 12% - 3%
= 9%. Project Old Tech must be discounted at 12%.
Tabular solution:
PVNew Tech = -$1,500 - $315(PVIFA9%,4) = -$1,500 - $315(3.2397)
= -$2,520.51.
PVOld Tech = -$600 - $600(PVIFA12%,4) = -$600 - $600(3.0373) = -$2,422.38.
PVOld Tech is a smaller outflow than NPVNew Tech, thus, Project Old Tech is the
better project.
Financial calculator solution:
Project New Tech: Inputs: CF0 = -1,500; CF1 = -315; Nj = 4; I = 9.
Output: NPV = -$2,520.51.
Project Old Tech: Inputs: CF0 = -600; CF1 = -600; Nj = 4; I = 12.
Output: NPV = -$2,422.41.
Time line:
0 1 2 3 Years
k = 14%

-42,000 14,280 16,200 11,400


TV = 18,120
x x x 29,520
Time line:
0 1 2 3 Years
k = 10%

-62,000 19,920 22,800 15,600


TV = 15,680
31,280
NPV
$

B
A

Expected increase in annual net income k


ARR = 0 10%
190 Initial (or average) investment IRRB IRRA

NPV
($)

Discount rate (%)


0 16% 17% 18% 30%

NPV
$
NPV ($)
P
C

k 10% k
10% 15% 20% %

NPV NPV
$

A A

B
B n
Sigma CF t
k k t=1
0 7% 12% 15% 0 7% 12% 14%
(1 + K)t
NPV
$

X
Crossover

k
10% IRRY 12% IRRX

0 k = 12% 1 2 3 4 5 6

CFsA -15,000 4,000 4,000 4,000 4,000 4,000 4,000


Project B:
0 k = ? 1 2 6 Years
  
CFsB -14,815 5,100 5,100 5,100
Time lines:
Project A:
0 1 2 3 4 Years
k = 12%

CFsA -25,000 13,000 13,000 13,000 13,000


NPVA = ? = 17,663 Terminal value = 5,000
CF 4 = 18,000
Project B:
0 1 2 3 4 Years
k = ?

CFsB -25,000 15,247 15,247 15,247 15,247


NPVA = NPVB = 17,663 Terminal value = 0
CF 4 = 15,247
Time line:
0 1 2 3 Years
k = 18%

-3,000 1,728 1,920 1,152


Time line:
0 1 2 3 4 5 Years
k = 12%

-45,000 7,800 10,680 7,560 5,880 -1,920


Time line:
0 k = 9% 1 2 3 4 5 Years

-40,000 9,800 11,720 9,640 8,520 15,320


Time lines:
Project A:
0 k = 14% 1 2 3 4 Years

CFsA -200,000 71,104 71,104 71,104 71,104


Project B:
0 k = 10% 1 2 3 4 Years

CFsB -200,000 0 0 146,411 146,411


Time line:
0 k = 12% 1 2 10 Years
  
-50,000 6,000 6,000 6,000
Time lines:
Project A:
0 k = 12% 1 2 3 Periods

CFsA -5,000 2,000 2,500 2,250


Project B:
0 k = 14% 1 2 3 Periods

CFsB -5,000 3,000 2,600 2,900


0 1 2 3 4
| | | | |
CFsNew Tech -1,500 -315 -315 -315 -315
. Risky Projects Answer: d Diff: M
Look at the NPV, IRR, and hurdle rate for each project:
Project A B C D E
Hurdle 9.00% 9.00% 11.00% 13.00% 13.00%
NPV $13,822 $11,998
IRR 12.11% 14.04% 10.85% 16.64% 11.63%

Projects A and B are mutually exclusive, so we pick project A because it has


the largest NPV. Projects C, D, and E are independent so we pick the ones
whose IRR exceeds the cost of capital, in this case, just D. Therefore, the
projects undertaken are A and D.
Time line:
0 1 2 3 Years
k = 14%

-42,000 14,280 16,200 11,400


TV = 18,120
x x x 29,520
Time line:
0 1 2 3 Years
k = 10%

-62,000 19,920 22,800 15,600


TV = 15,680
31,280
NPV
$

B
A

Expected increase in annual net income k


ARR = 0 10%
191 Initial (or average) investment IRRB IRRA

NPV
($)

Discount rate (%)


0 16% 17% 18% 30%

NPV
$
NPV ($)
P
C

k 10% k
10% 15% 20% %

NPV NPV
$

A A

B
B n
Sigma CF t
k k t=1
0 7% 12% 15% 0 7% 12% 14%
(1 + K)t
NPV
$

X
Crossover

k
10% IRRY 12% IRRX

0 k = 12% 1 2 3 4 5 6

CFsA -15,000 4,000 4,000 4,000 4,000 4,000 4,000


Project B:
0 k = ? 1 2 6 Years
  
CFsB -14,815 5,100 5,100 5,100
Time lines:
Project A:
0 1 2 3 4 Years
k = 12%

CFsA -25,000 13,000 13,000 13,000 13,000


NPVA = ? = 17,663 Terminal value = 5,000
CF 4 = 18,000
Project B:
0 1 2 3 4 Years
k = ?

CFsB -25,000 15,247 15,247 15,247 15,247


NPVA = NPVB = 17,663 Terminal value = 0
CF 4 = 15,247
Time line:
0 1 2 3 Years
k = 18%

-3,000 1,728 1,920 1,152


Time line:
0 1 2 3 4 5 Years
k = 12%

-45,000 7,800 10,680 7,560 5,880 -1,920


Time line:
0 k = 9% 1 2 3 4 5 Years

-40,000 9,800 11,720 9,640 8,520 15,320


Time lines:
Project A:
0 k = 14% 1 2 3 4 Years

CFsA -200,000 71,104 71,104 71,104 71,104


Project B:
0 k = 10% 1 2 3 4 Years

CFsB -200,000 0 0 146,411 146,411


Time line:
0 k = 12% 1 2 10 Years
  
-50,000 6,000 6,000 6,000
Time lines:
Project A:
0 k = 12% 1 2 3 Periods

CFsA -5,000 2,000 2,500 2,250


Project B:
0 k = 14% 1 2 3 Periods

CFsB -5,000 3,000 2,600 2,900


0 1 2 3 4
| | | | |
CFsNew Tech -1,500 -315 -315 -315 -315

. Scenario analysis Answer: c Diff: M


Calculate expected value of NPV:
Probability of Unit Sales Sales NPV
Outcome, Pi Volume Price (In 1000s) Pi(x)
Worst case 0.30 6,000 $3,600 -$6,000 0.3(-6,000) = -1,800
Base case 0.50 10,000 4,200 13,000 0.5(13,000) = 6,500
Best case 0.20 13,000 4,400 28,000 0.2(28,000) = 5,600
Expected NPV = $10,300

Calculate standard deviation of NPV (in thousands):


Pi(x - x )2 (x - x )2 Pi(x - x )2
_____________________________________ ____________________ _________________________

Worst case 0.3(-6 - 10.3)2 265.69 79.707


Base case 0.5(13 - 10.3)2 7.29 3.645
Best case 0.2(28 - 10.3)2 313.29 62.658
Sum 146.01
(146.01)½ = 12,083 thousand.

Calculate coefficient of variation (CV) of NPV:


CVNPV = σNPV/E(NPV) = $12,083/$10,300 = 1.17.
Time line:
0 1 2 3 Years
k = 14%

-42,000 14,280 16,200 11,400


TV = 18,120
29,520
Time line:
0 1 2 3 Years
k = 10%

-62,000 19,920 22,800 15,600


TV = 15,680
31,280
NPV
$

B
A

Expected increase in annual net income k


ARR = 0 10%
192 Initial (or average) investment IRRB IRRA

NPV
($)

Discount rate (%)


0 16% 17% 18% 30%

NPV
$
NPV ($)
P
C

k 10% k
10% 15% 20% %
NPV NPV
$

A A

B
B n
Sigma CF t
k k t=1
0 7% 12% 15% 0 7% 12% 14%
(1 + K)t
NPV
$

X
Crossover

k
10% IRRY 12% IRRX

0 k = 12% 1 2 3 4 5 6

CFsA -15,000 4,000 4,000 4,000 4,000 4,000 4,000


Project B:
0 k = ? 1 2 6 Years
  
CFsB -14,815 5,100 5,100 5,100
Time lines:
Project A:
0 1 2 3 4 Years
k = 12%

CFsA -25,000 13,000 13,000 13,000 13,000


NPVA = ? = 17,663 Terminal value = 5,000
CF 4 = 18,000
Project B:
0 1 2 3 4 Years
k = ?

CFsB -25,000 15,247 15,247 15,247 15,247


NPVA = NPVB = 17,663 Terminal value = 0
CF 4 = 15,247
Time line:
0 k = 18% 1 2 3 Years

-3,000 1,728 1,920 1,152


Time line:
0 1 2 3 4 5 Years
k = 12%

-45,000 7,800 10,680 7,560 5,880 -1,920


Time line:
0 1 2 3 4 5 Years
k = 9%

-40,000 9,800 11,720 9,640 8,520 15,320


Time lines:
Project A:
0 k = 14% 1 2 3 4 Years

CFsA -200,000 71,104 71,104 71,104 71,104


Project B:
0 k = 10% 1 2 3 4 Years

CFsB -200,000 0 0 146,411 146,411


Time line:
0 k = 12% 1 2 10 Years
  
-50,000 6,000 6,000 6,000
Time lines:
Project A:
0 k = 12% 1 2 3 Periods

CFsA -5,000 2,000 2,500 2,250


Project B:
0 k = 14% 1 2 3 Periods

CFsB -5,000 3,000 2,600 2,900


0 1 2 3 4
| | | | |
CFsNew Tech -1,500 -315 -315 -315 -315 x x x .
New project investment Answer: a Diff: E
Initial investment:
Cost ($40,000)
Change in NOWC (2,000)
($42,000)
Time line:
0 1 2 3 Years
k = 14%

-42,000 14,280 16,200 11,400


TV = 18,120
29,520
Time line:
0 1 2 3 Years
k = 10%

-62,000 19,920 22,800 15,600


TV = 15,680
31,280
NPV
$

B
A

Expected increase in annual net income k


ARR = 0 10%
193 Initial (or average) investment IRRB IRRA

NPV
($)

Discount rate (%)


0 16% 17% 18% 30%

NPV
$
NPV ($)
P
C

k 10% k
10% 15% 20% %

NPV NPV
$

A A

B
B n
Sigma CF t
k k t=1
0 7% 12% 15% 0 7% 12% 14%
(1 + K)t
NPV
$

X
Crossover

k
10% IRRY 12% IRRX

0 k = 12% 1 2 3 4 5 6

CFsA -15,000 4,000 4,000 4,000 4,000 4,000 4,000


Project B:
0 k = ? 1 2 6 Years
  
CFsB -14,815 5,100 5,100 5,100
Time lines:
Project A:
0 1 2 3 4 Years
k = 12%

CFsA -25,000 13,000 13,000 13,000 13,000


NPVA = ? = 17,663 Terminal value = 5,000
CF 4 = 18,000
Project B:
0 1 2 3 4 Years
k = ?

CFsB -25,000 15,247 15,247 15,247 15,247


NPVA = NPVB = 17,663 Terminal value = 0
CF 4 = 15,247
Time line:
0 1 2 3 Years
k = 18%

-3,000 1,728 1,920 1,152


Time line:
0 1 2 3 4 5 Years
k = 12%

-45,000 7,800 10,680 7,560 5,880 -1,920


Time line:
0 k = 9% 1 2 3 4 5 Years

-40,000 9,800 11,720 9,640 8,520 15,320


Time lines:
Project A:
0 k = 14% 1 2 3 4 Years

CFsA -200,000 71,104 71,104 71,104 71,104


Project B:
0 k = 10% 1 2 3 4 Years

CFsB -200,000 0 0 146,411 146,411


Time line:
0 k = 12% 1 2 10 Years
  
-50,000 6,000 6,000 6,000
Time lines:
Project A:
0 k = 12% 1 2 3 Periods

CFsA -5,000 2,000 2,500 2,250


Project B:
0 k = 14% 1 2 3 Periods

CFsB -5,000 3,000 2,600 2,900


0 1 2 3 4
| | | | |
CFsNew Tech -1,500 -315 -315 -315 -315 x x x

. Operating cash flow Answer: e Diff: M


Depreciation schedule:
Depreciable basis = $40,000.
MACRS
Depreciation Depreciable Annual
Year Rate Basis Depreciation
1 0.33 $40,000 $13,200
2 0.45 40,000 18,000
3 0.15 40,000 6,000
4 0.07 40,000 2,800
$40,000

Operating cash flows:


Year 1 2 3
1) Increase in revenues $20,000 $20,000 $20,000
2) Increase in costs (5,000) (5,000) (5,000)
3) Before-tax change in
earnings 15,000 15,000 15,000
4) After-tax change in
earnings (line 3  0.60) 9,000 9,000 9,000
5) Depreciation 13,200 18,000 6,000
6) Tax savings deprec.
(line 6  0.40) 5,280 7,200 2,400
7) Net operating CFs
(line 4 + 6) $14,280 $16,200 $11,400

Time line:
0 1 2 3 Years
k = 14%

-42,000 14,280 16,200 11,400


TV = 18,120
29,520
Time line:
0 1 2 3 Years
k = 10%

-62,000 19,920 22,800 15,600


TV = 15,680
31,280
NPV
$

B
A

Expected increase in annual net income k


ARR = 0 10%
194 Initial (or average) investment IRRB IRRA

NPV
($)

Discount rate (%)


0 16% 17% 18% 30%

NPV
$
NPV ($)
P
C

k 10% k
10% 15% 20% %

NPV NPV
$

A A

B
B n
Sigma CF t
k k t=1
0 7% 12% 15% 0 7% 12% 14%
(1 + K)t
NPV
$

X
Crossover

k
10% IRRY 12% IRRX

0 k = 12% 1 2 3 4 5 6

CFsA -15,000 4,000 4,000 4,000 4,000 4,000 4,000


Project B:
0 k = ? 1 2 6 Years
  
CFsB -14,815 5,100 5,100 5,100
Time lines:
Project A:
0 1 2 3 4 Years
k = 12%

CFsA -25,000 13,000 13,000 13,000 13,000


NPVA = ? = 17,663 Terminal value = 5,000
CF 4 = 18,000
Project B:
0 1 2 3 4 Years
k = ?

CFsB -25,000 15,247 15,247 15,247 15,247


NPVA = NPVB = 17,663 Terminal value = 0
CF 4 = 15,247
Time line:
0 1 2 3 Years
k = 18%

-3,000 1,728 1,920 1,152


Time line:
0 1 2 3 4 5 Years
k = 12%

-45,000 7,800 10,680 7,560 5,880 -1,920


Time line:
0 k = 9% 1 2 3 4 5 Years

-40,000 9,800 11,720 9,640 8,520 15,320


Time lines:
Project A:
0 k = 14% 1 2 3 4 Years

CFsA -200,000 71,104 71,104 71,104 71,104


Project B:
0 k = 10% 1 2 3 4 Years

CFsB -200,000 0 0 146,411 146,411


Time line:
0 k = 12% 1 2 10 Years
  
-50,000 6,000 6,000 6,000
Time lines:
Project A:
0 k = 12% 1 2 3 Periods

CFsA -5,000 2,000 2,500 2,250


Project B:
0 k = 14% 1 2 3 Periods

CFsB -5,000 3,000 2,600 2,900


0 1 2 3 4
| | | | |
CFsNew Tech -1,500 -315 -315 -315 -315 x x x .
Non-operating cash flows Answer: a Diff: M
Additional Year 3 cash flows:
3
Salvage value $25,000
Tax on Salvage value (8,880)*
Recovery of NOWC 2,000
$18,120

*(Market value - Book value)(Tax rate)


($25,000 - $2,800)(0.40) = $8,880.

Time line:
0 1 2 3 Years
k = 14%

-42,000 14,280 16,200 11,400


TV = 18,120
29,520
Time line:
0 1 2 3 Years
k = 10%

-62,000 19,920 22,800 15,600


TV = 15,680
31,280
NPV
$

B
A

Expected increase in annual net income k


ARR = 0 10%
195 Initial (or average) investment IRRB IRRA

NPV
($)

Discount rate (%)


0 16% 17% 18% 30%

NPV
$
NPV ($)
P
C

k 10% k
10% 15% 20% %

NPV NPV
$

A A

B
B n
Sigma CF t
k k t=1
0 7% 12% 15% 0 7% 12% 14%
(1 + K)t
NPV
$

X
Crossover

k
10% IRRY 12% IRRX

0 k = 12% 1 2 3 4 5 6

CFsA -15,000 4,000 4,000 4,000 4,000 4,000 4,000


Project B:
0 k = ? 1 2 6 Years
  
CFsB -14,815 5,100 5,100 5,100
Time lines:
Project A:
0 1 2 3 4 Years
k = 12%

CFsA -25,000 13,000 13,000 13,000 13,000


NPVA = ? = 17,663 Terminal value = 5,000
CF 4 = 18,000
Project B:
0 1 2 3 4 Years
k = ?

CFsB -25,000 15,247 15,247 15,247 15,247


NPVA = NPVB = 17,663 Terminal value = 0
CF 4 = 15,247
Time line:
0 1 2 3 Years
k = 18%

-3,000 1,728 1,920 1,152


Time line:
0 1 2 3 4 5 Years
k = 12%

-45,000 7,800 10,680 7,560 5,880 -1,920


Time line:
0 k = 9% 1 2 3 4 5 Years

-40,000 9,800 11,720 9,640 8,520 15,320


Time lines:
Project A:
0 k = 14% 1 2 3 4 Years

CFsA -200,000 71,104 71,104 71,104 71,104


Project B:
0 k = 10% 1 2 3 4 Years

CFsB -200,000 0 0 146,411 146,411


Time line:
0 k = 12% 1 2 10 Years
  
-50,000 6,000 6,000 6,000
Time lines:
Project A:
0 k = 12% 1 2 3 Periods

CFsA -5,000 2,000 2,500 2,250


Project B:
0 k = 14% 1 2 3 Periods

CFsB -5,000 3,000 2,600 2,900


0 1 2 3 4
| | | | |
CFsNew Tech -1,500 -315 -315 -315 -315 x x x .
New project NPVAnswer: c Diff: M
Time line:
0 1 2 3 Years
k = 14%

-42,000 14,280 16,200 11,400


TV = 18,120
29,520
Tabular solution:
NPVk = 14% = -$42,000 + $14,280(PVIF14%,1)
+ $16,200(PVIF14%,2) + $29,520(PVIF14%,3)
= -$42,000 + $14,280(0.8772) + $16,200(0.7695)
+ $29,520(0.6750) = $2,918.32.

Financial calculator solution:


Inputs: CF0 = -42,000; CF1 = 14,280; CF2 = 16,200; CF3 = 29,520; I = 14.
Output: NPV = $2,916.85  $2,917.

Note: Tabular solution differs from calculator solution due to interest


factor rounding.
Time line:
0 1 2 3 Years
k = 10%

-62,000 19,920 22,800 15,600


TV = 15,680
31,280
NPV
$

B
A

Expected increase in annual net income k


ARR = 0 10%
196 Initial (or average) investment IRRB IRRA

NPV
($)

Discount rate (%)


0 16% 17% 18% 30%

NPV
$
NPV ($)
P
C

k 10% k
10% 15% 20% %

NPV NPV
$

A A

B
B n
Sigma CF t
k k t=1
0 7% 12% 15% 0 7% 12% 14%
(1 + K)t
NPV
$

X
Crossover

k
10% IRRY 12% IRRX

0 k = 12% 1 2 3 4 5 6

CFsA -15,000 4,000 4,000 4,000 4,000 4,000 4,000


Project B:
0 k = ? 1 2 6 Years
  
CFsB -14,815 5,100 5,100 5,100
Time lines:
Project A:
0 1 2 3 4 Years
k = 12%

CFsA -25,000 13,000 13,000 13,000 13,000


NPVA = ? = 17,663 Terminal value = 5,000
CF 4 = 18,000
Project B:
0 1 2 3 4 Years
k = ?

CFsB -25,000 15,247 15,247 15,247 15,247


NPVA = NPVB = 17,663 Terminal value = 0
CF 4 = 15,247
Time line:
0 1 2 3 Years
k = 18%

-3,000 1,728 1,920 1,152


Time line:
0 1 2 3 4 5 Years
k = 12%

-45,000 7,800 10,680 7,560 5,880 -1,920


Time line:
0 k = 9% 1 2 3 4 5 Years

-40,000 9,800 11,720 9,640 8,520 15,320


Time lines:
Project A:
0 k = 14% 1 2 3 4 Years

CFsA -200,000 71,104 71,104 71,104 71,104


Project B:
0 k = 10% 1 2 3 4 Years

CFsB -200,000 0 0 146,411 146,411


Time line:
0 k = 12% 1 2 10 Years
  
-50,000 6,000 6,000 6,000
Time lines:
Project A:
0 k = 12% 1 2 3 Periods

CFsA -5,000 2,000 2,500 2,250


Project B:
0 k = 14% 1 2 3 Periods

CFsB -5,000 3,000 2,600 2,900


0 1 2 3 4
| | | | |
CFsNew Tech -1,500 -315 -315 -315 -315 x x x
Time line:
0 1 2 3 Years
k = 14%

-42,000 14,280 16,200 11,400


TV = 18,120
29,520
. New project investment Answer: d Diff: E
Initial investment:
Cost ($50,000)
Modification (10,000)
Change in NOWC (2,000)
Total net investment = ($62,000)
Time line:
0 1 2 3 Years
k = 10%

-62,000 19,920 22,800 15,600


TV = 15,680
31,280
NPV
$

B
A

Expected increase in annual net income k


ARR = 0 10%
197 Initial (or average) investment IRRB IRRA

NPV
($)

Discount rate (%)


0 16% 17% 18% 30%
NPV
$
NPV ($)
P
C

k 10% k
10% 15% 20% %

NPV NPV
$

A A

B
B n
Sigma CF t
k k t=1
0 7% 12% 15% 0 7% 12% 14%
(1 + K)t
NPV
$

X
Crossover

k
10% IRRY 12% IRRX

0 k = 12% 1 2 3 4 5 6

CFsA -15,000 4,000 4,000 4,000 4,000 4,000 4,000


Project B:
0 k = ? 1 2 6 Years
  
CFsB -14,815 5,100 5,100 5,100
Time lines:
Project A:
0 1 2 3 4 Years
k = 12%

CFsA -25,000 13,000 13,000 13,000 13,000


NPVA = ? = 17,663 Terminal value = 5,000
CF 4 = 18,000
Project B:
0 1 2 3 4 Years
k = ?

CFsB -25,000 15,247 15,247 15,247 15,247


NPVA = NPVB = 17,663 Terminal value = 0
CF 4 = 15,247
Time line:
0 k = 18% 1 2 3 Years

-3,000 1,728 1,920 1,152


Time line:
0 1 2 3 4 5 Years
k = 12%

-45,000 7,800 10,680 7,560 5,880 -1,920


Time line:
0 1 2 3 4 5 Years
k = 9%

-40,000 9,800 11,720 9,640 8,520 15,320


Time lines:
Project A:
0 k = 14% 1 2 3 4 Years

CFsA -200,000 71,104 71,104 71,104 71,104


Project B:
0 k = 10% 1 2 3 4 Years

CFsB -200,000 0 0 146,411 146,411


Time line:
0 k = 12% 1 2 10 Years
  
-50,000 6,000 6,000 6,000
Time lines:
Project A:
0 k = 12% 1 2 3 Periods

CFsA -5,000 2,000 2,500 2,250


Project B:
0 k = 14% 1 2 3 Periods

CFsB -5,000 3,000 2,600 2,900


0 1 2 3 4
| | | | |
CFsNew Tech -1,500 -315 -315 -315 -315 x x x
Time line:
0 1 2 3 Years
k = 14%

-42,000 14,280 16,200 11,400


TV = 18,120
29,520
. Operating cash flow Answer: c Diff: M
Depreciation schedule:
Depreciable basis = $60,000.
MACRS
Depreciation Depreciable Annual
Year Rate Basis Depreciation
1 0.33 $60,000 $19,800
2 0.45 60,000 27,000
3 0.15 60,000 9,000
4 0.07 60,000 4,200
$60,000

Operating cash flows:


Year 1 2 3
1) Before-tax cost reduction $20,000 $20,000 $20,000
2) After-tax cost reduction
(line 1  0.6) 12,000 12,000 12,000
3) Depreciation 19,800 27,000 9,000
4) Tax savings from deprec.
(line 3  0.4) 7,920 10,800 3,600
5) Net operating CFs $19,920 $22,800 $15,600
Time line:
0 1 2 3 Years
k = 10%

-62,000 19,920 22,800 15,600


TV = 15,680
31,280
NPV
$

B
A

Expected increase in annual net income k


ARR = 0 10%
198 Initial (or average) investment IRRB IRRA

NPV
($)

Discount rate (%)


0 16% 17% 18% 30%

NPV
$
NPV ($)
P
C

k 10% k
10% 15% 20% %
NPV NPV
$

A A

B
B n
Sigma CF t
k k t=1
0 7% 12% 15% 0 7% 12% 14%
(1 + K)t
NPV
$

X
Crossover

k
10% IRRY 12% IRRX

0 k = 12% 1 2 3 4 5 6

CFsA -15,000 4,000 4,000 4,000 4,000 4,000 4,000


Project B:
0 k = ? 1 2 6 Years
  
CFsB -14,815 5,100 5,100 5,100
Time lines:
Project A:
0 1 2 3 4 Years
k = 12%

CFsA -25,000 13,000 13,000 13,000 13,000


NPVA = ? = 17,663 Terminal value = 5,000
CF 4 = 18,000
Project B:
0 1 2 3 4 Years
k = ?

CFsB -25,000 15,247 15,247 15,247 15,247


NPVA = NPVB = 17,663 Terminal value = 0
CF 4 = 15,247
Time line:
0 k = 18% 1 2 3 Years

-3,000 1,728 1,920 1,152


Time line:
0 1 2 3 4 5 Years
k = 12%

-45,000 7,800 10,680 7,560 5,880 -1,920


Time line:
0 1 2 3 4 5 Years
k = 9%

-40,000 9,800 11,720 9,640 8,520 15,320


Time lines:
Project A:
0 k = 14% 1 2 3 4 Years

CFsA -200,000 71,104 71,104 71,104 71,104


Project B:
0 k = 10% 1 2 3 4 Years

CFsB -200,000 0 0 146,411 146,411


Time line:
0 k = 12% 1 2 10 Years
  
-50,000 6,000 6,000 6,000
Time lines:
Project A:
0 k = 12% 1 2 3 Periods

CFsA -5,000 2,000 2,500 2,250


Project B:
0 k = 14% 1 2 3 Periods

CFsB -5,000 3,000 2,600 2,900


0 1 2 3 4
| | | | |
CFsNew Tech -1,500 -315 -315 -315 -315 x x x
Time line:
0 1 2 3 Years
k = 14%

-42,000 14,280 16,200 11,400


TV = 18,120
29,520

. Non-operating cash flows Answer: c Diff: M


Additional Year 3 cash flows:
3
Salvage value $20,000
Tax on salvage value (6,320)*
Recovery of NOWC 2,000
Total terminal year CF $15,680

*(Market value - Book value)(Tax rate)


($20,000 - $4,200)(0.40) = $6,320.
Time line:
0 1 2 3 Years
k = 10%

-62,000 19,920 22,800 15,600


TV = 15,680
31,280
NPV
$

B
A

Expected increase in annual net income k


ARR = 0 10%
199 Initial (or average) investment IRRB IRRA

NPV
($)

Discount rate (%)


0 16% 17% 18% 30%

NPV
$
NPV ($)
P
C

k 10% k
10% 15% 20% %

NPV NPV
$

A A

B
B n
Sigma CF t
k k t=1
0 7% 12% 15% 0 7% 12% 14%
(1 + K)t
NPV
$

X
Crossover

k
10% IRRY 12% IRRX

0 k = 12% 1 2 3 4 5 6

CFsA -15,000 4,000 4,000 4,000 4,000 4,000 4,000


Project B:
0 k = ? 1 2 6 Years
  
CFsB -14,815 5,100 5,100 5,100
Time lines:
Project A:
0 1 2 3 4 Years
k = 12%

CFsA -25,000 13,000 13,000 13,000 13,000


NPVA = ? = 17,663 Terminal value = 5,000
CF 4 = 18,000
Project B:
0 1 2 3 4 Years
k = ?

CFsB -25,000 15,247 15,247 15,247 15,247


NPVA = NPVB = 17,663 Terminal value = 0
CF 4 = 15,247
Time line:
0 1 2 3 Years
k = 18%

-3,000 1,728 1,920 1,152


Time line:
0 1 2 3 4 5 Years
k = 12%

-45,000 7,800 10,680 7,560 5,880 -1,920


Time line:
0 k = 9% 1 2 3 4 5 Years

-40,000 9,800 11,720 9,640 8,520 15,320


Time lines:
Project A:
0 k = 14% 1 2 3 4 Years

CFsA -200,000 71,104 71,104 71,104 71,104


Project B:
0 k = 10% 1 2 3 4 Years

CFsB -200,000 0 0 146,411 146,411


Time line:
0 k = 12% 1 2 10 Years
  
-50,000 6,000 6,000 6,000
Time lines:
Project A:
0 k = 12% 1 2 3 Periods

CFsA -5,000 2,000 2,500 2,250


Project B:
0 k = 14% 1 2 3 Periods

CFsB -5,000 3,000 2,600 2,900


0 1 2 3 4
| | | | |
CFsNew Tech -1,500 -315 -315 -315 -315 x x x
Time line:
0 1 2 3 Years
k = 14%

-42,000 14,280 16,200 11,400


TV = 18,120
29,520
. New project NPVAnswer: a Diff: M
Time line:
0 1 2 3 Years
k = 10%

-62,000 19,920 22,800 15,600


TV = 15,680
31,280
Tabular solution:
NPVk = 10% = -$62,000 + $19,920(PVIF10%,1) + $22,800(PVIF10%,2)
+ $31,280(PVIF10%,3)
= -$62,000 + $19,920(0.9091) + $22,800(0.8264)
+ $31,280(0.7513) = -$1,548.14.

Financial calculator solution:


Inputs: CF0 = -62,000; CF1 = 19,920; CF2 = 22,800; CF3 = 31,280; I = 10.
Output: NPV = -$1,546.81  -$1,547.

Note: Tabular solution differs from calculator solution due to interest


factor rounding.

You might also like